You are on page 1of 190

CBSE Term II

2022

Contents
CHAPTER
Quadratic Equations -
CHAPTER
Arithmetic Progressions -
CHAPTER
Circles -
CHAPTER
Constructions -
CHAPTER
Applications of Trigonometry -

CHAPTER
Surface Areas and Volumes -

CHAPTER
Statistics -

Practice Papers - -

CLICK HERE TO GET MORE TERM 2 AND NTSE MATERIALS

Watch Free Learning Videos


Subscribe arihant Channel

þ Video Solutions of CBSE Sample Papers


þ Chapterwise Important MCQs
þ CBSE Updates
CBSE Term II
2022

Syllabus
CBSE Term II Class XII
One Paper Max Marks:
No. Units Marks
I. Algebra Cont.
II. Geometry Cont.
III. Trigonometry Cont.
IV. Mensuration Cont.
V. Statistics Probability Cont.

Total
Internal Assessment
Total

UNIT-I ALGEBRA
. Quadratic Equations Periods
Standard form of a quadratic equation ax + bx + c = , a ≠ . Solutions of quadratic equations
only real roots by factorisation, and by using quadratic formula. Relationship between
discriminant and nature of roots. Situational problems based on quadratic equations related to
day to day activities problems on equations reducible to quadratic equations are excluded

. Arithmetic Progressions
Motivation for studying Arithmetic Progression Derivation of the nth term and sum of the first n
terms of AP and their application in solving daily life problems. Applications based on sum to n
terms of an AP. are excluded

UNIT-II GEOMETRY
. Circles
Tangent to a circle at point of contact
. Prove The tangent at any point of a circle is perpendicular to the radius through the
point of contact.
. Prove The lengths of tangents drawn from an external point to a circle are equal.
CBSE Term II
2022

. Constructions
. Division of a line segment in a given ratio internally .
. Tangents to a circle from a point outside it.

UNIT-III TRIGONOMETRY
. Some Applications of Trigonometry
HEIGHTS AND DISTANCES-Angle of elevation, Angle of Depression.
Simple problems on heights and distances. Problems should not involve more than two right
triangles. Angles of elevation depression should be only , , .

UNIT-IV MENSURATION
. Surface Areas and Volumes
. Surface areas and volumes of combinations of any two of the following: cubes, cuboids,
spheres, hemispheres and right circular cylinders cones.
. Problems involving converting one type of metallic solid into another and other mixed
problems. Problems with combination of not more than two different solids be taken .

UNIT-V STATISTICS PROBABILITY


. Statistics
Mean, median and mode of grouped data bimodal situation to be avoided . Mean by Direct
Method and Assumed Mean Method only

Internal Assessment Marks Total Marks

Periodic Test 3
10 Marks
Multiple Assessments 2
for the
Portfolio 2
Term
Student Enrichment Activities-practical work 3
CBSE Circular
Acad - 51/2021, 05 July 2021

Exam Scheme Term I & II

dsUnh;
z ek/;fed f'k{kk cksMZ
(f'k{kk ea=ky;] Hkkjr ljdkj ds v/khu ,d Lok;r laxBu)

Hkkjr CENTRAL BOARD OF SECONDARY EDUCATION


(An Autonomous Organisation under the Ministryof Education, Govt. of India)
dsUnh;
z ek/;fed f'k{kk cksMZ
(f'k{kk ea=ky;] Hkkjr ljdkj ds v/khu ,d Lok;r laxBu)
Hkkjr
CENTRAL BOARD OF SECONDARY EDUCATION
(An Autonomous Organisation under the Ministryof Education, Govt. of India)
dsUnh;
z ek/;fed f'k{kk cksMZ
(f'k{kk ea=ky;] Hkkjr ljdkj ds v/khu ,d Lok;r laxBu)

Hkkjr CENTRAL BOARD OF SECONDARY EDUCATION


(An Autonomous Organisation under the Ministryof Education, Govt. of India)

To cover this
situation, we
have given
both MCQs and
Subjective
Questions in
each Chapter.
dsUnh;
z ek/;fed f'k{kk cksMZ
(f'k{kk ea=ky;] Hkkjr ljdkj ds v/khu ,d Lok;r laxBu)

Hkkjr CENTRAL BOARD OF SECONDARY EDUCATION


(An Autonomous Organisation under the Ministryof Education, Govt. of India)
CBSE Term II Mathematics X (Standard) 1

CHAPTER 01

Quadratic
Equations
CLICK HERE TO GET MORE TERM 2 AND NTSE MATERIALS
In this Chapter...
! Quadratic Equation and its Solutions
! Solution of a Quadratic Equation by Factorisation
! Solution of a Quadratic Equation by Quadratic Formula
! Relationship between Discriminant and Nature of Roots

An equation of the form ax 2 + bx + c = 0 is called quadratic Solutions or Roots of a Quadratic Equation


equation in variable x, where a , b and c are real numbers and All the values of variable which satisfy the given quadratic
a ≠ 0. equation, are called roots or zeroes or solutions of given
quadratic equation.
e.g. 2 x 2 + x − 100 = 0, − x 2 + 1 + 300 x = 0,
In other words, a real number α is said to be a root or zero or
4 x − 3 x 2 + 7 = 0, 4 x 2 − 25 = 0 are quadratic equations. solution of a quadratic equation ax 2 + bx + c = 0 , a ≠ 0,
The form ax 2 + bx + c = 0, a ≠ 0 is called the standard form of if a(α ) 2 + b(α ) + c = 0.
a quadratic equation. Any quadratic equation can have atmost two roots.
To express a quadratic equation in its standard form, write Method to Check Whether the Given Value is a
the terms of given equation in the descending order of their Solution of the Given Quadratic Equation
degrees.
Let p( x) = 0 be the given quadratic equation and x = α be the
e.g. 3 x 2 + x + 2 = 0 and x 2 − 2 x + 6 = 0, are in standard form given value of x.
whereas, x 2 − 3 + 4 x = 0 and x + x 2 + 8 = 0 are not in their To check whether x = α is a solution of the given equation or
standard form. not, use the following steps
Method to Check Whether a Given Equation is Case I Write the given equation in the form, p( x) = 0.
Quadratic or Not Case II Now, put x = α in p( x). If p(α ) = 0, then x = α is the
To check whether a given equation is quadratic or not, first solution of given equation, otherwise not.
write the given equation in its simplest form and then Method to Determine An Unknown Constant in a
compare the equation with the standard form of a quadratic
equation, Quadratic Equation when its Solution or Root is Given

i.e. ax 2 + bx + c = 0, a ≠ 0. I. Sometimes, given quadratic equation involves one unknown


constant and its solution or root is given. Then, to find the
If the given equation follows the form of quadratic equation value of unknown constant, we put the value of root or
( ax 2 + bx + c = 0 , a ≠ 0 ), then it is a quadratic equation solution in given quadratic equation and simplify it to get the
otherwise not. required unknown constant.
2 CBSE Term II Mathematics X (Standard)

II. Sometimes, quadratic equation involves two unknown constants So, the quadratic equation has no real roots or
and its both roots are given. Then, to find unknowns we put both
roots one-by-one in the quadratic equation and get two linear
imaginary roots or we can say that roots of
equations in two unknowns. On solving these equations, we get quadratic equation does not exist. This can be
the required values of unknown constants. explained using the flow chart.

Solution of a Quadratic Equation Quadratic equation


ax2+bx+c=0, a ≠ 0
by Factorisation
To find the solution of a quadratic equation by factorisation Find discriminant, D
method, we use the following steps.
Step I Write the given equation in standard form
i.e. ax 2 + bx + c = 0 (if not given in standard form) and D=0 D>0
D<0
find the value of a , b and c. ⇒ Roots are real ⇒ Roots are
⇒ Roots are
and equal imaginary or
Step II Find the product of a and c and write it as a sum of its real and distinct
real roots
two factor such that sum is equal to b. i.e. write does not exist
ac = p × q and p + q = b where, p and q are factors of ac.
Step III Put the value of b obtained from step II in given Method to Determine The Value of Unknown
equation and write it LHS as product of two linear when Nature of Roots is Given
factors. If nature of roots of a quadratic equation is given and
Step IV Now, equate each factor equal to zero and get desired quadratic equation involves an unknown. Then to find the
roots of given quadratic equation. value of unknown, first we find the value of discriminant
in terms of unknown. After that use the given condition
Solution of a Quadratic Equation by i.e. D > 0 or D = 0 or D < 0 and simplify it.
Quadratic Formula Some Important Points
In a quadratic equation ax 2 + bx + c = 0, a ≠ 0, if b 2 − 4 ac ≥ 0 ,
(i) Three consecutive numbers are x,( x +1 ) and (x + 2),
then the roots of the quadratic equation are given by
respectively.
− b ± b 2 − 4 ac −b ± D (ii) Three consecutive even and odd numbers are
x= or x =
2a 2a 2 x,(2 x + 2 ), (2 x + 4 ) and (2 x + 1 ), (2 x + 3 ), (2 x + 5 ),
respectively.
where, D = b 2 − 4 ac is known as discriminant. This result is
known as quadratic formula or Sridharacharya formula. (iii) Pythagoras theorem,
(Hypotenuse) 2 = (Perpendicular) 2 + (Height) 2
Relationship between Discriminant and
1
Nature of Roots (iv) Area of triangle =
2
× Base × Height
The nature of roots depends upon the value of the discriminant
(v) Area of right angled triangle
D, whereas, D can be zero, positive or negative, so three cases 1
may arise. = × Base × Perpendicular
2
Case I When D = 0 i.e. b 2 − 4 ac = 0.
(vi) Area of rectangle = Length × Breadth
−b ± 0 b b
If D = b 2 − 4 ac = 0, then x = ⇒x= − ,− (vii) Perimeter of rectangle = 2 × (Length + Breadth)
2a 2a 2a
Distance
So, the quadratic equation has two equal real roots or (viii) Speed =
Time
repeated roots or coincident roots.
Case II When D > 0 i.e. b 2 − 4 ac > 0. (ix) Two-digit number = 10x + y, where x and y are the
digits of ten’s place and unit place, respectively.
−b + D −b − D
If D = b 2 − 4 ac > 0, then x = and On reversing the digits, new number = 10y + x
2a 2a
(x) If speed of stream be x km/h and speed of boat in
So, the quadratic equation has two distinct real roots.
still water be y km/h. Then speed of boat in
Case III When D < 0 i.e. b 2 − 4 ac < 0. upstream = ( y − x) km/h and speed of boat in
If D = b 2 − 4 ac < 0, then D can not be evaluated as downstream = ( y + x) km/h.
square root of negative value is not defined.
CBSE Term II Mathematics X (Standard) 3

Solved Examples
1+2 −3
Example 1. Check whether the following equations are = =0
3
quadratic or not. 1
So, x = is a root of the given equation.
3 3
(i) x + = x 2 2 2
(ii) 2 x − 5 x = x − 2 x + 3
x 1
(iii) On putting x = − in Eq. (i), we get
2 1 2
(iii) x − 2 = 5 (iv) x 2 − 3 x − x + 4 = 0 2
x ⎛ 1⎞ ⎛ 1⎞ ⎛ 1⎞
p ⎜ − ⎟ = 3⎜ − ⎟ + 2 ⎜ − ⎟ − 1
Sol. 3 ⎝ 2⎠ ⎝ 2⎠ ⎝ 2⎠
(i) Given that, x + = x2
x 3
= −1−1
⇒ x2 + 3 = x3 4
⇒ x3 − x2 − 3 = 0 3
= −2 =
3 − 8 −5
= ≠0
Which is not of the form ax 2 + bx + c, a ≠ 0. 4 4 4
1
Thus, the equation is not a quadratic equation. So, x = − is not a root of the given equation.
2
(ii) Given that, 2 x 2 − 5x = x 2 − 2 x + 3
(iv) On putting x = 2 in Eq. (i), we get
⇒ 2 x 2 − x 2 − 5x + 2 x − 3 = 0
p(2 ) = 3(2 )2 + 2(2 ) − 1
⇒ x 2 − 3x − 3 = 0
= 12 + 4 − 1 = 15 ≠ 0
Which is of the form ax 2 + bx + c, a ≠ 0. So, x = 2 is not a root of the given equation.
Thus, the equation is a quadratic equation.
1
Example 3. In each of the following equations, find the
(iii) Given that, x 2 − 2 = 5 value of unknown constant(s) for which the given
x
value(s) is (are) solution of the equations.
⇒ x 4 − 1 = 5x 2
⇒ x 4 − 5x 2 − 1 = 0
(i) x 2 − k 2 = 0; x = 0. 3
Which is not of the form ax 2 + bx + c, a ≠ 0. −1
(ii) 3 x 2 + 2 ax − 3 = 0; x =
Thus, the equation is not a quadratic equation. 2
(iv) Given that, x 2 − 3x − x + 4 = 0 Sol. (i) We have, x 2 − k 2 = 0, here k is unknown.

Which is not of the form ax 2 + bx + c, a ≠ 0. Since, x = 0. 3 is a solution of given equation, so it will


satisfy the given equation,
Thus, the equation is not a quadratic equation.
On putting x = 0. 3 in the given equation, we get
Example 2. Which of the following are the roots of ( 0. 3)2 − k 2 = 0
3x 2 + 2 x − 1 = 0 ? ⇒ k 2 = ( 0. 3)2
1 ⇒ k = ± 0. 3
(i) x = − 1 (ii) x =
3 (ii) We have, 3x 2 + 2 ax − 3 = 0, here a is unknown.
1
(iii) x = − (iv) x = 2 1
2 Since, x = − is a solution of given equation, so it will
2
Sol. Given equation is of the form p( x ) = 0, where satisfy the given equation.
p( x ) = 3x 2 + 2 x − 1 …(i) 1
On putting x = − in the given equation, we get
(i) On putting x = − 1 in Eq. (i), we get 2
2
p( − 1) = 3( − 1)2 + 2( − 1) − 1 ⎛ 1⎞ ⎛ 1⎞
3⎜ − ⎟ + 2 a ⎜ − ⎟ − 3 = 0
= 3−2 −1= 0 ⎝ 2⎠ ⎝ 2⎠
So, x = − 1 is a root of the given quadratic equation. 3
⇒ −a −3=0
1 4
(ii) On putting x = in Eq. (i), we get
3 3
2 ⇒ a= −3
⎛ 1⎞ ⎛ 1⎞ ⎛ 1⎞ 4
p ⎜ ⎟ = 3⎜ ⎟ + 2 ⎜ ⎟ − 1
⎝ 3⎠ ⎝ 3⎠ ⎝ 3⎠ 3 − 12 9
⇒ a= =−
1 2 4 4
= + −1
3 3
4 CBSE Term II Mathematics X (Standard)

x + 5 − ( x − 3) 1
Example 4. Find the roots of the quadratic equation ⇒ =
( x − 3)( x + 5) 6
5
2x 2 + x − 2 = 0 by factorisation method. x+ 5−x+ 3 1
3 ⇒ = ⇒ 8 × 6 = ( x − 3) ( x + 5 )
5 ( x − 3) ( x + 5) 6
Sol. Given equation is 2 x 2 +x −2 = 0
3 ⇒ 48 = x 2 + 2 x − 15
On multiplying by 3 both sides, we get ⇒ x 2 + 2 x − 63 = 0
2
6x + 5x − 6 = 0 ⇒ x 2 + 9x − 7 x − 63 = 0 [by splitting the middle term]
2
⇒ 6x + ( 9x − 4x ) − 6 = 0 ⇒ x( x + 9) − 7( x + 9) = 0
[by splitting the middle term] ⇒ ( x − 7 ) ( x + 9) = 0
⇒ 6x 2 + 9x − 4x − 6 = 0 ⇒ x = 7 and x = − 9
⇒ 3 x ( 2 x + 3 ) − 2 ( 2 x + 3) = 0 ∴Sum of roots = 7 + ( − 9) = − 2
⇒ (2 x + 3 ) ( 3 x − 2 ) = 0 Example 7. Using the quadratic formula, solve the
Now, 2x + 3 = 0 quadratic equation.
⇒ x=−
3 x 2 + 2 2 x − 6 = 0.
2 Sol. Given equation is x 2 + 2 2 x − 6 = 0 .
and 3x − 2 = 0
On comparing with ax 2 + bx + c = 0, we get
2
⇒ x=
3 a = 1, b = 2 2 and c = − 6
2 5 −b± b 2 − 4ac
Hence, the roots of the equation 2 x + x − 2 = 0 are
3 By quadratic formula, x =
2a
−3 2
and . − (2 2 ) ± (2 2 )2 − 4 (1) ( −6)
2 3 =
2 (1 )
Example 5. Solve the quadratic equation by
factorisation method. − 2 2 ± 8 + 24
=
2
3 2 x 2 − 5x − 2 = 0
−2 2 ± 32 − 2 2 ± 4 2
Sol. Given equation is 3 2 x 2 − 5x − 2 = 0 = =
2 2
3 2 x 2 − ( 6x − x ) − 2 = 0 −2 2 + 4 2 −2 2 − 4 2
= ,
[by splitting the middle term] 2 2
3 2 x 2 − 6x + x − 2 = 0 = 2, − 3 2
3 2 x2 − 3 2 ⋅ 2 x + x − 2 = 0 So, 2 and − 3 2 are the roots of the given equation.
⇒ 3 2 x (x − 2) + 1 (x − 2 ) = 0 Example 8. Find discriminant of the quadratic equation
⇒ (x − 2) (3 2x + 1) = 0 3x 2 + 4x − 5 = 0.
Now, x− 2 =0 ⇒ x= 2 Sol. Comparing the given quadratic equation
and 3 2 x+1=0 3x 2 + 4x − 5 = 0
1 − 2 with standard quadratic equation ax 2 + bx + c = 0, we get
⇒ x=− =
3 2 6 a = 3, b = 4 and c = − 5
2
Hence, the roots of the equation 3 2 x − 5x − 2 = 0 are ∴ Discriminant (D ) = b 2 − 4ac
2 = ( 4)2 − 4 × ( 3) × ( − 5) = 16 + 60 = 76
− and 2 .
6
Example 9. Check whether the quadratic equation has
Example 6. Find the sum of the roots of the equation, real roots. If real roots exist, find them
⎡ 1 1 1⎤ 8x 2 + 2x − 3 = 0
⎢ x − 3 − x + 5 = 6 ⎥.
⎣ ⎦ Sol. Given equation is 8x 2 + 2 x − 3 = 0.
⎡ 1 1 ⎤ 1
Sol. Given ⎢ − = On comparing with ax 2 + bx + c = 0, we get
⎣ x − 3 x + 5 ⎥⎦ 6
a = 8, b = 2 and c = − 3
∴Discriminant, D = b 2 − 4ac
CBSE Term II Mathematics X (Standard) 5

= ( 2 ) 2 − 4 ( 8 ) ( − 3) On comparing with ax 2 + bx + c = 0, we get


= 4 + 96 = 100 > 0 a = 2 , b = 1 and c = − 1
Therefore, the equation 8x 2 + 2 x − 3 = 0 has two distinct real ∴ Discriminant,
roots as the discriminant greater than zero. D = b 2 − 4ac = (1 )2 − 4 (2 ) ( − 1 )
− b ± D − 2 ± 100 − 2 ± 10 = 1 + 8 = 9 > 0 i.e. D > 0
Thus roots, x = = =
2a 16 16 Hence, the equation 2 x 2 + x − 1 = 0 has two distinct
− 2 + 10 − 2 − 10 real roots.
= ,
16 16 9
(iii) Given equation is 2 x 2 − 6x + = 0.
8 12 1 3 2
= ,− = ,−
16 16 2 4 On comparing with ax 2 + bx + c = 0, we get
Example 10. Find the nature of roots of the quadratic a = 2 , b = − 6 and c =
9
equation 3x 2 − 4 3x + 4 = 0. 2
∴ Discriminant, D = b 2 − 4ac
If the roots are real, find them. [CBSE 2020 (Standard)]
⎛ 9⎞
Sol. Given quadratic equation is = ( − 6 ) 2 − 4 (2 ) ⎜ ⎟
2 ⎝ 2⎠
3x − 4 3x + 4 = 0
= 36 − 36 = 0
Compare with standard quadratic equation
i.e. D = 0
ax 2 + bx + c = 0, we get
9
a = 3, b = −4 3 and c = 4 Hence, the equation 2 x 2 − 6x + = 0 has equal and
2
Now, discriminant = b 2 − 4ac real roots.
= ( − 4 3 )2 − 4 × 3 × 4 Example 12. The quadratic equation x 2 − 4x + k = 0 has
= 48 − 48 = 0 distinct real roots, if k = 4. Why or why not?
Hence, roots are real and equal. Sol. Given quadratic equation is x 2 − 4x + k = 0
By using Sridharacharya formula, Compare with standard equation ax 2 + bx + c = 0, we get
−b ± D a = 1, b = − 4 and c = k
x=
2a The condition for distinct real root is b 2 − 4ac > 0
− ( −4 3 ) ± 0
= ⇒ ( − 4) 2 − 4 × 1 × k > 0
2×3
⇒ 16 − 4k > 0
4 3 2 3
= = ⇒ 16 > 4k
2×3 3
16
2 3 2 3 ⇒ k< ⇒ k<4
Hence, roots of given quadratic equation are and . 4
3 3
Example 13. Find the value of k, for which the
Example 11. State whether the following quadratic quadratic equation ( k + 4)x 2 + ( k + 1 )x + 1 = 0 has
equations have two distinct real roots. Justify your
equal roots. [CBSE 2020 (Standard)]
answer.
Sol. Given, quadratic equation is
(i) x 2 − 3 x + 4 = 0
( k + 4 )x 2 + ( k + 1 ) x + 1 = 0
(ii) 2 x 2 + x − 1 = 0
Compare with ax 2 + bx + c = 0, we get
9
(iii) 2 x 2 − 6 x + = 0 a = k + 4, b = k + 1 and c = 1
2
Condition for equal roots, b 2 − 4ac = 0
Sol. (i) Given equation is x 2 − 3x + 4 = 0.
∴ ( k + 1)2 − 4 × ( k + 4)(1) = 0
On comparing with ax 2 + bx + c = 0, we get
⇒ k 2 + 12 + 2 k − 4k − 16 = 0
a = 1, b = − 3 and c = 4
[Q( a + b )2 = a 2 + b 2 + 2 ab ]
∴ Discriminant,
D = b 2 − 4ac = ( −3)2 − 4 (1 ) ( 4) ⇒ k 2 − 2 k − 15 = 0
2
= 9 − 16 = − 7 < 0 ⇒ k − ( 5 − 3)k − 15 = 0
i.e. D<0 [by splitting middle term]
Hence, the equation x 2 − 3x + 4 = 0 has no real root . ⇒ k 2 − 5k + 3k − 15 = 0
(ii) Given equation is 2 x 2 + x − 1 = 0 ⇒ k( k − 5) + 3( k − 5) = 0
6 CBSE Term II Mathematics X (Standard)

⇒ ( k + 3) ( k − 5) = 0 would have been 11 more than five times her actual


⇒ k = − 3, 5 age, what is her age now?
Example 14. The denominator of a fraction is 3 more Sol. Let the actual age of Zeba = x yr
than its numerator. The sum of the fraction and its Her age when she was 5 yr younger = ( x – 5) yr
29 Now, by given condition,
reciprocal is . Find the fraction.
10 Square of her age = 11 more than five times her actual age
Sol. Let numerator = x ( x – 5)2 = 5 × actual age + 11
Then denominator = x + 3 ⇒ ( x – 5)2 = 5x + 11
x
∴ The fraction is the form of ⇒ x 2 + 25 – 10x = 5x + 11
x+3
⇒ x 2 – 15x + 14 = 0
According to the question,
2
x x + 3 29 ⇒ x – 14x – x + 14 = 0 [by splitting the middle term]
+ =
x+3 x 10 ⇒ x ( x – 14) – 1 ( x – 14) = 0
x 2 + ( x + 3)2 29 ⇒ ( x – 1) ( x – 14) = 0
⇒ =
x ( x + 3) 10 ⇒ x = 14
⇒ 10 ( x 2 + x 2 + 9 + 6x ) = 29 ( x 2 + 3x ) [here, x ≠ 1 because her age is x – 5. So, x – 5 = 1 – 5 = – 4
i.e. age cannot be negative]
⇒ 20x 2 + 60x + 90 = 29x 2 + 87 x Hence, required Zeba’s age now is 14 yr.
⇒ 9x 2 + 27 x − 90 = 0
Example 17. A two-digit number is such that the
⇒ x 2 + 3x − 10 = 0 [divide by 9] product of its digit is 35. When 18 is added to the
⇒ x 2 + 5x − 2 x − 10 = 0 [by splitting middle term] number the digits interchange their places. Find
the number.
⇒ x ( x + 5) − 2 ( x + 5) = 0
Sol. Let the ten’s digit number be x.
⇒ ( x + 5) ( x − 2 ) = 0
According to the question,
⇒ x + 5 = 0 and x − 2 = 0
Product of the digits = 35
⇒ x = − 5 and x = 2
i.e. Ten’s digits × Unit digit = 35
Example 15. Find a natural number whose square 35
⇒ Units digit =
diminished by 84 is equal to thrice of 8 more than x
the given number. 35
∴ Two digit number = 10x +
Sol. Let n be a required natural number. x
Square of a natural number diminished by 84 = n 2 – 84 Also it is given that if 18 is added to the number, the digits
And thrice of 8 more than the natural number = 3 ( n + 8) gets interchange.
Now, by given condition, 35 35
∴ 10x + + 18 = 10 × +x
n 2 – 84 = 3 ( n + 8) x x
10x 2 + 35 + 18x 350 + x 2
⇒ n 2 – 84 = 3n + 24 ⇒ =
x x
⇒ n 2 – 3n – 108 = 0
⇒ 9x 2 + 18x − 315 = 0
⇒ n 2 – 12 n + 9n – 108 = 0
⇒ x 2 + 2 x − 35 = 0 [divide by 9]
[by splitting the middle term]
2
⇒ n ( n – 12 ) + 9 ( n – 12 ) = 0 ⇒ x + 7 x − 5x − 35 = 0
⇒ ( n – 12 ) ( n + 9) = 0 ⇒ x (x + 7) − 5 (x + 7) = 0
⇒ n = 12 ⇒ ( x − 5) ( x + 7 ) = 0
[Q n ≠ – 9 because n is a natural number] ⇒ x = 5, − 7
Hence, the required natural number is 12. But a digit can never be negative.
So, x = − 7 is rejected.
Example 16. If Zeba were younger by 5 yr than what
∴ The required number is
she really is, then the square of her age (in years)
10 × 5 + 5 = 50 + 5
= 55
CBSE Term II Mathematics X (Standard) 7

Chapter
Practice
PART 1
Objective Questions
!
Multiple Choice Questions 8. A quadratic equation with integral coefficient has
1. Which of the following is a quadratic equation? integral roots.
[NCERT Exemplar] (a) True (b) False
(a) x 2 + 2 x + 1 = ( 4 − x )2 + 3 (c) Can’t determined (d) None of these

⎛ 2⎞ 9. If b = 0, c < 0, then the roots of x 2 + bx + c = 0 are


(b) − 2 x 2 = ( 5 − x ) ⎜2 x − ⎟
⎝ 5⎠ numerically equal and opposite in sign.
2 3 [NCERT Exemplar]
(c) ( k + 1 ) x + x = 7, where k = − 1
2 (a) True (b) False
(d) x 3 − x 2 = ( x − 1)3 (c) Can’t determined (d) None of these

2. Which of the following is not a quadratic equation? 10. The roots of the quadratic equation
[NCERT Exemplar] x 2 − 8 x − 20 = 0 are
(a) 2 ( x – 1 )2 = 4x 2 – 2 x + 1 (b) 2 x – x 2 = x 2 + 5 (a) 5, − 4 (b) − 4, 5 (c) 10, − 2 (d) − 10, 2
2 2 2 2 4 2
(c) ( 2 x + 3 ) = 3x – 5x (d) ( x + 2 x ) = x + 3 + 4x 11. Which constant must be added and subtracted to
3
3. If a number x is added to twice its square, then the solve the quadratic equation 9x 2 + x − 2 = 0.
resultant is 21. Then the quadratic representation of 4
[NCERT Exemplar]
this statement is 1 1 1 9
2
(a) 2x − x + 21 = 0 2
(b) 2x + x − 21 = 0 (a) (b) (c) (d)
8 64 4 64
(c) 2x 2 − x − 20 = 0 (d) None of these
12. Solve 12 x 2 + 5x − 3 = 0.
4. Which of the following equations has 2 as a root? 1 4
(a) ,
1 3
(b) , (c) −
1 3
,
1
(d) , −
3
2 2 3 3 2 4 3 4 3 4
(a) x − 4x + 5 = 0 (b) x + 3x − 12 = 0
(c) 2 x 2 − 7 x + 6 = 0 (d) 3x 2 − 6x − 2 = 0 13. The discriminant of the quadratic
1 5 equation x 2 − 4x + 1 = 0 is [CBSE 2013]
5. If is a root of the equation x 2 + kx − = 0, then
2 4 (a) 2 3 (b) 4 (c) 12 (d) 16
the value of k is [NCERT Exemplar] 14. If the discriminant of the equation
1 1
(a) 2 (b) −2 (c) (d) 6x 2 − bx + 2 = 0 is 1, then the value of b is [CBSE 2012]
4 2 (a) 7 (b) − 7
6. Which of the following equation has root as 3? (c) Both (a) and (b) (d) None of these
(a) x 2 – 5x + 6 = 0 (b) – x 2 + 3x – 3 = 0 15. Value(s) of k for which the quadratic equation
3
(c) 2 x 2 – x+1=0 2
(d) 3x – 3x + 3 = 0 2 x 2 − kx + k = 0 has equal roots is/are [NCERT]
2
(a) 0 (b) 4 (c) 8 (d) 0, 8
7. 0.2 is a root of the equation x 2 − 0. 4 = 0? 2
16. The quadratic equation 2 x − 5x + 1 = 0 has
[NCERT Exemplar]
(a) True (b) False (a) two distinct real roots (b) two equal real roots
(c) Can’t determined (d) None of these (c) no real roots (d) more than 2 real roots
8 CBSE Term II Mathematics X (Standard)

17. If the discriminant of the equation (ii) Which of the following quadratic equation
describe the speed of Raj’s car?
kx 2 − 3 2 x + 4 2 = 0 is 14, then the value of k is
(a) x 2 − 5x − 500 = 0 (b) x 2 + 4x − 400 = 0
1 1 1
(a) 2 (b) (c) (d) 2
(c) x + 5x − 500 = 0 (d) x 2 − 4x + 400 = 0
3 2 2 4 2
18. Which of the following equations has two distinct (iii) What is the speed of Raj’s car?
(a) 20 km/h (b) 15 km/h
real roots? [NCERT Exemplar]
9 (c) 25 km/h (d) 10 km/h
(a) 2 x 2 − 3 2 x +
=0 (b) x 2 + x − 5 = 0
4 (iv) How much time took Ajay to travel 400 km?
2
(c) x + 3x + 2 2 = 0 (d) 5x 2 − 3x + 1 = 0 (a) 20 h (b) 40 h (c) 25 h (d) 16 h
(v) How much time took Raj to travel 400 km?
19. Which of the following equations has no real roots? (a) 15 h (b) 20 h (c) 18 h (d) 22 h
(a) x 2 − 4x + 3 2 = 0 (b) x 2 + 4x − 3 2 = 0
(c) x 2 − 4x − 3 2 = 0 (d) 3x 2 + 4 3x + 4 = 0
25. The speed of a motor boat is 20 km/h. For covering
the distance of 15 km the boat took 1 h more for
20. ( x 2 + 1 )2 − x 2 = 0 has [NCERT Exemplar] upstream than downstream. [CBSE Question Bank]
(a) four real roots (b) two real roots
(c) no real roots (d) one real root
21. The sum of the squares of three consecutive
integers is 110, then the smallest positive integer is
[NCERT Exemplar]
(a) 6 (b) 5 (c) 7 (d) 4
Downstream (a) Upstream (b)
22. A line segment AB is 8 cm in length. AB is
produced to P such that BP 2 = AB ⋅ AP. Then, the
length of BP is [NCERT Exemplar]
(a) 5( 5 + 1) (b) 5 + 1
(c) 4( 5 + 1) (d) 3 + 1

23. One year ago, a man was 8 times as old as his son.
Direction of boat Direction of boat
Now, his age is equal to the square of his son’s age. Direction of stream Direction of stream
Present age of man is
(a) 49 yr (b) 37 yr (i) Let speed of the stream be x km/h, then speed of
(c) 59 yr (d) 39 yr the motorboat in upstream will be
(a) 20 km/h (b) (20 + x ) km/h
!
Case Based MCQs (c) (20 − x ) km/h (d) 2 km/h
24. Raj and Ajay are very close friends. Both the (ii) What is the relation between speed, distance and
families decide to go to Ranikhet by their own cars. time?
Distance
Raj’s car travels at a speed of x km/h while Ajay’s (a) Speed =
car travels 5 km/h faster than Raj’s car. Raj took Time
Speed
4 h more than Ajay to complete the journey of (b) Distance =
Time
400 km. [CBSE Question Bank]
(c) Time = Speed × Distance
(d) Speed = Distance × Time
(iii) Which is the correct quadratic equation for the
speed of the current ?
(a) x 2 + 30x − 200 = 0 (b) x 2 + 20x − 400 = 0
(c) x 2 + 30x − 400 = 0 (d) x 2 − 20x − 400 = 0
(iv) What is the speed of current ?
(a) 20 km/h (b) 10 km/h
(i) What will be the distance covered by Ajay’s car in (c) 15 km/h (d) 25 km/h
two hours? (v) How much time boat took in downstream?
(a) 2 ( x + 5) km (b) ( x – 5) km (a) 90 min (b) 15 min
(c) 2 ( x + 10) km (d) (2 x + 5) km (c) 30 min (d) 45 min
CBSE Term II Mathematics X (Standard) 9

26. By quadratic formula, the roots of the quadratic


equation ax 2 + bx + c = 0, a ≠ 0 are given by PART 2
x=
−b ± b 2 − 4ac −b ± D
or x = Subjective Questions
2a 2a
where, D = b 2 − 4ac is called discriminant. !
Short Answer Type Questions
(i) The roots of the quadratic equation 1. Check whether the following are quadratic
8 x 2 − 22 x − 21 = 0 are equations or not.
7 3 7 3 (i) ( x − 1 ) ( x + 2 ) = ( x − 3 ) ( x + 1 )
(a) − ,− (b) ,
2 4 2 4
7 3 7 3
(ii) ( x + 2 ) 2 = 4 ( x + 3 )
(c) , − (d) − ,
2 4 2 4 1
2
2. If x = is root of the equation
(ii) The discriminant of x + x + 7 = 0 is 3
(a) 27 (b) − 27 Px 2 + ( 3 − 2 ) x − 1 = 0, then find the value of
(c) 27 (d) −27
P 2 + 1. [NCERT Exemplar]
(iii) Roots of 4 x 2 − 2 x = 3 are
(a) Real and distinct (b) Real and equal 3. In each of the following equations, determine the
(c) Imaginary (d) More than two real roots value of k for which the given value is a solution of
the equation.
(iv) The value of k for which
4 x 2 + kx + 9 = 0 has real and equal roots is (i) kx 2 + 2 x − 3 = 0 , x = 2
(a) 12 (b) − 12 (ii) x 2 + 2 ax − k = 0 , x = − a
(c) Both (a) and (b) (d) None of these
4. Find the value of k in the following equations
(v) The least positive value of k for which
x 2 + kx + 16 = 0 has real roots, is (i) x 2 − 2 kx − 6 = 0, when x = 3
5 1
(a) 18 (b) 4 (ii) x 2 − kx − = 0, when x =
(c) 2 (d) 8 4 2
27. Seven years ago, Varun’s age was five times the −1 1
5. Determine whether x = , x = are the solutions
square of Swati’s age. Three years hence, Swati’s 2 3
age will be two-fifth of Varun’s age. of the given equation 6x 2 − x − 2 = 0, or not.
(i) If seven years ago, Swati’s age be 6. Solve the quadratic equation by factorisation
x yr, then Varun’s age is
method.
(a) ( 5x − 7 )2 yr (b) 5x 2 yr
2 4 3x 2 + 5x − 2 3 = 0
(c) ( 5x + 7 ) yr (d) ( 5x 2 − 7 ) yr
16 15
(ii) After three years, Swati’s age is 7. Solve for x : − 1 = ; x ≠ 0 , − 1.
(a) ( x + 3) yr (b) ( x − 3) yr
x x +1
(c) ( x + 7 ) yr (d) ( x + 10) yr 8. Find the roots of the equation
(iii) The quadratic equation related to the given ax 2 + a = a 2 x + x. [CBSE 2012]
problem is
9. Solve for x , 6x + 7 − (2 x − 7 ) = 0 [CBSE 2016]
(a) 2 x 2 − x − 6 = 0 (b) 5x 2 − x + 6 = 0
2 2
(c) 3x − 2 x + 5 = 0 (d) 7 x − 3x + 1 = 0 10. Find the numerical difference of the
(iv) Present age of Varun’s is roots of equation x 2 − 7 x − 18 = 0. [CBSE 2015]
(a) 27 yr (b) 20 yr 11. Using the quadratic formula, solve the quadratic
(c) 30 yr (d) 37 yr equation.
(v) If Swati’s present age 10 yr, then present age of 3x 2 + 11 x + 6 3 = 0
Varun’s is
(a) 40 yr (b) 47 yr 12. If the discriminant of the equation 5x 2 − sx + 4 = 0
(c) 45 yr (d) 52 yr is 1, then find the value of s.
10 CBSE Term II Mathematics X (Standard)

13. Show that ( x 2 + 1 )2 − x 2 = 0 has no real roots. 27. The sum of the reciprocals of Anjali’s age 3 yr ago
[NCERT Exemplar] 1
and 5 yr from now is . Find the present age of
14. Find the value of k for which the quadratic equation 3
Anjali.
2 x 2 − kx + k = 0 has equal roots. [NCERT Exemplar]
28. ‘A two-digit number is such that the product of the
15. Find the values of k for which the equation digits is 12. When 36 is added to the number the
9x 2 + 3kx + 4 = 0 has real roots. digits interchange their places. Find the two-digit
16. If the equation (1 + m 2 )x 2 + (2 mc )x + ( c 2 − a 2 ) = 0 number.
has equal roots, then prove that c 2 = a 2 (1 + m 2 ). 29. ‘‘John and Janvi together have 45 marbles. Both of
them lost 5 marbles each and the product of the
17. The sum of two numbers is 11 and the sum of their number of marbles they now have, is 124. Find out
11 how many marbles they had to start with?’’
reciprocals is . Find the numbers.
28 [CBSE 2013]
30. The hypotenuse of right angled triangle is 6 m more
18. In a cricket match. Harbhajan took three wickets than twice the shortest side. If the third side is 2 m
less than twice the number of wickets taken by less than the hypotenuse, then find all sides of the
Zaheer. The product of the numbers of wickets triangle. [CBSE 2020 (Standard)]
taken by these two is 20. Represent the above 31. At present Asha’s age (in years) is 2 more than the
situation in the form of a quadratic equation. square of her daughter Nisha’s age. When Nisha
[CBSE 2015]
grows to her mother’s present age. Asha’s age
!
Long Answer Type Questions would be one year less than 10 times the present
age of Nisha. Find the present ages of both Asha
19. If x = 2 and x = 3 are roots of the equation and Nisha. [NCERT Exemplar]
3x 2 − 2 ax + 2 b = 0, then find the values of a and b. 32. The speed of a boat in still water is 15 km/h. It can
20. Find the nature of roots of the following quadratic go 30 km upstream and return downstream to the
equations. If the real roots exist, then also find the original point in 4 h and 30 min. Find the speed of
roots. stream.
(i) 4 x 2 + 12 x + 9 = 0 (ii) 3 x 2 + 5 x − 7 = 0 7
33. Two water taps together can fill a tank in 1 h.
8
21. Find the value of k for which the given equation has The tap with longer diameter takes 2 h less than
equal roots. the tap with smaller one to fill the tank separately.
( k − 12 ) x 2 + 2( k − 12 ) x + 2 = 0 Find the time in which each tap can fill the tank
separately. [CBSE 2019]
22. If x = − 2 is a root of the equation 3x 2 + 7 x + p = 0.
Find the values of k, so that the roots of the equation !
Case Based Questions
x 2 + k ( 4x + k − 1 ) + p = 0 are equal. [CBSE 2015] 34. In the centre of a rectangular lawn of dimensions
23. Find two consecutive odd natural numbers, sum of 50 m × 40 m, a rectangular pond has to be
whose squares is 130. [CBSE 2013] constructed, so that the area of the grass
24. A piece of cloth costs ` 200. If the piece was 5 m surrounding the pond would be 1184 m 2
longer and each metre of cloth costs ` 2 less, the
cost of the piece would have remained unchanged.
How long is the piece and what is the original rate
per metre? [CBSE 2015]
25. The difference of two numbers is 4. If the
4
difference of their reciprocals is , the find the
21 (i) If the distance between pond and lawn is x m. Find
two numbers. [CBSE 2008] the length and breadth of rectangular pond.
26. The perimeter of a right angled triangle is 70 units (ii) Find the quadratic equation related to the given
and its hypotenuse is 29 units we would like to find problem.
the length of the other sides. (iii) Find the length and breadth of the pond.
CBSE Term II Mathematics X (Standard) 11

SOLUTIONS
Objective Questions ⇒ 4x 3 – 3 = 0
1. (d) (a) Given that, which is not of the form ax 2 + bx + c = 0, a ≠ 0.
x 2 + 2 x + 1 = ( 4 − x )2 + 3 Thus, the equation is not quadratic.
⇒ x 2 + 2 x + 1 = 16 + x 2 − 8x + 3 This is a cubic equation.
⇒ 10x − 18 = 0 3. (b) Let the number be x.
Then according to the given condition,
which is not of the form ax 2 + bx + c = 0, a ≠ 0.
2 x 2 + x = 21
Thus, the equation is not a quadratic equation.
2
⎛ 2⎞ ⇒ 2 x + x − 21 = 0
(b) Given that, − 2 x 2 = ( 5 − x ) ⎜2 x − ⎟
⎝ 5⎠ 4. (c) (a) Substituting x = 2 in x 2 − 4x + 5 , we get
2x (2 )2 − 4 (2 ) + 5 = 4 − 8 + 5 = 1 ≠ 0.
⇒ − 2 x 2 = 10x − 2 x 2 − 2 +
5 So, x = 2 is not a root of x 2 − 4x + 5 = 0.
⇒ 50x + 2 x − 10 = 0 (b) Substituting x = 2 in x 2 + 3x − 12, we get
⇒ 52 x − 10 = 0
(2 )2 + 3(2 ) − 12
which is also not a quadratic equation.
3 = 4 + 6 − 12 = −2 ≠ 0
(c) Given that, x 2 ( k + 1) + x = 7
2 So, x = 2 is not a root of x 2 + 3x − 12 = 0.
Given, k = −1 (c) Substituting x = 2 in 2 x 2 − 7 x + 6, we get
2 3 2 (2 )2 − 7 (2 ) + 6 = 2 ( 4) − 14 + 6
⇒ x ( − 1 + 1) + x = 7
2 = 8 − 14 + 6 = 14 − 14 = 0
⇒ 3x − 14 = 0 So, x = 2 is root of the equation 2 x 2 − 7 x + 6 = 0.
which is also not a quadratic equation.
(d) Substituting x = 2 in 3x 2 − 6x − 2 , we get
(d) Given that, x 3 − x 2 = ( x − 1 )3
3(2 )2 − 6(2 ) − 2 = 12 − 12 − 2 = −2 ≠ 0
⇒ x 3 − x 2 = x 3 − 3 x 2 ( 1 ) + 3 x ( 1 ) 2 − (1 ) 3
So, x = 2 is not a root of 3x 2 − 6x − 2 = 0.
[Q ( a − b )3 = a 3 − b 3 + 3ab 2 − 3a 2b]
1 5
⇒ x 3 − x 2 = x 3 − 3x 2 + 3x − 1 5. (a) Since, is a root of the quadratic equation x 2 + kx − = 0.
2 4
⇒ − x + 3x 2 − 3x + 1 = 0 ⇒ 2 x 2 − 3x + 1 = 0
2
2
⎛ 1⎞ ⎛ 1⎞ 5
Then, ⎜ ⎟ + k ⎜ ⎟ − = 0
which represents a quadratic equation because it has ⎝ 2⎠ ⎝ 2⎠ 4
the quadratic form ax 2 + bx + c = 0, a ≠ 0. 1 k 5
⇒ + − =0
2. (d) (a) Given that, 2 ( x – 1)2 = 4x 2 – 2 x + 1 4 2 4
⇒ 2 ( x 2 + 1 – 2 x ) = 4x 2 – 2 x + 1 ⇒
1 + 2k − 5
=0
⇒ 2 x 2 + 2 – 4x = 4x 2 – 2 x + 1 4
⇒ 2k − 4 = 0
⇒ 2 x2 + 2 x – 1 = 0
⇒ 2k = 4
which represents a quadratic equation because it has
⇒ k =2
the quadratic form ax 2 + bx + c = 0, a ≠ 0.
6. (a) (a) Given that, x 2 – 5x + 6 = 0
(b) Given that, 2 x – x 2 = x 2 + 5
Put x = 3, we get
⇒ 2 x2 – 2 x + 5 = 0
( 3)2 − 5( 3) + 6 = 9 − 15 + 6 = 0
which also represents a quadratic equation because it
Hence, x = 3 is a root of the equation.
has the quadratic form ax 2 + bx + c = 0, a ≠ 0.
(b) – x 2 + 3x – 3 = 0
(c) Given that, ( 2 ⋅ x + 3 ) 2 = 3 x 2 – 5x
Put x = 3, we get
⇒ 2 ⋅ x 2 + 3 + 2 6 ⋅ x = 3x 2 – 5x −( 3)2 + 3( 3) − 3 = −9 + 9 − 3 = −3 ≠ 0
⇒ x2 – ( 5 + 2 6 ) x – 3 = 0 Hence, x = 3 is not a root of the equation.
which also represents a quadratic equation because it 3
(c) 2 x 2 – x+1=0
has the quadratic form ax 2 + bx + c = 0, a ≠ 0. 2
(d) Given that, ( x 2 + 2 x )2 = x 4 + 3 + 4x 2 Put x = 3, we get
3( 3)
⇒ x 4 + 4x 2 + 4x 3 = x 4 + 3 + 4x 2 2 ( 3) 2 − +1
2 ( 3)
12 CBSE Term II Mathematics X (Standard)

9 9 2
⇒9 2 − +1= +1≠ 0 ⎛ 1⎞ 1 + 64⋅ 2
2 2 ⎜y + ⎟ =
⎝ 8⎠ 64
Hence, x = 3 is not a root of the equation. 1
(d) 3x 2 – 3x + 3 = 0 Thus, must be added and subtracted to solve the given
64
Put x = 3, we get equation.
3( 3)2 − 3( 3) + 3 = 27 − 9 + 3 = 21 ≠ 0. 12. (d) Given quadratic equation is 12 x 2 + 5x − 3 = 0.
Hence, x = 3 is not a root of the equation. On comparing the given equation with
7. (b) False, since 0.2 does not satisfy the ax 2 + bx + c = 0, we get a = 12 , b = 5 and c = −3
equation i.e. ( 0.2 )2 − 0. 4 = 0. 04 − 0. 4 ≠ 0. On substituting the values of a = 12 , b = 5 and c = −3 in
8. (b) False, consider the quadratic equation 2 x 2 + x − 6 = 0 quadratic formula,
with integral coefficient. The roots of the given quadratic − b ± b 2 − 4ac
x= , we get
3 2a
equation are −2 and which are not integrals.
2 −5 ± ( 5)2 − 4 × 12 × ( −3)
9. (a) Given that, b = 0 and c < 0 and quadratic equation x=
2 × 12
x 2 + bx + c = 0 ...(i) −5 ± 25 + 144
⇒ x=
Put b = 0 in Eq. (i), we get 24
x2 + 0 + c = 0 − 5 ± 169
=
⎡here, c > 0⎤ 24
⇒ x2 = – c ⎢∴ − c > 0 ⎥ − 5 ± 13
⎣ ⎦ =
24
∴ x=± −c
−5 + 13 8 1
2
So, the roots of x + bx + c = 0 are numerically equal and Now, x= = = [taking +ve sign]
24 24 3
opposite in sign. −5 − 13 18 3
Or x= = − =− [taking −ve sign]
10. (c) Given, quadratic equation is x 2 − 8x − 20 = 0, which is 24 24 4
already in its standard form. 1 3
Hence, the roots of the given equation are and − .
On comparing it with ax 2 + bx + c = 0, we get 3 4
a = 1, b = −8 and c = −20 13. (c) Given quadratic equation is x 2 − 4x + 1 = 0.
Here, ac = 1 × ( −20) = −20. On comparing with ax 2 + bx + c = 0, we get
Here, ac has −ve sign, a = 1, b = −4 and c = 1
So, let p = −10 and q = 2 as p × q = −20 and p + q = −8 Now, discriminant (D ) = b 2 − 4ac
∴ x 2 − 8x − 20 = 0 = ( − 4) 2 − 4 × 1 × 1
⇒ x 2 − 10x + 2 x − 20 = 0 = 16 − 4 = 12
⇒ x ( x − 10) + 2( x − 10) = 0 14. (c) Given, 6x 2 − bx + 2 = 0
⇒ ( x − 10)( x + 2 ) = 0 On comparing with Ax 2 + Bx + C = 0, we get
Now, put x − 10 = 0 or x + 2 = 0 A = 6, B = − b and C = 2
⇒ x = 10 or x = −2 We know that,
Thus, 10 and −2 are the required roots of a given quadratic Discriminant, D = B 2 − 4AC
equation. ⇒ 1 = ( − b )2 − 4 × 6 × 2 [given, D = 1]
3
11. (b) Given equation is 9x 2 + x − 2 = 0. ⇒ 1 = b 2 − 48
4
2 1 ⇒ b 2 = 49
( 3x ) + ( 3x ) − 2 = 0
4 ⇒ b=±7 [taking square root on both sides]
On putting 3x = y, we have Hence, the required value of b is −7 or 7.
1
y2 + y − 2 = 0 15. (d) Given equation is 2 x 2 − kx + k = 0
4 On comparing with ax 2 + bx + c = 0, we get
2 2
1 ⎛ 1⎞ ⎛ 1⎞ a = 2, b = − k and c = k
y2 + y + ⎜ ⎟ − ⎜ ⎟ − 2 = 0
4 ⎝ 8⎠ ⎝ 8⎠ For equal roots, the discriminant must be zero.
2
⎛ 1⎞ 1 i.e. D = b 2 − 4ac = 0
⎜y + ⎟ = + 2
⎝ 8⎠ 64 ⇒ ( − k ) 2 − 4 (2 ) k = 0
2 2 2
[Q ( a + b ) = a + b + 2 ab ]
CBSE Term II Mathematics X (Standard) 13

⇒ k 2 − 8k = 0 (d) Given equation is, 5x 2 − 3x + 1 = 0


⇒ k ( k − 8) = 0 On comparing with ax 2 + bx + c = 0, we get
∴ k = 0, 8 a = 5, b = − 3, c = 1
Hence, the required values of k are 0 and 8. Now, D = b 2 − 4ac = ( −3)2 − 4( 5)(1) = 9 − 20 <0
16. (c) Given equation is 2 x 2 − 5x + 1 = 0. Hence, roots of the equation are not real.
On comparing with ax 2 + bx + c = 0, we get 19. (a) (a) The given equation is x 2 − 4x + 3 2 = 0.
On comparing with ax 2 + bx + c = 0, we get
a = 2 , b = − 5 and c = 1
a = 1, b = − 4 and c = 3 2
∴ Discriminant,
The discriminant of x 2 − 4x + 3 2 = 0 is
D = b 2 − 4ac
D = b 2 − 4ac
= ( − 5 ) 2 − 4 × ( 2 ) × (1 ) = 5 − 8 = − 3 < 0
= ( − 4 ) 2 − 4 (1) ( 3 2 )
Since, discriminant is negative, therefore
= 16 − 12 2 = 16 − 12 × (1 . 41)
quadratic equation 2 x 2 − 5x + 1 = 0 has no real roots i.e.
imaginary roots. = 16 − 16. 92 = − 0. 92
17. (d) Given quadratic equation is ⇒ b 2 − 4ac < 0
kx 2 − 3 2 x + 4 2 = 0 (b) The given equation is x 2 + 4x − 3 2 = 0
On comparing the equation with ax 2 + bx + c = 0, we get
On comparing with
a = 1, b = 4 and c = −3 2
ax 2 + bx + c = 0, we get
Then, D = b 2 − 4ac = ( 4)2 − 4(1)( −3 2 )
a = k , b = − 3 2 and c = 4 2
= 16 + 12 2 > 0
Q Discriminant, D = b 2 − 4ac
Hence, the equation has real roots.
⇒ 14 = ( − 3 2 )2 − 4 × k × 4 2 (c) Given equation is x 2 − 4x − 3 2 = 0
⇒ 14 = 18 − 16 2 k On comparing the equation with ax 2 + bx + c = 0,
we get
⇒ 16 2 k = 4
a = 1, b = − 4 and c = – 3 2
1
⇒ k= Then, D = b 2 − 4ac = ( −4)2 − 4(1)( −3 2 )
4 2
= 16 + 12 2 > 0
18. (b)
(a) Given equation is 2 x 2 − 3 2 x + 9 / 4 = 0, we get Hence, the equation has real roots.
(d) Given equation is 3x 2 + 4 3x + 4 = 0.
On comparing with ax 2 + bx + c = 0
a = 2, b = −3 2 and c = 9/4 On comparing the equation with ax 2 + bx + c = 0,we get
Now, D = b 2 − 4ac a = 3, b = 4 3 and c = 4
= ( −3 2 )2 − 4(2 )( 9 / 4) Then, D = b 2 − 4ac = ( 4 3 )2 − 4( 3)( 4)
= 18 − 18 = 0 = 48 − 48 = 0
Thus, the equation has real and equal roots. Thus, the equation has real roots.
(b) The given equation is x 2 + x − 5 = 0 Hence, x 2 − 4x + 3 2 = 0 has no real roots.
On comparing with ax 2 + bx + c = 0, we get 20. (c) Given equation is ( x 2 + 1 )2 − x 2 = 0
a = 1, b = 1 and c = − 5 ⇒ x4 + 1 + 2 x2 − x2 = 0
The discriminant of x 2 + x − 5 = 0 is [Q( a + b )2 = a 2 + b 2 + 2 ab]
2 2 4 2
D = b − 4ac = (1) − 4 (1) ( −5) ⇒ x + x +1=0
= 1 + 20 = 21 Let x2 = y
⇒ b 2 − 4ac > 0 2 2 2
∴ (x ) + x + 1 = 0
So, x 2 + x − 5 = 0 has two distinct real roots.
⇒ y2 + y + 1 = 0
(c) Given equation is x 2 + 3x + 2 2 = 0
On comparing with ay 2 + by + c = 0, we get
On comparing with ax 2 + bx + c = 0, we get
a = 1, b = 3 and c = 2 2 a = 1, b = 1 and c = 1
Now, D = b 2 − 4ac = ( 3)2 − 4(1)(2 2 ) Discriminant, D = b 2 − 4ac
= 9−8 2 < 0 = ( 1 ) 2 − 4 ( 1 ) (1 )
∴ Roots of the equation are not real. =1− 4= −3
14 CBSE Term II Mathematics X (Standard)

Since, D<0 which is the required quadratic equation.


∴ y + y + 1 = 0 i.e. x 4 + x 2 + 1 = 0
2
Now, x 2 − 7 x − x + 7 = 0 [by factorisation]
or ( x 2 + 1)2 − x 2 = 0 has no real roots. ⇒ x( x − 7 ) − 1( x − 7 ) = 0
21. (b) Let the smallest integer be x. Then, three consecutive ⇒ ( x − 7 )( x − 1) = 0
integers are x, x + 1, x + 2. ⇒ x − 7 = 0 or x − 1 = 0
From the question, ⇒ x = 7 or x = 1
x 2 + ( x + 1)2 + ( x + 2 )2 = 110 But x = 1 is not possible because if x = 1, then present age of
the son and father are same.
⇒ x 2 + x 2 + 1 + 2 x + x 2 + 4 + 4x = 110
So, x = 7.
⇒ 3x 2 + 6x − 105 = 0
Hence, present age of his son = 7 yr
−6± 62 − 4 ⋅ 3 ⋅ ( − 105) and present age of man = 8 × 7 − 7 = 49 yr.
∴ x= [using formula]
2×3 24. (i) (a) Given, Raj’s car travel at a speed of x km/h. Then
−6± 36 + 1260 Ajay’s car travels a distance in one hour is ( x + 5) km.
= Therefore, Ajay’s car travels a distance in two hours is
6 2( x + 5) km.
− 6 ± 1296 − 6 ± 36 Distance
= = (ii) (c)Q Time =
6 6 Speed
− 6 + 36 − 6 − 36 30 − 42
= , = , = 5, − 7 Time taken by Ajay and Raj to complete the 400 km
6 6 6 6 journey
When x = 5, 400 400
t1 = and t 2 =
x+1 = 5+1 = 6 x+5 x
x+2 = 5+2 =7 According to the question,
When x = −7 t 2 = t1 + 4
x + 1 = −7 + 1 = −6 400 400
∴ = +4
x + 2 = −7 + 2 = −5 x x+5
∴Three consecutive integers are 5, 6, 7 or − 7, − 6, − 5. 100 100
⇒ = + 1 (divide by 4)
Hence, smallest positive integer is 5. x x+5
22. (c) Let BP = x cm ⇒ 100( x + 5) = 100x + x( x + 5)
Then, AP = AB + BP = ( 8 + x ) cm ⇒ 100x + 500 = 100x + x 2 + 5x
8 cm x cm ⇒ x 2 + 5x − 500 = 0
A B P (iii) (a) Consider the quadratic equation x 2 + 5x − 500 = 0
Now, BP 2 = AB ⋅ AP ⇒ x 2 = 8 ⋅ ( 8 + x ) On comparing with ax 2 + bx + c = 0, we get
2 a = 1, b = 5 and c = − 500
⇒ x − 8x − 64 = 0
2
− ( − 8) ± ( − 8) − 4 ⋅ 1 ⋅ ( − 64) − b ± b 2 − 4ac
∴ x= Q x=
2 2a
8± 64 × 5 − 5 ± ( 5)2 − 4 × (1)( − 500)
or x= =
2 2 ×1
8± 8 5 − 5 ± 25 + 2000 − 5 ± 2025
= =4± 4 5 = =
2 2 2
But the length of BP is positive. −5 ± 45 −50 40
= = , = − 25, 20
2 2 2
So, x = ( 4 + 4 5 ) cm = 4( 5 + 1) cm
Since, speed cannot be negative, so we consider only,
23. (a) Let present age of his son = x yr x = 20.
One year ago, his son’s age = ( x − 1) yr Hence, speed of Raj’s car is 20 km/h.
One year ago, man’s age = 8( x − 1) yr (iv) (d) To travel 400 km, time taken by Ajay
= ( 8x − 8) yr 400 400 400
t1 = = = = 16 h
Present age of man = ( 8x − 8 + 1) yr ( x + 5) 20 + 5 25
= ( 8x − 7 ) yr (v) (b) To travel 400 km, time taken by Raj,
According to the question, 400 400
t2 = = = 20 h
8x − 7 = x 2 ⇒ x 2 − 8x + 7 = 0 x 20
CBSE Term II Mathematics X (Standard) 15

25. (i) (c) Since, the speed of stream be x km/h and speed of 11 + 17 28 7
Now, x= = = [taking + ve sign]
motorboat is 20 km/h. Therefore, the speed of 8 8 2
motorboat in upstream will be (20 − x ) km/h. 11 − 17 6 3
or x= =− =− [taking − ve sign]
(ii) (a) The relation between speed, distance and time is 8 8 4
Distance 7 3
Speed = Hence, the roots of the given equation are and − .
Time 2 4
(iii) (c)Q Time =
Distance (ii) (b) Given quadratic equation is x 2 + x + 7 = 0.
Speed On comparing with ax 2 + bx + c, we get
Here, distance = 15 km/h a = 1, b = 1 and c = 7
Speed of motorboat in downstream = (20 + x ) km/h Now, discriminant (D ) = b 2 − 4ac = 12 − 4 × 1 × 7
and speed of motorboat in upstream = (20 − x ) km/h
= 1 − 28 = − 27
Time taken by motorboat in downstream and upstream
(iii) (a) Given equation is 4x 2 − 2 x − 3 = 0
15 15
are t1 = h and t 2 = h. On comparing with ax 2 + bx + c = 0, we get
20 + x 20 − x
a = 4, b = − 2 and c = − 3
According to the question,
t 2 = 1 + t1 ∴ Discriminant (D ) = b 2 − 4ac
15 15 = ( −2 )2 − 4 × 4 × ( −3) = 4 + 48 = 52 > 0
∴ =1+
20 − x 20 + x So, 4x 2 − 2 x = 3 has two distinct real roots.
15 15 (iv) (c) Given equation is 4x 2 + kx + 9 = 0.
⇒ − =1
20 − x 20 + x On comparing with ax 2 + bx + c = 0, we get
⇒ 15 (20 + x − 20 + x ) = (20 + x )(20 − x )
a = 4, b = k and c = 9
⇒ 15(2 x ) = 400 − x 2
2
Now, D = b 2 − 4ac
⇒ x + 30x − 400 = 0
= k 2 − 4 × 4 × 9 = k 2 − 144
(iv) (b) Consider quadratic equation,
Since, roots of given equation are real and equal.
x 2 + 30x − 400 = 0
∴ D=0
⇒ x 2 + ( 40 − 10)x − 400 = 0
⇒ k 2 − 144 = 0 ⇒ k 2 = 144
⇒ x 2 + 40x − 10x − 400 = 0 ⇒ k = ± 12
⇒ x( x + 40) − 10 ( x + 40) = 0
(v) (d) Given equation is x 2 + kx + 16 = 0
⇒ ( x − 10) ( x + 40) = 0
On comparing with ax 2 + bx + c = 0, we get
⇒ x = 10, − 40
Since, speed cannot be negative, so we consider only a = 1, b = k and c = 16
positive value. Now, D = b 2 − 4ac
∴ x = 10 = k 2 − 4 × 1 × 16 = k 2 − 64
Hence, speed of current is 10 km/h. Since, roots of given equation are real.
(v) (c) The time taken by motorboat in downstream ∴ D≥0
15
t1 = ⇒ k 2 − 64 ≥ 0
20 + x
⇒ k 2 ≥ 64
15 15 1
= = = h = 30 min ⇒ k ≥ 8 and k ≤ −8
20 + 10 30 2
Hence, positive least value of k is 8.
26. (i) (c) Given quadratic equation is
27. (i) (b) Seven years ago,
8x 2 − 22 x − 21 = 0 Swati’s age = x yr
On comparing the given equation with Varun’s age = 5x 2 yr
ax 2 + bx + c = 0, we get (ii) (d) Swati’s present age = ( x + 7 ) yr
a = 8, b = − 22 and c = − 21 and Varun’s present age = ( 5x 2 + 7 ) yr
By quadratic formula, After three years, we have
− ( − 22 ) ± ( − 22 )2 − 4 × 8 × ( − 21) Swati’s age = ( x + 7 + 3) = ( x + 10) yr
x=
2×8 Varun’s age = ( 5x 2 + 7 + 3) = ( 5x 2 + 10)yr
22 ±
484 + 672 22 ± 1156 (iii) (a) According to the question,
= = 2
16 16 x + 10 = ( 5x 2 + 10)
22 ± 34 11 ± 17 5
⇒ x= = ⇒ 2 x2 − x − 6 = 0
16 8
16 CBSE Term II Mathematics X (Standard)

(iv) (a) Now, 2 x2 − x − 6 = 0 2


⇒ P= ×3
2 3
⇒ 2 x − 4x + 3 x − 6 = 0
⇒2x (x − 2) + 3 (x − 2) = 0 ⇒ P= 2 × 3= 6
⇒ (2 x + 3 ) ( x − 2 ) = 0 ∴ P 2 + 1 = ( 6 )2 + 1
3 = 6 + 1=7
⇒ x=− ,2
2 3. (i) We have,
∴ x =2 [Q age can’t negative] kx 2 + 2 x − 3 = 0, here k is unknown.
⇒ Present age of Varun’s = ( 5x 2 + 7 ) yr Since, x = 2 is a solution of given equation, so it will
= ( 5 × 4 + 7 ) yr satisfy the given equation.
= (20 + 7 ) yr On putting x = 2 in the given equation, we get
= 27 yr k(2 )2 + 2(2 ) − 3 = 0
(v) (d) Here, Swati’s present age = 10 yr ⇒ 4k + 4 − 3 = 0
⇒ x + 7 = 10 ⇒ 4k + 1 = 0
⇒ x=3 −1
⇒ k=
So, Varun’s present age = ( 5x 2 + 7 ) yr 4
= 5 ( 3) 2 + 7 (ii) We have, x 2 + 2 ax − k = 0, here k is unknown.
= 45 + 7 = 52 yr Since, x = − a is a solution of given equation, so it will
satisfy the given equation.
Subjective Questions
On putting x = − a in the given equation, we get
1. (i) Given, ( x − 1)( x + 2 ) = ( x − 3)( x + 1) ...(i)
( − a )2 + 2 a ( − a ) − k = 0
LHS = ( x − 1) ( x + 2 ) = x 2 + 2 x − x − 2
⇒ a2 − 2a2 − k = 0
= x2 + x − 2
⇒ − a2 − k = 0
RHS = ( x − 3) ( x + 1) = x 2 + x − 3x − 3 ⇒ k = − a2
= x2 − 2 x − 3 4. (i) Given quadratic equation is
On substituting these values in Eq. (i), we get x 2 − 2 kx − 6 = 0 …(i)
x2 + x − 2 = x2 − 2 x − 3 Since, x = 3 is one of the root of the given quadratic
⇒ x2 − x2 + x + 2 x − 2 + 3 = 0 equation. Then, it satisfies the given equation.
So, put x = 3 in Eq. (i), we get
⇒ 3x + 1 = 0
( 3)2 − 2 k( 3) − 6 = 0
It is not of the form ax 2 + bx + c = 0, a ≠ 0.
⇒ 9 − 6k − 6 = 0
As a = 0 and it is an equation of degree 1.
⇒ 6k = 3
Hence, the given equation does not represent a
1
quadratic equation. ⇒ k=
(ii) Given, ( x + 2 )2 = 4( x + 3) ...(ii) 2
x 2 + 4 + 4x = 4x + 12 (ii) Given quadratic equation is
5
[Q( a + b )2 = a 2 + b 2 + 2 ab ] x 2 − kx − = 0
4
⇒ x 2 + 4x − 4x + 4 − 12 = 0 1
⇒ x 2 − 8 = 0 or x 2 + 0x − 8 = 0 Put x = , we get
2
It is of the form ax 2 + bx + c = 0, a ≠ 0. 2
⎛ 1⎞ ⎛ 1⎞ 5
Hence, given equation represents a quadratic equation. ⎜ ⎟ − k⎜ ⎟ − = 0
⎝ 2⎠ ⎝ 2⎠ 4
2. Given equation is 1 k 5
Px 2 + ( 3 − 2 ) x − 1 = 0 ⇒ − − =0
4 2 4
1 1 − 2k − 5
and x = is a root of the equation. ⇒ =0
3 4
2
⎛ 1 ⎞ 1 ⇒ 2k = − 4
∴ P⎜ ⎟ + ( 3 − 2) −1= 0
⎝ 3⎠ 3 ⇒ k = −2
P 3− 2 − 3 5. Given equation is in the form p( x ) = 0, where
⇒ + =0
3 3 p( x ) = 6x 2 − x − 2 ...(i)
P 2 −1
⇒ − =0 On putting x = in Eq. (i), we get
3 3 2
CBSE Term II Mathematics X (Standard) 17

2
⎛ −1⎞ ⎛ −1⎞ ⎛ −1⎞ 8. Given that, ax 2 + a = a 2x + x
p ⎜ ⎟ = 6⎜ ⎟ − ⎜ ⎟ − 2 2 2
⎝2⎠ ⎝2⎠ ⎝2⎠ ⇒ ax − a x − x + a = 0
6 1 6 + 2 − 8 8−8 ⇒ ax ( x − a ) − 1 ( x − a ) = 0
= + −2= =
4 2 4 4 ⇒ ( ax − 1) ( x − a ) = 0
⎛ −1⎞ 1
⇒ p⎜ ⎟ = 0 ⇒ x= ,a
⎝2⎠ a
−1 9. Given that, 6x + 7 − (2 x − 7 ) = 0
So, x = is a solution of the given equation.
2 ⇒ 6x + 7 = 2 x − 7
1
Now, on putting x = in Eq. (i), we get On squaring both sides, we get
3
2 6 x + 7 = (2 x − 7 ) 2
⎛ 1⎞ ⎛ 1⎞ ⎛ 1⎞
p⎜ ⎟ = 6⎜ ⎟ − ⎜ ⎟ − 2
⎝ 3⎠ ⎝ 3⎠ ⎝ 3⎠ 6x + 7 = 4x 2 + 49 − 28x
⇒ 4x 2 − 34x + 42 = 0
1 1 6 1
=6× − −2 = − − 2 ⇒ 2 x 2 − 17 x + 21 = 0 [divide by 2]
9 3 9 3
6 − 3 − 18 −15 ⇒ 2 x 2 + 14x + 3x + 21 = 0
= = ≠0
9 9 ⇒ 2 x ( x + 7 ) + 3( x + 7 ) = 0
⎛ 1⎞ ⇒ (2 x + 3 ) ( x + 7 ) = 0
⇒ p⎜ ⎟ ≠ 0
⎝ 3⎠ 3
⇒ x = − ,−7
1 2
So, x = is not a solution of the given equation. 10. Given equation is x 2 − 7 x − 18 = 0
3
6. Given, 4 3x 2 + 5x − 2 3 = 0. ⇒ x 2 − 9x + 2 x − 18 = 0
On comparing with standard form of quadratic equation ⇒ x ( x − 9) + 2 ( x − 9 ) = 0
i.e. ax 2 + bx + c = 0, we get ⇒ ( x + 2 ) ( x − 9) = 0
a = 4 3, b = 5 and c = −2 3 ⇒ x = − 2, 9
Here, ac = 4 3 × ( −2 3 ) = −24 So, the roots of given equation are − 2 and 9.
∴ Required numerical difference of the roots
Then, factors of ac are 8 and −3.
= 9 − ( − 2 ) = 11
∴ 4 3 x 2 + ( 8 − 3)x − 2 3 = 0
11. The given equation is 3x 2 + 11x + 6 3 = 0.
⇒ 4 3 x 2 + 8 x − 3x − 2 3 = 0
On comparing with ax 2 + bx + c = 0, we get
⇒ 4x( 3x + 2 ) − 3 ( 3x + 2 ) = 0 a = 3, b = 11 and c = 6 3
⇒ ( 4x − 3 ) ( 3x + 2 ) = 0 On substituting the values of a, b and c in the quadratic
⇒ 4x − 3 = 0 formula,
and 3x + 2 = 0 − b ± b 2 − 4ac
x=
3 2a
⇒ x=
4 − 11 ± (11)2 − 4( 3 )( 6 3 )
⇒ x=
−2 2( 3 )
or x=
3 − 11 ± 121 − 72
=
3 2 3
Hence, roots of equation 4 3x 2 + 5x − 2 3 = 0 are
4 −11 ± 49
−2 =
and . 2 3
3 −11 ± 7
16 15 =
7. Given, −1 = 2 3
x x+1 −11 + 7 −4 −2
16 15 16( x + 1) − 15x ⇒ x= = = [taking +ve sign]
⇒ − =1 ⇒ =1 2 3 2 3 3
x x+1 x ( x + 1) −11 − 7 −18 −9
and x= = = [taking −ve sign]
⇒ 16x + 16 − 15x = x 2 + x 2 3 2 3 3
⇒ x 2 = 16 −2 −9 −2 3
Hence, and (or and −3 3) are the required
⇒ x2 = ± 4 3 3 3
Hence, the roots are 4 and −4. solutions of the given equation.
18 CBSE Term II Mathematics X (Standard)

12. Given equation is 5x 2 − sx + 4 = 0 16. Given equation is


2
On comparing with ax + bx + c = 0, we get (1 + m 2 )x 2 + (2 mc )x + ( c 2 − a 2 ) = 0
a = 5, b = − s and c = 4 On comparing with Ax 2 + Bx + C = 0, we get
∴ Discriminant (D ) = b 2 − 4ac A = (1 + m 2 ), B = 2 mc and C = ( c 2 − a 2 )
= ( − s )2 − 4 × 5 × 4 Since, the given equation has equal roots.
2
= s − 80 ∴ Discriminant, D = 0 ⇒ B 2 − 4AC = 0
Given, D =1 ⇒ (2 mc )2 − 4(1 + m 2 ) ( c 2 − a 2 ) = 0
⇒ s 2 − 80 = 1 ⇒ 4m c − 4 ( c 2 − a 2 + m 2c 2 − m 2a 2 ) = 0
2 2

⇒ s 2 = 81 ⇒ m 2c 2 − ( c 2 − a 2 + m 2c 2 − m 2a 2 ) = 0 [dividing by 4]
⇒ s=± 9 ⇒ m 2c 2 − c 2 + a 2 − m 2c 2 + m 2a 2 = 0
13. Given that, ⇒ − c 2 + a 2 + m 2a 2 = 0
( x 2 + 1) 2 − x 2 = 0
⇒ − c 2 + a 2(1 + m 2 ) = 0
⇒ ( x 2 + 1) 2 = x 2 ⇒ x
⇒ − c 2 = − a 2(1 + m 2 )
2
⇒ x +1=±x
⇒ c 2 = a 2(1 + m 2 ) Hence proved.
⇒ x2 m x + 1 = 0
17. Let one number be x.
On comparing with ax 2 + bx + c, we get Then, another number = (11 − x )
a = 1, b = m 1 and c = 1 [Q sum of two numbers = 11, given]
∴ D = b 2 − 4ac According to the question,
= ( m 1) 2 − 4 × 1 × 1 1 1 11
+ =
x (11 − x ) 28
=1 − 4= − 3 < 0
11 − x + x 11
∴ It has no real roots. ⇒ =
x(11 − x ) 28
14. Given equation is 2 x 2 − kx + k = 0
⇒ x(11 − x ) = 28
On comparing with ax 2 + bx + c = 0, we get
⇒ x 2 − 11x + 28 = 0
a = 2, b = − k and c = k
⇒ x 2 − (7 + 4)x + 28 = 0
∴ D = b 2 − 4ac
⇒ x 2 − 7 x − 4x + 28 = 0
= ( − k )2 − 4 × 2 × k ⇒ x( x − 7 ) − 4( x − 7 ) = 0
= k 2 − 8k ⇒ ( x − 7 )( x − 4) = 0
Since, the given equation has equal roots. ⇒ x = 4 or x = 7
∴ D=0 When x = 4, then 11 − x = 11 − 4 = 7
⇒ k 2 − 8k = 0 When x = 7, then 11 − x = 11 − 7 = 4
⇒ k ( k − 8) = 0 Hence, the numbers are 4 and 7.
18. Let the number of wickets taken by Zaheer in a cricket
⇒ k = 0, 8
match are x, then number of wickets taken by Harbhajan
15. Given quadratic equation is
= 2x − 3
9x 2 + 3kx + 4 = 0
According to the question,
On comparing with ax 2 + bx + c = 0, we get x (2 x − 3) = 20
a = 9, b = 3k and c = 4 ⇒ 2 x 2 − 3x = 20
Now, D = b 2 − 4ac = ( 3k )2 − 4( 9) ( 4) ⇒ 2 x 2 − 3x − 20 = 0
2
= 9k − 144 2
19. Given, 3x − 2 ax + 2 b = 0 ...(i)
Since, roots of given equation are real. Here, a and b are unknown constants. Since, x = 2 and x = 3
∴ D ≥ 0 ⇒ 9k 2 − 144 ≥ 0 are the solutions of given equation, so it will satisfy the given
equation.
⇒ 9( k 2 − 16) ≥ 0
On putting x = 2 and x = 3 one-by-one,
∴ k 2 − 16 ≥ 0 [Q9 ≠ 0]
in Eq. (i), we get
⇒ k 2 − ( 4) 2 ≥ 0
3(2 )2 − 2 a × (2 ) + 2 b = 0
⇒ ( k − 4 ) ( k + 4) ≥ 0 [Q a 2 − b 2 = ( a − b )( a + b )]
⇒ 3 × 4 − 4a + 2 b = 0
⇒ k ≤ − 4 or k ≥ 4 ⇒ 12 − 4a + 2 b = 0
CBSE Term II Mathematics X (Standard) 19

⇒ − 2 (2 a − b − 6 ) = 0 21. Given quadratic equation is


⇒ 2a − b = 6 [Q − 2 ≠ 0] ...(ii) ( k − 12 ) x 2 + 2( k − 12 ) x + 2 = 0
and 3( 3)2 − 2 a × 3 + 2 b = 0 On comparing with ax 2 + bx + c = 0 , we get
⇒ 27 − 6a + 2 b = 0
a = k − 12 , b = 2( k − 12 ) and c = 2
⇒ 6a − 2 b = 27 … (iii)
Now, D = b 2 − 4ac
On multiplying Eq. (ii) by 2 and then subtract it from
Eq. (iii), we get = [2( k − 12 )]2 − 4( k − 12 ) (2 )
6a − 2 b − 4a + 2 b = 27 − 12 = 4( k − 12 )2 − 8( k − 12 )
15
⇒ 2 a = 15 ⇒ a = = ( k − 12 ) [ 4( k − 12 ) − 8]
2
15 = ( k − 12 ) ( 4k − 48 − 8)
On substituting a = in Eq. (ii), we get = ( k − 12 ) ( 4k − 56)
2
15 Since, roots of given equation are equal.
2× −b=6
2 ∴ D=0
⇒ 15 − b = 6 ⇒ ( k − 12 ) ( 4k − 56) = 0
⇒ b = 15 − 6 = 9
⇒ k − 12 = 0 or 4k − 56 = 0
⇒ b=9 56
Hence, the required values of a and b are 15/2 and 9, ⇒ k = 12 or k =
4
respectively.
⇒ k = 12 or k = 14
20. (i) Given quadratic equation is But k = 12 does not satisfy the given equation because if
4x 2 + 12 x + 9 = 0 k = 12, then coefficients of x 2 and x become zero.
On comparing with ax 2 + bx + c = 0, we get Hence, required value of k is 14.
a = 4, b = 12 and c = 9 22. Given equation is 3x 2 + 7 x + p = 0
Now, D = b 2 − 4ac = (12 )2 − 4( 4) ( 9) Since, x = − 2 is a root of the given equation, so it will satisfy
= 144 − 144 = 0 the given equation.
Since, D = 0, so given quadratic equation has two equal On putting x = − 2 in the given equation, we get
and real roots which are given by 3 ( − 2 )2 + 7 ( − 2 ) + p = 0
− b ± D −12 ± 0 ⇒ 12 − 14 + p = 0
x= =
2a 2( 4) ⇒ −2 + p = 0
−12 + 0 −12 − 0 ⇒ p =2
⇒ x= or x =
8 8 On putting p = 2 in x 2 + k ( 4x + k − 1) + p = 0, we get
3 3
⇒ x = − or x = − x 2 + k ( 4x + k − 1 ) + 2 = 0
2 2
−3 3 ⇒ x 2 + 4kx + ( k 2 − k + 2 ) = 0
Hence, the roots are and − .
2 2 On comparing with ax 2 + bx + c = 0, we get
(ii) Given quadratic equation is a = 1, b = 4k and c = k 2 − k + 2
3x 2 + 5x − 7 = 0
∴ D = b 2 − 4ac
On comparing with ax 2 + bx + c = 0, we get
= ( 4k ) 2 − 4 × 1 × ( k 2 − k + 2 )
a = 3, b = 5 and c = − 7
= 16k 2 − 4k 2 + 4k − 8
Now, D = b 2 − 4ac = ( 5)2 − 4( 3) ( −7 ) = 25 + 84 = 109
= 12 k 2 + 4k − 8
Since, D > 0, so given quadratic equation has two
distinct real roots which are given by Since, roots are equal.
− b ± D −5 ± 109 ∴ D=0
x= = ⇒ 12 k 2 + 4k − 8 = 0 [divide by 4]
2a 2( 3)
2
−5 + 109 ⇒ 3k + k − 2 = 0
⇒ x= [taking +ve sign] 2
6 ⇒ 3k + 3k − 2 k − 2 = 0
−5 − 109 ⇒ 3k ( k + 1) − 2 ( k + 1 ) = 0
or x= [taking −ve sign]
6 ⇒ ( 3 k − 2 ) ( k + 1) = 0
−5 + 109 −5 − 109 2
Hence, the roots are and . ⇒ k = ,−1
6 6 3
20 CBSE Term II Mathematics X (Standard)

23. Let two consecutive odd natural numbers are x and x + 2. When x = − 7, then second number = − 7 + 4 = − 3
Then according to the given condition, When x = 3, then second number = 3 + 4 = 7
x 2 + ( x + 2 )2 = 130 Hence, two numbers are − 7 , − 3 or 3, 7.
⇒ x 2 + x 2 + 4x + 4 = 130 26. Let one side = x.
C
⇒ 2 x 2 + 4x − 126 = 0
⇒ x 2 + 2 x − 63 = 0 [divide by 2]
2
⇒ x + 9x − 7 x − 63 = 0 x 29
⇒ x ( x + 9) − 7 ( x + 9) = 0
⇒ ( x − 7 ) ( x + 9) = 0
⇒ x = 7, − 9 A B
Since, natural number cannot be negative. Now, perimeter of a triangle,
So, we neglect x = − 9. 70 = x + 29 + AB
Thus, x = 7 and x + 2 = 7 + 2 = 9 ⇒ AB = 70 − 29 − x = 41 − x.
Hence, two consecutive odd numbers are 7 and 9. In right ΔABC, use Pythagoras theorem,
24. Let the length of piece be x m. BC 2 = AC 2 + AB 2
200
Then, rate = ` per m ⇒ (29)2 = x 2 + ( 41 − x )2
x
Now, new length = ( x + 5) m ⇒ 841 = x 2 + 1681 + x 2 − 82 x
Since, the cost remains same. ⇒ 2 x 2 − 82 x + 840 = 0
200
∴ New rate = ` per m ⇒ x 2 − 41x + 420 = 0 [divide by 2]
x+5
2
According to the given condition, ⇒ x − 21x − 20x + 420 = 0
200 200 ⇒ x ( x − 21) − 20 ( x − 21) = 0
= −2
x+5 x ⇒ ( x − 20) ( x − 21) = 0
200 ⎛ 100 − x ⎞ ⇒ x = 20, 21
⇒ =2⎜ ⎟
x+5 ⎝ x ⎠ Hence, length of other sides of a ΔABC are 20 units, 21 units.
⇒ 100x = ( x + 5) (100 − x ) 27. Let present age of Anjali be x yr.
⇒ 100x = 100x − x 2 + 500 − 5x ∴ Anjali’s age 3 yr ago = ( x − 3) yr
⇒ x 2 + 5x − 500 = 0 and Anjali’s age 5 yr from now = ( x + 5) yr
2
⇒ x + 25x − 20x − 500 = 0 According to the question,
⇒ x ( x + 25) − 20 ( x + 25) = 0 1 1 1
+ =
⇒ ( x − 20) ( x + 25) = 0 x−3 x+ 5 3
⇒ x = 20, − 25 x+ 5+ x−3 1
⇒ =
Since, length of piece cannot be negative, so neglect x = − 25. ( x − 3)( x + 5) 3
2x + 2 1
Thus, x = 20 ⇒ =
200 200 x 2 − 3x + 5x − 15 3
Now, rate = = = ` 10
x 20 ⇒ 3(2 x + 2 ) = x 2 + 2 x − 15
Hence, length of piece is ` 20 m and rate per metre is ` 10. ⇒ 6x + 6 = x 2 + 2 x − 15
25. Let first number be x. ⇒ x 2 + 2 x − 15 − 6x − 6 = 0
Then, second number = x + 4 ⇒ x 2 − 4x − 21 = 0,
[Q difference of two numbers = 4]
which is the required quadratic equation.
According to the question,
1 1 4 Now, by factorisation method, we get
− = x 2 − 7 x + 3x − 21 = 0
x x + 4 21
( x + 4) − x 4 4 4 ⇒ x( x − 7 ) + 3( x − 7 ) = 0
⇒ = ⇒ 2 =
x ( x + 4) 21 x + 4x 21 ⇒ ( x − 7 )( x + 3) = 0
⇒ x − 7 = 0 or x + 3 = 0
⇒ x 2 + 4x = 21 ⇒ x 2 + 4x − 21 = 0
⇒ x = 7 or x = − 3
⇒ x 2 + (7 − 3)x − 21 = 0 ⇒ x 2 + 7 x − 3x − 21 = 0 But x = − 3 is not possible because age cannot be negative.
⇒ x ( x + 7 ) − 3 ( x + 7 ) = 0 ⇒ ( x − 3) ( x + 7 ) = 0 ∴ x =7
⇒ x = − 7, 3 Hence, Anjali’s present age is 7 yr.
CBSE Term II Mathematics X (Standard) 21

28. Let the ten’s digit of the number be x. 30. Let length of the shortest side = x m.
According to the question, Then, hypotenuse = (2 x + 6) m and
Product of the digits = 12 third side = (2 x + 6 − 2 ) m = (2 x + 4) m
i.e. Ten’s digit × Unit’s digit = 12 By Pythagoras theorem,
12 (2 x + 6 ) 2 = x 2 + ( 2 x + 4) 2
⇒ Unit’s digit = [Q ten’s digit = x]
x
12 [Q (Hypotenuse) 2 = (Perpendicular) 2 + (Base) 2]
∴ Two-digit number = 10x +
x ⇒ 4x 2 + 24x + 36 = x 2 + 4x 2 + 16x + 16
Also, it is given that if 36 is added to the number, the digits [Q( a + b )2 = a 2 + 2 ab + b 2]
get interchange.
12 12 ⇒ x 2 + 4x 2 + 16x + 16 − 4x 2 − 24x − 36 = 0
∴ 10x + + 36 = 10 × +x
x x ⇒ x 2 − 8x − 20 = 0
⇒ 10x 2 + 12 + 36x = 120 + x 2 By quadratic formula,
⇒ 9x 2 − 108 + 36x = 0 − ( −8) ± ( − 8)2 − 4 × 1 × ( −20)
x=
⇒ x 2 + 4x − 12 = 0 [divide both sides by 9] 2 ×1
which is the required quadratic equation. ⎡ − b ± b 2 − 4ac
⎢Q x = ; here a = 1, b = −8 and c = −20]
By factorisation method, we get ⎢⎣ 2a
x 2 + 6x − 2 x − 12 = 0
8 ± 64 + 80 8 ± 144
⇒ x( x + 6) − 2( x + 6) = 0 ⇒ x= ⇒ x=
2 2
⇒ ( x + 6)( x − 2 ) = 0 8 ± 12
⇒ x=
⇒ x + 6 = 0 or x − 2 = 0 2
⇒ x = − 6 or x = 2 8 + 12 8 − 12
⇒ x= or x =
But a digit can never be negative. 2 2
So, x = 2. 20 −4
⇒ x= or x =
2 2
Hence, the required two-digit number
12 ⇒ x = 10 or x = − 2
= 10 × 2 + = 20 + 6 = 26 But length of side cannot be negative.
2
29. Given, John and Janvi together have 45 marbles. ∴ x = 10
Let John has x marbles. Hence, shortest side is 10 m, hypotenuse is 2 × 10 + 6 = 26 m
and third side = 2 × 10 + 4 = 24 m.
Then, number of marbles Janvi has = 45 − x
31. Let Nisha’s present age be x yr.
Q Both of them lost 5 marbles each.
∴ The number of marbles John has = x − 5 Then, Asha’s present age = x 2 + 2 [by given condition]
and the number of marbles Janvi has = 45 − x − 5 = 40 − x Now, when Nisha grows to her mother’s present age.
Now, product of the number of marbles = 124 Then, Asha’s age will be [( x 2 + 2 ) – x ] yr.
∴ ( x − 5) ( 40 − x ) = 124 Again by given condition,
⇒ 40x − x 2 − 200 + 5x = 124 Age of Asha = One year less than 10 times the present age of
Nisha
⇒ − x 2 + 45x − 200 − 124 = 0
( x 2 + 2 ) + {( x 2 + 2 ) – x} = 10x – 1
⇒ − x 2 + 45x − 324 = 0
⇒ 2 x 2 – x + 4 = 10x – 1
⇒ x 2 − 45x + 324 = 0 [multiplying by ( −1)] ⇒ 2 x 2 – 11x + 5 = 0
which is the required quadratic equation. ⇒ 2
2 x − 10x – x + 5 = 0
Now, by factorisation method, we get ⇒ 2 x ( x – 5) – 1 ( x – 5 ) = 0
x 2 − 36x − 9x + 324 = 0 ⇒ ( x – 5 ) (2 x – 1 ) = 0
⇒ x( x − 36) − 9( x − 36) = 0 ∴ x=5
⇒ ( x − 36)( x − 9) = 0 1 1
[here, x = cannot be possible, because at x = ,
⇒ x − 36 = 0 or x − 9 = 0 2 2
1
⇒ x = 36 or x = 9 Asha’s age is 2 yr which is not possible]
4
when John has 36 marbles, then
Hence, required age of Nisha = 5 yr
Janvi has = 45 − 36 = 9 marbles.
and required age of Asha = x 2 + 2
when John has 9 marbles, then
Janvi has = 45 − 9 = 36 marbles. = ( 5)2 + 2 = 25 + 2 = 27 yr
22 CBSE Term II Mathematics X (Standard)

32. Let speed of the stream = x km/h When, x = 5


Given, speed of boat in still water = 15 km/h Time taken by smaller tap = 5 h
∴ Speed of boat upstream = (15 − x ) km/h Time taken by larger tap = x − 2 = 5 − 2 = 3 h
and speed of boat downstream = (15 + x ) km/h 3
When, x =
According to the question, 4
30 30 1 3
+ =4 Time taken by smaller tap = h
15 − x 15 + x 2 4
⎡ distance Time taken by larger tap = x − 2
⎢Q time = speed and distance = 30 km 3 −5
⎣ = − 2 = , which is not solution.
⎛ 30⎞ 1 ⎤ 4 4
and also, 4h 30 min = ⎜ 4 + ⎟ h = 4 h
⎝ 60⎠ 2 ⎥⎦ Hence, time taken by smaller tap = 5 h and time taken by
30(15 + x ) + 30(15 − x ) 9 larger tap = 3 h.
⇒ = 34. (i) Given that a rectangular pond has to be constructed in the
(15 − x )(15 + x ) 2
centre of a rectangular lawn of dimensions 50 m × 40 m .
450 + 30x + 450 − 30x 9
⇒ = x
(15)2 − x 2 2
2 2 x
[Q ( A − B )( A + B ) = A − B ] 40 m
x
900 9
⇒ =
225 − x 2 2 x
900 × 2
⇒ = 225 − x 2 ⇒ 200 = 225 − x 2 ⇒ x 2 = 25 50 m
9
⇒ x=±5 [taking square root on both sides] Now, length of rectangular lawn
But speed cannot be negative. ( l1 ) = 50 m
∴ x=5 and breadth of rectangular lawn
Hence, speed of stream is 5 km/h. ( b1 ) = 40 m
33. Let the time taken by smaller tap to fill tank completely = x h ∴ Length of rectangular pond
1 ( l2 ) = 50 – ( x + x ) = 50 – 2 x
So, volume of tank filled by smaller tap in 1 h =
x and breadth of rectangular pond
1 ( b 2 ) = 40 – ( x + x ) = 40 – 2 x
Volume of tank filled by larger tap in 1 h =
x −2 (ii) Given, area of the grass surrounding the pond = 1184 m2
7 15 ∴ Area of rectangular lawn – Area of rectangular pond
Now, time taken by both taps to fill = 1 = h
8 8 = Area of grass surrounding the pond
15 1 15 15 l1 × b1 – l2 × b 2 = 1184
Tank filled by smaller tap in h= × =
8 x 8 8x [Q area of rectangle = length × breadth]
15 1 15 15
Tank filled by larger tap in h= × = ⇒ 50 × 40 − ( 50 − 2 x ) ( 40 − 2 x ) = 1184
8 x −2 8 8( x − 2 ) ⇒ 2000 − (2000 − 80x − 100x + 4x 2 ) = 1184
15 15 15 ⎡ 1 1 ⎤ ⇒ 80x + 100x – 4x 2 = 1184
Therefore, + =1 ⇒ + =1
8x 8( x − 2 ) 8 ⎢⎣ x x − 2 ⎥⎦
⇒ 4x 2 − 180x + 1184 = 0
2( x − 1) 8
⇒ = ⇒ 15( x − 1) = 4( x 2 − 2 x ) ⇒ x 2 – 45x + 296 = 0 [divide by 4]
x 2 − 2 x 15
2 (iii) Now, x − 4.5x + 296 = 0 ⇒ x 2– 37 x – 8x + 296 = 0
2
⇒ 15x − 15 = 4x − 8x
[by splitting the middle term]
⇒ 23x = 4x 2 + 15 ⇒ x ( x – 37 ) – 8 ( x – 37 ) = 0
2
⇒ 4x − 23x + 15 = 0 ⇒ ( x – 37 ) ( x – 8) = 0
By using quadratic formula ∴ x=8
[at x = 37, length and breadth of pond are –24 and –34,
− ( −23) ± ( −23)2 − 4 ⋅ 4 ⋅ 15 23 ± 529 − 240
x= ⇒ x= respectively but length and breadth cannot
2⋅4 8 be negative. So, x = 37 cannot be possible]
23 ± 289 23 ± 17 ∴ Length of pond = 50 – 2 x = 50 – 2 ( 8)
⇒ x= ⇒ x=
8 8 = 50 – 16 = 34 m
23 + 17 40 and breadth of pond = 40 – 2 x
Taking positive sign, x = = =5
8 8 = 40 – 2( 8) = 40 – 16 = 24 m
23 − 17 6 3 Hence, required length and breadth of pond are 34 m
Taking negative sign, x = = =
8 8 4 and 24 m, respectively.
Chapter Test
Multiple Choice Questions (ii) 15 y 2 − 41y − 14 = 0
1. Is − 8 a solution of the equation 3
(a) ,
2
(b)
3
,
7
3 x 2 + 8 x + 2 = 0? [NCERT Exemplar] 7 3 5 2
2 7 2 7
(a) Yes (c) , (d) ,
(b) No 3 5 5 3
(c) Cannot be determined 1
(iii) 21x 2 − 2x + =0
(d) None of the above 21
2. Solve the quadratic equation 1 1
(a) 21, 3 (b) ,
2 21 21
x − 14 x + 24 = 0 [CBSE 2013] 1 1 3
(c) − 21, (d) ,
(a) 2, 12 (b) 3, 8 21 21 21
(c) 8, 3 (d) None of these
(iv) 6 x 2 − 31x + 40 = 0
2
3. The roots of a equation 2x + 5 2x + 5 = 0 are (a)
5 8
, (b) −
5 8
,−
2 3 2 3
−5 2 ± 5 −5 2 ± 10
(a) (b) 5 3 2 5
2 4 (c) − , (d) ,
2 8 3 8
−5 2 ± 10
(c) (d) None of these (v) 3 x 2 + 2 5x − 5 = 0
5
5
4. The quadratic equation 7 y 2 − 4 y + 5 = 0 has (a) 2 5 , − 5 (b) 5 , −
3
(a) Real and distinct
5
(b) Real and equal (c) − 5 , 3 5 (d) − 5 ,
(c) Imaginary 3
(d) More than 2 real roots Short Answer Type Questions
5. If a number is added to twice its square, then 7. Find the roots of the equation x 2 + 182 = 27 x
the resultant is 21. The quadratic representation
of this situation is [CBSE 2014, 15]
8. Find the roots of the quadratic equation
(a) 2 x2 + x − 21 = 0 a2b 2x 2 + b 2x − a2x − 1 = 0 [CBSE 2012, 11]
(b) 2 x2 + x + 21 = 0 9. If the roots of the equation x 2 + 2cx + ab = 0 are
(c) 2 x2 − x + 21 = 0
real and unequal, then prove that the equation
(d) 2 x2 − x − 21 = 0 x 2 − 2(a + b) x + a2 + b 2 + 2c2 = 0 has no real roots.
Case Study MCQs Long Answer Type Questions
6. Sohan is preparing for UPSC exam. For this, he 10. If x = − 5 is a root of the quadratic equation
has to practice the chapter of quadratic
2x 2 + px − 15 = 0 and the quadratic equation
equations. So, he started with factorisation
method. p(x 2 + x) + k = 0 has equal roots, then find the
value of k. [CBSE 2016]
Let two roots of ax 2 + bx + c be p and q.
∴ ac = p × q and p + q = b
11. A rectangular park is to be designed whose
breadth is 3 m less than its length. Its area is to
Now, factorize each of the following quadratic be 4 sq m more than the area of a park that has
equations and find the roots. already been made in the shape of an isosceles
(i) x 2 − 10x + 21 = 0 triangle with its base as the breadth of the
(a) 9, 3 (b) 21, 1
rectangular park and of altitude 12 m. Find its
length and breadth of the rectangular park.
(c) 3, 7 (d) 3, 9

Answers
1. (b) 2. (a) 3. (b) 4. (c) 5. (a) 6. (i) (c) (ii) (d) (iii) (b) (iv) (a) (v) (d) For Detailed Solutions
1 1
7. 13 ,14 8. x = + ,− 10. 7/4 Scan the code
b2 a2

11. Length = 7 m and Breadth = 4 m


24 CBSE Term II Mathematics X (Standard)

CHAPTER 02

Arithmetic
Progressions
In this Chapter...
! Arithmetic Progression
! nth Term of an AP
! Sum of n-Term of an AP
! Arithmetic Mean

Sequence Some numbers arranged in definite order, In general, a , a + d , a + 2 d , a + 3 d , … represent an arithmetic


according to a definite rule are said to form a sequence. progression, where a is the first term and d is the common
Progression Sequences which follow a definite pattern are difference. This is called general form of an AP.
called progressions. If number of terms in an AP is finite, then it is called a finite
AP, otherwise it is called an infinite AP and such AP ’s do not
Arithmetic Progression have a last term.
An Arithmetic Progression (AP) is a list of numbers in which Method to Check an AP
each term is obtained by adding a fixed number to the When a List of Numbers is Given
preceding term except the first term.
Sometimes, a list of numbers or sequence is given and we
This fixed number is called the common difference ( d ) of the have to check that this sequence is an AP or not. For this, we
AP. It can be positive, negative or zero. find the differences of consecutive terms. If these differences
In other words, a list of numbers a 1 , a 2 , a 3 ,. . . , a n is called are same, then given list of numbers or sequence is an AP,
an arithmetic progression (AP), if there exists a constant otherwise not.
number d (called common difference) such that
Method to Write an AP When First Term and
a 2 − a1 = d Common Difference are Given
a3 − a2 = d
To write an AP, the minimum information required to know
a4 − a3 = d
the first term a and the common difference d of the
M arithmetic progression. Then, we put the values of a and d in
a n − a n − 1 = d and so on. a , a + d , a + 2 d , a + 3 d , … to get the required AP.
Each of the number in this list is called a term.
CBSE Term II Mathematics X (Standard) 25

nth Term of an AP Sum of First n-Terms of an AP


If the first term of an AP is ‘a’ and its common difference is If first term of an AP is ‘a’ and its common difference is ‘d’,
‘d’, then its nth term is given by the formula then the sum of its first n terms S n , is given by the formula
a n = a + ( n − 1 )d n
Sn = [2 a + ( n − 1 ) d ]
The nth term of an AP is also called its general term. 2
In an AP, nth term is known as last term of an AP and it is n
or Sn = [a + a n ]
denoted by l, which is given by the formula 2
l = a + ( n − 1 )d where, a n = nth term of an AP.
(i) If l is the last term of an AP having n terms, then sum of
nth Term from the End of an AP
all the terms is given by this formula
Let ‘a’ be the first term, ‘d’ be the common difference and ‘l’ n
be the last term of an AP, then nth term from the end can be Sn = [a + l ]
found by the formula 2
(ii) If S n and S n −1 are the sums of first n and ( n −1 ) terms of
nth term from the end = l − ( n − 1 )d
an AP respectively, then its nth term a n is given by
Selection of Terms in an AP
a n = Sn − Sn − 1
Number of terms Terms Common difference
3 a − d, a , a + d d
Arithmetic Mean
If a , b and c are in AP, then b is known as arithmetic mean of
4 a − 3 d, a − d, a + d, a + 3 d 2d a+c
a and c, i.e. b = .
5 a − 2 d, a − d, a , a + d, a + 2 d d 2
26 CBSE Term II Mathematics X (Standard)

Solved Examples
Example 1. Examine that the sequence 13, 10, 7, 4,... is Example 5. How many terms are there in the sequence
an AP. 3, 6, 9, 12, ..., 111?
Sol. Given, AP is 13, 10, 7, 4, …… Sol. Given, sequence is 3, 6, 9, 12, ..., 111.
Here, a1 = 13, a 2 = 10, a 3 = 7, a 4 = 4, ……… Here, 6 − 3 = 9 − 6 = 12 − 9 ... = 3
Here, we have a 2 − a1 = 10 − 13 = −3,
So, it is an AP with first term, a = 3 and common difference,
a 3 − a 2 = 7 − 10 = −3, d = 3. Let there be n terms in the given sequence.
a 4 − a 3 = 4 − 7 = −3 and so on.
Then, nth term = 111
Since, difference of any two consecutive terms is same.
So, the given sequence is an AP. ⇒ a + ( n − 1)d = 111 [Q a n = ( a + ( n − 1)d)]

Example 2. Find the common difference of the following ⇒ 3 + ( n − 1) × 3 = 111


AP’s . ⇒ 3(1 + n − 1) = 111
(i) 3 , − 2 , − 7 , − 12 ,. . . (ii) 11, 11, 11, 11, ... 111
⇒ n= ⇒ n = 37
3
1 1 1 1
(iii) 5 , 9 ,13 ,17 …… Hence, the given sequence contains 37 terms.
2 2 2 2
Sol. (i) Given, AP is 3, − 2 , − 7 , − 12 ,... Example 6. Which term of the AP: 21 , 18 , 15,. . . is −81 ?
Here, a1 = 3, a 2 = −2 , a 3 = −7, a 4 = −12 and so on. Sol. Given, AP is 21, 18, 15,... .
∴ Common difference ( d) = a 2 − a1 = −2 − 3 = −5 Here, a = 21 and d = 18 − 21 = −3
(ii) Given, AP is 11, 11, 11, 11, ... Let nth term of given AP be − 81
Here, a1 = 11, a 2 = 11, a 3 = 11, a 4 = 11 and so on. Then, a n = −81
∴Common difference ( d) = a 2 − a1 = 11 − 11 = 0 ⇒ a + ( n − 1)d = −81 [Q a n = a + ( n − 1)d ]
1 1 1 1 On putting the values of a and d, we get
(iii) Given, AP is 5 , 9 ,13 ,17 ,……
2 2 2 2 21 + ( n − 1)( −3) = −81 ⇒ 21 − 3n + 3 = −81
1 1 1 1 ⇒ 24 − 3n = −81 ⇒ −3n = −81 − 24 = −105
Here, a1 = 5 , a 2 = 9 , a 3 = 13 , a 4 = 17 and so on.
2 2 2 2 − 105
1 1 ⇒ n= = 35
∴ Common difference (d) = a 2 − a1 = 9 − 5 −3
2 2 Hence, 35th term of given AP is − 81.
19 11 8
= −
2 2 2
= =4 Example 7. How many numbers of two digits are
divisible by 7?
Example 3. Write an AP having 4 as the first term and Sol. Two-digits numbers are 10, 11, 12 , 13, 14, 15,... , 97 , 98, 99
− 3 as the common difference. in which only 14, 21,28,..., 98 are divisible by 7.
Sol. Given, first term ( a ) = 4 and common difference ( d) = − 3 Here, 21 − 14 = 28 − 21... = 7.
On putting the values of a and d in general form So, this list of numbers forms an AP, whose first term
a , a + d, a + 2 d, a + 3d,... , we get ( a ) = 14, common difference ( d) = 7.
4, 4 − 3, 4 + 2( −3), 4 + 3( −3), ... Let there are n terms in the above sequence, then a n = 98
4, 1, 4 – 6, 4 – 9, … or 4, 1, − 2 , − 5,... ⇒ a + ( n − 1)d = 98 [Q a n = a + ( n − 1)d]
Which is the required AP. ⇒ 14 + ( n − 1)7 = 98 ⇒ 14 + 7 n − 7 = 98
91
⇒ 7 n = 91 ⇒ n = = 13
Example 4. Find the 20th term of the sequence 7
7, 3, −1 , −5 . . . Hence, 13 numbers of two digits are divisible by 7.
Sol. Given, sequence is 7 , 3, − 1, − 5,... .
Example 8. Determine the 10 th term from the end of
Here, 3 − 7 = − 4, −1 − 3 = − 4, −5 + 1 = − 4 and so on.
the AP : 4, 9, 14, ..., 254.
So, given sequence is an AP, in which a = 7 and d = − 4.
Sol. Given, AP is 4, 9, 14,..., 254.
Since, nth term, a n = a + ( n − 1)d
Here, l = last term = 254
On putting n = 20, we get
d = common difference = 9 − 4 = 5
a 20 = a + (20 − 1)d = 7 + 19 ( −4) [Q a = 7 , d = −4]
∴ 10th term from the end = l − (10 − 1)d = l − 9d
= 7 − 19 × 4 = 7 − 76 = − 69
= 254 − 9 × 5 = 254 − 45 = 209
Hence, 20th term of given sequence is − 69.
CBSE Term II Mathematics X (Standard) 27

Alternate Method Rate of interest, R = 7% per year; Time, T = 1, 2, 3, 4,...


On reversing the given AP, new AP is 254, ..., 14, 9, 4. We know that, simple interest is given by the following formula
Here, first term ( a ) = 254 and PRT
SI =
common difference ( d) = 4 − 9 = −5 100
2000 × 7 × 1
Now, 10th term of new AP = a10 ∴SI at the end of 1st year = = ` 140
100
= 254 + (10 − 1)( −5)
2000 × 7 × 2
= 254 − 9 × 5 = 209 SI at the end of 2nd year = = ` 280
100
Hence, 10th term from the end of given AP is 209. 2000 × 7 × 3
SI at the end of 3rd year = = ` 420
Example 9. Determine the general term of an AP 100
whose 7th term is − 1 and 16 th term is 17. Thus, required list of numbers is 140, 280, 420, ... .
Sol. Let a be the first term and d be the common difference of Here, 280 − 140 = 420 − 280 K = 140
the AP, whose 7th term is −1 and 16th term is 17. So, above list of numbers forms an AP, whose first term
Since, a 7 = − 1 and a16 = 17 ( a ) = 140 and common difference ( d) = 140.
∴ We have, a + (7 − 1)d = − 1 ⇒ a + 6d = − 1 ...(i) Now, SI at the end of 20th year will be equal to 20th term of
and a + (16 − 1)d = 17 ⇒ a + 15d = 17 ...(ii) the above AP.
[Q a n = a + ( n − 1)d] Q a 20 = a + (20 − 1) d = 140 + 19 × 140 = 140 + 2660 = 2800
On subtracting Eq. (i) from Eq. (ii), we get Hence, the interest at the end of 20th year will be ` 2800.
a + 15d − a − 6d = 17 + 1 Example 12. Each year, a tree grow 5 cm less than the
⇒ 9d = 18 ⇒ d = 2 preceding year. If it grew by 1m in the first year,
On substituting d = 2 in Eq. (i), we get then in how many years will it have ceased
a + 6 × 2 = −1 growing? [CBSE 2015]
⇒ a + 12 = − 1 Sol. Given that, tree grow 5 cm or 0.05 m less than preceding
⇒ a = − 13 year.
Hence, general term, ∴The following sequence can be formed.
a n = a + ( n − 1 )d 1, (1 − 0. 05), (1 − 2 × 0. 05), ... ,0
= − 13 + ( n − 1)2 [Q a = − 13 and d = 2] i.e. 1, 0.95, 0.90, ... ,0 which is an AP.
= − 13 + 2 n − 2 = 2 n − 15 Here, a = 1, d = 0. 95 − 1 = − 0. 05 and l = 0
Example 10. Find four numbers in AP whose sum is 20 Let l = a n = a + ( n − 1 )d
and the sum of whose squares is 120. Then, 0 = 1 + ( n − 1) ( − 0. 05)
Sol. Let the numbers be a − 3d, a − d, a + d and a + 3d. ⇒ ( n − 1) ( 0. 05) = 1
Then, according to the given condition, we have 1
⇒ n −1=
( a − 3d) + ( a − d) + ( a + d) + ( a + 3d) = 20 …(i) 0. 05
and ( a − 3d)2 + ( a − d)2 + ( a + d)2 + ( a + 3d)2 = 120 …(ii) 1
⇒ n − 1 = × 100
5
From Eq. (i), we get
⇒ n − 1 = 20
4a = 20 ⇒ a = 5
⇒ n = 21
From Eq. (ii), we get
Hence, in 21 yr, tree will have ceased growing.
a 2 + 9d2 − 6da + a 2 + d2 − 2 ad + a 2 + d2 + 2 ad
+ a 2 + 9d2 + 6ad = 120 Example 13. The eighth term of an AP is half its second
2 2 term and the eleventh term exceeds one-third of its
⇒ 4a + 20d = 120
fourth term by 1. Find the 15th term.
⇒ a 2 + 5d2 = 30 [NCERT Exemplar]
⇒ 25 + 5d2 = 30 [Q a = 5]
2 2
Sol. Let a and d be the first term and last term of an AP. Then,
⇒ 5d = 5 ⇒ d = 1 ⇒ d = ±1 1 1
If d = 1, then the numbers are 2, 4, 6, 8 and if d = −1, then a 8 = a 2 and a11 = a 4 + 1
2 3
the numbers are 8, 6, 4, 2.
1
Hence, the numbers are 2, 4, 6, 8 or 8, 6, 4, 2. ⇒ a + ( 8 − 1 ) d = [ a + (2 − 1 ) d ]
2
Example 11. A sum of ` 2000 is invested at 7% simple 1
interest per year. Calculate the interest at the end and a + (11 − 1) d = [( a + ( 4 − 1) d) + 1]
3
of each year. Do these interest form an AP? If so, 1
then find the interest at the end of 20th year ⇒ a + 7 d = ( a + d)
2
making use of this fact. 1
Sol. Given, initial money P = ` 2000 and a + 10d = [( a + 3d) + 1]
3
28 CBSE Term II Mathematics X (Standard)

⇒ 2 a + 14d − a − d = 0 10 140
⇒ [2 a + (10 − 1)d] = 140 ⇒ 2 a + 9d =
and 3a + 30d = a + 3d + 1 2 5
⇒ a + 13d = 0 …(i) ⎡ n ⎤
Q S = [2 a + ( n − 1 )d ]
and 2 a + 27 d − 1 = 0 …(ii) ⎢⎣ n 2 ⎥⎦
On solving Eqs. (i) and (ii), we get ⇒ 2 a + 9d = 28 ...(i)
a = − 13, d = 1 Also, given sum of first 16 terms, (S16 ) = 320
∴ a15 = a + (15 − 1) (1) 16
⇒ [2 a + (16 − 1)d] = 320
= − 13 + 14 = 1 2
320
Example 14. The fourth term of an AP is 11. The sum ⇒ 2 a + 15d = ⇒ 2 a + 15d = 40 ...(ii)
8
of the fifth and seventh terms of the AP is 24. Find On subtracting Eq. (i) from Eq. (ii), we get
its common difference. [CBSE 2015] 6d = 12 ⇒ d = 2
Sol. Let a be the first term and d be the common difference. On putting d = 2 in Eq. (i), we get
Then, 2 a + 9(2 ) = 28 ⇒ 2 a = 28 − 18
a 4 = 11 ⇒ a + ( 4 − 1) d = 11 10
⇒ a= =5
⇒ a + 3d = 11 …(i) 2
Also, given a 5 + a 7 = 24 Thus, a = 5 and d = 2.
⇒ a + ( 5 − 1) d + a + (7 − 1) d = 24 m
Hence, sum of first m terms, (Sm ) = [2 a + ( m − 1)d]
⇒ a + 4d + a + 6d = 24 2
⇒ 2 a + 10d = 24 m
= [2( 5) + ( m − 1)2 ] = m [ 5 + ( m − 1)]
⇒ a + 5d = 12 [divide by 2] …(ii) 2
On subtracting Eq. (i) from Eq. (ii), we get = m ( 5 + m − 1) = m ( m + 4) = m 2 + 4m
1
2d = 1 ⇒ d =
2
Example 18. Find the sum of all three-digit natural
1 numbers, which are multiples of 11. [CBSE 2009]
Hence, common difference is . Sol. All three-digit natural numbers, multiples of 11 are 110, 121,
2
132, …, 990.
Example 15. Find the sum of the first 22 terms of the Here, common difference, 121 − 110 = 132 − 121 = ... = 11.
AP : 8, 3, − 2, ... So, it is an AP with first term, a = 110, common difference,
Sol. Given, AP is 8, 3, − 2, ... d = 11 and last term, l = 990.
Here, first term, ( a ) = 8 Let l = a n = a + ( n − 1 )d
Common difference, ( d) = 3 − 8 = − 5 and n = 22 ∴ 990 = 110 + ( n − 1) × 11
n ⇒ 990 = 110 + 11n − 11
Q Sum of first n terms, (Sn ) = [2 a + ( n − 1) d]
2 ⇒ 11n = 891 ⇒ n = 81
22 n
∴ Sum of first 22 terms, (S22 ) = [2 × 8 + (22 − 1) × ( − 5)] Q Sn = [ a + l ]
2 2
= 11 [16 + 21 × ( − 5)] 81
∴ S81 = [110 + 990]
= 11 [16 − 105] 2
= 11 ( − 89) = − 979 81
= × 1100 = 81 × 550 = 44550
Hence, sum of first 22 terms of an AP is − 979. 2

Example 16. Find the sum of first 24 terms of an AP, Example 19. If S n , the sum of first n terms of an AP is
whose nth term is given by a n = 3 + 2 n. given by S n = 3n 2 − 4n, find the nth term.[CBSE 2019]
Sol. Given, nth term of an AP, a n = 3 + 2 n Sol. Given, S n = 3n 2 − 4n …(i)
Clearly, sum of first 24 terms, (S24 )
On replacing n by ( n − 1) in Eq. (i), we get
24
= ( a + a 24 ) = 12( 5 + 51) Sn −1 = 3( n − 1)2 − 4( n − 1)
2
[Q a1 = 3 + 2 = 5 and a 24 = 3 + 2 × 24 = 3 + 48 = 51] nth term of the AP a n = Sn − Sn −1
= 12 × 56 = 672 ∴ a n = ( 3n 2 − 4n ) − [ 3( n − 1)2 − 4( n − 1)]
Example 17. If the sum of first 10 terms of an AP is 140 ⇒ a n = 3[ n 2 − ( n − 1)2 ] − 4[ n − ( n − 1)]
and the sum of first 16 terms is 320, then find the ⇒ a n = 3 [ n 2 − n 2 + 2 n − 1 ] − 4 [ n − n + 1]
sum of first m terms. ⇒ a n = 3 (2 n − 1) − 4
Sol. Let the first term of this AP be a and common difference be d. ⇒ a n = 6n − 3 − 4 ⇒ a n = 6n − 7
Given, sum of first 10 terms, (S10 ) = 140 Thus, the nth term of the AP = 6n − 7.
CBSE Term II Mathematics X (Standard) 29

Chapter
Practice
PART 1
Objective Questions
!
Multiple Choice Questions 7. The first four terms of an AP whose first term is − 2
1. Which of the following form of an AP? and the common difference is −2, are
[NCERT Exemplar] [NCERT Exemplar]
(a) − 1, − 1 , − 1 , − 1 , ... (b) 0, 2 , 0, 2, … (a) − 2 , 0, 2 , 4 (b) − 2 , 4, − 8, 16
1 1 1 (c) − 2 , − 4, − 6, − 8 (d) − 2 , − 4, − 8, − 16
(c) 1 , 1 , 2 , 2 , 3, 3, K (d) , , , K
2 3 4 8. Let a be a sequence defined by a1 = 1, a 2 = 1 and
2. Which of the following is not an AP? a
a n = a n − 1 + a n − 2 for all n > 2, then the value of 4 is
[CBSE 2020 (Standard)] a3
(a) −1.2 , 0.8, 2.8, ... 2 5 4 3
(b) 3, 3 + 2 , 3 + 2 2 , 3 + 3 2 , … (a) (b) (c) (d)
3 4 5 2
4 7 9 12
(c) , , , , .... 9. If an AP have 8 as the first term and −5 as the
3 3 3 3
−1 −2 −3 common difference and its first three terms are
(d) , , , .... 8, A , B, then ( A + B ) is equal to
5 5 5
(a) 0 (b) −1
5 (c) 1 (d) 2
3. If − , a, 2 are consecutive terms in an Arithmetic
7 10. In an AP, if d = −4, n = 7 and a n = 4, then a is equal to
Progression, then the value of ‘a’ is
[CBSE 2020 (Standard)] (a) 6 (b) 7
9 9 (c) 20 (d) 28
(a) (b) −5 5
7 14 11. The 11th term of an AP − 5, , 0, , . . .
(c)
19
(d)
19 2 2
7 14 [NCERT Exemplar]
(a) − 20 (b) 20
4. The common difference of an AP, whose nth term is (c) −30 (d) 30
a n = ( 3n + 7 ), is
(a) 3 (b) 7 12. The 21st term of an AP whose first two terms are
(c) 10 (d) 6 − 3 and 4, is [NCERT Exemplar]
(a) 17 (b) 137
5. The value of x for which 2 x ,( x + 10) and ( 3x + 2 ) are (c) 143 (d) − 143
the three consecutive terms of an AP, is
[CBSE 2020 (Standard)]
13. If the 2nd term of an AP is 13 and 5th term is 25,
(a) 6 (b) − 6
what is its 7th term?
(c) 18 (d) −18 (a) 30 (b) 33
(c) 37 (d) 38
6. The value of p for which (2 p + 1 ), 10 and ( 5p + 5) are
14. Which term of an AP : 21, 42, 63, 84, ... is 210?
three consecutive terms of an AP is [NCERT Exemplar]
(a) − 1 (b) − 2 (a) 9th (b) 10th
(c) 1 (d) 2 (c) 11th (d) 12th
30 CBSE Term II Mathematics X (Standard)

15. If the common difference of an AP is 5, then what !


Case Based MCQs
is a18 − a13 ?
(a) 5 (b) 20 27. In a flower bed, there are 43 rose plants in the first
(c) 25 (d) 30 row, 41 in the second, 39 in the third and so on.
16. What is the common difference of an AP in which (i) If there are 11 rose plants in the last row, then
a18 − a14 = 32 ? [NCERT Exemplar] number of rose required are
(a) 16 (b) 15
(a) 8 (b) − 8
(c) 17 (d) 10
(c) − 4 (d) 4
(ii) Difference of rose plants in 7th row and 13th row is
17. Two APs have the same common difference. (a) 11 (b) 12
The first term of one of these is − 1 and that of (c) 13 (d) 14
the other is − 8. The difference between their 4th
(iii) If there are x rose plants in 15 rose, then x is
terms is [NCERT Exemplar]
equal to
(a) − 1 (b) − 8
(a) 10 (b) 12
(c) 7 (d) − 9
(c) 13 (d) 15
18. If 7 times the 7th term of an AP is equal to 11 times (iv) The rose plants in 6th row is
its 11th term, then its 18th term will be (a) 35 (b) 37
(a) 7 (b) 11 (c) 33 (d) 31
(c) 18 (d) 0
(v) The total number of rose plants in 5th and
19. The 4th term from the end of an AP 8th row is
− 11 , − 8 , − 5, . . . , 49 is (a) 64 (b) 54
(a) 37 (b) 40 (c) 46 (d) 45
(c) 43 (d) 58
28. The sum of the first five terms of an AP and the
20. Which term of the AP 5, 15, 25, ... will be 130 more sum of the first seven terms of the same AP is 167.
than its 31st term? The sum of the first ten terms of this AP is 235.
(a) 42 (b) 44 (i) Let first term and common difference of an AP be
(c) 46 (d) 48 a and d, respectively. Then pair of linear equations
21. The number of terms of an AP 5, 9, 13, ..., 185 is for given problem is
[NCERT Exemplar, CBSE 2020 (Standard)] (a) 13a + 31d = 167, 2 a + 9d = 47
(a) 31 (b) 51 (b) 13a + 31d = 169, 2 a + 9d = 45
(c) 41 (d) 46 (c) 12 a + 31d = 167 , 2 a + 9d = 47
22. The sum of AP, sequence −37 , − 33, − 29, . . . . . . . . . (d) 12 a + 31d = 169, 2 a + 9d = 45
(ii) Common difference of given AP is
upto 12 term is
(a) 5 (b) 7
(a) 180 (b) −180
(c) 170 (d) −170 (c) 9 (d) 11
(iii) First term of given AP is
23. The sum of first 16 terms of the AP 10, 6, 2, ... is
(a) 3 (b) 4
[NCERT Exemplar]
(c) 2 (d) 1
(a) − 320 (b) 320
(c) − 352 (d) − 400 (iv) Fourth term of the AP is
(a) 15 (b) 16
24. If the first term of an AP is − 5 and the common
(c) 17 (d) 18
difference is 2, then the sum of the first 6 terms is
(v) Sum of first twenty terms is
(a) 0 (b) 5
(a) 970 (b) 990
(c) 6 (d) 15
(c) 950 (d) 980
25. In an AP, if a = 1 , a n = 20 and S n = 399, then n is
29. India is competitive manufacturing location due to
equal to the low cost of manpower and strong technical and
(a) 19 (b) 21 engineering capabilities contributing to higher
(c) 38 (d) 42
quality production runs. The production of TV sets
26. The sum of first five multiples of 3 is in a factory increases uniformly by a fixed number
[NCERT Exemplar] every year. It produced 16000 sets in 6th year and
(a) 45 (b) 55 (c) 65 (d) 75 22600 in 9th year.
CBSE Term II Mathematics X (Standard) 31

(ii) What is the minimum number of days he needs to


practice till his goal is achieved?
(a) 10 (b) 12
(c) 11 (d) 9
(iii) Which of the following term is not in the AP of the
above given situation?
(a) 41 (b) 30
(c) 37 (d) 39
(iv) If nth term of an AP is given by a n = 2 n + 3, then
common difference of an AP is
(a) 2 (b) 3
Based on the above information, answer the (c) 5 (d) 1
following questions: (v) The value of x, for which 2 x, x + 10 , 3 x + 2 are three
(i) Find the production during first year. consecutive terms of an AP, is
(a) 4000 sets (b) 5000 sets (a) 6 (b) − 6
(c) 6000 sets (d) 7000 sets (c) 18 (d) − 18
(ii) Find the production during 8th year. 31. Your elder brother wants to buy a car and plans to
(a) 48000 sets (b) 20400 sets take loan from a bank for his car. He repays his total
(c) 43000 sets (d) None of these loan of ` 118000 by paying every month starting
(iii) Find the production during first 3 years. with the first installment of ` 1000. If he increases
(a) 20000 sets (b) 25000 sets the installment by ` 100 every month, answer the
(c) 31000 sets (d) 21600 sets following :
(iv) In which year, the production is ` 29200.
(a) 11 (b) 12
(c) 10 (d) 8
(v) Find the difference of the production during
7th year and 4th year.
(a) 5500 (b) 6700
(c) 5400 (d) 6600
30. Your friend Veer wants to participate in a 200 m
race. He can currently run that distance in 51
seconds and with each day of practice it takes him
2 seconds less. He wants to do in 31 seconds. [CBSE Question Bank]
(i) The amount paid by him in 30th installment is
(a) 3900 (b) 3500
(c) 3700 (d) 3600
(ii) The amount paid by him in the 30 installments is
(a) 37000 (b) 73500
(c) 75300 (d) 75000
(iii) What amount does he still have to pay after 30th
installment?
(a) 45500 (b) 49000
(c) 44500 (d) 54000
(iv) If total installments are 40, then amount paid in
the last installment?
(a) 4900 (b) 3900
[CBSE Question Bank] (c) 5900 (d) 9400
(i) Which of the following terms are in AP for the (v) The ratio of the 1st installment to the last
given situation? installment is
(a) 51, 53, 55…. (b) 51, 49, 47…. (a) 1 : 49 (b) 10 : 49
(c) − 51, − 53, − 55…. (d) 51, 55, 59… (c) 10 : 39 (d) 39 : 10
32 CBSE Term II Mathematics X (Standard)

15. Split 207 into three parts such that these are in AP
PART 2 and the product of the two smaller parts is 4623.
[NCERT Exemplar]
Subjective Questions 16. Find the 12th term from the end of the AP
− 2 , − 4, − 6, . . . , − 100 . [NCERT Exemplar]
!
Short Answer Type Questions
17. How many numbers lie between 10 and 300, which
1. Justify whether it is true to say that divided by 4 leave a remainder 3?
−3 5
−1 , , − 2 , , . . . forms an AP as a 2 − a1 = a 3 − a 2 . 18. If m times the mth term of an AP is equal to n
2 2 times its nth term, show that the ( m + n )th term of
[NCERT Exemplar]
the AP is zero.
2. Find the values of a, b and c if it is given that the
19. Find the sum of first 20 terms of the following AP
numbers a, 7, b, 23, c are in AP. [CBSE 2020 (Standard)]
sequence 1, 4, 7, 10, ……
3. The angles of a triangle are in AP. The greatest angle
20. Which term of the AP : 120, 116, 112, … is first
is twice the least. Find all the angles of the triangle.
[NCERT Exemplar]
negative term? [CBSE 2012]

4. The taxi fare after each km, when the fare is ` 15 for 21. How many terms of AP 18, 16, 14, ... should be taken,
the first kilometre and ` 8 for each additional so that their sum is zero? [CBSE 2013]
kilometre, does not form an AP as the total fare 22. Find the sum of first 8 multiples of 3. [CBSE 2018]
(in `) after each kilometre is 15, 8, 8, 8, … . Is the
23. Subha Rao started work in 1995 at an annual salary
statement true? Give reasons.
of ` 5000 and received an increment of ` 200 each
5. Determine k, so that k 2 + 4k + 8, 2 k 2 + 3k + 6 and year. In which year did his income reach ` 7000?
3k 2 + 4k + 4 are three consecutive terms of an AP. [NCERT Exemplar]
[NCERT Exemplar] 24. Ramkali saves ` 5 in the first week of a year and
6. Show that ( a − b )2 ,( a 2 + b 2 ) and ( a + b )2 are in AP. then increased her weekly savings by ` 1.75. If in
[CBSE 2020 (Standard)] the nth week, her weekly saving becomes ` 20.75.
Find n. [NCERT Exemplar]
7. Find the 11th term from the last term (towards the
1 + 3 + 5 + Kupto n terms
first term) of the AP 12, 8, 4, .., − 84. 25. If = 9, then find the
[CBSE 2020 (Standard)] 2 + 5 + 8 + . . . upto 8 terms
8. For the AP −3, − 7 , − 11 ,K can we find directly value of n.
a 30 − a 20 without actually finding a 30 and a 20 ? 26. In an AP, if S n = 3n 2 + 5n and a k = 164, then find
Give reason for your answer. [NCERT Exemplar] the value of k. [NCERT Exemplar]
9. Is 0 a term of the AP 31, 28, 25,…? Justify your 27. Find the sum ( −5) + ( −8 ) + ( −11 ) + K + ( −230).
answer. [CBSE 2020 (Standard)]
10. If four numbers are in AP such that their sum is 50 28. Sum of the first n terms of an AP is 5n 2 − 3n. Find
and the greatest number is 4 times the least, then the AP and also find its 16th term. [CBSE 2010]
find the numbers.
29. The sum of the first n terms of an AP whose first
11. Find the 20th term of the AP whose 7th term is 24 term is 8 and the common difference is 20, is equal
less than the 11th term, first term being 12. to the sum of first 2n terms of another AP whose
[NCERT Exemplar]
first term is −30 and the common difference is 8.
12. If the 9th term of an AP is zero, then prove that its Find the value of n. [NCERT Exemplar]
29th term is twice its 19th term.
30. Find the sum of 10 terms of an AP.
13. The 16th term of an AP is 1 more than twice its 8th
term. If the 12th term of an AP is 47, then find its 2 , 8 , 18 , 32 , …,
nth term. 31. Find the sum of all multiples of 7 lying between
14. Find the 19th term of the following sequence. 500 and 900.
⎧ n 2 , where n is even 32. Find the sum of all the two digit numbers which leave
tn = ⎨ 2 the remainder 2 when divided by 5. [CBSE 2019]
⎩n − 1, where n is odd [CBSE 2015]
CBSE Term II Mathematics X (Standard) 33

33. If S n denotes the sum of first n terms of an AP, then ⎛ 1⎞ ⎛ 2⎞ ⎛ 3⎞


(ii) ⎜ 4 − ⎟ + ⎜4 − ⎟ + ⎜4 − ⎟ + . . . upto n terms.
prove that S12 = 3 (S 8 − S 4 ). [NCERT Exemplar] ⎝ n⎠ ⎝ n⎠ ⎝ n⎠
[NCERT Exemplar]
34. Find the sum of last ten terms of the AP 8, 10,
12,..., 126. [NCERT Exemplar] 48. Find the sum of the two middle most terms of an
4 2 1
35. Find the sum of the first 100 natural numbers. AP − , − 1 , − ,. . . , 4 .
[CBSE 2020 (Standard)] 3 3 3
36. Find the sum of first seven numbers which are 49. Find the sum of first 17 terms of an AP whose 4th
multiples of 2 as well as of 9. [NCERT Exemplar] and 9th terms are − 15 and − 30, respectively.
37. For an AP, it is given that the first term ( a) = 5, 50. The sum of first n terms of three AP’s are S1 , S 2
common difference ( d) = 3, and the nth term and S 3 . The first term of each AP is unity and their
( a n ) = 50. Find n and sum of first n terms (S n ) of common differences are 1, 2 and 3, respectively.
the AP. [CBSE 2020 (Standard)] Prove that
38. Find the sum of first 16 terms of an Arithmetic S 1 + S 3 = 2S 2 . [CBSE 2016]
Progression whose 4th and 9th terms are −15 and 51. If the sum of first four terms of an AP is 40 and that
− 30, respectively. [CBSE 2020 (Standard)] of first 14 terms is 280. Find the sum of its first n
terms. [CBSE 2019]
39. If the sum of first 14 terms of an Arithmetic
Progression is 1050 and its fourth term is 40, find 52. The ratio of the 11th term to the 18th term of an AP is
its 20th term. [CBSE 2020 (Standard)] 2 : 3. Find the ratio of the 5th term to the 21st term
and also the ratio of the sum of the first five terms to
!
Long Answer Type Questions the sum of the first 21 terms. [NCERT Exemplar]

40. If the nth terms of the two AP’s 9, 7, 5, ... and 24, 53. The sum of four consecutive numbers in AP is
21, 18, ... are the same, then find the value of n. 32 and the ratio of the product of the first and
Also, that term. [NCERT Exemplar] last terms to the product of two middle terms is
41. The 26th, 11th and the last terms of an AP are, 0, 3 7 : 15. Find the numbers. [CBSE 2020 (Standard)]
1 54. Show that the sum of an AP whose first term is a,
and − , respectively. Find the common difference
5 the second term b and the last term c, is equal to
and the number of terms. [NCERT Exemplar] ( a + c ) ( b + c − 2 a)
. [NCERT Exemplar]
42. The 4th term of an AP is zero. Prove that the 25th 2 ( b − a)
term of the AP is three times its 11th term. 1 2
[CBSE 2016] 55. How many terms of the AP 20, 19 , 18 , ... must
3 3
1 1
43. If the mth term of an AP is and nth term is , be taken, so that their sum is 300?
n m
then show that its mnth term is 1. !
Case Base Questions
44. In an AP given that the first term ( a) = 54, the 56. Kanika was given her pocket money on Jan 1st,
common difference ( d) = − 3 and the n th term 2008. She puts ` 1 on day 1, ` 2 on day 2, ` 3 on day
( a n ) = 0, find n and the sum of first nterms (S n ) of 3 and continued doing so till the end of the month.
the AP. [CBSE 2020 (Standard)] From this money into her piggy bank, she also
45. Solve 1 + 4 + 7 + 10 + . . . + x = 287. spent ` 204 of her pocket money and found that at
the end of the month she still had ` 100 with her.
[CBSE 2020 (Standard)]
[NCERT Exemplar]
46. Solve the equation: (i) How much Kanika take till the end of the month
1 + 5 + 9 + 13 + K + x = 1326 from pocket money?
[CBSE 2020 (Standard)]
(ii) How much was pocket money for the month?
47. Find the sum (iii) What is the amount saved by Kanika, till
(i) 1 + ( − 2 ) + ( − 5 ) + ( − 8 ) + . . . + ( − 236 ) January 13th, 2008?
34 CBSE Term II Mathematics X (Standard)

SOLUTIONS
Objective Questions −1 −2 −3
(d) We have, , , , ....
1. (a) (a) Here, t1 = − 1, t 2 = − 1, t 3 = − 1 and t 4 = − 1 5 5 5
1 2 −3
Now, t 2 − t1 = − 1 + 1 = 0 Here, a1 = − , a 2 = − , a 3 =
t3 − t2 = − 1 + 1 = 0 5 5 5
2 ⎛ −1⎞ 1
t4 − t3 = − 1 + 1 = 0 Now, a 2 − a1 = − − ⎜ ⎟ = −
5 ⎝ 5⎠ 5
Clearly, the difference of successive terms is same,
therefore given list of numbers forms an AP. −3 ⎛ 2 ⎞ −3 2 1
a3 − a2 = − ⎜− ⎟ = + =−
(b) Here, t1 = 0, t 2 = 2 , t 3 = 0 and t 4 = 2 5 ⎝ 5⎠ 5 5 5
Now, t 2 − t1 = 2 − 0 = 2 Thus, given series is an AP.
t3 − t2 = 0 − 2 = − 2 Hence, in the given options, option (c) is not an AP.
5
t4 − t3 = 2 − 0 = 2 3. (b) Given, − , a , 2 are consecutive terms in AP.
7
Clearly, the difference of successive terms is not same,
therefore given list of numbers does not form an AP. ⎛ 5⎞
∴ a − ⎜− ⎟ = 2 − a [Q In AP, a 2 − a1 = a 3 − a 2]
⎝ 7⎠
(c) Here, t1 = 1, t 2 = 1, t 3 = 2 and t 4 = 2
5 9 9
Now, t 2 − t1 = 1 − 1 = 0 ⇒ 2 a = 2 − ⇒2 a = ⇒ a =
7 7 14
t3 − t2 = 2 − 1 = 1
4. (a) Given, nth term of an AP is
t4 − t2 = 2 − 2 = 0
a n = ( 3n + 7 )
Clearly, the difference of successive terms is not same,
therefore given list of numbers does not form an AP. ∴ The common difference of an AP = a n − a n − 1
1 1 1 = ( 3n + 7 ) − [ 3( n − 1) + 7 ]
(d) , , , …
2 3 4 = 3n + 7 − ( 3n + 4) = 7 − 4 = 3
1 1 1 a+c
Here, t1 = , t 2 = and t 3 = 5. (a) If a , b , c are in AP, then b = [Q b − a = c − b]
2 3 4 2
1 1 2−3 1 Given, 2 x , ( x + 10) and ( 3x + 2 ) are in AP.
Now, t 2 − t1 = − = =−
3 2 6 6 2 x + ( 3x + 2 )
∴ x + 10 =
1 1 3−4 1 2
t3 − t2 = − = =−
4 3 12 12 5x + 2
⇒ x + 10 = ⇒ 2 x + 20 = 5x + 2
Clearly, the difference of successive terms is not same, 2
therefore given list of numbers does not form an AP. ⇒ 5x − 2 x = 20 − 2 ⇒ 3x = 18
2. (c) The condition for given series is not AP is the common 18
⇒ x= =6
difference of two consecutive terms is not constant. 3
(a) We have, − 1.2 , 0.8, 2.8, ... 6. (d) Let a1 = 2 p + 1, a 2 = 10 and a 3 = 5p + 5.
Here, a1 = − 1.2, a 2 = 0.8, a 3 = 2.8 Given that three consecutive terms are in AP.
Now, a 2 − a1 = 0.8 − ( −1.2 ) = 2.0 ∴ a 2 − a1 = a 3 − a 2
and a 3 − a 2 = 2.8 − 0.8 = 2 ⇒ 10 − (2 p + 1) = 5p + 5 − 10
Thus, given series is an AP. ⇒ 10 + 10 = 5p + 5 + 2 p + 1
(b) We have, 3, 3 + 2 , 3 + 2 2 , 3 + 3 2 , … ⇒ 20 = 7 p + 6
Here, a1 = 3, a 2 = 3 + 2 , a 3 = 3 + 2 2 ⇒ 7 p = 20 − 6
⇒ 7 p = 14
Now, a 2 − a1 = 3 + 2 − 3 = 2 14
⇒ p= =2
and a 3 − a 2 = 3 + 2 2 − ( 3 + 2 ) = 2 7
Thus, given series is an AP. 7. (c) Let the first four terms of an AP are
4 7 9 12 a , a + d, a + 2 d and a + 3d.
(c) We have, , , , ....
3 3 3 3 Given, that first term, a = − 2 and common difference,
4 7 9 d = − 2, then we have an AP as follows
Here, a1 = , a 2 = , a 3 = − 2 , − 2 − 2 , − 2 + 2 ( −2 ), − 2 + 3( −2 ) i.e. − 2 , − 4 , − 6, − 8
3 3 3
7 4 3 8. (d) We have, a1 = 1, a 2 = 1 and a n = a n −1 + a n − 2 for all n > 2
Now, a 2 − a1 = − = = 1
3 3 3 On putting n = 3 and 4, we get
9 7 2 a 3 = a 2 + a1 = 1 + 1 = 2
a3 − a2 = − =
3 3 3 a4 = a3 + a2 = 2 + 1 = 3
Thus, given series is not an AP, as common difference is a4 3
Now, =
not constant. a3 2
CBSE Term II Mathematics X (Standard) 35

9. (c) Given, first term ( a ) = 8, common difference ( d) = − 5 17. (c) Let the common difference of two APs are d1 and d2,
On putting the values of a and d in general form, respectively.
a , a + d, a + 2 d, a + 3d, ..., we get By condition, d1 = d2 = d ...(i)
8, 8 − 5, 8 + 2( −5), 8 + 3( −5), ... or 8, 3, −2 , − 7 , ... Let the first term of first AP ( a1 ) = − 1
On comparing with given terms 8, A ,B , ... , we get and the first term of second AP ( a 2 ) = − 8
A = 3, B = − 2 We know that, the nth term of an AP, Tn = a + ( n − 1 ) d
∴ A + B = 3 + ( −2 ) = 3 − 2 = 1 ∴ 4th term of first AP, T4 = a1 + ( 4 − 1) d = − 1 + 3 d
10. (d) In an AP, an = a + ( n − 1) d and 4th term of second AP, T4ʹ = a 2 + ( 4 − 1 ) d = − 8 + 3 d
⇒ 4 = a + ( 7 − 1 ) ( − 4) [by given condition] Now, the difference between their 4th terms is
⇒ 4 = a + 6 ( − 4) ⇒ 4 + 24 = a |T4 − T4ʹ| = ( − 1 + 3 d ) − ( − 8 + 3 d )
∴ a = 28 = − 1 + 3 d + 8 − 3d = 7
5 5 Hence, the required difference is 7.
11. (b) Given AP, − 5, − , 0 , , ……
2 2 18. (d) According to the question,
−5 5 7 a 7 = 11 a11
Here, a = − 5, d = + 5=
2 2 ⇒ 7 [ a + ( 7 − 1 ) d ] = 11 [ a + (11 − 1 ) d ]
∴ a11 = a + (11 − 1 ) d [Q a n = a + ( n − 1 ) d ] [Q a n = a + ( n − 1 ) d]
5
= − 5 + (10 ) × = − 5 + 25 = 20 ⇒ 7 ( a + 6 d) = 11 ( a + 10 d)
2
⇒ 7 a + 42 d = 11a + 110 d
12. (b) Given, first two terms of an AP are a = − 3 and a + d = 4.
⇒ 4a + 68 d = 0
⇒ −3 + d = 4
⇒ 4 ( a + 17 d ) = 0
Common difference, d = 7
∴ a 21 = a + (21 − 1 ) d[Q a n = a + ( n − 1 ) d ] ⇒ a + 17 d = 0 …(i)
= − 3 + (20 ) 7 ∴ 18th term of an AP, a18 = a + (18 − 1 ) d
= − 3 + 140 = 137 = a + 17 d = 0 [from Eq. (i)]
13. (b) Given, a 2 = 13 and a 5 = 25 19. (b) We know that, the n th term of an AP from the end is
⇒ a + (2 − 1 ) d = 13 [Q a n = a + ( n − 1 ) d ] a n = l − ( n − 1) d …(i)
and a + ( 5 − 1 ) d = 25 Here, l = Last term and l = 49 [given]
⇒ a + d = 13 …(i) Common difference, d = − 8 − ( − 11 )
and a + 4 d = 25 …(ii) = − 8 + 11 = 3
On subtracting Eq. (i) from Eq. (ii), we get From Eq. (i), a 4 = 49 − ( 4 − 1 ) 3 = 49 − 9 = 40
3 d = 25 − 13 = 12 ⇒ d = 4 20. (b) We have, a = 5 and d = 10
From Eq. (i), a = 13 − 4 = 9 ∴ a 31 = a + 30d = 5 + 30 × 10 = 305
∴ a 7 = a + ( 7 − 1 ) d = 9 + 6 × 4 = 33 Let nth term of the given AP be 130 more than its 31st term.
14. (b) Let nth term of the given AP be 210. Then, a n = 130 + a 31
Here, first term, a = 21 ⇒ a + ( n − 1)d = 130 + 305
and common difference, ⇒ 5 + 10( n − 1) = 435
d = 42 − 21 = 21 and a n = 210 ⇒ 10( n − 1) = 430
Q a n = a + (n − 1) d
⇒ n − 1 = 43
⇒ 210 = 21 + ( n − 1 ) 21
⇒ n = 44
⇒ 210 = 21 + 21 n − 21
Hence, 44th term of the given AP is 130 more than its
⇒ 210 = 21 n ⇒ n = 10 31st term.
Hence, the 10th term of an AP is 210. 21. (d) Given, AP sequence is 5, 9, 13, ..., 185.
15. (c) Given, the common difference of AP i.e. d = 5 Here, first term a = 5
Now, a18 − a13 = a + (18 − 1 ) d − [ a + (13 − 1 ) d]
Common difference, d = 9 − 5 = 4
[Q a n = a + ( n − 1 ) d ]
and last term, l = 185
= a + 17 × 5 − a − 12 × 5 = 85 − 60 = 25
Q l = a + ( n − 1 )d
16. (a) Given, a18 − a14 = 32
∴ 185 = 5 + ( n − 1)4
⇒ a + (18 − 1 ) d − [ a + (14 − 1 ) d] = 32 [Q a n = a + ( n − 1 ) d ] ⇒ 180 = ( n − 1)4
⇒ a + 17 d − a − 13 d = 32 180
⇒ 4 d = 32 ⇒ ( n − 1) =
4
∴ d=8 ⇒ ( n − 1) = 45
Which is the required common difference of an AP. ⇒ n = 45 + 1 ⇒ n = 46
36 CBSE Term II Mathematics X (Standard)

22. (b) Given sequence is Let n be the number of rows required.


−37 , − 33, − 29, …… upto 12 terms ∴ a n = 11
Here a = −37 , d = −33 − ( −37 ) = 4 ⇒ a + ( n − 1) d = 11
n ⇒ 43 + ( n − 1) ( − 2 ) = 11
Q S n = [2 a + ( n − 1 )d ]
2 ⇒ − 2 ( n − 1) = − 32
12 ⇒ n − 1 = 16 ⇒ n = 17
∴ S12 = [2 × ( −37 ) + (12 − 1)4]
2 (ii) (b) Number of rose plants in 7th row = a 7
= 6[ −74 + 44] = a + 6d = 43 + 6 ( − 2 ) = 43 − 12 = 31
= 6 × ( −30) = −180 Number of rose plants in 13th row = a 13
23. (a) Given, AP is 10, 6, 2 , ... = a + 12 d = 43 + 12 ( − 2 ) = 43 − 24 = 19
Here, first term a = 10, common difference, d = − 4 ∴ Required difference = 31 − 19 = 12
16 (iii) (d) Here, n = 15
∴ S16 = [2 a + (16 − 1 ) d ]
2 ∴ a 15 = a + 14d = 43 + 14 ( − 2 ) = 43 − 28 = 15
⎡ n ⎤
Q Sn = {2 a + ( n − 1) d} (iv) (c) Number of rose plants in 6th row
⎣⎢ 2 ⎦⎥ = a 6 = a + 5d
= 8 [2 × 10 + 15 ( − 4)] = 43 + 5 ( − 2 )
= 8 (20 − 60 ) = 8 ( − 40 ) = − 320 = 43 − 10 = 33
24. (a) Given, a = − 5 and d = 2 (v) (a) Number of rose plants in 5th row
6 = a 5 = a + 4d
∴ S 6 = [ 2 a + ( 6 − 1) d ]
2 = 43 + 4 ( − 2 )
⎡ n ⎤
Q Sn = {2 a + ( n − 1) d} = 43 − 8 = 35
⎣⎢ 2 ⎦⎥ Number of rose plants in 8th row
= 3 [ 2 ( − 5) + 5 (2 )] = a 8 = a + 7d
= 3 ( − 10 + 10 ) = 0 = 43 + 7 ( − 2 )
n
25. (c)Q S n = [2 a + ( n − 1 ) d ] = 43 − 14 = 29
2 ∴ Required sum = 35 + 29 = 64
n
399 = [2 × 1 + ( n − 1 ) d ] 28. (i) (c) Let the number of terms of AP be n.
2
Q Sum of first n terms of an AP,
798 = 2 n + n ( n − 1 ) d …(i)
n
and a n = 20 S n = [2 a + ( n − 1 ) d ] ...(i)
2
⇒ a + ( n − 1 ) d = 20 [Q a n = a + ( n − 1 ) d ]
∴ Sum of first five terms of an AP,
⇒ 1 + ( n − 1 ) d = 20 ⇒ ( n − 1) d = 19 …(ii) 5
Using Eq. (ii) in Eq. (i), we get S 5 = [2 a + ( 5 − 1 ) d ] [from Eq.(i)]
2
798 = 2 n + 19 n 5
⇒ 798 = 21 n = (2 a + 4 d ) = 5 ( a + 2 d )
2
798
∴ n= = 38 ⇒ S 5 = 5a + 10 d ...(ii)
21
and sum of first seven terms of an AP,
26. (a) The first five multiples of 3 are 3, 6, 9, 12 and 15. 7
Here, first term, a = 3, common difference, d = 6 − 3 = 3 and S 7 = [2 a + ( 7 − 1 ) d ]
2
number of terms, n = 5 7
5 = [2 a + 6 d ] = 7 ( a + 3 d )
∴ S 5 = [2 a + ( 5 − 1 ) d ] 2
2 ⇒ S 7 = 7 a + 21 d ...(iii)
⎡ n ⎤
Q Sn = {2 a + ( n − 1) d} Now, by given condition,
⎣⎢ 2 ⎦⎥
S 5 + S 7 = 167
5
= [2 × 3 + 4 × 3 ] ⇒ 5a + 10 d + 7 a + 21 d = 167
2
5 ⇒ 12 a + 31 d = 167 ...(iv)
= ( 6 + 12 ) = 5 × 9 = 45 Given that, sum of first ten terms of this AP is 235.
2
∴ S10 = 235
27. (i) (c) Number of rose plants in 1st, 2nd and 3rd row ...... are
10
43, 41, 39, ...... ⇒ [2 a + (10 − 1 ) d ] = 235
2
So, it forms an AP with first term,
⇒ 5 (2 a + 9 d ) = 235
a = 43 and common difference,
⇒ 2 a + 9 d = 47 …(v)
d = 41 − 43 = − 2
CBSE Term II Mathematics X (Standard) 37

(ii) (a) On multiplying Eq. (v) by 6 and then subtracting it 24200


⇒ ( n − 1) =
into Eq. (iv), we get 2200
12 a + 54 d = 282 ⇒ n − 1 = 11 ⇒ n = 12
12 a + 31 d = 167 Hence, production is ` 29200 in 12th year.
− − − (v) (d) The difference of the production during 7th year and
23 d = 115 4th year = T7 − T4
⇒ d=5 = a + (7 − 1 ) d − [ a + ( 4 − 1) d ]
(iii) (d) Now, put the value of d in Eq. (v), we get = 6d − 3d = 3d = 3 × 2200 = 6600
2 a + 9 ( 5) = 47 ⇒ 2 a + 45 = 47 30. (i) (b) In first day, Veer takes 51 seconds to complete the
⇒ 2 a = 47 − 45 = 2 ⇒ a = 1 200 m race. But in each day he takes 2 seconds lesser
(iv) (b) a 4 = a + 3d than the previous days.
= 1 + 3 ( 5) = 1 + 15 = 16 Thus, AP series will formed
(v) (a) Sum of first twenty terms of this AP, 51, 49, 47, …
20 (ii) (c) Since, Veer wants to achieve the race in 31 seconds.
S20 = [2 a + (20 − 1 ) d ] Let Veer takes n days to achieve the target.
2
= 10 [2 × ( 1 ) + 19 × ( 5)] = 10 (2 + 95) ∴ Tn = a + ( n − 1)d
= 10 × 97 = 970 Here, a = 51, d = 49 − 51 = − 2
Hence, the required sum of its first twenty terms is 970. ∴ 31 = 51 + ( n − 1)( −2 )
29. (i) (b) Let the production of TV sets in first year be ‘a’ units. ⇒ ( n − 1)2 = 20 ⇒ ( n − 1) = 10 ⇒ n = 11
Then, production in the next consecutive years are Hence, he needs minimum 11 days to achieve the goal.
a + d, a + 2 d, …. (iii) (b) In an AP series, we get the series of odd terms.
Thus, we get the sequence, a , a + d, a + 2 d, … Hence, term 30 is not an AP.
This is an AP sequence, whose first term = a (iv) (a) Given, a n = 2 n + 3
and common difference = d. ∴Common difference = a n + 1 − a n
Given, T6 = 16000 and T9 = 22600 = 2 ( n + 1 ) + 3 − ( 2 n + 3)
∴ a + ( 6 − 1)d = 16000 = 2n + 2 + 3 − 2n − 3 = 2
and a + ( 9 − 1)d = 22600 [Q Tn = a + ( n − 1) d] (v) (a) Given, terms 2 x , x + 10, 3x + 2 are in AP.
2 x + ( 3x + 2 )
⇒ a + 5d = 16000 …(i) ∴ x + 10 =
and a + 8 d = 22600 …(ii) 2
⇒ 2 x + 20 = 5x + 2 ⇒ 3x = 18 ⇒ x = 6
On subtracting Eq. (i) from Eq. (ii), we get
31. (i) (a) Since, he pays first installment of ` 1000 and next
3d = 22600 − 16000
consecutive months he pay the installment are
⇒ 3d = 6600 1100, 1200, 1300, … .
⇒ d = 2200 Thus, we get the AP sequence,
Put d = 2200 in Eq. (i), we get 1000, 1100, 1200, …
a + 5 × 2200 = 16000 Here, a = 1000, d = 1100 − 1000 = 100
⇒ a = 16000 − 11000 = 5000 Now, T30 = a + ( 30 − 1) d
Hence, the production during first year is 5000 sets. = 1000 + 29 × 100 = 1000 + 2900 = 3900
(ii) (b) The production during 8th year is Hence, the amount paid by him in 30th installment is
T8 = a + ( 8 − 1) d ` 3900.
= 5000 + 7 × 2200 30
(ii) (b) Now, S30 = [2 a + ( 30 − 1) d]
= 5000 + 15400 = 20400 2
Hence, production during 8th year is 20400 sets. = 15(2 × 1000 + 29 × 100)
(iii) (d) The production during first 3 years, = 15(2000 + 2900)
3 = 15 × 4900 = ` 73500
S 3 = [2 a + ( 3 − 1 ) d ]
2 (iii) (c) After 30th installment, he still have to pay
3 = 118000 − 73500 = 44500
= [2 × 5000 + 2 × 2200]
2 (iv) (a) The amount in last 40th installment is
= 3[ 5000 + 2200] T40 = a + ( 40 − 1) d
= 3 × 7200 = 21600 = 1000 + 39 × 100
(iv) (b) Let in nth year, the production is 29200 = 1000 + 3900 = ` 4900
Q Tn = a + ( n − 1) d (v) (b) The ratio of 1st installment to the last installment
∴ 29200 = 5000 + ( n − 1)2200 1000 10
is i.e. .
⇒ ( n − 1)2200 = 24200 4900 49
38 CBSE Term II Mathematics X (Standard)

Subjective Questions 5. Since, k 2 + 4k + 8, 2 k 2 + 3k + 6 and 3k 2 + 4k + 4 are


−3 5 consecutive terms of an AP.
1. Here, a1 = − 1, a 2 = , a 3 = − 2 and a 4 =
2 2 ∴ 2 k 2 + 3k + 6 − ( k 2 + 4k + 8) = 3 k 2 + 4k + 4
−3 1 − (2 k 2 + 3k + 6 ) = Common difference
Now, a 2 − a1 = +1=−
2 2
⇒ 2 k + 3k + 6 − k 2 − 4k − 8 = 3k 2 + 4k + 4 − 2 k 2 − 3k − 6
2
3 1
a3 − a2 = − 2 + = − ⇒ k2 − k − 2 = k2 + k − 2
2 2
5 9 ⇒ − k = k ⇒ −2 k = 0 ⇒ k = 0
a4 − a3 = + 2 =
2 2 6. Let a1 = ( a − b )2, a 2 = a 2 + b 2 and a 3 = ( a + b )2.
Clearly, the difference of successive terms is not same, Now, a 2 − a1 = a 2 + b 2 − ( a − b )2
although, a 2 − a1 = a 3 − a 2 but a 3 − a 2 ≠ a 4 − a 3., therefore it = a 2 + b 2 − ( a 2 + b 2 − 2 ab ) = 2ab …(i)
does not form an AP.
and a 3 − a 2 = ( a + b )2 − ( a 2 + b 2 )
2. Given, sequence a, 7, b, 23, c is an AP.
= a 2 + b 2 + 2 ab − ( a 2 + b 2 ) = 2ab …(ii)
Since, a, 7, b is in AP.
From Eqs. (i) and (ii), we get
a+b x + z⎤
∴ 7= [Q If x , y , z in AP, then, y = a 2 − a1 = a 3 − a 2
2 2 ⎦⎥
Hence, given terms are in AP.
⇒ a + b = 14 ...(i) 7. Given, sequence of an AP is 12, 8, 4, ..., − 84.
Since, 7, b, 23 is in AP. Here, first term is a = 12
7 + 23 30 Common difference is d = 8 − 12 = − 4 and last term, l = − 84
∴ b= ⇒b = ⇒ b = 15 ...(ii)
2 2 The nth term from the last term of an AP is l − ( n − 1)d.
Since, b, 23, c is in AP. ∴The 11th term from the last term of an AP
b+c = l − (11 − 1)d
∴ 23 =
2 = − 84 − (10) × ( − 4)
⇒ 23 × 2 = 15 + c [from Eq. (ii), b = 15] = − 84 + 40 = − 44
⇒ c = 46 − 15 8. Q nth term of an AP, a n = a + ( n − 1 ) d
⇒ c = 31 ∴ a 30 = a + ( 30 − 1 ) d = a + 29 d
Put b = 15 in Eq. (i), we get and a 20 = a + (20 − 1 ) d = a + 19 d ...(i)
a + 15 = 14 Now, a 30 − a 20 = ( a + 29 d ) − ( a + 19 d ) = 10 d
⇒ a = 14 − 15 and from given AP common difference,
⇒ a = −1 d = − 7 − ( − 3) = − 7 + 3 = − 4
Hence, values of a, b and c are respectively −1, 15 and 31. ∴ a 30 − a 20 = 10 ( − 4) = − 40 [from Eq. (i)]
3. Let the angles are ( a − d)° , a ° , ( a + d)°. 9. Let 0 be the nth term of given AP. i.e. a n = 0.
Then, we get ( a − d) + a + ( a + d) = 180 Given that, first term a = 31,
and a + d = 2( a − d) Common difference, d = 28 − 31 = − 3
⇒ 3a = 180° ⇒ a = 60° The n th term of an AP, is a n = a + ( n − 1 ) d
and 60° + d = 2 ( 60° − d) ⇒ 0 = 31 + ( n − 1 ) ( −3)
⇒ 60° + d = 120° − 2 d 31
⇒ 3 ( n − 1 ) = 31 ⇒ n − 1 =
⇒ 3d = 60° 3
⇒ d = 20° 31 34 1
∴ n= + 1= = 11
∴ The angles of an AP are 3 3 3
a − d = 60° − 20° = 40° Since, n should be positive integer. So, 0 is not a term of the
given AP.
a = 60°
and a + d = 60° + 20° = 80° 10. Let four numbers in AP are a , a + d, a + 2 d, a + 3d. Then,
Hence, angles of an AP are 40°, 60°, 80°. ( a ) + ( a + d) + ( a + 2 d) + ( a + 3d) = 50 ⇒ 4a + 6d = 50
4. No, because the total fare (in `) after each kilometre is ⇒ 2 a + 3d = 25 …(i)
and ( a + 3d ) = 4 ( a )
15, (15 + 8), (15 + 2 × 8), (15 + 3 × 8),… or 15, 23, 31, 39,…
⇒ a=d …(ii)
Let t1 = 15, t 2 = 23, t 3 = 31 and t 4 = 39
On solving Eq. (i) and Eq. (ii), we get
Now, t 2 − t1 = 23 − 15 = 8
a=d=5
t 3 − t 2 = 31 − 23 = 8
∴ The four numbers in AP are
t 4 − t 3 = 39 − 31 = 8
a = 5, a + d = 5 + 5 = 10
Since, all the successive terms of the given list have same
a + 2 d = 5 + 10 = 15, a + 3d = 5 + 15 = 20
difference i.e. common difference = 8
Hence, four numbers in AP are 5, 10, 15 and 20.
Hence, the total fare after each killometre form an AP.
CBSE Term II Mathematics X (Standard) 39

11. Let the first term, common difference and number of terms 16. Given AP, −2 , − 4, − 6,... , − 100
of an AP are a , d and n, respectively. Here, first term ( a ) = − 2, common difference
Given that, first term ( a ) = 12. ( d ) = − 4 − ( − 2 ) = − 2 and the last term ( l ) = − 100.
Now by condition, We know that, the nth term a n of an AP from the end is
7th term (T7 ) = 11th term (T11 ) − 24 a n = l − ( n − 1) d, where l is the last term and d is the
[Q nth term of an AP, Tn = a + (n − 1) d] common difference.
⇒ a + (7 − 1) d = a + (11 − 1) d − 24 ∴ 12th term from the end,
⇒ a + 6 d = a + 10 d − 24 a12 = − 100 − (12 − 1) ( − 2 )
⇒ 24 = 4 d ⇒ d = 6 = − 100 + (11) (2 ) = − 100 + 22 = − 78.
∴ 20th term of AP, T20 = a + (20 − 1) d = 12 + 19 × 6 = 126 Hence, the 12th term from the end is – 78.
Hence, the required 20th term of an AP is 126. 17. Here, the first number is 11, which divided by 4 leave
remainder 3 between 10 and 300. Last term before 300 is
12. Let the first term, common difference and number of terms
299, which divided by 4 leave remainder 3.
of an AP are a , d and n, respectively.
∴ Required AP is 11, 15, 19, 23,... , 299
Given that, 9th term of an AP, T9 = 0
Here, first term ( a ) = 11, common difference d = 15 − 11 = 4
[Q nth term of an AP, Tn = a + ( n − 1) d]
Q n th term, a n = a + ( n − 1 ) d = l [last term]
⇒ a + ( 9 − 1) d = 0 ⇒ a + 8 d = 0 ⇒ a = − 8 d ... (i)
⇒ 299 = 11 + ( n − 1 ) 4
Now, its 19th term, T19 = a + (19 − 1) d
⇒ 299 − 11 = ( n − 1 ) 4
= − 8 d + 18 d [from Eq. (i)]
⇒ 4 ( n − 1 ) = 288 ⇒ ( n − 1 ) = 72
⇒ T19 = 10 d ... (ii) ∴ n = 73
and its 29th term, T29 = a + (29 − 1) d 18. Let first term of an AP is a and common difference is d.
= − 8 d + 28 d [from Eq. (i)]
The n th term of an AP is
= 20d = 2 × (10 d)
a n = a + ( n − 1 )d
⇒ T29 = 2 × T19 [from Eq. (ii)]
According to the given condition,
Hence, its 29th term is twice its 19th term. Hence proved.
m × am = n × an
13. Let first term and common difference of an AP are a and d.
∴ m × [ a + ( m − 1 ) d ] = n × [ a + ( n − 1 )d ]
According to the given condition,
⇒ a ( m − n ) = [( n 2 − m 2 ) + ( − n + m )]d
a12 = 47
⇒ a + 11d = 47 ...(i) ⇒ a( m − n ) = [( n − m ) ( n + m ) + ( m − n )]d
and a16 = 1 + 2 a 8 [by given condition] ⇒ a = [ − ( n + m ) + 1] d ...(i)
⇒ [ a + (16 − 1) d] = 1 + 2 [ a + ( 8 − 1) d] [divide both sides by m − n ]
⇒ a − d = −1 ...(ii) Now, ( m + n )th term of an AP is
On solving Eqs. (i) and (ii), we get a m + n = a + ( m + n − 1 )d
d = 4 and a = 3 = [ − ( n + m ) + 1 ]d + ( m + n − 1 ) d
∴ a n = 3 + ( n − 1) 4 = 4 n − 1 =0 Hence proved.
⎧ n 2 , where n is even 19. Given, AP sequence is 1, 4, 7, 10, .... whose first term is a =1
14. We have, t n = ⎨ 2
⎩n − 1, where n is odd and common difference, d = 4 − 1 = 3.
Q Sum of n terms of an AP is
For 19th term, i.e. for n = 19 which is odd, we take n
S n = [2 a + ( n − 1 )d ]
t n = n 2 − 1 = (19)2 − 1 = 360 2
15. Let the three parts of the number 207 are ( a − d) , a and 20
∴ S20 = [2 × 1 + (20 − 1) × 3] [put, d = 3]
( a + d), which are in AP. 2
Now, by given condition, = 10 [2 + 19 × 3] = 10 [2 + 57 ] = 590
Sum of these parts = 207 Hence, sum of the first 20 terms of an AP is 590.
⇒ a − d + a + a + d = 207 20. Given sequence in AP is 120, 116, 112, ...
⇒ 3a = 207 Here, a = 120, d = 116 − 120 = − 4
a = 69 The nth term of an AP is
Given that, product of the two smaller parts = 4623 a n = a + ( n − 1) d
⇒ a ( a − d) = 4623 a n = 120 + ( n − 1) ( − 4)
⇒ 69 ⋅ ( 69 − d) = 4623 ⇒ 69 − d = 67 For, first negative term, a n < 0
⇒ d = 69 − 67 = 2 ∴ 120 + ( n − 1) ( − 4) < 0
So, first part = a − d = 69 − 2 = 67, ⇒ 4 ( n − 1) > 120
second part = a = 69 ⇒ ( n − 1) > 30
and third part = a + d = 69 + 2 = 71, ⇒ n > 31
Hence, required three parts are 67, 69, 71. ∴ The first negative term is 32.
40 CBSE Term II Mathematics X (Standard)

21. Here, a = 18 and d = −2 27. Given series is ( −5) + ( −8) + ( −11) + .... + ( −230)
Let n terms are taken, so that their sum is zero. Here, first term, a = − 5 and common difference,
Then, we have d = − 8 − ( − 5) = − 8 + 5 = − 3
Sn = 0 Q a n = a + ( n − 1 )d
n
⇒ [ 2 a + ( n − 1 )d ] = 0 ∴ ( −230) = − 5 + ( n − 1)( −3)
2
⇒ ( n − 1)( −3) = − 230 + 5
⇒ 2 a + ( n − 1 )d = 0 −225
⇒ 2 × 18 + ( n − 1)( −2 ) = 0 ⇒ ( n − 1) =
−3
⇒ n − 1 = 18 ⇒ n = 19 ⇒ n − 1 = 75
22. First 8 multiples of 3 are 3, 6, 9, 12, 15, 18, 21, 24. ⇒ n = 75 + 1 = 76
∴The sum of first 8 multiples of 3 Q The sum of n th term is
n 8 n
= [ a + l ] = [ 3 + 24] = 4 × 27 = 108 Sn = [ a + l ]
2 2 2
23. The annual salary received by Subha Rao in the years 76
∴ Sn = [ −5 + ( −230)]
1995, 1996, 1997 etc., is ` 5000, ` 5200, ` 5400, …, ` 7000 2
Hence, the list of numbers 5000, 5200, 5400, …, 7000 forms = 38 [ −235] = − 8930
an AP 28. S n = 5n 2 − 3n
Q a 2 − a1 = a 3 − a 2 = 200 Now, a n = Sn − Sn −1
Let nth term of an AP, a n = 7000 = 5n 2 − 3n − [ 5( n − 1)2 − 3( n − 1)]
⇒ 7000 = a + ( n − 1)d [Q a n = a + ( n − 1)d]
= 5n 2 − 3n − [ 5( n 2 + 1 − 2 n ) − 3n + 3]
⇒ 7000 = 5000 + ( n − 1)(200)
⇒ a n = 10n − 8 …(i)
⇒ 200( n − 1) = 7000 − 5000 = 2000
2000 Clearly, a16 = 10 × 16 − 8 = 160 − 8 = 152
⇒ n −1 = = 10 Now, for finding AP, put n = 1, 2, 3, 4 …… in Eq. (i).
200
⇒ n = 10 + 1 = 11 So, from Eq. (i), we have
Thus, 11th year of his service or in 2005 Subha Rao received a1 = 2 , a 2 = 12 , a 3 = 22
an annual salary ` 7000. The AP is 2, 12, 22, ...... .
24. Ramkali’ savings in the subsequent weeks are respectively 29. Given, a1 = 8, d1 = 20, a 2 = −30, d2 = 8
` 5, ` 5 + ` 1.75, ` 5 + 2 × ` 1.75, ` 5 + 3 × 1.75 … Sn = S2n
In nth week her saving will be ` 5 + (n − 1) × ` 1.75 n 2n
⇒ 5 + ( n − 1) × 1.75 = 20.75 [given] [2 × 8 + ( n − 1) × 20] = [2 × ( −30) × 30 + (2 n − 1) × 8]
2 2
⇒ ( n − 1) × 1.75 = 20.75 −5 = 15.75 ⇒ [16 + ( n − 1)20] = 2 [ −60 + (2 n − 1)8]
15.75 ⇒ 16 + 20n − 20 = −120 + 32 n − 16
⇒ n −1 = =9
1.75 ⇒ 12 n = 132 ⇒ n = 11
⇒ n = 9 + 1 = 10 30. Here, a1 = 2 , a 2 = 8 = 2 2 , a 3 = 3 2
1 + 3 + 5 + K upto n terms ∴ a = 2 , d = a 2 − a1 = 2 2 − 2 = 2
25. Given, =9
2 + 5 + 8 + ... upto 8 term 10
n ∴ S10 = [2 × 2 + (10 − 1)( 2 )]
[2(1) + ( n − 1)2 ] 2
⇒ 2 =9
8 = 5[2 2 + 9 2 ] = 55 2
[ 2 ( 2 ) + ( 8 − 1) 3]
2 31. The multiples of 7 lying between 500 and 900 are 504, 511,
n (2 n ) 518, ..., 896.
⇒ =9
8(25) Clearly, it forms an AP.
⇒ n 2 = 9 × 100 ⇒ n 2 = 900 ⇒ n = 30 Here, a = 504 and d = 511 − 504 = 7
Let there are n terms, i.e. a n = 896
26. Q n th term of an AP,
⇒ a + ( n − 1) d = 896
a n = Sn − Sn −1
⇒ 504 + ( n − 1)7 = 896
= 3n 2 + 5n − 3 ( n − 1 ) 2 − 5 ( n − 1 )
⇒ ( n − 1)7 = 392
[Q Sn = 3n 2 + 5n (given)] ⇒ n − 1 = 56
2 2
= 3 n + 5 n − 3 n − 3 + 6 n − 5n + 5 ⇒ n = 57
a n = 6n + 2 …(i) n 57
Now, S57 = ( a + l ) = ( 504 + 896)
or a k = 6 k + 2 = 164 [Q a k = 164 (given)] 2 2
⇒ 6k = 164 − 2 = 162 57
= × 1400 = 39900
∴ k = 27 2
CBSE Term II Mathematics X (Standard) 41

32. The sequence of two digit number which divided by 5 and common difference ( d) = 36 − 18 = 18
leave the remainder 2 is n
∴ S 7 = [2 a + ( n − 1 ) d ]
12, 17, 22, ..., 97 which is an AP 2
Here, a = 12 , d = 17 − 12 = 5 and l = 97 7
= [2 (18) + (7 − 1) 18]
∴ l = a + ( n − 1) d 2
∴ 97 = 12 + ( n − 1)5 7
= [ 36 + 6 × 18]
⇒ 85 = ( n − 1) 5 2
⇒ ( n − 1) = 17 = 7 (18 + 3 × 18)
⇒ n = 17 + 1 = 18 = 7 (18 + 54)
∴ Required sum of two digit number which divided by = 7 × 72 = 504
n 37. Given, first term of an AP, a = 5
5 and leave the remainder 2 is ( a + l )
2 Common difference, d = 3
18 nth term of an AP, a n = 50
= (12 + 97 ) = 9 × 109 = 981
2 ∴ a + ( n − 1)d = 50
33. Q Sum of n terms of an AP, ⇒ 5 + ( n − 1)3 = 50
n ⇒ ( n − 1)3 = 50 − 5
S n = [2 a + ( n − 1 ) d ] …(i)
2 45
8 ⇒ n −1=
∴ S 8 = [2 a + ( 8 − 1 ) d ] 3
2 ⇒ n − 1 = 15
= 4 (2 a + 7 d ) = 8a + 28d ⇒ n = 15 + 1 = 16
4
and S 4 = [2 a + ( 4 − 1 ) d ] ∴ The sum of nth term of an AP is
2 n
= 2 (2 a + 3 d ) = 4 a + 6 d S n = [2 a + ( n − 1 )d ]
2
Now, S8 − S4 = 8a + 28d − 4a − 6d = 4a + 22 d …(ii) 16
12 ∴ S16 = [2 × 5 + (16 − 1) × 3 ]
and S12 = [2 a + (12 − 1 ) d ] = 6 (2 a + 11 d ) 2
2
= 8 [10 + 15 × 3]
= 3 ( 4a + 22 d) = 3 (S8 − S4 ) [from Eq. (ii)]
= 8 [10 + 45]
∴ S12 = 3(S8 − S4 ) Hence proved.
= 8 × 55 = 440
34. For finding, the sum of last ten terms, we write the given AP 38. Let a and d be the first term and common difference of an
in reverse order. AP. Then,
i.e. 126, 124, 122 , ... , 12 , 10, 8 a 4 = − 15 and a 9 = − 30
Here, first term ( a ) = 126, ⇒ a + ( 4 − 1) d = − 15
common difference, and a + ( 9 − 1)d = − 30
( d) = 124 − 126 = − 2 ⇒ a + 3d = − 15 …(i)
10 ⎡ n ⎤
∴ S10 = [2 a + (10 − 1 ) d] Q Sn = [2 a + ( n − 1) d] and a + 8d = −30 …(ii)
2 ⎢⎣ 2 ⎥⎦
On subtracting Eq. (i) from Eq. (ii), we get
= 5 {2 (126) + 9 ( −2 )} 8d − 3d = − 30 − ( −15)
= 5 (252 − 18) ⇒ 5d = − 30 + 15
= 5 × 234 15
⇒ d=−
= 1170 5
35. Let the sequence of 100 natural numbers be 1, 2, 3, ...., 100 ⇒ d = −3
Here, a = 1, d = 2 − 1 = 3 − 2 = 1 Put d = − 3 in Eq. (i), we get
Thus, natural number sequence is an AP. a + 3( −3) = − 15
Now, sum of first 100 natural number is ⇒ a = − 15 + 9
100 ⎡ n ⎤ ⇒ a = −6
S100 = [2 × 1 + (100 − 1)1] Sn = [2 a + ( n − 1)d]
2 ⎣⎢ 2 ⎦⎥ ∴The sum of first 16 terms of an AP is
= 50 [2 + 99] n
S n = [2 a + ( n − 1 ) d ]
= 50 × 101 = 5050 2
36. For finding, the sum of first seven numbers which are multiples 16
⇒ S16 = [2 ( −6) + 15 ( −3)]
of 2 as well as of 9. Take LCM of 2 and 9 which is 18. 2
So, the series becomes 18, 36, 54,... = 8 [−12 − 45]
Here, first term ( a ) = 18, = 8 × ( −57 ) = − 456
42 CBSE Term II Mathematics X (Standard)

39. Let a and d be the first term and common difference of an and nth term of an AP is
AP. Then, Tn = a + ( n − 1) d ...(ii)
S14 = 1050 and T4 = 40 [given] Given that, 26th term of an AP = 0
14 ⇒ T26 = a + (26 − 1) d = 0 [from Eq. (i)]
⇒ [2 a + (14 − 1) d] = 1050 and a + ( 4 − 1) d = 40
2 ⇒ a + 25 d = 0 ...(iii)
⇒ 7 [2 a + 13d] = 1050 and a + 3d = 40 11th term of an AP = 3
⇒ 2 a + 13d = 150 …(i) ⇒ T11 = a + (11 − 1) d = 3 [from Eq. (ii)]
and a + 3d = 40 …(ii) ⇒ a + 10 d = 3 ... (iv)
Multiply Eq. (ii) by 2 and subtract Eq. (ii) from Eq. (i), and last term of an AP = − 1 / 5
13d − 6d = 150 − 80 ⇒ l = a + ( n − 1) d [from Eq. (i)]
⇒ 7 d = 70 ⇒ − 1 / 5 = a + ( n − 1) d ...(v)
⇒ d = 10 Now, subtracting Eq. (iv) from Eq. (iii),
Put d = 10 in Eq. (i), we get a + 25 d = 0
2 a + 13 × 10 = 150 a + 10 d = 3
− − −
⇒ 2 a = 150 − 130 15 d = − 3
⇒ 2 a = 20 1
⇒ a = 10 ⇒ d=−
5
∴The 20th term of an AP is Put the value of d in Eq. (iii), we get
a 20 = a + (20 − 1) d ⎛ 1⎞
= 10 + 19 × 10 a + 25 ⎜ − ⎟ = 0
⎝ 5⎠
= 10 + 190 = 200 ⇒ a −5=0
40. Let the first term, common difference and number of terms ⇒ a=5
of the AP 9, 7, 5, ... are a1 , d1 and n1, respectively.
Now, put the value of a , d in Eq. (v), we get
i.e. first term ( a1 ) = 9 and common difference ( d1 )
− 1 / 5 = 5 + ( n − 1) ( − 1 / 5)
= 7 − 9 = − 2.
⇒ − 1 = 25 − ( n − 1)
∴ Its nth term, Tʹ n = a1 + ( n − 1) d1
⇒ − 1 = 25 − n + 1
⇒ Tʹn = 9 + ( n − 1) ( − 2 ) ⇒ n = 25 + 2 = 27
⇒ Tʹn = 9 − 2 n + 2 Hence, the common difference and number of terms are
⇒ Tʹn = 11 − 2 n ...(i) − 1 / 5 and 27, respectively.
Let the first term, common difference and the number of 42. Let a and d be the first term and common difference of the
terms of the AP 24, 21, 18, ... are a 2 , d2 and n 2, respectively. given AP, respectively.
i.e. first term, ( a 2 ) = 24 and common difference Given a4 = 0
( d2 ) = 21 − 24 = − 3. ⇒ a + 3d = 0
∴ Its nth term, Tnʹʹ = a 2 + ( n − 1) d2 ⇒ a = − 3d …(i)
⇒ Tnʹʹ = 24 + ( n − 1) ( − 3) Now, a 25 = a + 24d
⇒ Tnʹʹ = 24 − 3n + 3 = − 3d + 24d [from Eq. (i)]
⇒ Tnʹʹ = 27 − 3n ...(ii) ⇒ a 25 = 21d …(ii)
Now, by given condition, Also, a11 = a + 10d
nth terms of the both APs are same, = − 3d + 10d [From Eq. (i)]
i.e. Tʹn = Tʹʹ ⇒ a11 = 7 d
n
11 − 2 n = 27 − 3n [from Eqs. (i) and (ii)] ⇒ 3a11 = 21d …(iii)
⇒ n = 16 From Eqs. (ii) and (iii), we get
∴ nth term of first AP, a 25 = 3a11 Hence proved.
Tʹ n = 11 − 2 n = 11 − 2 (16) 43. Let a and d be the first term and common difference of an
= 11 − 32 = − 21 AP.
and nth term of second AP, 1
Then, Tm =
Tʹʹ n = 27 − 3n n
1
= 27 − 3 (16) = 27 − 48 = − 21 ⇒ a + ( m − 1 )d = …(i)
n
Hence, the value of n is 16 and that term i.e. nth term is –21.
1
41. Let the first term, common difference and number of terms and Tn =
m
of an AP are a, d and n, respectively.
1
We know that, if last term of an AP is known, then ⇒ a + ( n − 1 )d = …(ii)
m
l = a + ( n − 1) d ...(i)
CBSE Term II Mathematics X (Standard) 43

Subtracting Eq. (ii) from Eq. (i), we get Now, sum of AP series is
1 1 n
[( m − 1) − ( n − 1)]d = − S n = [2 a + ( n − 1 ) d ]
n m 2
m −n 1 ⎛ x + 2⎞ ⎡ ⎛x+2 ⎞ ⎤
⇒ ( m − n )d = = ⎜ ⎟ 2 ×1 + ⎜ − 1⎟ 3⎥
mn 2 ⎝ 3 ⎠ ⎢⎣ ⎝ 3 ⎠ ⎦
1
⇒ d= x+2
mn = [2 + x − 1 ]
6
1
Put d = in Eq (i), we get x+2
mn ⇒ 287 = × ( x + 1)
1 1 6
a + ( m − 1) = 2
⇒ 1722 = x + 3x + 2
mn n
1 1 1 ⇒ x 2 + 3x − 1720 = 0
⇒ a= − ( m − 1) =
n mn mn ⇒ x 2 + ( 43 − 40)x − 1720 = 0 [splitting middle term]
∴ a mn = a + ( mn − 1)d 2
⇒ x + 43x − 40x − 1720 = 0
1 1
= + ( mn − 1) × ⇒ x ( x + 43) − 40( x + 43) = 0
mn mn
⇒ ( x − 40) ( x + 43) = 0
1 mn
= [1 + mn − 1] = =1 ⇒ x = 40, − 43
mn mn
But x = − 43 is not possible, because it is an increasing AP.
44. Given first term of an AP, a = 54
Hence, required value of x is 40.
Common difference, d = − 3
46. Given, equation is 1 + 5 + 9 + 13 + K + x = 1326
and nth term of an AP,
Here, first term is a1 = 1
an = 0
last term is a n = l = x
⇒ a + ( n − 1 )d = 0
Difference of two consecutive terms,
⇒ 54 + ( n − 1)( −3) = 0
5 − 1 = 4 and 9 − 5 = 4, which is same.
⇒ 54 − 3n + 3 = 0
Thus, given series is an AP.
⇒ 3n = 57
Then, nth term of given AP is
57
⇒ n= a n = a + ( n − 1 )d
3
∴ x = 1 + ( n − 1 )4
⇒ n = 19
⇒ ( n − 1) 4 = x − 1
Now, sum of first 19 terms of given AP is x −1
n ⇒ n −1=
S n = [2 a + ( n − 1 )d ] 4
2 x −1
19 ⇒ n= +1
∴ S19 = [2 × 54 + (19 − 1)( −3)] 4
2 x+3
19 ⇒ n=
= [108 − 54] 4
2 Now, sum of given AP is
= 513 n
S n = [ 2 a + ( n − 1 )d ]
45. Given equation is 1 + 4 + 7 + 10 + .... + x = 287 2
Consider series, 1 + 4 + 7 + 10 + .... + x x+ 3⎡ x+3 ⎤
∴ 1326 = 2 ×1+ ×4
Here, a1 = 1, a 2 = 4, a 3 = 7 4 × 2 ⎢⎣ 4 ⎥⎦
Now, a 2 − a1 = 4 − 1 = 3 ⇒ 1326 × 8 = ( x + 3) [2 + x + 3]
a3 − a2 = 7 − 4 = 3 ⇒ 10608 = ( x + 3) ( x + 5)
It implies that common difference is constant say 3. So, it is ⇒ x 2 + 8x + 15 − 10608 = 0
an AP series, whose first term is a = 1 and common ⇒ x 2 + 8x − 10593 = 0
difference d = 3. 2
Here, last term of an AP is l = x ⇒ x + (107 − 99)x − 10593 = 0
Q l = a + ( n − 1) d [by splitting middle term]
∴ x = 1 + ( n − 1) × 3 ⇒ x 2 + 107 x − 99x − 10593 = 0
⇒ x = 1 + 3n − 3 ⇒ x( x + 107 ) − 99( x + 107 ) = 0
⇒ x = 3n − 2 ⇒ ( x − 99) ( x + 107 ) = 0
x+2 ⇒ x = 99, − 107
⇒ n=
3
44 CBSE Term II Mathematics X (Standard)

47. (i) Here, first term ( a ) = 1 4 2 1


48. Given AP sequence is − , − 1, − , .... , 4
and common difference 3 3 3
( d) = ( − 2 ) − 1 = − 3 4
Here, first term ( a ) = − ,
Q Sum of n terms of an AP, 3
n 4 1
S n = [2 a + ( n − 1 ) d ] common difference ( d ) = − 1 + =
2 3 3
n 1 13
⇒ Sn = [2 × 1 + ( n − 1) × ( − 3)] and the last term ( l ) = 4 =
2 3 3
n Q nth term of an AP,
⇒ S n = (2 − 3 n + 3 )
2 l = an = a + (n − 1) d
n 13 4 1
⇒ S n = ( 5 − 3n ) ...(i) ⇒ = − + (n − 1)
2 3 3 3
We know that, if the last term ( l) of an AP is known, then ⇒ 13 = − 4 + ( n − 1 )
l = a + ( n − 1) d ⇒ n − 1 = 17
⇒ − 236 = 1 + ( n − 1) ( − 3) [Q l = − 236, given ] ⇒ n = 18 [even]
⇒ − 237 = − ( n − 1) × 3 ⎛ n⎞ ⎛n ⎞
So, the two middle most terms are ⎜ ⎟ th and ⎜ + 1⎟ th.
⇒ n − 1 = 79 ⎝2⎠ ⎝2 ⎠
⇒ n = 80 ⎛ 18⎞ ⎛ 18 ⎞
i.e. ⎜ ⎟ th and ⎜ + 1⎟ th terms
Now, put the value of n in Eq. (i), we get ⎝2⎠ ⎝2 ⎠
80 i.e. 9th and 10th terms.
Sn = [ 5 − 3 × 80]
2 4 ⎛ 1 ⎞ −4 + 8 4
∴ a9 = a + 8d = − + 8⎜ ⎟ = =
= 40 ( 5 − 240) 3 ⎝ 3⎠ 3 3
= 40 × ( − 235) −4 ⎛ 1 ⎞ −4 + 9 5
= − 9400 and a10 = a + 9d = + 9⎜ ⎟ = =
3 ⎝ 3⎠ 3 3
Hence, the required sum is − 9400. So, sum of the two middle most terms
Alternate Method = a 9 + a10
Given, a = 1, d = − 3 and l = − 236 4 5 9
∴ Sum of n terms of an AP, = + = =3
3 3 3
n
Sn = [ a + l] 49. Let the first term, common difference and the number of
2 terms in an AP are a, d and n, respectively.
80 We know that, the n th term of an AP,
= [1 + ( − 236)] [Q n = 80]
2 Tn = a + ( n − 1) d ... (i)
= 40 × ( − 235) ∴ 4th term of an AP,
= − 9400 T4 = a + ( 4 − 1 ) d = − 15 [given]
1
(ii) Here, first term, a = 4 − ⇒ a + 3 d = − 15 ...(ii)
n
and 9th term of an AP,
Common difference, T9 = a + ( 9 − 1 ) d = − 30 [given]
⎛ 2⎞ ⎛ 1 ⎞ − 2 1 −1 ⇒ a + 8 d = − 30 ...(iii)
d = ⎜4 − ⎟ − ⎜4 − ⎟ = + =
⎝ n⎠ ⎝ n⎠ n n n Now, subtract Eq. (ii) from Eq. (iii), we get
Q Sum of n terms of an AP, a + 8 d = − 30
n a + 3d = − 15
S n = [2 a + ( n − 1 ) d ] − − +
2
5 d = − 15
n ⎡ ⎛ 1⎞ ⎛ −1⎞ ⎤
⇒ Sn = ⎢2 ⎜ 4 − ⎟ + ( n − 1 ) ⎜ ⎟ ⎥ ⇒ d=−3
2 ⎣ ⎝ n⎠ ⎝ n ⎠⎦
Put the value of d in Eq. (ii), we get
n ⎧ 2 1⎫
= ⎨8 − − 1 + ⎬ a + 3 ( − 3) = − 15
2 ⎩ n n⎭
⇒ a − 9 = − 15
n ⎛ 1⎞ ⇒ a = − 15 + 9
= ⎜7 − ⎟
2 ⎝ n⎠ ⇒ a=−6
n ⎛ 7 n − 1⎞ Q Sum of first n terms of an AP,
= ×⎜ ⎟
2 ⎝ n ⎠ n
S n = [2 a + ( n − 1 ) d ]
7n − 1 2
=
2
CBSE Term II Mathematics X (Standard) 45

∴ Sum of first 17 terms of an AP, On substituting d = 2 in Eq. (i), we get


17 ⇒ 2 a + 3 × 2 = 20
S17 = [2 × ( − 6) + (17 − 1) ( − 3)]
2 ⇒ 2 a = 14
17 a =7
= [ − 12 + (16) ( − 3)]
2 n n
Now, Sn = [2 a + ( n − 1)d] = [2(7 ) + ( n − 1)2 ]
17 2 2
= ( − 12 − 48)
2 n
= [14 + 2 n − 2 ] = n[ 6 + n ] = 6n + n 2
17 2
= × ( − 60 )
2 Hence, the sum of first n terms is n 2 + 6n .
= 17 × ( − 30 ) 52. Let a and d be the first term and common difference of an AP.
= − 510 Given that, a11 : a18 = 2 : 3
Hence, the required sum of first 17 terms of an AP is − 510. a + 10 d 2
⇒ =
50. Given first term of each sum of an AP is 1 and common ratio a + 17 d 3
of each sum are 1, 2 and 3, respectively.
⇒ 3a + 30 d = 2 a + 34 d
n
∴ S 1 = [2(1) + ( n − 1)1] ⇒ a = 4d …(i)
2
Now, a 5 = a + 4 d = 4 d + 4 d = 8 d [from Eq. (i)]
n
= [2 + n − 1 ] and a 21 = a + 20 d = 4 d + 20 d = 24 d [from Eq. (i)]
2
n ∴ a 5 : a 21 = 8d : 24 d = 1 : 3
= ( n + 1) Now, sum of the first five terms,
2
n 5
∴ S 2 = [2(1) + ( n − 1)2 ] S 5 = [2 a + ( 5 − 1 ) d ]
2 2
n 5
= [2 + 2 n − 2 ] = [2 ( 4 d ) + 4 d ] [from Eq. (i)]
2 2
5 5
= n2 = ( 8 d + 4 d ) = × 12 d = 30 d
2 2
n
and S 3 = [2 (1 ) + ( n − 1) 3] and sum of the first 21 terms,
2 21
n S21 = [2 a + (21 − 1 ) d ]
= [2 + 3 n − 3] 2
2 21
n = [2 ( 4 d ) + 20 d ] [from Eq. (i)]
= ( 3n − 1) 2
2 21
LHS = S1 + S 3 = (28 d ) = 294 d
2
n n So, ratio of the sum of the first five terms to the sum of the
= ( n + 1 ) + ( 3 n − 1)
2 2 first 21 terms
n S 5 : S21 = 30 d : 294 d = 5 : 49
= [ n + 1 + 3 n − 1]
2 53. Let the four consecutive number of an AP be
n a , a + d, a + 2 d and a + 3d.
= × 4n
2 Since, sum of four consecutive number in AP is 32.
= 2 n 2 = 2S 2 Hence proved. ∴ a + a + d + a + 2 d + a + 3d = 32
⇒ 4a + 6d = 32
51. Given, S4 = 40 and S14 = 280
⇒ 2 a + 3d = 16 [divide by 2]
Let a be the first term and d be the common difference of 16 − 3d
4 ⇒ a= …(i)
given AP. Then, S4 = 40 ⇒ [2 a + ( 4 − 1)d] = 40 2
2
n According to the question,
⎡ ⎤
Q S = {2 a + ( n − 1) d} Product of first and last terms 7
⎢⎣ n 2 ⎥⎦ =
Product of two middle terms 15
⇒ 2[2 a + 3d] = 40 ⇒ 2 a + 3d = 20 …(i)
a ( a + 3d ) 7
and S14 = 280 =
14 ( a + d) ( a + 2 d) 15
⇒ [2 a + (14 − 1) d] = 280
2 ⇒ 15a ( a + 3d) = 7 ( a + d) ( a + 2 d)
⇒ 2 a + 13d = 40 …(ii) ⇒ 15a 2 + 45ad = 7 a 2 + 21ad + 14d2
On subtracting Eq. (i) from Eq. (ii), we get ⇒ 8a 2 + 24ad − 14d2 = 0
10d = 20 ⇒ d = 2 ⇒ 4a 2 + 12 ad − 7 d2 = 0 [divide by 2]
46 CBSE Term II Mathematics X (Standard)

⇒ 4a 2 + (14 − 2 )ad − 7 d2 = 0 1 2
55. Given AP is 20, 19 , 18 ,... .
[by splitting middle term] 3 3
⇒ 4a 2 + 14ad − 2 ad − 7 d2 = 0 1 58 58 − 60 −2
Here, a = 20 and d = 19 − 20 = − 20 = =
3 3 3 3
⇒ 2 a(2 a + 7 d) − d(2 a + 7 d) = 0
⇒ (2 a + 7 d ) (2 a − d ) = 0 Let n terms of given AP be required to get sum 300.
7d d n
⇒ a=− and a = …(ii) We know that, Sn = [2 a + ( n − 1)d]
2 2 2
7d n ⎡ ⎛ −2 ⎞ ⎤
Put a = − in Eq. (i), we get ⇒ 300 = ⎢2(20) + ( n − 1)⎜ ⎟ ⎥
2 2 ⎣ ⎝ 3 ⎠⎦
7 d 16 − 3d [Qa = 20 and d = − 2 / 3]
− =
2 2 ⎡ 2 2⎤
⇒ 600 = n 40 − n +
⇒ −7 d = 16 − 3d ⎢⎣ 3 3 ⎥⎦
⇒ 4d = − 16 ⇒ d = − 4 1
d ⇒ 600 = [120n − 2 n 2 + 2 n ]
Now, put a = in Eq. (i), we get 3
2 ⇒ 600 × 3 = 122 n − 2 n 2
d 16 − 3d
= ⇒ 1800 + 2 n 2 − 122 n = 0
2 2
⇒ d = 16 − 3d ⇒ 2 [ n 2 − 61n + 900 ] = 0
⇒ 4d = 16 ⇒ d = 4 ⇒ n 2 − 61n + 900 = 0 [divide by 2]
7d ⇒ 2
n − 36n − 25n + 900 = 0
For a = − and d = − 4, then
2
⇒ n ( n − 36) − 25( n − 36) = 0
7 × ( − 4)
a=− = 14 ⇒ ( n − 36)( n − 25) = 0 ⇒ n = 36 or 25
2
d 4 Since, a is positive and d is negative, so both values of n
For a = and d = 4, then a = = 2 are possible.
2 2
Hence, sum of 25 terms of given AP
Therefore, the four consecutive numbers in an AP are
= Sum of 36 terms of given AP = 300.
14, [14 + ( −4)] , [14 + (2 × − 4)] , [14 + ( 3 × − 4)]
56. (i) Now, she takes ` 1 on day 1, ` 2 on day 2, ` 3 on day 3
or 2 , (2 + 4), (2 + 2 × 4), (2 + 3 × 4). and so on till the end of the month, from this money.
Hence, the numbers are 14, 10, 6, 2 or 2, 6, 10, 14. i.e. 1 + 2 + 3 + 4 + ... + 31.
54. Given that, the AP is a , b , c. which form an AP in which terms are 31 and first term
Here, first term = a, common difference = b − a ( a ) = 1, common difference ( d) = 2 − 1 = 1
and last term, l = a n = c ∴ Sum of first 31 terms = S31
Q an = l = a + (n − 1) d Sum of n terms,
⇒ c = a + ( n − 1) ( b − a ) n
c−a S n = [2 a + ( n − 1 ) d ]
⇒ (n − 1) = 2
b−a 31
∴ S31 = [2 × 1 + ( 31 − 1) × 1]
c−a 2
⇒ n= +1
b−a 31 31 × 32
= (2 + 30) = = 31 × 16 = 496
c − a + b − a c + b − 2a 2 2
⇒ n= = …(i)
b−a b−a So, Kanika takes ` 496 till the end of the month from this
money.
∴ Sum of an AP,
n (ii) Let her pocket money be ` x.
Sn = [2 a + ( n − 1 ) d ] Now, she spent ` 204 of her pocket money and found that
2
at the end of the month she still has ` 100 with her.
(b + c − 2a) ⎡ ⎧b + c − 2a ⎫ ⎤
Now, according to the condition,
= ⎢2a + ⎨ − 1 ⎬ ( b − a )⎥
2 (b − a) ⎣ ⎩ b−a ⎭ ⎦ ( x − 496) − 204 = 100

(b + c − 2a) c−a ⎤ ⇒ x − 700 = 100
=
2 (b − a)⎢2 a + b − a ⋅ ( b − a )⎥ ∴ x = ` 800
⎣ ⎦
Hence, ` 800 was her pocket money for the month.
( b + c − 2a)
= (2 a + c − a ) (iii) Here, a = 1, d = 1, n = 13
2 (b − a)
Now, a n = a + ( n − 1) d
(b + c − 2a)
= ⋅ ( a + c) Hence proved. ⇒ a 13 = 1 + 12(1) = 1 + 12 = 13
2( b − a )
So, Kanika, saved ` 13 till January 13th, 2008.
Chapter Test
Multiple Choice Questions (iv) Total amount paid in 13th and 17th
installment is
1. The list of numbers − 10, − 6, − 2, 2, ... is (a) ` 380
[NCERT Exemplar] (b) ` 300
(a) an AP with d = − 16 (b) an AP with d = 4 (c) ` 360
(c) an AP with d = − 4 (d) not an AP (d) ` 340
2. In an AP, if a = 3.5, d = 0 and n = 101, then an will be (v) If he increases the installment by ` 6 every
month, then the amount he will pay in
(a) 0 (b) 3.5 53th installment is
(c) 103.5 (d) 104.5 (a) ` 314 (b) ` 360
(c) ` 412 (d) ` 416
3. Is an sequence defined by an = 2n2 + 1 forms an
Short Answer Type Questions
AP?
(a) Yes 7. Two AP’s have the same common difference.
(b) Not The first term of one AP is 2 and that of the
(c) Cannot be determined other is 7.
(d) None of the above
The difference between their 10th terms is the
4. The sum of first 20 terms of an AP in which same as the difference between their 21st
a = 1 and 20th term = 58 is terms, which is the same as the difference
(a) 590 (b) 580 between any two corresponding terms?
[NCERT Why?
Exemplar]
(c) 570 (d) 560
8. Determine the AP whose fifth term is 19 and
5. The 10th term of an AP is 52 and 16th term is 82, the difference of the eighth term from the
then 32nd term of the AP is [NCERT Exemplar] thirteenth term is 20.
(a) 152 (b) 159
(c) 162 (d) 156 9. Find the sum of all the 11 terms of an AP
whose middle most term is 30.
Case Based MCQs
Long Answer Type Questions
6. Kartik starts repaying a loan as first installment
of 10. An AP consists of 37 terms. The sum of the
` 100. He increases the installment by ` 5 every three middle most terms is 225 and the sum
month. of the last three terms is 429. Find the AP.
(i) AP formed from the given situation is 11. If sum of first 6 terms of an AP is 36 and that
(a) 105, 110, 115, ...... (b) 100, 105, 110, ...... of the first 16 terms is 256, then find the sum
(c) 95, 100, 105, ...... (d) 110, 115, 120, ...... of first 10 terms.
(ii) The amount Kartik will pay in 30th 12. Which term of the AP : 121, 117, 113, ... is its
installment is second negative term?
(a) ` 265 (b) ` 235
(c) ` 255 (d) ` 245 13. The sum of the third and the seventh terms of
(iii) If Kartik pays ` 795, then it is an AP is 6 and their product is 8. Find the sum
(a) 140th installment of first sixteen terms of the AP.
(b) 150th installment
14. Solve the equation −4 + (−1) + 2 + ..... + x = 437.
(c) 160th installment
(d) 170th installment

Answers
1. (b) 2. (b) 3. (b) 4. (c) 5. (a) 6. (i) (b) (ii) (d) (iii) (a) (iv) (d) (v) (c) For Detailed Solutions
7. (37) 8. 3, 7, 11, 15 9. 330 10. 3, 7, 11, 15 Scan the code
11. 100 12. 33rd term 13. 20 or 76 14. x = 50
48 CBSE Term II Mathematics X (Standard)

CHAPTER 03

Circles

In this Chapter...
! Circle
! Tangent of a Circle
! Theorem Related to Tangent of a Circle

A circle is a collection of all points in a plane which are at a Semi-circle


constant distance i.e. radius from a fixed point i.e. centre.
A diameter of a circle divides it into two equal parts or
In the given figure, O is the centre of circle and OA is the radius of in two equal arcs. Each of these two arcs is called a
the circle. Also, AB is the diameter of the circle. semi-circle.

Circumference
B A
O The length of the complete circle is called the
circumference of the circle.

Two or more circles having the same centre are called concentric Arc
circles. A continuous piece of a circle is called an arc. In
adjoining figure, P and Q are two points on a circle
Some Important Terms Related to Circle which divide it into two parts, called the arcs. The
Chord larger part is called the major arc QRP and the smaller
part is called the minor arc PMQ.
A line segment joining any two points on the circumference of the
R
circle is called a chord of the circle. If this chord passes through the
centre, then this chord (or diamter) is the longest chord of the circle. Major arc

O
P Q
Minor arc
A B
M
CBSE Term II Mathematics X (Standard) 49

Segment Tangent to a Circle


The region between a chord and either of its arcs is called a
A line which touches the circle at a point, is called tangent to
segment of the circular region or simply a segment of the
a circle.
circle. The segment formed by minor arc along with chord, is
called minor segment and the segment formed by major arc, is In the figure, O is the centre of circle, AB is a tangent line
called the major segment. and P is a point of contact.
!
There is only one tangent at a point of the circle.
!
A circle can have maximum two parallel tangents which
Major
can be drawn to the opposite sides of the centre.
segment

A B
O

Minor segment
A B (Tangent line)
P
Sector (Point of contact)

The region between an arc and the two radii, joining the ends
of the arc to the centre, is called a sector. Length of a Tangent
The length of the segment of the tangent, between the given
Major sector point (on the tangent) and the point of contact, is called the
O
length of tangent from the given point.

Minor
A sector B O

The sector formed by minor arc, is called minor sector and


the sector formed by major arc, is called major sector.
A B
Segment of the tangent between
Important Results Related to Circle Tangent line the given point B and point of contact A
(i) The perpendicular drawn from the centre of a circle to
a chord bisects it and vice-versa. In the above figure, AB is called the length of tangent.
(ii) Equal chords of a circle are equidistant from the centre. ∴ Length of tangent to the circle from an exterior point,
(iii) The angle subtended by an arc (or corresponding AB =
chord) at the centre of the circle is twice the angle (Distance of exterior point from centre) 2 − (Radius) 2
subtended by the same arc at any point on the
remaining part of the circle. Number of Tangent from a Point on a Circle
C (i) If point P lies outside the circle, then two tangents can
θ be drawn to the circle, i.e. PT1 and PT2 .
ent)
O ang
T1 (T

A B P

(iv) Equal chords of a circle subtend equal angles at T2 (T


ang
the centre. ent)

(v) The angle in a semi-circle is a right angle.


(ii) If point P lies on the circle, then there is one and only
(vi) Angles in the same segment of a circle are equal. one tangent to a circle passing through point P.
(vii) The sum of any pair of opposite angles of a cyclic
quadrilateral is 180°.
(viii) If two circles intersect at two points, then the line
through the centres is the perpendicular bisector of the
common chord.
P Tangent
50 CBSE Term II Mathematics X (Standard)

(iii) If a point P lies inside the circle, then there is no Important Results Related to Tangent to a Circle
tangent to a circle passing through a point lying
(i) If two circles touch internally or externally, then point of
inside the circle.
contact lies on the straight line through the two centres.

No Tangent

P O Oʹ
O Oʹ P
P

(ii) A pair of tangents drawn at two points of a circle are either


parallel or they intersect each other at a point outside the
Theorems Related to Tangent of Circle circle.
Theorem 1 The tangent at any point of a circle is (iii) If two tangents drawn to a circle are parallel to each other,
perpendicular to the radius through the point of contact. then the line segment joining their point of contact is a
diameter of the circle.
O
(iv) If two tangents are drawn to a circle from an external point,
then
(a) They subtend equal angles at the centre,
A B i.e. ∠POA = ∠POB.
P
A
Here, O is centre of circle and AB is tangent of circle at P
and it is point of contact and OP is radius. P O
∴ OP ⊥ AB.
Theorem 2 A perpendicular drawn from the end point of B
radius is tangent to the circle. If OP ⊥ AB, then AB is (b) They are equally inclined to the segment joining
tangent to circle. the centre to that point,
i.e. ∠APQ = ∠BPQ.
O

A B
P

Theorem 3 The lengths of two tangents drawn from an


external point to a circle are equal.
A (v) The opposite sides of a quadrilateral circumscribing a circle
subtend supplementary angles at the centre of the circle.
O P D C

B
O
Here, P is exterior point and PA and PB are tangents.
A B
∴ PA = PB
CBSE Term II Mathematics X (Standard) 51

Solved Examples
Example 1. Prove that a tangent to a circle is From Eqs. (i) and (ii), we get
perpendicular to the radius through the point of TP = TQ
contact. [CBSE 2020 (Standard)] Hence, T is the mid-point of the line segment PQ.
Sol. Given A circle with centre O and a tangent AB at a point
P on the circle.
Example 3. In figure, PQ is tangent to the circle with
To prove OP ⊥ AB centre O, at the point B. If ∠AOB = 100°, then find
Construction Take any point Q, other than P on the tangent ∠ABP. [CBSE 2020 (Standard)]
AB and join OQ.

O
A 10

O
Q
B
R
P
A P Q B
Sol. Given, ∠AOB = 100° ,
Proof Here, Q is a point on the tangent AB, other than the
point of contact P. So, Q lies outside the circle (if Q lies
inside the circle, then AB becomes a secant and not a A
tangent to the circle). 100°
O
Let OQ intersects the circle at R .
Then, OP = OR [radii of the circle] B
Now, OQ = OR + RQ P Q
⇒ OQ > OR
In ΔOAB,
⇒ OQ > OP or OP < OQ [Q OP = OR ]
OA = OB [radii of the circle]
Thus, OP is shorter than any other segment joining O to any
point of AB. Also, we know that the shortest distance ⇒ ∠OBA = ∠OAB
between a point and a line is perpendicular distance from [angles opposite to equal sides are equal] …(i)
the point to the line. In ΔOAB,
So, OP is perpendicular to AB. ∠AOB + ∠OAB + ∠OBA = 180°
i.e. OP ⊥ AB Hence proved. [by angle sum property of triangle]
⇒ 100° + ∠OBA + ∠OBA = 180° [from Eq. (i)]
Example 2. In given figure, two circles touch each ⇒ 2 ∠OBA = 180° − 100°
other at the point C. Prove that the common 80°
tangent to the circles at C, bisects the common ⇒ ∠OBA =
2
tangent at P and Q.
⇒ ∠OBA = 40° …(ii)
P
T Q We know that, radius of circle is perpendicular to the
tangent.
A C B ∴ ∠OBP = 90° ⇒ ∠OBA + ∠ABP = 90°
⇒ 40° + ∠ABP = 90°
⇒ ∠ABP = 90° − 40° ⇒ ∠ABP = 50°
Sol. We know that, tangents drawn from an external point are of Example 4. In below figure, PA is a tangent from
equal length. Therefore, according to the given figure,
an external point P to a circle with centre O.
TP = TC …(i) [Q point T is external]
If ∠POB = 115°, find ∠APO. [CBSE 2020 (Standard)]
and TQ = TC …(ii) [Q point T is external]
A
P
T Q
P O
A B 115°
C
B
52 CBSE Term II Mathematics X (Standard)

Sol. Given, ∠POB = 115° ⇒ AP 2 = OA 2 − OQ2


A [Q OP = OQ = radii of a circle] …(ii)
P Now, in right angled ΔOQA,
O OA 2 = OQ2 + AQ2
115°
⇒ AQ2 = OA 2 − OQ2 ...(iii)
B
From Eqs. (ii) and (iii), we get
Since, AB is a straight line. AP 2 = AQ2
∴ ∠POB + ∠AOP = 180°
⇒ AP = AQ Hence proved.
⇒ 115° + ∠AOP = 180°
⇒ ∠AOP = 180° − 115° Example 6. Prove that the angle between the two
⇒ ∠AOP = 65° tangents drawn from an external point to a circle is
We know that, radius line is perpendicular to the tangent. supplementary to the angle subtended by the line
∴ ∠PAO = 90° segment joining the points of contact at the centre.
In ΔAOP, Sol. Let PQ and PR be two tangents drawn from an external point
∠PAO + ∠AOP + ∠APO = 180° P to a circle with centre O.
[Q sum of all angles of a triangle is 180°] Q
∴ 90° + 65° + ∠APO = 180°
⇒ ∠APO = 180° − ( 65° + 90° )
P O
= 180° − 155° = 25°
Example 5. Prove that the length of tangents drawn
from an external point to a circle are equal. R
[CBSE 2020 (Standard)] To prove ∠QOR = 180° − ∠QPR
Sol. Let AP and AQ are two tangents drawn from a point A to a
or ∠QOR + ∠QPR = 180°
circle with centre O.
Proof In Δ OQP and ΔORP,
P
PQ = PR [Q tangents drawn from an
external point are equal in length]
A O OQ = OR [radii of circle]
OP = OP [common sides]
Q
∴ ΔOQP ≅ ΔORP [by SSS congruence rule]
Then, ∠ QPO = ∠ RPO [by CPCT]
To prove AP = AQ and ∠ POQ = ∠ POR [by CPCT]
Construction Join OP, OQ and OA. ⇒ ∠ QPR = 2 ∠ OPQ⎫
⎬ …(i)
and ∠ QOR = 2 ∠ POQ ⎭
Proof We know that, a tangent at any point of a circle is
perpendicular to the radius through the point of contact. Now, in right angled ΔOQP,
Here, AP is a tangent and OP is the radius of the circle ∠ QPO + ∠ QOP = 90°
through P. ⇒ ∠ QOP = 90° − ∠ QPO
∴ OP ⊥ AP ⇒ 2 ∠QOP = 180° − 2 ∠QPO
Similarly, OQ ⊥ AQ [multiplying both sides by 2]
⇒ ∠OPA = ∠OQA = 90° ...(i) ⇒ ∠ QOR = 180° − ∠ QPR [from Eq. (i)]
First Method ⇒ ∠ QOR + ∠ QPR = 180° Hence proved.
In ΔOPA and ΔOQA, we have
OP = OQ [radii of a circle]
Example 7. In figure, find the perimeter of ΔABC,
∠OPA = ∠OQA = 90° [from Eq. (i)]
if AP = 12 cm.
OA = OA [common sides] A

So, ΔOPA ≅ ΔOQA [by RHS congruence rule]


∴ AP = AQ [by CPCT] D
B C
Second Method
In right angled ΔOPA, P Q
OA 2 = OP 2 + AP 2
[by Pythagoras theorem]
⇒ AP 2 = OA 2 − OP 2
CBSE Term II Mathematics X (Standard) 53

Sol. Given, AP = 12 cm ∴ AC = AR + RC
⇒ AQ = AP = 12 cm ⇒ 11 = 4 + RC
A ⇒ RC = 11 − 4 = 7 cm
Now, BC = BQ + QC
= 3 + 7 = 10 cm
D
B C Hence, length of BC is 10 cm.
P Q Example 9. In the given figure, from an external point
P, two tangents PQ and PR are drawn to a circle of
radius 4 cm with centre O. If ∠QPR = 90°, then
find the length of PQ.
[Q tangents drawn from an external point Q
are equal in lengths]

4 cm
P
Also, BD = BP
[Q B is an external point] …(i) O R
and CD = CQ
[Q C is an external point] …(ii)
Now, AP = AB + BP
⇒ 12 = AB + BD [from Eq. (i)] …(iii) Sol. We know that, if pair of tangents are drawn from an external
point P, then line joining from centre O to the point P,
and AQ = AC + CQ
bisects the angle P.
⇒ 12 = AC + CD [from Eq. (i)] …(iv)
Perimeter of ΔABC Q
= AB + BC + AC 4 cm
= AB + BD + DC + AC 45°
P
= 12 + 12 O
90°
[from Eqs. (iii) and (iv)]
= 24 cm R
Hence, perimeter of a ΔABC is 24 cm.
∠QPR
∴ ∠OPQ =
Example 8. In below figure, ΔABC is circumscribing a 2
circle, the length of BC is …… cm. 90°
= = 45°
[CBSE 2020 (Standard)] 2
A
Also, radius of circle OQ is perpendicular to the tangent line
QP.
m
4c

Now, in right angled ΔOQP,


11

OQ
tan 45° =
cm

P R QP
m

4
3c

⇒ 1=
QP
B Q C
⇒ QP = 4 cm
Sol. We know that, the tangents drawn from an external point to Hence, length of PQ is 4 cm.
a circle are equal. Therefore,
A Example 10. In the given figure, if tangents PA and PB
from an external point P to a circle with centre O,
m

are inclined to each other at an angle of 80°, then


4c
4c

find ∠AOB.
m

[CBSE 2020 (Standard)]


P R A
m

7c

P
80º
3c

B 3 cm Q 7 cm C O

BP = BQ , [point B is an external] B
AP = AR [point A is an external]
and CQ = CR [point C is an external]
54 CBSE Term II Mathematics X (Standard)

Sol. Given, ∠APB = 80°. Example 12. In given figure, two tangents TP and TQ
We know that, line drawn from centre of a circle to the are drawn to a circle with centre O from an external
tangent is perpendicular. point T. Prove that ∠PTQ = 2 ∠OPQ.
Since, OA⊥PA and OB⊥PB. [CBSE 2020 (Standard)]
Then, ∠OAP = ∠OBP = 90° …(i) P
A
P T
80º O
O
Q
B
Sol. Given, TP and TQ are two tangents of a circle with centre O
and points P and Q are point of contact.
To prove ∠ PTQ = 2 ∠OPQ
Since, OAPB is a quadrilateral. Proof Let ∠PTQ = θ
By using angle sum property of a quadrilateral,
As we know that, the length of tangents drawn from an
∠AOB + ∠OBP + ∠APB + ∠OAP = 360° external point to a circle are equal.
⇒ ∠AOB + 90° + 80° + 90° = 360° So, ΔTPQ is an isosceles triangle.
⇒ ∠AOB = 360° − 260° Therefore, according to the given figure,
⇒ ∠AOB = 100° 1 θ
∠TPQ = ∠TQP = (180° − θ ) = 90°−
2 2
Example 11. In given figure, PA and PB are tangents to
As we know that, the tangents at any point of a circle is
the circle with centre O, such that ∠APB = 50°,
perpendicular to the radius through the point of contact.
then the measure of ∠OAB is ...... P
A

T !
P 50° O
O

Q
B
∴ ∠OPT = 90°
Sol. Given, ∠APB = 50° Now, ∠OPQ = ∠OPT − ∠TPQ
A θ ⎞ θ ∠PTQ

= 90° − ⎜ 90° − ⎟ = =
⎝ 2⎠ 2 2
P 50° O ⇒ ∠PTQ = 2 ∠OPQ Hence proved.

B
Example 13. In figure, a quadrilateral ABCD is drawn
to circumscribe a circle. Prove that
Since, P is an external point of a circle.
AB + CD = BC + AD
Therefore, PA = PB [CBSE 2020 (Standard)]
[Q tangents drawn from an external to a circle are equal] A
⇒ ∠PBA = ∠PAB B
[Q angles opposite to equal sides are equal] ...(i)
In ΔAPB,
∠APB + ∠PBA + ∠PAB = 180°
D
[Q sum of all angles of a triangle is 180°] C
∴ 50° + 2 ∠PAB = 180° [from Eq. (i)] Sol. Given A quadrilateral ABCD is circumscribing a circle.
⇒ 2 ∠PAB = 130° To prove AB + CD = AD + BC
⇒ ∠PAB = 65° ...(ii) Proof Let P , Q, R and S be the point of contact.
Also, radius OA is perpendicular to the tangent of a circle. A P
Therefore, B
∠OAP = 90° S
⇒ ∠OAB + ∠PAB = 90° Q
⇒ ∠OAB + 65° = 90° [from Eq. (ii)] D
⇒ ∠OAB = 90° − 65° = 25° R C
CBSE Term II Mathematics X (Standard) 55

We know that, the length of tangents drawn from an external Now, ∠OPQ = 90° − 60° = 30°
point to a circle are equal. [Q ∠OPT = 90°, as radius line OP is perpendicular
∴ AP = AS to the tangent]
[Q both are tangents to a circle from point A] …(i) ⇒ ∠OQP = 30°
Similarly, BP = BQ, …(ii) [angles opposite to equal sides are equal]
CR = CQ …(iii) In ΔOPQ, using angle sum property of a triangle,
and DR = DS …(iv) ∠POQ + ∠OPQ + ∠OQP = 180°
Adding Eqs. (i), (ii), (iii) and (iv), we get ⇒ ∠POQ + 30° + 30° = 180°
( AP + BP ) + ( CR + DR ) = ( AS + BQ) + ( CQ + DS) ⇒ ∠POQ = 180° − 60° = 120°
⇒ AB + CD = ( AS + DS) + (BQ + CQ)
⇒ ∠PQʹ Q = 60°
⇒ AB + CD = AD + BC Hence proved.
[angle subtended by an arc at centre is twice the angle
Example 14. Prove that the tangents at the extremities subtended at remaining part of circle]
of any chord of a circle make equal angles with the ⇒ ∠PRQ = 180°−∠PQʹ Q = 120°
chord. [CBSE 2020 (Standard)] [Q opposite angles are supplementary in a cyclic
quadrilateral PQʹ QR ]
Sol. Let AB be a chord of a circle having centre O. Let AP and
BP be the tangents at A and B, which intersect at point P. Example 16. In given figure, AB is a chord of circle
To prove ∠PAC = ∠PBC with centre O, AOC is diameter and AT is tangent
Construction Join points C and P. at A. Prove that ∠BAT = ∠ACB.
Proof We know that, tangents drawn from an external point C
are equal.
A
O B

O C
P
A T
B Sol. Given AB is a chord of a circle, AOC is a diameter of the
∴ In ΔPCA and ΔPCB, circle having centre O and line AT is tangent at A.
PA = PB, [QP is an external point of a circle] To prove ∠BAT = ∠ACB
∠APC = ∠BPC C
[Q AP and BP are equally inclined to OP]
and PC = PC [common sides] B
O
∴ ΔPAC ~ ΔPBC [by SAS similarity rule]
⇒ ∠PAC = ∠PBC [by CPCT]
A T
Example 15. In given figure, PQ is a chord of a circle
and PT is tangent at P such that ∠QPT = 60°, then Proof We know that, diameter of a circle subtends 90° to the
the measure of ∠PRQ is ....... [CBSE 2020 (Standard)] semi-circle.
∴ ∠ABC = 90°
Q Let ∠ACB = θ,
O then ∠CAB = 180° − ( 90° + θ)
60° [by using angle sum property of triangle]
R
P ⇒ ∠CAB = 90° − θ ...(i)
T We know that, radius of a circle is perpendicular to the
Sol. Take a point Qʹ on circle and join PQʹ and QQʹ. tangent.
∴ ∠OAT = 90°
⇒ ∠OAB + ∠BAT = 90°

Q ⇒ ∠CAB + ∠BAT = 90° [Q ∠OAT = ∠CAT]
O ⇒ 90° − θ + ∠BAT = 90°
⇒ ∠BAT = θ
º R
60 ⇒ ∠BAT = ∠ACB Hence proved.
A P T
56 CBSE Term II Mathematics X (Standard)

Chapter
Practice
PART 1 A

Objective Questions O
115°
!
Multiple Choice Questions B
P
1. If radii of two concentric circles are 4 cm and 5 cm,
then length of each chord of one circle, which is (a) 25° (b) 20° (c) 30° (d) 65°
tangent to the other circle, is [NCERT Exemplar] 8. In figure, AT is a tangent to the circle with centre O
(a) 3 cm (b) 6 cm (c) 9 cm (d) 1 cm such that OT = 4 cm and ∠ OTA = 30° . Then, AT is
2. The length of tangent from an external point P on a equal to [NCERT Exemplar]
circle with centre O is always less than OP.
[NCERT Exemplar]
(a) True (b) False O
(c) Can’t determined (d) None of these
4 cm
3. The length of the tangents to the circle from a point
30°
at any distance of 5 cm from centre of the circle of A T
radius 3 cm is
(a) 2 cm (b) 4 cm (a) 4 cm (b) 2 cm (c) 2 3 cm (d) 4 3 cm
(c) 8 cm (d) None of these 9. PQ is a tangent drawn from a point P to a circle
4. The length of the tangent drawn from a point 8 cm with centre O and QOR is a diameter of the circle
away from the centre of circle of radius 6 cm is such that ∠POR = 135°, then ∠OPQ is
(a) 7 cm (b) 2 7 cm (c) 10 cm (d) 5 cm (a) 60° (b) 45° (c) 30° (d) 90°
5. PQ is a tangent to a circle with centre O at the 10. A tangent PQ at a point P of a circle of radius 6 cm
point P. If ΔOPQ is an isosceles triangle, then meets a line through the centre O at a point Q, so
∠OPQ is equal to that OQ = 14 cm, then length of PQ is
(a) 30° (b) 45° (c) 60° (d) 90°
(a) 4 10 cm (b) 6 10 cm
6. In figure, if O is the centre of a circle, PQ is a chord (c) 5 10 cm (d) 7 10 cm
and the tangent PR at P makes an angle of 50° with
PQ, then ∠POQ is equal to [NCERT Exemplar] 11. In figure, if PA and PB are tangents to the circle
P R
with centre O such that ∠ APB = 50°, then ∠OAB is
50° equal to [NCERT Exemplar]
A

Q P 50° O

(a) 100° (b) 80° (c) 90° (d) 75°


7. In the given figure, PA is a tangent from an external B
point P to a circle with centre O. If ∠POB = 115°, (a) 25° (b) 30°
then ∠APO is (c) 40° (d) 50°
CBSE Term II Mathematics X (Standard) 57

12. If angle between two tangents drawn from a point P (a) 41 cm (b) 41 cm
to a circle of radius a and centre O is 90°, then (c) 40 cm (d) 40 cm
OP = a 2. [NCERT Exemplar] 18. PA is a tangent to the circle with centre O.
(a) True (b) False If BC = 3 cm, AC = 4 cm and ΔACB ~ΔPAO, then
(c) Can’t say (d) Partially true or false OA is equal to [CBSE 2013]
B
13. In the given figure, find the value of x°.
A C

O

30°
B
O

A P

(a) 130° (b) 75° (c) 120° (d) 60° (a) 2.7 cm (b) 5 cm
5
14. From the given figure, find the value of x ° + y°. (c) 5 cm (d) cm
2
P
19. In the given figure, if ∠ACB = 50°, then ∠ATO is
A
x° y°

O Q R
C
O T

(a) 270° (b) 180°


(c) 90° (d) None of these B

15. At one end A of a diameter AB of a circle of radius (a) 30°


5 cm, tangent XAY is drawn to the circle. The (b) 50°
length of the chord CD parallel to XY and at a (c) 40°
(d) Can’t be determined
distance 8 cm from A is [NCERT Exemplar]
(a) 4 cm (b) 5 cm (c) 6 cm (d) 8 cm 20. In the adjoining figure, PQ is a chord of a circle
16. In figure, AB is a chord of the circle and AOC is its with centre O and PT is a tangent at P such that
diameter such that ∠ ACB = 50°. If AT is the tangent ∠QPT = 60°, then ∠PRQ =
to the circle at the point A, then ∠BAT is equal to
[NCERT Exemplar] Q
C
O

R
O B 60°
A P T
(a) 120° (b) 160° (c) 130° (d) 150°
A T 21. In figure, if PQR is the tangent to a circle at Q,
(a) 45° (b) 60° (c) 50° (d) 55° whose centre is O, AB is a chord parallel to PR and
17. In the adjoining figure, AD = 8 cm, AC = 6 cm and ∠BQR = 70°, then ∠AQB is equal to
TB is the tangent at B to the circle with centre O. [NCERT Exemplar]
If BT is 4 cm, then OT = [CBSE 2013] A D B
A

C D
O
70°
P Q R

B 4 cm T (a) 20° (b) 40° (c) 35° (d) 45°


58 CBSE Term II Mathematics X (Standard)

22. From an external point P, tangents PA and PB are 28. In figure, if ∠AOB = 125°, then ∠COD is equal to
drawn to a circle with centre O. If CD is the [NCERT Exemplar]
tangent to the circle at a point E and PA = 14 cm, A
then perimeter of ΔPCD is
B
(a) 14 cm (b) 21 cm (c) 28 cm (d) 35 cm
125°
23. Tangents AP and AQ are drawn to circle with
O
centre O from an external point A, then ∠PAQ is
equal to
∠OPQ ∠OPQ ∠OPQ C
(a) 2∠OPQ (b) (c) (d) D
2 3 4
(a) 62.5° (b) 45°
24. In the given figure, two tangents AB andAC are (c) 35° (d) 55°
drawn to a circle with centre O such that
∠BAC = 120°, then OA is equal to !
Case Based MCQs
O
29. A playground is in the shape of a triangle with right
angle at B , AB = 3 m and BC = 4 m. A pit was dig
inside it such that it touches the walls AC , BC and
B C AB at P , Q and R, respectively such that AP = x m.
120°
A
(a) 2AB (b) 3AB (c) 4AB (d) 5AB A
25. In the given figure, O is the centre of a circle, BOA
is its diameter and the tangent at the point P meets
BA extended at T. If ∠PBO = 30°, then ∠PTA = P
r
[CBSE 2016]
P R
O

30° B C
B O T Q
A

(a) 40° (b) 50° (c) 30° (d) 20° Based on the above information, answer the
26. In adjoining figure, PQ and PR are tangents to the following questions.
circle with centre O and S is a point on the circle (i) The value of AR =
such that ∠SQL = 50° and ∠SRM = 60°. Then, (a) 2x m (b) x / 2 m
∠QSR [NCERT Exemplar] (c) x m (d) 3x m
L
(ii) The value of BQ =
50° Q
(a) 2x m (b) ( 3 − x ) m
(c) (2 − x ) m (d) 4x m
S O P
(iii) The value of CQ =
60° R
(a) ( 4 + x ) m (b) ( 5 − x ) m
M (c) (1 + x ) m (d) Both (b) and (c)
(a) 40° (b) 50° (c) 60° (d) 70° (iv) Which of the following is correct?
(a) Quadrilateral AROP is a square
27. In given figure, AB is diameter of a circle with
(b) Quadrilateral BROQ is a square
centre O and AT is tangent. If ∠AOQ = 58 ° , then
(c) Quadrilateral CQOP is a square
∠ATQ = [CBSE 2015]
B
(d) None of the above
(v) Radius of the pit is
(a) 1 m (b) 3 m
O
Q (c) 4 m (d) 5 m
58°
30. A student draws two circles that touch each other
A T externally at point K with centres A and B and radii
(a) 52° (b) 58° (c) 61° (d) 62° 6 cm and 4 cm, respectively as shown in the figure.
CBSE Term II Mathematics X (Standard) 59

T 3c 5. If a number of circles touch a given line segment


m
PQ at a point A, then their centres lie on the
X K
P Q perpendicular bisector of PQ. Why or why not?
6 cm A 4 cm B Y [NCERT Exemplar]
8
cm

6. Out of the two concentric circles, the radius of


S the outer circle is 5 cm and the chord AC of
Based on the above information, answer the following length 8 cm is a tangent to the inner circle. Find
questions. the radius of the inner circle.
(i) The value of PA = 7. If a chord AB subtends an angle of 60° at the
(a) 10 cm (b) 5 cm centre of a circle, then find the angle between the
(c) 13 cm (d) Can’t be determined tangents at A and B. [NCERT Exemplar]
(ii) The value of BQ = 8. From an external point P, two tangents, PA and
(a) 4 cm (b) 5 cm PB are drawn to a circle with centre O. At one
(c) 6 cm (d) 18 cm point E on the circle tangent is drawn, which
(iii) The value of PK = intersects PA and PB at C and D, respectively.
(a) 13 cm (b) 15 cm If PA = 10 cm, find the perimeter of the trianlge
(c) 16 cm (d) 18 cm PCD.
(iv) The value of QY = 9. Prove that the centre of a circle touching two
(a) 2 cm (b) 5 cm intersecting lines lies on the angle bisector of the
(c) 1 cm (d) 3 cm lines. [NCERT Exemplar]
(v) If two circles touch externally, then the number of 10. If from an external point B of a circle with centre
common tangents can be drawn is O, two tangents BC and BD are drawn, such that
(a) 1 (b) 2 ∠ DBC = 120°, prove that BC + BD = BO i.e.
(c) 3 (d) None of these BO = 2 BC.
11. In figure, AB and CD are common tangents to
PART 2 two circles of equal radii. Prove that AB = CD.
[NCERT Exemplar]
Subjective Questions A B

!
Short Answer Type Questions
1. If PQ is a tangent to a circle with centre O and radius
6 cm such that ∠PQO = 60°, then find the length of a C D
tangent PQ and a line OQ.
12. In figure, common tangents AB and CD to two
2. The tangent to the circumcircle of an isosceles ΔABC circles intersect at E. Prove that AB = CD.
at A, in which AB = AC, is parallel to BC. [NCERT Exemplar]
[NCERT Exemplar] A
D
3. If AB is a chord of a circle with centre O, AOC is a
diameter and AT is the tangent at A as shown in
figure. Prove that ∠ BAT = ∠ ACB. [NCERT Exemplar] E
C
B
C

O B
13. If PA and PB are two tangents drawn from a point
P to a circle with centre O touching it at A and B,
prove that OP is perpendicular bisector of AB.
[CBSE 2008]
A T
14. Tangents AP and AQ are drawn to circle with
4. Prove that a diameter AB of a circle bisects all
centre O from an external point A. Prove that
those chords, which are parallel to the tangent at
∠PAQ = 2 ∠OPQ. [CBSE 2013, 12,11, 09]
the point A. [NCERT Exemplar]
60 CBSE Term II Mathematics X (Standard)

15. In the given figure, ∠ADC = 90°, BC = 38 cm, 21. If a hexagon ABCDEF circumscribe a circle,
CD = 28 cm and BP = 25 cm, then find the radius of prove that
the circle. [CBSE 2011] AB + CD + EF = BC + DE + FA [NCERT Exemplar]
B 22. In the given figure, AD is a diameter of a circle with
Q
centre O and AB is a tangent at A. C is a point on the
C
circle such that DC produced intersects the
O
tangent at B and ∠ABD = 50°. Find ∠COA. [CBSE 2015]
R
P D

D A
S O C

16. ΔABC is a right angled triangle with ∠B = 90°,


50°
BC = 3 cm and AB = 4 cm. A circle with centre O B
A
and radius r cm has been inscribed in ΔABC. Find
the radius of the incircle. 23. Tangents PQ and PR are drawn to a circle such that
17. The radii of two concentric circles are 13 cm and ∠RPQ = 30°. A chord RS is drawn parallel to the
8 cm. AB is a diameter of the bigger circle. BD is a tangent PQ. Find ∠RQS. [CBSE 2015]
tangent to the smaller circle touching it at D.
Find the length of AD. [CBSE 2010] 24. PA and PB are the tangents to a circle, which
circumscribes an equilateral ΔABQ. If ∠PAB = 60°,
18. A circle is inscribed in a ΔABC having sides as shown in the figure, prove that QP bisects AB at
AB = 8 cm, BC = 10 cm and CA = 12 cm, as shown in right angle. [CBSE 2015]
figure. Find AD, BE and CF. [CBSE 2012]
A
A

60°
Q P
F D M

B
C B
E 25. Two circles with centres O and Oʹ of radii 3 cm and
4 cm, respectively intersect at two points P and Q,
!
Long Answer Type Questions such that OP and O ʹ P are tangents to the two
circles. Find the length of the common chord PQ.
19. Let s denotes the semi-perimeter of a ΔABC, in [NCERT Exemplar]
which BC = a, CA = b and AB = c. If a circle touches
26. If an isosceles ΔABC in which AB = AC = 6 cm, is
the sides BC , CA , AB at D , E, F , respectively. Prove
that BD = s − b. [NCERT Exemplar] inscribed in a circle of radius 9 cm, find the area of
the triangle.
20. AC and AD are tangents at C and D, respectively.
If ∠BCD = 44° , then find∠CAD , ∠ADC , ∠CBD 27. In a figure, the common tangents AB and CD of
and ∠ACD. two circles with centres O and Oʹ intersect at E.
C Prove that the points O , E and O ʹ are collinear.
A [NCERT Exemplar]

44° A
D
O
D O Oʹ
E

C B
B
CBSE Term II Mathematics X (Standard) 61

28. In figure, O is the centre of a circle of radius 5 cm, (ii) A circle of radius 3 cm is inscribed in a right
T is a point such that OT = 13 and OT intersects the angled ΔBAC such that BD = 9 cm and DC = 3 cm.
circle at E, if AB is the tangent to the circle at E, Find the length of AB.
find the length of AB. [NCERT Exemplar] A

P
A

F
O T
E
R O E
B
Q
B D C
(a) 6 cm (b) 12 cm
!
Case Based Questions (c) 15 cm (d) 10 cm
29. Dheeraj loves geometry. So, he was curious to know (iii) In the given figure, what is the length of CD?
more about the concepts of circles. His grand father A 4 cm 2 cm
is a mathematicians. So, he reached to his grand B
P
father to learn something interesting about tangents
and circles. His grand father gave him knowledge 5 cm
10 cm
on circles and tangents and ask him to solve the S Q
following questions. R
C
D
(a) 11 cm (b) 9 cm
(c) 7 cm (d) 13 cm
(iv) If PA and PB are two tangents to a circle with
centre O from an external point P such that
∠OPB = 50 °, then find ∠BPA
(a) 60° (b) 50°
(c) 120° (d) 100°
(v) In the given figure, P is an external point from,
(i) In the given figure, AP , AQ and BC are tangents to which tangents are drawn to two externally
the circle such that AB = 7 cm, BC = 4 cm and touching circles. If PA = 11 cm, then find PC.
AC = 9 cm. Find AP P
A

D
B C
A
P Q B C

(a) 3.5 cm (b) 4 cm


(a) 12 cm (b) 15 cm (c) 13 cm (d) 10 cm (c) 11 cm (d) Can’t be determined
62 CBSE Term II Mathematics X (Standard)

SOLUTIONS
Objective Questions 4. (b) Since, tangent to a circle is perpendicular to the radius
1. (b) Let O be the centre of two concentric circles C1 and C 2, through the point of contact.
whose radii are r1 = 4 cm and r2 = 5 cm . Now, we draw a ∴ ∠OTP = 90°
chord AC of circle C 2, which touches the circle C1 at B.
Also, join OB, which is perpendicular to AC. T
[Q tangent at any point of circle is perpendicular to radius
6 cm
through the point of contact]
P 8 cm O
C2

C1
O
In ΔOTP, we have
A C OP 2 = OT 2 + PT 2
B
⇒ ( 8)2 = ( 6)2 + PT 2
⇒ PT 2 = 64 − 36 = 28
Now, in right angled ΔOBC, by using Pythagoras theorem,
⇒ PT = 28 = 2 7 cm
OC 2 = BC 2 + BO 2
5. (b) Since, PQ is a tangent to a circle from a point P and
[Q(hypotenuse) 2 = (base) 2 + (perpendicular) 2]
centre of circle is O.
⇒ 5 2 = BC 2 + 4 2 ∴ ΔOPQ is an isosceles triangle.
⇒ BC 2 = 25 − 16 = 9 Q
⇒ BC = 3 cm
∴ Length of chord AC = 2 BC
P O
= 2 × 3 = 6 cm
2. (a)

∠OQP = 90°
O
OP = QP
P ∴ ∠POQ = ∠OPQ
In ΔOPQ,
T ∠POQ + ∠OQP + ∠OPQ = 180°
PT is a tangent drawn from external point P. Join OT. ⇒ 2 ∠OPQ = 180° − 90° [Q ∠POQ = ∠OPQ]
Q OT ⊥ PT ⇒ 2 ∠OPQ = 90° ⇒ ∠OPQ = 45°
So, ΔOPT is a right angled triangle formed. 6. (a) Given, ∠ QPR = 50°
In right angled triangle, hypotenuse is always greater than We know that, the tangent at any point of a circle is
any of the two sides of the triangle. perpendicular to the radius through the point of contact.
∴ OP > PT ∴ ∠OPR = 90°
or PT < OP ⇒ ∠OPQ + ∠QPR = 90° [from figure]
3. (b) Given, OB = 5 cm and radius OA = 3 cm ⇒ ∠OPQ = 90° − 50° = 40° [Q ∠QPR = 50°]
By Pythagoras theorem, in right angled ΔOAB, Now, OP = OQ = Radius of circle
∴ ∠OQP = ∠ OPQ = 40°
O [since, angles opposite to equal sides are equal]
In Δ OPQ, ∠ O + ∠ P + ∠ Q = 180°
5 3
[since, sum of all angles of a triangle = 180°]
B
A ⇒ ∠ O = 180° − ( 40° + 40° ) [Q ∠P = 40° = ∠Q]
= 180° − 80° = 100°
OB 2 = OA 2 + AB 2
7. (a) Here, ∠OAP = 90° [Q tangent at any point of a circle is
AB 2 = ( 5)2 − ( 3)2 [Q OB = 5, AB = 3]
perpendicular to the radius]
AB 2 = 25 − 9 Now, ∠AOP + ∠BOP = 180°
AB 2 = 16 ⇒ ∠AOP + 115° = 180°
AB = 4 cm ⇒ ∠AOP = (180° − 115° ) = 65°
CBSE Term II Mathematics X (Standard) 63

And also, ∠OAP + ∠AOP + ∠APO = 180° ⇒ PQ2 = 160


[angle sum property of triangle]
⇒ PQ = 16 × 10 = 4 10 cm
⇒ 90°+ 65°+∠APO = 180°
⇒ 155° + ∠APO = 180° 11. (a) Given, PA and PB are tangent lines.
⇒ ∠APO = 180°−155°= 25° ∴ PA = PB
8. (c) Join OA. [since, the length of tangents drawn from an
external point to a circle is equal]
⇒ ∠PBA = ∠ PAB = θ [say]
In ΔPAB, ∠ P + ∠ A + ∠ B = 180°
O
[since, sum of all angles of a triangle = 180°]
4 cm ⇒ 50° + θ + θ = 180°
30° ⇒ 2θ = 180° − 50° = 130°
A T
⇒ θ = 65°
We know that, the tangent at any point of a circle is Also, OA ⊥ PA
perpendicular to the radius through the point of contact. [since, tangent at any point of a circle is perpendicular
∴ ∠OAT = 90° to the radius through the point of contact]
AT ∴ ∠ PAO = 90°
In Δ OAT, cos 30° =
OT ⇒ ∠ PAB + ∠ BAO = 90°
3 AT ⇒ 65° + ∠ BAO = 90°
⇒ =
2 4 ⇒ ∠ BAO = 90° − 65° = 25°
⇒ AT = 2 3 cm 12. (a) From point P, two tangents are drawn.
9. (b) Given, PQ is a tangent from point P, centre O and QOR Given, OT = a
as diameter. Also, line OP bisects the ∠RPT.
R ∴ ∠ TPO = ∠ RPO = 45°
Also, OT ⊥ PT
T
135°
O
a
45°
P 90° O
P
Q

In ΔPQO, ∠ROP = ∠OPQ + 90°


[Q exterior angle of a triangle is equal to the sum R
of opposite angles] In right angled Δ OTP,
∠OPQ = 135° − 90° OT
sin 45° =
= 45° OP
10. (a) Here, OP = 6 cm and OQ = 14 cm 1 a
⇒ =
2 OP
⇒ OP = a 2
O
13. (c) Given, ∠OBA = 30°
6 cm 14
cm In ΔABO,
x ° = ∠ABO + 90°
P Q
[Q external angle = sum of opposite internal angles]
We know that, tangent at any point of a circle is x° = 30° + 90° = 120°
perpendicular to the radius through the point of contact.
14. (a) In ΔPOQ,
So, OP ⊥ PQ
x ° = ∠PQO + 90°
Now, in right angled ΔOPQ,
[Q external angle = sum of opposite internal angles]
OQ2 = OP 2 + PQ2 [by Pythagoras theorem]
= (180° − y ° ) + 90°
⇒ (14)2 = ( 6)2 + PQ2 = 270° − y °
⇒ PQ2 = 196 − 36 x ° + y ° = 270°
64 CBSE Term II Mathematics X (Standard)

15. (d) First, draw a circle of radius 5 cm having centre O. 18. (d) In ΔACB,
A tangent XY is drawn at point A. ∠BCA = 90° [angle in a semi-circle]
C ∴ AB 2 = AC 2 + BC 2 [by Pythagoras theorem]
B ⇒ AB 2 = 42 + 32

5 cm
X E ⇒ AB 2 = 16 + 9 cm
cm ⇒ AB 2 = 25 cm
cm O 3 D
5 ⇒ AB = 5 cm
cm 5
A 8
⇒ OA = cm
2
19. (c) ∠OAT = 90° [Q angle between radius and tangent]
Y
Now, ∠BOA = 100° [angle subtended by an arc at
A chord CD is drawn, which is parallel to XY and at a centre is twice the angle
distance of 8 cm from A. subtended at remaining part of circle]
Also, AE = 8 cm. Join OC ⇒ ∠ATO = 180° − ( ∠TOA + ∠OAT )
Now, in right angled ΔOEC, [angles property of a triangle]
OC 2 = OE 2 + EC 2 = 180° − ( 50° + 90° )
⇒ EC 2 = OC 2 − OE 2 = 180° − 140° = 40°
20. (a) Take a point Qʹ on circle and join PQʹ and QQʹ.
[by Pythagoras theorem]
= 52 − 32

[Q OC = radius = 5 cm, OE = AE − AO = 8 − 5 = 3 cm]
Q
= 25 − 9 = 16 O
⇒ EC = 4 cm
Hence, length of chord CD = 2 CE 60
º R
= 2 × 4 = 8 cm
A P T
[since, perpendicular from centre to the
chord bisects the chord] Now, ∠OPQ = 90°−60° = 30° [Q ∠OPT = 90°]
16. (c) In figure, AOC is a diameter of the circle. We know that, ⇒ ∠OQP = 30° [angles opposite to equal
diameter subtends an angle 90° at the circle. sides are equal]
So, ∠ ABC = 90° ⇒ ∠POQ = 120°
In ΔACB, ∠A + ∠B + ∠C = 180° [angle sum property of a triangle]
⇒ ∠PQʹ Q = 60°
[since, sum of all angles of a triangle is 180°]
[angle subtended by an arc at centre is twice the angle
⇒ ∠ A + 90° + 50° = 180°
subtended at remaining part of circle]
⇒ ∠A + 140 = 180
⇒ ∠PRQ = 120° [Q opposite angles
⇒ ∠ A = 180° − 140° = 40° are
∠A or ∠OAB = 40° supplementary in a cyclic quadrilateral PQQʹ R ]
Now, AT is the tangent to the circle at point A. So, OA is 21. (b) Given, AB ||PR
perpendicular to AT.
A D B
∴ ∠ OAT = 90° [from figure]
⇒ ∠ OAB + ∠ BAT = 90°
On putting ∠OAB = 40°, we get O
⇒ ∠ BAT = 90° − 40° = 50°
Hence, the value of ∠BAT is 50°.
70°
17. (b) Clearly, ∠CAD = 90° [angle in a semi-circle]
P Q R
So, in ΔACD, CD 2 = AC 2 + AD 2 = 36 + 64 = 100
∴ ∠ ABQ = ∠ BQR = 70° [alternate angles]
[by Pythagoras theorem]
Also, QD is perpendicular to AB and QD bisects AB.
⇒ CD = 10 cm
In ΔQDA and ΔQDB,
Therefore, OC = OD = OB = 5 cm [Q radius of a circle]
∠QDA = ∠QDB [each 90°]
Since, ∠OBT = 90°
AD = BD
[angle between radius and tangent]
QD = QD [common side]
So, in ΔOBT, OT 2 = OB 2 + BT 2
∴ Δ ADQ ~ Δ BDQ
= 25 + 16 = 41 [by Pythagoras theorem]
[by SAS similarity criterion]
⇒ OT = 41 cm
CBSE Term II Mathematics X (Standard) 65

Then, ∠QAD = ∠ QBD [by CPCT] ...(i) 1 AB


⇒ =
Also, ∠ABQ = ∠BQR [alternate interior angle] 2 OA
∴ ∠ABQ = 70° [Q ∠BQR = 70°] ⇒ OA = 2 AB
Hence, ∠QAB = 70° [from Eq. (i)] 25. (c) ∠OPB = 30°
Now, in Δ ABQ, ∠ A + ∠ B + ∠ Q = 180° [Q angles opposite to equal sides are equal]
⇒ ∠ Q = 180° − ( 70° + 70° ) = 40° and ∠OPT = 90°
22. (c) We have, PA = PB = 14 cm Now, ∠PTA = 180°−( ∠OBP + ∠BPT )
[angle sum property of a ΔBPT]
A
C = 180° − ( 30° + 120° )
cm
14 [Q ∠BPT = 90° + 30° = 120° ]
P E = 180° − 150°
O
= 30°
D
B 26. (d) ∠OQS = ∠OQL − ∠SQL [since, OQ ⊥ LP]
= 90°−50° = 40°
Also, CD is tangent at point E on the circle. Similarly, ∠ORS = 30°
So, CA and CE are tangents to the circle from point C. Now, ∠QSR = ∠OSR + ∠OSQ = ∠ORS + ∠OQS
Therefore, CA = CE, similarly DB = DE [Q angle opposite to equal sides are equal]
Now, perimeter of ΔPCD = 30° + 40°
= PC + CD + PD = PC + CE + ED + PD = 70°
= PC + CA + PD + DB 1 1
27. (c) ∠ABQ = ∠AOQ = ( 58° ) = 29°
[Q CA = CE and DE = DB] 2 2
= PA + PB = 14 + 14 = 28 cm [angle subtended by an arc at the centre is twice the angle
subtended at remaining part of circle]
23. (a) Here, AP = AQ
and ∠BAT = 90° [angle between radius and tangent]
⇒ ∠AQP = ∠APQ = x (say) In ΔABT, we get,
[Q angles opposite to equal sides of a triangle are equal] ∠ATQ = 180° − (29°+90° ) = 61°
P
[angle sum property of a triangle]
28. (d) We know that, the opposite sides of a quadrilateral
O A circumscribing a circle subtend supplementary angles at the
centre of the circle.
Q i.e. ∠AOB + ∠COD = 180°
⇒ ∠ COD = 180° − ∠AOB
In ΔAPQ, ∠PAQ = 180° − ( ∠APQ + ∠AQP )
= 180° − 125° = 55°
= 180° − ( x + x ) = 180° − 2x
29. Here, in right angled ΔABC, AB = 3 m and BC = 4 m.
Q OP ⊥ AP
∴ ∠OPA = 90° ∴By Pythagoras theorem,
⇒ ∠OPQ + ∠APQ = 90° AC = ( AB )2 + (BC )2
⇒ ∠OPQ + x = 90° = ( 3) 2 + ( 4 ) 2
⇒ ∠OPQ = 90° − x = 9 + 16 = 25 = 5 m
⇒ 2 ∠OPQ = 180°−2 x [multiplying by 2] Also, AP = x m
⇒ ∠PAQ = 2 ∠OPQ (i) (c) AR = AP = x m … (i)
24. (a) In ΔOAB and ΔOAC, we have [since, length of tangents drawn from an
∠OBA = ∠OCA = 90° external point are equal]
OA = OA [common] (ii) (b) BQ = BR = AB − AR = ( 3 − x ) m [using Eq. (i)]
and OB = OC [radii of circle] (iii) (d) CQ = CP = AC − AP = ( 5 − x ) m
So, by RHS congruence criterion, Also, CQ = BC − BQ = BC − BR
ΔOBA ≅ ΔOCA = 4 − ( 3 − x) = 1 + x
⇒ ∠OAB = ∠OAC (iv) (b) Since, CQ = 5 − x = 1 + x
1 ⇒ 4 = 2x ⇒ x = 2
= × 120° = 60°
2 ∴ AR = AP = 2 m, BR = BQ = 1 m
In ΔOBA, we have and CP = CQ = 3 m
AB
cos 60° =
OA
66 CBSE Term II Mathematics X (Standard)

A 3 6 ⎡ 3⎤

2
=
OQ ⎢Q sin 60°= 2 ⎥
⎣ ⎦
2×6 3
P ⇒ OQ = × [rationalising]
r 3 3
R O ⇒ OQ = 4 3 cm
Hence, length of a tangent PQ is 2 3 cm and a line OQ
B C is 4 3 cm.
Q
2. Let EAF be tangent to the circumcircle of ΔABC.
Also, OQ ⊥ BQ and OR ⊥ BR A
E F
∴ BROQ is a square.
(v) (a) Radius of the pit, OR = BR = 1 m
30. Here, AS = 6 cm, BT = 4 cm [Q radii of circles]
(i) (c) Since, radius at point of contact is perpendicular to
tangent.
B C
∴ By Pythagoras theorem, we have
PA = PS2 + AS2
To prove EAF ||BC
= 82 + 62 ∠EAB = ∠ABC
= 64 + 36 Here, AB = AC
= 100 = 10 cm ⇒ ∠ACB = ∠ABC …(i)
(ii) (b) Again, by Pythagoras theorem, we have [angle between tangent and its chord equal to angle made by
chord in the alternate segment]
BQ = TQ2 + BT 2 = 32 + 42
∴ Also, ∠ EAB = ∠ BCA …(ii)
= 9 + 16 = 25 = 5 cm From Eqs. (i) and (ii), we get
(iii) (c) PK = PA + AK = 10 + 6 = 16 cm ∠ EAB = ∠ ABC
(iv) (c) QY = BQ − BY = 5 − 4 = 1 cm ⇒ EAF ||BC
(v) (b) If two circles touch externally, then the number of 3. Since, AC is a diameter line, so angle in semi-circle makes
common tangents can be drawn is 2. an angle 90°.
∴ ∠ ABC = 90°
Subjective Questions
In Δ ABC, ∠ CAB + ∠ ABC + ∠ ACB = 180°
1. Given, PQ is a tangent, OP = 6 cm and ∠PQO = 60° [Q sum of all interior angles of any triangle is 180°]
We know that, tangent at any point of a circle is ⇒ ∠ CAB + ∠ ACB = 180° − 90° = 90° …(i)
perpendicular to the radius through the point of contact.
Since, diameter of a circle is perpendicular to the tangent.
∴ OP ⊥ PQ
i.e. CA ⊥ AT
∴ ∠ CAT = 90°
⇒ ∠ CAB + ∠ BAT = 90° …(ii)
O From Eqs. (i) and (ii), we get
∠ CAB + ∠ ACB = ∠ CAB + ∠ BAT
⇒ ∠ ACB = ∠ BAT Hence proved.
60°
Q 4. Given, AB is a diameter of the circle.
P
A tangent is drawn from point A. Draw a chord CD parallel
Now, in right angled ΔOPQ, to the tangent MAN.
OP ⎡ perpendicular ⎤ M C
tan 60° = Q tan θ =
PQ ⎢⎣ base ⎥⎦
6
⇒ 3= [Q tan 60° = 3] O
PQ A B
E
6 3
⇒ PQ = × [rationalising]
3 3
N D
OP
⇒ PQ = 2 3 cm and sin 60° = So, CD is a chord of the circle and OA is a radius of the circle.
OQ
∠ MAO = 90°
⎡ perpendicular ⎤
⎢Q sin θ = hypotenuse ⎥ [tangent at any point of a circle is perpendicular to the radius
⎣ ⎦ through the point of contact]
CBSE Term II Mathematics X (Standard) 67

∠ CEO = ∠ MAO [corresponding angles] ⇒ DO2 = 52 − 42 = 25 − 16 = 9


∴ ∠CEO = 90° ⇒ DO = 3 cm
Thus, OE bisects CD, [perpendicular from centre of circle to ∴ Radius of the inner circle OD = 3 cm
the chord bisects the chord] 7. Since, a chord AB subtends an angle of 60° at the centre of a
Similarly, the diameter AB bisects all chords, which are circle.
parallel to the tangent at the point A.
5. Given that, PQ is any line segment and S1 , S2 , S3 , S4 , ...
circles are touch a line segment PQ at a point A. Let the O
centres of the circles S1 , S2 , S3 , S4, ... be C1 , C 2 , C 3 , C 4,...
respectively. 60°
A B

S4
C4 S3
C3 S2 C
C2 S1
C1 i.e. ∠ AOB = 60°
As, OA = OB = Radius of the circle
P Q
A ∴ ∠ OAB = ∠ OBA = 60°
To prove Centres of these circles lie on the perpendicular The tangent at points A and B is drawn, which intersects
bisector of PQ. at C.
Now, joining each centre of the circles to the point A on the We know, OA ⊥ AC and OB ⊥ BC.
line segment PQ by a line segment, i.e. ∴ ∠ OAC = 90° and ∠OBC = 90°
C1A , C 2A , C 3A , C 4A ,... so on.
⇒ ∠ OAB + ∠ BAC = 90°
We know that, if we draw a line from the centre of a circle to
and ∠ OBA + ∠ABC = 90°
its tangent line, then the line is always perpendicular to the
tangent line. But it not bisect the line segment PQ. ⇒ ∠ BAC = 90° − 60° = 30°
So, C1A ⊥ PQ [for S1] and ∠ ABC = 90° − 60° = 30°
C 2A ⊥ PQ [for S2] In Δ ABC, ∠ BAC + ∠ CBA + ∠ ACB = 180°
C 3A ⊥ PQ [for S3] [since, sum of all interior angles of a triangle is 180°]
C 4A ⊥ PQ [for S4] ⇒ ∠ ACB = 180° − ( 30° + 30° ) = 120°
... so on. 8. Two tangents PA and PB are drawn to a circle with centre O
Since, each circle is passing through a point A. Therefore, all from an external point P.
the line segments C1A , C 2A , C 3A , C 4A,..., so on are coincident. A
C
So, centre of each circle lies on the perpendicular line of PQ
but they do not lie on the perpendicular bisector of PQ. E
O P
Hence, a number of circles touch a given line segment PQ at
a point A, then their centres lie. D
6. Let C1 and C 2 be the two circles having same centre O. AC is B
a chord which touches the C1 at point D. Perimeter of ΔPCD = PC + CD + PD
C2
= PC + CE + ED + PD
C1 = PC + CA + DB + PD
[Q CE = CA, DE = DB]
O = PA + PB
= 2PA = 2(10) [PA = PB tangents from
external point to a circle are equal]
= 20 cm
A D C

Join OD. 9. Given Two tangents PQ and PR are drawn from an external
Also, OD ⊥ AC point P to a circle with centre O.
∴ AD = DC = 4 cm R
[perpendicular line OD bisects the chord]
In right angled ΔAOD, O P
OA 2 = AD 2 + DO2 [by Pythagoras theorem,
i.e. (hypotenuse)2 =(base)2 +(perpendicular)2] Q
68 CBSE Term II Mathematics X (Standard)

To prove Centre of a circle touching two intersecting lines Also, ∠ OAB + ∠ OCD = 180°
lies on the angle bisector of the lines. ∴ AB||CD
In ∠RPQ.
Similarly, BD is a straight line.
Construction Join OR and OQ. and ∠ Oʹ BA = ∠ Oʹ DC = 90°
In ΔPOR and ΔPOQ, Also, AC = BD [radii of two circles are equal]
∠ PRO = ∠ PQO = 90° In quadrilateral ABCD,
[tangent at any point of a circle is perpendicular ∠ A = ∠ B = ∠ C = ∠ D = 90°
to the radius through the point of contact] and AC = BD
OR = OQ [radii of same circle] ABCD is a rectangle
Since, OP is common. Hence, AB = CD
∴ Δ PRO ≅ Δ PQO [by RHS] [opposite sides of rectangle are equal]
Hence, ∠ RPO = ∠ QPO [by CPCT] 12. Given Common tangents AB and CD of two circles
Thus, O lies on angle bisecter of PR and PQ. Hence proved. intersecting at E.
10. Two tangents BD and BC are drawn from an external point B. To prove AB = CD
C A
D

B 120°
O
E
D B
C
To prove BO = 2BC Proof EA = EC ...(i)
Given, ∠ DBC = 120° [the lengths of tangents drawn from an external
Join OC , OD and BO. point to a circle are equal]
Since, BC and BD are tangents. EB = ED ...(ii)
∴ OC ⊥ BC and OD ⊥ BD On adding Eqs. (i) and (ii), we get
We know, OB is a angle bisector of ∠DBC. EA +EB = EC + ED
∴ ∠ OBC = ∠ DBO = 60° ⇒ AB = CD Hence proved.
In right angled Δ OBC, 13. Let OP intersect AB at a point C.
BC Clearly, ∠APO = ∠BPO …(i)
cos 60° =
OB [Q O lies on bisector of ∠APB]
1 BC
⇒ =
2 OB A
⇒ OB = 2 BC
Also, BC = BD P
[tangents drawn from external point to circle are equal] C O
∴ OB = BC + BC
⇒ OB = BC + BD B
11. Given AB and CD are tangents to two circles of equal radii.
To prove AB = CD Now, in ΔACP and ΔBCP,
A B AP = BP
[Q length of tangents drawn from an external point
to a circle are equal]
C1 O Oʹ C2
PC = PC [common sides]
and ∠APO = ∠BPO [from Eq. (i)]
C D ∴ ΔACP ≅ ΔBCP [by SAS congruence rule]
Construction Join Oʹ A , Oʹ C , OB and OD Then, AC = BC [by CPCT]
and ∠ACP = ∠BCP [by CPCT]
Proof Now, ∠ OAB = 90°
1
[tangent at any point of a circle is perpendicular to radius = × 180° = 90°
2
through the point of contact]
[Q AB is a straight line]
Thus, AC is a straight line.
Hence, OP is perpendicular bisector of AB. Hence proved.
CBSE Term II Mathematics X (Standard) 69

14. 1 1 5r 2
P and ar (ΔOAC ) = × OF × AC = × r × 5 = cm
2 2 2
∴ ar (ΔABC ) = ar (ΔOAB ) + ar (ΔOBC ) + ar (ΔOAC )
A
1 4r 3r 5r 1 12 r
O ⇒ AB × BC = + + ⇒ ×3×4 =
2 2 2 2 2 2
⇒ r = 1 cm
Q 17. Produce BD to meet the bigger circle at E. Join AE.
AP = AQ Then, ∠AEB = 90° [Q angle in semi-circle]
⇒ ∠APQ = ∠AQP = x [say]
A
[Q angles opposite to equal sides are equal]
O
In ΔAPQ, ∠PAQ = 180° − ( ∠APQ + ∠AQP )
[angle sum property of a triangle] B D E
= 180° − ( x + x ) = 180° − 2x
Q OP ⊥ AP Clearly, OD ⊥ BE [QBE is tangent to the smaller
[Q radius is perpendicular to the tangent circle at D and OD is its radius]
at the point of contact] ∴ BD = DE
∴ ∠OPA = 90° [QBE is a chord of the bigger circle and OD ⊥ BE]
⇒ ∠OPQ + ∠APQ = 90° Now, in ΔAEB, O and D are the mid-points of AB and BE,
⇒ ∠OPQ + x = 90° respectively.
⇒ ∠OPQ = 90° − x Therefore, by mid-point theorem, we have
∴ ∠PAQ = 2 ∠OPQ Hence proved. 1
OD = AE ⇒ AE = 2 × OD = 2 × 8 = 16 cm
15. CR = CQ = BC − BQ 2
[Q OD = radius of smaller circle = 8 cm]
= 38 − 25 = 13 cm In right angled ΔODB,
∴ RD = CD − CR = 28 − 13 = 15 cm OB 2 = OD 2 + BD 2 [by Pythagoras theorem]
Here, OR ⊥ RD and OS ⊥ DA. ⇒ BD 2 = 169 − 64 = 105
[Q tangent is perpendicular to the radius ⇒ BD = 105 cm = DE [QBD = DE]
through the point of contact]
Now, in right angled ΔAED,
Also, ∠ADC = 90°, then fourth angle in quadrilateral ORDS
AD 2 = AE 2 + ED 2 [by Pythagoras theorem]
will be 90°. Thus, ORDS will be a rectangle.
2 2
Q D is an external point of a circle. ⇒ AD = (16) + ( 105 )
∴ DR = DS = 256 + 105 = 361 = 19 cm
Also, opposites sides of rectangle are equal. 18. We know that, tangents drawn from an exterior point to a
∴ RD = OR = OS = SD circle are equal in length.
Hence, quadrilateral DROS is a square. ∴ AD = AF = x cm [say]
∴ Radius = OR = RD = 15 cm BD = BE = y cm [say]
16. Let D, E and F are the points, where the incircle touches the CE = CF = z cm [say]
sides AB, BC and CA, respectively. Join OA, OB and OC. Given, AB = 8 cm
A
⇒ AD + BD = 8 cm
⇒ x+y=8 ...(i)
BC = 10 cm
F
r ⇒ BE + CE = 10 cm
r O
D ⇒ y + z = 10 ...(ii)
r
and CA = 12 cm
B C
E ⇒ CF + AF = 12 cm
In Δ ABC, 2 2 2
AC = AB + BC [by Pythagoras theorem] ⇒ z + x = 12 ...(iii)
= 42 + 32 = 16 + 9 = 25 On adding Eqs. (i), (ii) and (iii), we get
∴ AC = 5 cm 2( x + y + z ) = 30
[taking positive square root, as length cannot be negative] ⇒ x + y + z = 15 ...(iv)
1 1 4r 2 On subtracting Eq. (ii) from Eq. (iv), we get
Now, ar (ΔOAB ) = × OD × AB = × r × 4 = cm ,
2 2 2 x = 15 − 10 = 5
1 1 3r 2 On subtracting Eq. (iii) from Eq. (iv), we get
ar (ΔOBC ) = × OE × BC = × r × 3 = cm
2 2 2 y = 15 − 12 = 3
70 CBSE Term II Mathematics X (Standard)

On subtracting Eq. (i) from Eq. (iv), we get 21. Given, hexagon ABCDEF circumscribe a circle.
z = 15 − 8 = 7 A
∴ AD = x cm = 5 cm, P Q
F B
BE = y cm = 3 cm
and CF = z cm = 7 cm U R
Hence, the length of AD, BE and CE are 5 cm, 3 cm and
E C
7 cm, respectively.
T S
19. A circle is inscribed in the ΔABC, which touches the BC , CA D
and AB. Since, tangents drawn from an external point to a circle are
A
equal in length
∴ AQ = AP, BQ = BR, CR = CS, DS = DT,
ET = EU, FP = FU
F E So, AB + CD + EF = ( AQ + QB ) + ( CS + SD ) + (EU + UF )
= AP + BR + CR + DT + ET + FP
= ( AP + FP ) + (BR + CR ) + (DT + ET )
B D C
⇒ AB + CD + EF = AF + BC + DE Hence proved.
Given, BC = a , CA = b and AB = c 22. ∠ DAB = 90°
By using the property, tangents are drawn from an external In ΔABD , ∠ DAB + ∠ ABD + ∠ ADB = 180°
point to the circle are equal in length.
⇒ ∠ ADB = 180° − 140° = 40°
∴ BD = BF = x [say]
In ΔODC, OD = OC [radii of same circle]
DC = CE = y [say]
⇒ ∠OCD = ∠CDO = 40°
and AE = AF = z [say]
[Q angles opposite to equal sides are equal]
Now, BC + CA + AB = a + b + c
∴ ∠DOC + ∠OCD + ∠ CDO = 180°
⇒(BD + DC ) + ( CE + EA ) + ( AF + FB ) = a + b + c
[Q sum of all angles in a triangle is 180°]
⇒ ( x + y ) + ( y + z) + ( z + x) = a + b + c
⇒ ∠ DOC = 100°
⇒ 2 ( x + y + z ) = 2s
Since, AD is a straight line.
[Q 2s = a + b + c = perimeter of ΔABC]
∴ ∠ DOC + ∠ COA = 180° ⇒ ∠ COA = 80°
⇒ s = x + y + z ⇒ x = s − ( y + z)
23. Let O be the centre of circle.
⇒ BD = s − b [Qb = AE + EC = z + y ]
Q
Hence proved.
20. ∠OCA = 90° [angle between tangent and radius]
Now, ∠OCA = ∠OCD + ∠ACD S O 30° P
⇒ ∠ACD = ∠OCA − ∠OCD
A
⇒ ∠ACD = 90° − 44° = 46°
R
As, AC = AD
[tangents drawn from an external point Join OQ and OR. Then,
are equal in length]
OQ ⊥ PQ and OR ⊥PR
So, ∠ADC = ∠ACD = 46° [Q tangent is perpendicular to the radius
[Q angles opposite to the equal sides are equal] at the point of contact]
Also, ∠CAD + ∠ADC + ∠ACD = 180° So, ∠ROQ + ∠RPQ = 180°
[angle sum property of a ΔACD] [Q sum of all interior angles of
⇒ ∠CAD = 180°− ( 46° + 46° ) = 88° quadrilateral is 360°]
Again, ∠COD = 180°− ∠CAD = 92 ° ⇒ ∠ROQ = 150°
Further, ∠OBD = ∠ODB 1
But ∠RSQ = ∠ROQ
[OB = OD radii of circle] 2
In ΔOBD, use exterior angle theorem 1
= × 150° = 75°
exterior angle ∠COD = ∠OBD + ∠ODB 2
= ∠OBD + ∠OBD Now, on extending QO to intersect RS at A, we get
⇒ 2∠OBD = ∠COD [exterior angle theorem] ∠OQP = ∠QAS = 90° [alternate interior angle]
1 [Q PQ||RS and ∠OQP = 90°]
⇒ ∠CBD = × 92 °= 46°
2 Therefore, from ΔQSA,
Hence, ∠CAD = 88°, ∠ADC = 46°, ∠CBD = 46° and ∠SQA = 180° − 90° − 75° = 15°
∠ACD = 46°
CBSE Term II Mathematics X (Standard) 71

Also, ∠PQR + ∠PRQ + ∠QPR = 180° Let ON = x, then NOʹ = 5 − x


⇒ ∠PQR + ∠PQR + 30° = 180° In right angled Δ OPN,
[Q ∠PQR = ∠PRQ because PQ = PR ] ( OP )2 = ( ON )2 + (NP )2 [by Pythagoras theorem]

⇒ ∠PQR =
150°
= 75° ⇒ (NP )2 = 32 − x 2 = 9 − x 2 …(i)
2 and in right angled ΔPNOʹ,
⇒ ∠AQR = ∠AQP − ∠PQR (POʹ )2 = (PN )2 + (NOʹ )2 [by Pythagoras theorem]
= 90° − 75° = 15°
⇒ ( 4)2 = (PN )2 + ( 5 − x )2
So, ∠RQS = ∠SQA + ∠AQR
⇒ (PN )2 = 16 − ( 5 − x )2 …(ii)
= 15° + 15° = 30°
From Eqs. (i) and (ii), we get
24. Clearly, ∠QAB = 60° and ∠QBA = 60°
9 − x 2 = 16 − ( 5 − x )2
[Q ΔABQ is an equilateral]
⇒ 7 + x 2 − (25 + x 2 − 10x ) = 0 ⇒ 10x = 18
So, ∠PAQ = ∠PAB + ∠QAB = 120°
∴ x = 1. 8
Similarly, ∠PBQ = 120°
Again, in right angled ΔOPN,
[Q∠PAB = ∠PBA, as PA = PB] …(i)
OP 2 = ( ON )2 + (NP )2 [by Pythagoras theorem]
Now, in ΔPAQ and ΔPBQ, ⇒ 32 = (1.8)2 + (NP )2
PA = PB ⇒ (NP )2 = 9 − 3.24 = 5.76
[tangents drawn from external point] ∴ (NP ) = 2. 4
⇒ AQ = BQ [ΔABQ is an equilateral] ∴ Length of common chord, PQ = 2 PN = 2 × 2.4 = 4.8 cm
⇒ ∠PAQ = ∠PBQ [each 120°, shown above] 26. In a circle, ΔABC is inscribed.
So, ΔPAQ ≅ ΔPBQ [by SAS similarity rule] Join OB , OC and OA.
⇒ ∠APQ = ∠BPQ [by CPCT] …(ii) Conside ΔABO and ΔACO
Let QP intersect AB at M. A
m
Now, in ΔPAM and ΔPBM, 6c 1 2
6 cm
∠APM = ∠BPM [from Eq. (ii)] B C
M
⇒ PA = PB 9 9
O
[tangents drawn from an external point]
⇒ PM = PM [common side]
So, ΔPAM ≅ ΔPBM [by SAS congruence rule]
⇒ AM = BM AB = AC [given]
BO = CO [radii of same circle]
and ∠AMP = ∠BMP [by CPCT] ...(iii) AO = AO [common side]
But ∠AMP + ∠BMP = 180° ∴ ΔABO ≅ Δ ACO [by SSS congruence rule]
⇒ ∠AMP + ∠AMP = 180° ⇒ ∠1 = ∠2 [CPCT]
⇒ ∠AMP = 90° Hence Proved. Now, in ΔABM and ΔACM,
25. Here, two circles are of radii OP = 3 cm and POʹ = 4 cm . AB = AC [given]
These two circles intersect at P and Q. ∠1 = ∠2 [proved above]
AM = AM [common side]
P
∴ ΔAMB ≅ ΔAMC[by SAS congruence rule]
3 cm 4 cm
⇒ ∠ AMB = ∠ AMC …(i) [CPCT]
O N Oʹ Also, ∠ AMB + ∠ AMC = 180° [linear pair]
⇒ ∠ AMB + ∠ AMB = 180° [from Eq. (i)]
Q
⇒ ∠ AMB = 90°
Here, OP and POʹ are two tangents drawn at point P. We know that a perpendicular from centre of circle bisects
∠ OPOʹ = 90° the chord. So, OA is perpendicular bisector of BC.
[tangent at any point of circle is perpendicular to radius Let AM = x, then OM = 9 − x [Q OA = radius = 9 cm]
through the point of contact] In right angled ΔAMC,
Join OOʹ and PN.
AC 2 = AM 2 + MC 2 [by Pythagoras theorem]
In right angled Δ OPOʹ,
( OOʹ )2 = ( OP )2 + (POʹ )2 [by Pythagoras theorem] i.e. (Hypotenuse)2 = (Base)2 + (Perpendicular)2
i.e. ( Hypotenuse)2 = (Base)2 + (Perpendicular)2 ⇒ MC 2 = 62 − x 2 …(i)
= ( 3)2 + ( 4)2 = 25 and in right angled Δ OMC,
⇒ OOʹ = 5 cm OC 2 = OM 2 + MC 2 [by Pythagoras theorem]
Also, PN ⊥ OOʹ ⇒ MC 2 = 92 − ( 9 − x )2 …(ii)
72 CBSE Term II Mathematics X (Standard)

From Eqs. (i) and (ii), 1


2 2 2 2
⇒ ∠DEOʹ = 90° − ∠DOʹ B …(v)
6 − x = 9 − ( 9 − x) 2
⇒ 36 − x 2 = 81 − ( 81 + x 2 − 18 x ) [since, Oʹ E is the angle bisector of ∠DEB i.e.
1
⇒ 36 = 18x ⇒ x = 2 ∠DEB = ∠DEOʹ]
∴ AM = x = 2 2
In right angled ΔABM, Similarly, ∠AEC = 180° − ∠AOC
Divided by 2 on both sides, we get
AB 2 = BM 2 + AM 2 [by Pythagoras theorem]
1 1
62 = BM 2 + 2 2 ∠AEC = 90° − ∠AOC
2 2
⇒ BM 2 = 36 − 4 = 32 ⇒ BM = 4 2 1
⇒ ∠AEO = 90° − ∠AOC …(vi)
∴ BC = 2 BM = 2 × 4 2 = 8 2 cm 2
1 [since, OE is the angle bisector of ∠AEC
∴ Area of ΔABC = × Base × Height 1
2 i.e. ∠AEC = ∠AEO]
2
1
= × BC × AM Now,
2 ⎛ 1 ⎞
1 ∠AED + ∠DEOʹ + ∠AEO = ∠AED + ⎜ 90° − ∠DOʹ B⎟
= × 8 2 × 2 = 8 2 cm 2 ⎝ 2 ⎠
2 1
⎛ ⎞
Hence, the required area of ΔABC is 8 2 cm 2. + ⎜ 90° − ∠AOC⎟
⎝ 2 ⎠
27. In the given figure, join AO, OC and Oʹ D , Oʹ B. 1
= ∠AED + 180° − ( ∠DOʹ B + ∠AOC )
Now, in ΔEOʹ D and ΔEOʹ B, 2
Oʹ D = Oʹ B [radius] 1
= ∠AED + 180° − ( ∠AED + ∠AED )
Oʹ E = Oʹ E [common side] 2
ED = EB [from Eqs. (iii) and (iv)]
1
[since, tangents drawn from an external point to the circle = ∠AED + 180° − (2 × ∠AED )
are equal in length] 2
= ∠AED + 180° − ∠AED = 180°
A ∴ ∠AEO + ∠AED + ∠DEOʹ = 180°
D
So, OEOʹ is straight line.
O Oʹ Hence, O, E and Oʹ are collinear. Hence proved.
E
B 28. Given, OT = 13 cm and OP = 5 cm
C Since, if we draw a line from the centre to the tangent of the
circle, then it is always perpendicular to the tangent i.e.
∴ ΔEOʹ D ≅ ΔEOʹ B [by SSS similarity rule] OP ⊥ PT.
⇒ ∠Oʹ ED = ∠Oʹ EB [by CPCT] P
Oʹ E is the angle bisector of ∠DEB. …(i) A
Similarly, OE is the angle bisector of ∠AEC.
Now, in quadrilateral DEBOʹ, O E
T
∠Oʹ DE = ∠Oʹ BE = 90°
[since, CED is a tangent to the circle and Oʹ D is the radius, B
i.e. Oʹ D ⊥ CED] Q
⇒ ∠Oʹ DE + ∠Oʹ BE = 180°
In right angled ΔOPT ,
∴ ∠DEB + ∠DOʹ B = 180° OT 2 = OP 2 + PT 2
[since, DEBOʹ is cyclic quadrilateral] …(ii)
[by Pythagoras theorem,
Since, AB is a straight line.
(hypotenuse)2 = (base)2 + (perpendicular)2]
∴ ∠AED + ∠DEB = 180°
⇒ PT = (13)2 − ( 5)2
2
⇒ ∠AED + 180° − ∠DOʹ B = 180° [from Eq. (ii)]
⇒ ∠AED = ∠DOʹ B …(iii) = 169 − 25 = 144
⇒ PT = 12 cm
Similarly, ∠AED = ∠AOC ...(iv)
Since, the length of pair of tangents from an external point T
Again from Eq. (ii),
is equal.
∠DEB = 180° − ∠DOʹ B ∴ QT = 12 cm
Divided by 2 on both sides, we get Now, TA = PT − PA
1 1 ⇒ TA = 12 − PA ...(i)
∠DEB = 90° − ∠DOʹ B
2 2 and TB = QT − QB
CBSE Term II Mathematics X (Standard) 73

⇒ TB = 12 − QB ...(ii) Given, BD = FB = 9 cm, CD = CE = 3 cm


Again, using the property,length of pair of tangents from an In ΔABC, AB 2 = AC 2 + BC 2
external point is equal.
⇒ ( AF + FB )2 = ( AE + EC )2 + (BD + CD )2
∴ PA = AE and QB = EB ...(iii)
∴ OT = 13 cm ⇒ ( x + 9)2 = ( x + 3)2 + 12 2
∴ ET = OT − OE [Q OE = 5 cm = radius] ⇒ x 2 + 81 + 18x = x 2 + 9 + 6x + 144
⇒ ET = 13 − 5 ⇒ ET = 8 cm ⇒ 18x + 81 = 6x + 9 + 144
Since, AB is a tangent and OE is the radius.
⇒ 12 x = 72 ⇒ x = 6 cm
∴ OE ⊥ AB
∴ AB = 6 + 9 = 15 cm
⇒ ∠OEA = 90°
∴ ∠AET = 180° − ∠OEA [linear pair] (iii) (b) As we know that, tangents drawn from an external
point are equal in length. Therefore, AP = AS = 4 cm
⇒ ∠AET = 90°
Now, in right angled ΔAET, ∴ DS = DR = 10 − 4 = 6 cm
( AT )2 = ( AE )2 + (ET )2 And BP = BQ = 2 cm. So, CR = CQ = 5 − 2 = 3 cm
[by Pythagoras theorem] So, CD = DR + CR = 6 + 3 = 9 cm
⇒ (PT − PA )2 = ( AE )2 + ( 8)2 (iv) (d) Here ∠OAP = 90°
A
⇒ (12 − PA )2 = (PA )2 + ( 8)2 [from Eq. (iii)]
⇒ 144 + (PA )2 − 24 ⋅ PA = (PA )2 + 64
10 P O
⇒ 24 ⋅ PA = 80 ⇒ PA = cm 50°
3
10
∴ AE = cm [from Eq. (iii)] B
3
10 In ΔAOP and ΔBOP,
Similarly BE = cm
3 ∠OAP = ∠OBP [90° each]
10 10 20 OA = OB [radii of circle]
Hence, AB = AE + EB = + = cm
3 3 3 PA = PB
20 [tangents drawn from an external point are equal]
Hence,the required length AB is cm .
3 ∴ ΔAOP ~ ΔBOP [by SAS similarity]
29. (i) (d) We have, AP = AQ, BP = BD, CQ = CD … (i) ∴ ∠APO = ∠OPB [by CPCT]
[Q tangents drawn from an external point = 50°
are equal in length] ∴ ∠BPA = 50° + 50° = 100°
Now, AB + BC + AC = 7 + 4 + 9 = 20 cm (v) (c) For bigger circle, PA = PB … (i)
⇒ AB + BD + CD + AC = 20 cm [Qtangents drawn from an external point are
⇒ AP + AQ = 20 cm equal in length] … (ii)
⇒ 2 AP = 20 cm ⇒ AP = 10 cm Similarly, for smaller circle, PB = PC
(ii) (c) Let AF = AE = x cm From Eqs. (i) and (ii), we get
[Q tangents drawn from an external point to PA = PB = PC = 11 cm
a circle are equal in length]
Chapter Test
Multiple Choice Questions (iv) In PA and PB are two tangents, drawn to a circle
with centre O from P such that ∠PBA = 60°, then
1. Two concentric circles are of radii 10 cm and
∠OAB is
8 cm, then the length of the chord of the larger
(a) 50° (b) 25° (c) 30° (d) 130°
circle, which touches the smaller circle is
(a) 6 cm (b) 12 cm (v) In the adjoining figure, if PC is the tangent at A
(c) 18 cm (d) 9 cm of the circle with ∠PAB = 62° and ∠AOB = 132°,
2. From a point P, which is at a distance of 13 cm then ∠ABC is
from the centre O of a circle of radius 5 cm, the B
pair of tangents PQ and PR to the circle is
O
drawn. Then, the area of the quadrilateral
PQOR is [NCERT Exemplar]
62°
(a) 60 cm2 (b) 65 cm2
P A C
(c) 30 cm2 (d) 32.5 cm2
(a) 18° (b) 20°
3. If two tangents inclined at an angle 60° are (c) 60° (d) Can’t be determined
drawn to a circle of radius 3 cm, then the Short Answer Type Questions
length of each tangent is [NCERT Exemplar]
3 5. Two tangents PQ and PR are drawn from an
(a) 3 cm (b) 6 cm
2 external point to a circle with centre O. Prove
(c) 3 cm (d) 3 3 cm that QORP is a cyclic quadrilateral.
Case Based MCQs 6. In figure, AB and CD are common tangents to
4. For revision of chapter circles, a teacher two circles of unequal radii. Prove that Exemplar]
[NCERT AB = CD.
planned a game with some questions written A
on the paper, which are to be answered by the B
students. For each correct answer, a student
will get a prize. Some of the questions are given
below. D
Answer the questions to check your C
knowledge. Long Answer Type Questions
(i) In the given figure, x + y is
7. A is a point at a distance 13 cm from the centre
(a) 60° O
O of a circle of radius 5 cm. AP and AQ are the
(b) 90° x
tangents to the circle at P and Q. If a tangent
(c) 120° y
(d) 145° BC is drawn at a point R lying on the minor arc
A C
PQ to intersect AP at B and AQ at C, find the
(ii) In the given figure, PQ and PR are two tangents perimeter of the ΔABC. [NCERT Exemplar]
to the circle, then ∠ROQ is
8. If a chord and a tangent intersect externally,
S then the product of the lengths of the
R
segments of the chord is equal to the square of
O the length of the tangent from the point of
contact to the point of intersection.
20°
P
Q
(a) 30° (b) 60° (c) 105° (d)
Answers
C
160° 1. (b) 2. (a) 3. (d)
(iii) In the adjoining figure, AB is a 45° 4. (i) (b) (ii) (d) (iii) (b) (iv) (c) (v) (b) 7. 24 cm
chord of the circle and AOC is O B
its diameter such that
For Detailed Solutions
∠ACB = 45°, then ∠BAT is
T
(a) 35° A Scan the code
(b) 45°
(c) 125°
(d) 110°
CBSE Term II Mathematics X (Standard) 75

CHAPTER 04

Constructions

In this Chapter...
! Division of a Line Segment Internally in the Given Ratio
! Construction of a Tangent to a Circle at a Point that lies on it
! Construction of Tangent to a Circle from a Point Outside the Circle
! Construction of Tangents to a Circle When Angle

Constructions 1 Justification
Division of a Line Segment Internally in the Given Ratio Since, A m C||A p B, so use the basic proportionality theorem
in ΔABA p .
To divide a line segment AB (say) internally in the given ratio
AA m AC
m : n, where m and n are both positive integers, we use the Then, = …(i)
following steps A m A p CB
Step I Draw the given line segment AB and any ray AX, By using construction, the ratio is
making an acute angle with the line segment AB. AA m m
This ray AX can be drawn above or below AB. = …(ii)
A m A p ( p − m)
Step II Mark m + n = p points
∴From Eqs. (i) and (ii),
(i.e. A 1 , A 2 , . . . , A m ,. . . , A p ) on the ray AX, such AC m
that AA 1 = A 1 A 2 = ... = A p − 1 A p =
CB ( p − m)
Step III Join BA p .
Alternate Method
Step IV Through the point A m , draw a line parallel to A p B To divide a line segment in the given ratio m : n, where
(by making an angle equal to ∠AA p B at A m ) which m and n are both positive integers, we can also use the
intersects the line segment AB at point C. Thus, following steps.
point C divides the line segment AB internally in the Step I Draw the given line segment AB (say) and any ray
ratio m : n, i.e. AC : CB = m : n. AX making an acute angle with the line
X segment AB.
n
)
+ Step II Draw another ray BY || AX by making
(A m
Ap
∠ABY = ∠BAX.
Am Step III Mark m points i.e. A1 , A 2 ,. . . , A m on AX
A2 and n points i.e. B1 , B 2 , . . . , B n on BY such that
A1
AA1 = A1 A 2 = . . . = A m − 1 A m
A B
C = BB1 = B1B 2 = . . . = B n − 1 B n
76 CBSE Term II Mathematics X (Standard)

Step IV Join A m B n which intersects line segment AB at the So, in ΔABA 5, by basic proportionality theorem, we get
AA 3 AC
point C. = ...(i)
A 3A 5 CB
Now, C is the required point which divides line
By construction, we have
segment AB internally in the ratio m : n. AA 3 3 3
X = = ...(ii)
Am
A 3A 5 ( 5 − 3 ) 2
Am – 1 On equating Eqs. (i) and (ii), we get
A2 AC 3
=
A1 BC 2
This shows that C divides AB internally in the ratio 3 : 2.
A B
C
B1
Construction 2
B2 Construction of a Tangent to a Circle at a Point
that lies on it
Bn – 1
Bn We can construct a tangent to a circle at a point that lies on it
Y
by two cases which are given below
Justification Case I By using the centre of circle
Step V Use the condition of similarity of two triangles in To construct a tangent to a circle by using the centre, we use
AA m AC the following steps.
ΔAA m C and ΔBB n C. Then, = ...(i)
BB n BC Step I Take a point O as centre and draw a circle of
Step VI Write the ratio by using construction, given radius.
AA m m Step II Take a point P on the circle, at which we want to
= ..(ii)
BB n n draw tangent.
AC m Step III Join OP, which is the radius of circle.
Step VII Equating Eqs. (i) and (ii), we get =
BC n
Step IV Take OP as base and construct ∠OPT = 90 ° at P.
Example 1. Draw a line segment AB = 8 cm and divide it
internally in the ratio 3 : 2 and also justify it. Step V Draw a ray PT and produce TP to Tʹ to get the
Sol. Steps of Construction required tangent TPTʹ.
(i) First, draw line segment, AB = 8 cm and draw a ray AX,
which makes an acute angle with line segment AB.
(ii) Mark m + n = 3 + 2 = 5 points i.e. A1 , A 2 , A 3 , A 4 and A 5
on the ray AX such that O
AA1 = A1A 2 = A 2A 3 = A 3A 4 = A 4A 5
(iii) Join BA 5.
Tʹ T
(iv) Through the point A 3 (Q m = 3 ), draw a line A 3C || A 5B P
(by making an angle equal to ∠AA 5 B at A 3), which
intersects the line segment AB at C. Example 2. Draw a circle of diameter AB = 5 cm with
X
A5
centre O and then draw a tangent to the circle at
A4 point A or B.
A3 Sol. Given, diameter of circle = AB = 5 cm and centre is O.
A2
A1 5
∴ Radius = OA = OB = = 2 . 5 cm
A B 2
C
8 cm Steps of Construction
Thus, point C divides the line segment AB internally in the (i) Take a point O as centre and draw a circle of radius
ratio 3 : 2. 2.5 cm.
Justification (ii) Draw diameter AOB.
Since, A 3C || A 5B. (iii) Take OA as base and construct ∠OAT = 90° at A.
CBSE Term II Mathematics X (Standard) 77

(iv) Produce TA to Tʹ to get the required tangent TATʹ. Construction 3


Similarly, we can draw a tangent at point B or any other
point on the circle. Construction of Tangents to a Circle from a Point
Outside the Circle
T
If a point lies outside the circle, then there will be two
tangents to the circle from this point.
2.5 cm
B A Case I When centre of circle is known
O
If centre of circle is known, then to draw tangents from a
given external point, we use the following steps
Tʹ Step I Draw a circle with centre O of given radius and take
Case II Without using the centre of circle a point P outside it.
To construct a tangent to a circle without using the Step II Join OP and bisect it. Let its mid-point be M. Then,
centre of circle, we use the following steps. MP = MO.
Step I Draw a circle of given radius and take a point P Step III On taking M as centre and MO or MP as radius,
(at which we want to draw tangent) on the circle. draw a dotted circle, which intersects the given
Step II Draw any chord PQ through the given point P on the circle at points Q and Q ʹ (say).
circle.
Q
Step III Take a point R in either the major arc or minor arc
and join PR and QR.
Step IV On taking PQ as base, construct ∠QPY equal to P
M
O

∠PRQ and on the opposite side of R.


Step V Draw a ray PY and produce YP upto X to get the Qʹ
required tangent YPX.
R Step IV Join PQ and PQʹ. Thus, PQ and PQʹ are the required
Q tangents drawn to the circle from the external point
P. Here, we observe that PQ = PQ ʹ.
Justification
X
Join OQ. Then, ∠PQO = 90 °, since it is constructed in the
P Y semi-circle of dotted circle. It shows that OQ ⊥ PQ. Also, OQ
Example 3. Draw a circle of radius 6 cm. Take a point P is radius of given circle, so PQ has to be a tangent of given
on it. Without using the centre of the circle, draw a circle. Similarly, PQʹ is also a tangent to the given circle.
tangent to the circle at point P. Example 4. Draw a circle of radius 3.5 cm. From a point
Sol. Given, radius of circle = 6 cm P, 6 cm from its centre, draw two tangents of the
Steps of Construction circle.
(i) Draw a circle of radius 6 cm and take a point P on the Sol. Given, a circle of radius 3.5 cm whose centre is O (say) and
circle. a point P, 6 cm away from its centre.
(ii) Draw a chord PQ through the point P on the circle.
(iii) Take a point R in the major arc and join PR and RQ.
A
(iv) On taking PQ as base, construct ∠QPY equal to
∠PRQ on the opposite side of the point R.
(v) Produce YP to X. Then, YPX is the required tangent at
point P. 6 cm
R O P
M C
3.5 cm

Q
X
B
P
Y
78 CBSE Term II Mathematics X (Standard)

Steps of Construction Sol. Given, radius of circle = 2 cm and distance between point P
(i) Draw a circle with O as centre and radius and centre = 6.5 cm
OC = 3.5 cm. Take a point P such that OP = 6 cm. Steps of Construction
(ii) Draw the bisector of OP which intersect OP at M. (i) Draw a circle of radius 2 cm with centre O.
(iii) Take M as centre and MO as radius, draw a dotted (ii) Take a point P outside it, such that its distance from
circle. Let this circle cuts the given circle at A and B. centre O is 6.5 cm.
(iv) Join PA and PB. (iii) Consider O and P as centre and draw arcs of radius
Hence, PA and PB are the required tangents. more than half of OP on both sides of OP which
intersect each other at R and S. Join RS which bisects
Case II When centre of circle is unknown
OP at M. Then, MP = MO.
If centre of the circle is unknown, then to draw tangents of
(iv) Consider M as centre and MO as radius, draw a dotted
the circle, by using the following steps circle which intersects given circle at Q and Qʹ.
Step I Firstly, draw the circle and then draw two (v) Join PQ and PQʹ.
non-parallel chords of the circle.
Hence, we get the required tangents drawn from point
Step II Draw the perpendicular bisectors of both chords P to the given circle.
which intersect each other at a point, say O. Then, R
this point O gives the centre of given circle. Now,
Q
we further use the steps given in case I to draw
tangents.
Alternate Method 1.1 cm M 2 cm
P O
If centre of circle is unknown, then we can draw tangents 6.5 cm
without finding centre of the circle. For this, we use the
following steps of construction.

Step I Draw a circle of given radius and take a point P
outside it. S

D
Construction 4
Construction of Tangents to a Circle When Angle
T1 between Them is Given
Sometimes, angle between two tangents (or pair of tangents)
K is given and we have to draw these tangents. Then, we use
C B
P A the following steps of construction.
T2
Step I First, draw the given circle with centre O and radius
r cm.
Step II Through P, draw a line (i.e. secant) intersecting the
A
given circle at points A and B, respectively and
O
produce it to C in opposite direction of AB such that r
AP = CP. α Q

Step III Now, bisect the segment CB at K. Then, take K as α


centre and KB (or KC) as radius, draw a semi-circle. P R
Step IV At point P, draw PD ⊥ CB which cuts the
Step II Draw any diameter say AOQ of this circle.
semi-circle at D.
Step III Make given angle α at centre O with OQ (say) as
Step V Take P as centre and PD as radius draw arcs to
intersect the given circle at points T1 and T2 . base which intersect the circle at point R (say) or
Step VI Join PT1 and PT2 which are the required tangents. draw the radius OR meets the circle at R such that
∠QOR = α.
Example 5. Draw a circle of radius 2 cm with centre O
and take a point P outside the circle such that Step IV Now, draw perpendiculars to OA at A and to OR at
OP = 6.5 cm. From P, draw two tangents to the R, which intersect the tangents each other at a point
circle. say P.
CBSE Term II Mathematics X (Standard) 79

Then, AP and RP are the required pair of tangents to given (iii) Draw the radius OR meets the circle at R such that
circle, inclined at an angle α, i.e. angle between pair of ∠QOR = 30°.
tangents is α.
Justification P
By construction, ∠OAP = 90 ° and OA is radius. 4 cm
O
So, PA is a tangent to the circle. 30° Q
Similarly, PR is a tangent to the circle. E 30°
Also, ∠AOR = 180 ° − ∠QOR [Q AOQ is a straight line] N R
D
= 180 ° − α
Now, in quadrilateral AORP, (iv) Draw PD ⊥ PQ and RE ⊥ OR , which intersect each
other at point N. Then, NP and NR are the required
∠APR + ∠PAO + ∠AOR + ∠PRO = 360 ° tangents to the given circle inclined to each other at an
⇒ ∠APR + 90 ° + 180 ° − α + 90 ° = 360 ° angle of 30°.
⇒ ∠APR = α Justification
Example 6. Draw a pair of tangents to a circle of radius By construction, ∠OPN = 90° and OP is radius.
4 cm which are inclined to each other at an angle of ∴ PN is a tangent to the circle.
30°. Similarly, NR is a tangent to the circle.
Sol. Given, a circle of radius 4 cm. We have to construct a pair of Now, ∠POR = 180° − 30°= 150°
tangents, which are inclined to each other at an angle of 30°. [QPOQ is a straight line and ∠QOR = 45°]
Steps of Construction In quadrilateral OPNR,
(i) Draw a circle with O as centre and radius 4 cm. ∠OPN = 90° , ∠POR = 150° and ∠ORN = 90°
(ii) Draw any diameter POQ of this circle. ∴ ∠PNR = 360° − ( 90° + 150° + 90° ) = 30°
80 CBSE Term II Mathematics X (Standard)

Chapter
Practice
6. To divide a line segment AB in the ratio 4 : 7, a ray
PART 1 AX is drawn first such that ∠ BAX is an acute angle
and then points A 1 , A 2 , A 3 ,. . . are located at equal
Objective Questions distances on the ray AX and the point B is joined to
(a) A12 (b) A11
(c) A10 (d) A 9
!
Multiple Choice Questions
7. To divide a line segment AB in the ratio 5 : 6, draw a
1. To divide a line segment AB in ratio m : n
ray AX such that ∠ BAX is an acute angle, then draw
(m and n are positive integers), draw a ray AX to
a ray BY parallel to AX and the points
that ∠BAX is an acute angle and the mark point on
A 1 , A 2 , A 3 ,. . . and B 1 , B 2 , B 3 ,. . . are located to
ray AX at equal distances such that the minimum
equal distances on ray AX and BY, respectively.
number of these points is
Then, the points joined are
(a) greater of m and n (b) m + n
(a) A 5 and B 6 (b) A 6 and B 5
(c) m + n − 1 (d) mn
(c) A 4 and B 5 (d) A 5 and B 4
2. To divide a line segment AB in the ratio 5 : 7, first 8. To divide a line segment AB in the ratio 6 : 7, a ray
a ray AX is drawn, so that ∠ BAX is an acute angle AX is drawn first such that ∠BAX is an acute angle
and then at equal distances points are marked on and then points A 1 , A 2 , A 3 , … are located equal
the ray AX such that the minimum number of distances on the ray AX and the point B is joined
these points is with
(a) 8 (b) 10 (c) 11 (d) 12
(a) A12 (b) A13 (c)A10 (d)A11
3. To divide a line segment AB in the ratio 3 : 5 first a 9. In the given figure, find the ratio, when P divides
ray AX is drawn so that ∠BAX is an acute angle AB internally.
and then at equal distances points are marked on
A3 X
the ray AX such that the minimum number of A2
these points is A1
(a) 8 (b) 9 (c) 10 (d) 11 A B
P
4. To divide a line segment AB in the ratio 4 : 5, first B1
B2
a ray AX is drawn making ∠BAX an acute angle B3
Y B4
and then points A 1 , A 2 , A 3 , . . at equal distances
are marked on the ray AX and the point B is joined (a) 3 : 2 (b) 2 : 3 (c) 4 : 3 (d) 3 : 4
to
10. From the following ratios, a line segment cannot be
(a) A 4 (b) A 5
divided into A ratio.
(c) A 9 (d) A 7
1 1 1
5. The ratio of division of the line segment AB by the (a) A → 5: (b) A → :
5 5 5
point P from A in the following figure is [CBSE 2012] 2 5 1
(c) A → : (d) A → : 1
A5 5 2 5
A4
A3 11. To draw a pair of tangents to a circle, which are
A2 inclined to each other at an angle of 60°, it is
A1
required to draw tangents at end points of those two
A
P B
radii of the circle, the angle between them should be
(a) 135° (b) 90° (c) 60° (d) 120°
(a) 2 : 3 (b) 3 : 2
(c) 3 : 5 (d) 2 : 5
CBSE Term II Mathematics X (Standard) 81

12. A pair of tangents can be constructed from a 7. Draw a circle of radius 3.5 cm. Take a point P outside
point P to a circle of radius 3.5 cm, situated at a the circle at a distance of 7 cm from the centre of the
distance of 3 cm from the centre. circle and construct a pair of tangents to the circle
(a) True from that point. [CBSE 2020 (Standard)]
(b) False
8. Draw a line segment AB of length 9 cm. Taking A as
(c) Can’t determined
(d) None of the above
centre, draw a circle of radius 5 cm and taking B as
centre, draw another circle of radius 3 cm. Construct
13. A pair of tangents can be constructed to a circle tangents to each circle from the centre of the other
inclined at an angle of 170°. circle. [CBSE 2020 (Standard)]
(a) True
(b) False 9. Draw a circle with the help of circular solid ring.
(c) Can’t determined Construct a pair of tangents from a point P outside the
(d) None of the above circle. Also, justify the construction.
!
Long Answer Type Questions
PART 2 10. Draw a circle of radius 4 cm. Construct a pair of
tangents to it, the angle between which is 60°.
Subjective Questions Also, justify the construction. Measure the distance
between the centre of the circle and the point of
!
Short Answer Type Questions intersection of tangents.
1. Draw a line segment of length 7 cm. Find a point 11. Construct a tangent to a circle of radius 1.8 cm from a
P on it, which divides it in the ratio 3 : 5. point on the concentric circle of radius 2.8 cm and
measure its length. Also, verify the measurement by
2. Draw a circle of diameter AB = 6 cm with centre actual calculation.
O and then draw a tangent to the circle at point A
or B. 12. Draw a circle of radius 2.8 cm. From an external point
P, draw tangents to the circle without using the centre
3. Draw a circle of radius 5 cm. Take a point P on it. of the circle.
Without using the centre of the circle, draw a
tangent to the circle at point P. 13. Draw a pair of tangents to a circle of radius 3 cm,
which are inclined to each other at an angle of 45°.
4. Draw a circle of radius 6 cm and draw a tangent
to this circle, making an angle of 30° with a line 14. Let ABC be a right angled triangle, in which
passing through the centre. AB = 6 cm, BC = 8 cm and ∠ B = 90°. BD is the
perpendicular from B on AC. The circle through
5. Draw a circle of radius 4 cm. From a point 6 cm B , C and D is drawn. Construct the tangents from A to
away from its centre, construct a pair of tangents this circle. Also, justify the construction.
to the circle and measure their lengths.
[CBSE 2019] 15. Draw a circle of radius 3 cm. Take two points P and Q
6. Draw a circle of radius 1cm. From a point P, on one of its extended diameter each at a distance of
2.2 cm apart from the centre of the circle, draw 7 cm from its centre. Draw tangents to the circle from
tangents to the circle. these two points P and Q. [NCERT]
82 CBSE Term II Mathematics X (Standard)

SOLUTIONS
Objective Questions 11. (d) The angle between them should be 120° because in that
1. (b) To divide a line segment in the ratio m : n , the maximum case the figure formed by the intersection point of pair of
tangent, the two end points of those two radii (at which
number of the points to mark are m + n .
tangents are drawn) and the centre of the circle is a
2. (d) We know that, to divide a line segment AB in the ratio quadrilateral.
m : n , first draw a ray AX, which makes an acute angle
∠BAX, then marked m + n points at equal distance. P
Here, m = 5, n = 7
So, minimum number of these points = m + n = 5 + 7 = 12.
3. (a) Minimum number of points = 3 + 5 = 8 θ R
O 60°
4. (c) Here, 4 + 5 = 9 points are located at equal distances on
the ray AX, so B is joined to last point A 9 .
Q
5. (b) The ratio of division of the line segment AB by the point
P from A is AP : BP = 3 : 2 . From figure, it is quadrilateral
6. (b) Here, minimum 4 + 7 = 11 points are located at equal ∠POQ + ∠ PRQ = 180°
distances on the ray AX and then B is joined to last point [Q sum of opposite angles are 180°]
is A 11. 60° + θ = 180°
7. (a) Given, a line segment AB and we have to divide it in the ∴ θ = 120
ratio 5 : 6. Hence, the required angle between them is 120°.
Y
B
6
12. (b) False, since, the radius of the circle is 3.5 cm
B
5 B i.e. r = 3. 5 cm and a point P is situated at a distance
4
B
3
of 3 cm from the centre i.e. d = 3 cm
B
2 B We see that, r > d
1
A i.e. a point P lies inside the circle. So, no tangent can be
C B
A1 drawn to a circle from a point lying inside it.
A2 13. True
A3
A4
A5 X

170°
Steps of Construction
1. Draw a ray AX, making an acute ∠BAX.
2. Draw a ray BY parallel to AX by making ∠ABY equal
to ∠BAX.
If the angle between the pair of tangents is always greater
3. Now, locate the points A 1, A 2, A 3, A 4 and A 5 ( m = 5) than 0 or less than 180°, then we can construct a pair of
on AX and B1 , B 2 , B 3 , B 4 , B 5 and B 6 ( n = 6 ) such that tangents to a circle.
all the points are at equal distance from each other. Hence, we can draw a pair of tangents to a circle inclined at
4. Join B 6 A 5, which intersect AB at a point C. an angle of 170°.
Then, AC : BC = 5 : 6
Subjective Questions
8. (b) A 6 + 7 i.e. A13 is joined to the point B.
1. Steps of Construction
9. (d) From given figure, it is clear that there are three points at 1. Draw a line segment AB = 7 cm.
equal distances on AX and four points at equal distances on
2. Draw a ray AX, making an acute ∠BAX.
BY. Here, P divides AB on joining A 3B 4. So, P divides AB
internally in the ratio 3 : 4.
3. Along AX, mark 3 + 5 = 8 points
10. (c) Since, i.e. A 1 , A 2 , A 3 , A 4 , A 5 , A 6 , A 7 and A 8 such that
1 1 1 AA 1 = A 1 A 2 = A 2 A 3 = A 3 A 4
(a) 5 : = 5:1 (b) : =1:1
5 5 5 = A 4A 5 = A 5A 6 = A 6A 7 = A 7A 8
2 5 1 4. Join A 8B.
(c) : =2 2 : 5 (d) :1 = 1: 5 5. From A 3 , draw A 3 C|| A 8 B, meeting AB at C.
5 2 5
Since, (a), (b) and (d) are the ratio of both integers. So, it is [by making an angle equal to ∠BA 8 A at A 3 ]
possible to divide a line segment into these points. Then, C is the point on AB, which divides it in the
Hence, option (c) is correct. ratio 3 : 5.
CBSE Term II Mathematics X (Standard) 83

Thus, AC : CB = 3 : 5 4. Steps of Construction


X (i) Draw a circle with centre O and radius 6 cm.

P
A8
B
A7 30° Q
A6 A
60°
A5 6 cm
O
A4
A3
A2
A1 (ii) Draw a radius OA and produce it to B.
(iii) Construct an ∠AOP equal to the complement of 30°
A C B i.e. equal to 60°.
7 cm (iv) Draw a perpendicular to OP at P, which intersects OB at
2. Given, diameter of circle = AB = 6 cm and centre is O. point Q.
6 Hence, PQ is the required tangent such that ∠OQP = 30°.
∴ Radius = OA = OB = = 3 cm
2 5. Given, a point Mʹ is at a distance of 6 cm from the centre of a
circle of radius 4 cm.
Steps of Construction
Steps of Construction
(i) Take a point O as centre and draw a circle of radius 3 cm.
(i) Draw a circle of radius 4 cm. Let centre of this circle is
(ii) Draw diameter AOB.
O.
(iii) Take OA as base and construct ∠OAT = 90° at A.
(ii) Join OMʹ and bisect it. Let M be mid-point of OMʹ.
(iv) Draw a ray AT and produce TA to Tʹ to get the required
tangent TATʹ.
Similarly, we can draw a tangent at point B or any other P
point on the circle.
T

M
Mʹ O
4 cm
3 cm
B A 6 cm
O

3. Given, radius of circle = 5 cm


(iii) Taking M as centre and MO as radius, draw a circle to
Steps of Construction intersect circle (0, 4) at two points, P and Q.
(i) Draw a circle of radius 5 cm and take a point P on the circle.
(iv) Join PMʹ and QMʹ. PMʹ and QMʹ are the required
(ii) Draw a chord PQ through the point P on the circle. tangents from Mʹ to circle C ( 0, 4).
(iii) Take a point R in the major arc and join PR and RQ. The measure length of the tangents are 4.48 cm.
(iv) On taking PQ as base, construct ∠QPY equal to ∠PRQ on
6. Given, radius of circle = 1 cm and distance between point P
the opposite side of the point R.
and centre = 2.2 cm.
(v) Draw a ray PY and produce YP to X. Then, YPX is the
Steps of Construction
required tangent at point P.
(i) Draw a circle of radius 1 cm with centre O.
R
(ii) Take a point P outside it such that its distance from
centre O is 2.2 cm.
Q (iii) Take O and P as centre and draw arcs of radius more
X than half of OP on both sides of OP, which intersect
each other at R and S. Join RS, which bisects OP at M.
P Then, MP = MO.
Y
84 CBSE Term II Mathematics X (Standard)

R
Q M
N
5 cm 3c
O m
A 9 cm B
1.1 cm M 1 cm
P O
2.2 cm Q
P

9. Steps of Construction

S (i) First, draw a circle with the help of given circular solid
ring and then draw two non-parallel chords AB and CD.
(iv) On taking M as centre and MO as radius, draw a dotted T A
circle, which intersects given circle at Q and Qʹ.
(v) Join PQ and PQʹ. Thus, we get the required tangents D
drawn from point P to the given circle. P
M O B
7. Steps of Construction
(i) Draw a circle with O as centre and radius
OC = 3.5 cm. Take a point P such that OP = 7 cm. Tʹ C
(ii) Draw the bisector of OP, which intersects OP at M. (ii) Draw perpendicular bisectors of AB and CD, which
(iii) On taking M as centre and MO as radius, draw a dotted intersect each other at point O. Then, O is the centre of
circle. Let this circle cuts the given circle at A and B. the circle.
(iv) Join PA and PB. (iii) Now, take a point P outside the circle and join OP.
Thus, PA and PB are the required tangents. (iv) Draw bisector of OP. Let its mid-point be M.
(v) On taking M as centre and MP as radius, draw a dotted
circle which intersect the given circle at T and Tʹ.
A
(vi) Join PT and PTʹ.
Then, PT and PTʹ are the required pair of tangents drawn to
the circle from P.
Justification
7 cm
O P Join OT.
3.5 cm M C
Then, ∠PTO = 90° [angle in semi-circle of dotted circle]
This shows that OT ⊥ PT.
Also, OT is radius of given circle, so PT has to be a tangent of
given circle. Similarly, PTʹ is also a tangent of given circle.
B
10. Steps of Construction
(i) Take a point O on the plane of the paper and draw a circle
with centre O and radius OA = 4 cm.
8. Given, a line segment AB = 9 cm, two circles with centres A
(ii) At O construct radii OA and OB such that ∠AOB equal to
and B of radii 5 cm and 3 cm, respectively. 120° i.e. supplement of the angle between the tangents.
We have to construct two tangents to each circle from the (iii) Draw perpendiculars to OA and OB at A and B,
centre of the other circle. respectively. Suppose these perpendiculars intersect at P.
Steps of Construction Then, PA and PB are required tangents.
(i) Draw a line segment AB = 9 cm.
(ii) Draw a circle with centre A and radius 5 cm and another A
circle with centre B and radius 3 cm. 4 cm
(iii) Now, bisect AB. Let O be the mid-point of AB.
(iv) Take O as centre and AO as radius and draw a dotted 60° 120° O
P
circle, which intersects the two given circles at N , Q, M
and P.
(v) Join AN , AQ, BM and BP. These are the required tangents B
to each circle from the centre of the other circle.
CBSE Term II Mathematics X (Standard) 85

The distance between the centre of the circle and the point
of intersection of tangents is 8 cm. D
Justification
In quadrilateral OAPB, we have M

∠OAP = ∠OBP = 90°


and ∠AOB = 120°
∴ ∠OAP + ∠OBP + ∠AOB + ∠APB = 360° C O A
B
P
⇒ 90° + 90° + 120° + ∠APB = 360°
∴ ∠APB = 360° − ( 90° + 90° + 120° ) N
= 360° − 300° = 60°
(iii) Now, bisect BC and take its mid-point as O. Draw a
11. Given, two concentric circles of radii 2.8 cm and 1.8 cm with semi-circle with centre O and radius OB (or OC ).
common centre say, O.
(iv) Draw PD ⊥ BC, which intersects the semi-circle at D.
A (v) With centre P and radius PD draw two arcs, which
C1 C2 intersects the given circle at points M and N.
P M O (vi) Join PM and PN. Thus, PM and PN are the required
tangents to the given circle.
B 13. Given, a circle of radius 3 cm. We have to construct a pair of
tangents, which are inclined to each other at an angle of 45°.
Steps of Construction Steps of Construction
(i) Draw two circles with common centre O and radii (i) Draw a circle with O as centre and radius 3 cm.
2.8 cm and 1.8 cm, respectively. (ii) Draw any diameter POQ of this circle.
(ii) Take a point P on the outer circle and join OP. (iii) Draw the radius OR meets the circle at R such that
(iii) Draw bisector of OP. Let mid-point of OP be M. ∠QOR = 45°.
(iv) Taking M as centre and PM as radius, draw a dotted circle,
which intersects the inner circle at two points say A and B. P
(v) Join AP and BP. Then, AP and BP are required tangents.
3 cm
On measuring the lengths, we get PA = PB = 2.14 cm O
Calculation 45° Q
Join OA. Then, OA = 1.8 cm 45°
[radius of inner circle C1] E
N R
OP = 2.8 cm D
[radius of outer circle C 2]
and ∠PAO = 90° (iv) Draw PD ⊥ PQ and RE ⊥ OR ,
[Q angle in semi-circle of constructed circle] which intersects each other at point N.
So, in ΔPAO, by Pythagoras theorem, Then, NP and NR are the required tangents to the given
OP 2 = OA 2 + AP 2 circle inclined to each other at an angle of 45°.
⇒ (2.8)2 = (1.8)2 + AP 2 Justification
By construction, ∠OPN = 90° and OP is radius.
⇒ 7.84 = 3.24 + AP 2
∴ PN is a tangent to the circle.
⇒ AP 2 = 7.84 − 3.24 = 4.6
Similarly, NR is a tangent to the circle.
⇒ AP = 2.14 cm Now, ∠POR = 180° − 45°= 135°
[taking positive square root, as length cannot be negative] [QPOQ is a straight line and ∠QOR = 45°]
⇒ PA = PB = 2.14 cm In quadrilateral OPNR,
Hence, the length of tangents is 2.14 cm. ∠OPN = 90° , ∠POR = 135°
12. Given, a circle of radius 2.8 cm and we have to draw and ∠ORN = 90°
tangents without using the centre. ∴ ∠PNR = 360° − ( 90° + 135° + 90° ) = 45°
Steps of Construction 14. Given, ABC is a right angled triangle, in which AB = 6 cm,
(i) First, draw a circle of radius 2.8 cm and take a point P BC = 8 cm, ∠B = 90° and BD is perpendicular to AC.
outside the circle.
Then, ∠ADB = ∠CDB = 90°
(ii) Through P, draw a secant PAB, which intersects the
circle at A and B and extend it to C in opposite
direction of AB such that PC = PA.
86 CBSE Term II Mathematics X (Standard)

Steps of Construction We have to construct the tangents to the circle from the
(i) Draw the line segments AB = 6 cm and BC = 8 cm given points P and Q.
perpendicular to each other. Join AC. Thus, ΔABC is
the given right angled triangle. M Pʹ
(ii) Draw perpendicular bisector of BC, which meets BC
at O.
3 cm
(iii) With O as centre and OB as radius draw a circle, O
P Q
which intersects AC at D, then ∠BDC = 90°. Thus, E F
BD is perpendicular to AC. 7 cm 7 cm
(iv) With A as centre and AB as radius draw an arc,
cutting the circle at M. N Qʹ
(v) Join AM. Thus, AB and AM are required tangents.
X Steps of Construction
(i) Draw a circle of radius 3 cm with centre at O.
A (ii) Produce its diameter on both sides and take points P and Q
on it such that OP = OQ = 7 cm
D M
6 cm (iii) Draw bisector of OP and OQ, which intersect OP and OQ at
E and F, respectively.
B
(iv) Now, take E as centre and OE as radius, draw a dotted circle
O C which intersects the given circle at two points M, N. Again,
8 cm take F as centre and OF as radius, draw another dotted circle
which intersects the given circle at two points Pʹ and Qʹ.
(v) Join PM , PN , QPʹ and QQʹ. These are the required tangents
Justification from P and Q to the given circle.
Since ΔABC is right angled triangle with ∠ABC = 90° Justification
∴ BO ⊥ AB. Join OM and ON. The ∠OMP is the angle that lies in the
semi-circle of the dotted circle with centre E. Therefore,
Also, BO is the radius of circle. So, AB has to be tangent of
∠OMP = 90° ⇒ OM ⊥PM
the circle. Similarly, AM is also a tangent to the circle.
Since, OM is radius of the circle. So, MP has to be a tangent to
15. Given, two points P and Q on the extended diameter of a
the circle. Similarly, PN, QPʹ and QQʹ are also tangents to the
circle with radius 3 cm such that OP = OQ = 7 cm given circle.
Chapter Test
Multiple Choice Questions these two radii of the circle, the angle between
1. To divide a line segment AB in the ratio 2 : 5, two radii is
(a) 105°
first a ray AX is drawn, so that ∠BAX is an acute
(b) 70°
angle and then at equal distances, then the
(c) 125°
number of points located on the ray AX is (d) 135°
[CBSE 2011]
Short Answer Type Questions
(a) 7 (b) 10
(c) 2 (d) 5 5. Draw a line segment AB = 6. 5 cm and divide it
2. To divide a line segment AB in the ratio 7 : 5, internally in the ratio 3 : 5.
first a ray AX is drawn, so that ∠BAX is acute 6. Draw two tangents at the end points of the
angle and then at equal distance points are diameter of a circle of radius 3.5 cm. Are these
marked. Then, the minimum number of these tangents parallel?
points is
Long Answer Type Questions
(a) 5 (b) 35
(c) 7 (d) 12 7. Draw two concentric circles of radii 3 cm and
3. By geometrical construction, it is possible to 5 cm. Taking a point on outer circle construct
the pair of tangents to the other. Measure the
1
divide a line segment in the ratio 3 : . length of a tangent and verify it by actual
3 calculation.
(a) True (b) False
8. Draw a line segment AB of length 7 cm. Taking
(c) Can’t determined (d) None of these A as centre, draw a circle of radius 3 cm and
4. To draw a pair of tangents to a circle, which are taking B as centre, draw another circle of radius
inclined to each other at an angle of 55°, it is 2 cm. Construct tangents to each circle from
required to draw tangents at the end points of the centre of the other circle.

Answers
1. (a) 2. (d) 3. (a) 4. (c) 6. These are parallel. 7. 4 cm For Detailed Solutions
Scan the code
88 CBSE Term II Mathematics X (Standard)

CHAPTER 05

Applications of
Trigonometry

In this Chapter...
! Line of Sight & Horizontal Line
! Angle of Elevation
! Angle of Depression

Line of Sight Some Important Points


The line of sight is the line drawn from the eye of an observer to (i) The angle of elevation of a point P as seen from a
the point where the object is viewed by the observer. point O is always equal to the angle of depression of
O as seen from P.
Horizontal Line
P
Angle of
The line which goes parallel from eye to ground, is called depression
horizontal line. ht
f sig
o
Angle of Elevation e
Lin Angle of
The angle of elevation of an object viewed, is the angle formed by elevation
A
O
the line of sight with the horizontal, when it is above the horizontal Horizontal line
level, i.e. the case when we raise our head to look at the object.
(ii) The angles of elevation and depression are always
P(object)
acute angles.
ht
sig (iii) If the observer moves towards the perpendicular
of
e line (tower/building), then angle of elevation
Lin
Angle of increases and if the observer moves away from the
O elevation
Horizontal line
A perpendicular line (tower/building), then angle of
Eye elevation decreases.
(iv) If the height of tower is doubled and the distance
Angle of Depression between the observer and foot of the tower is also
The angle of depression of an object Horizontal line doubled, then the angle of elevation remains same.
O A
viewed, is the angle formed by the line Angle of (v) If the angle of elevation of Sun, above a tower
depression
of sight with the horizontal, when it is Lin decreases, then the length of shadow of a tower
e
below the horizontal level, i.e. the case of
sig increases and vice-versa.
when we lower our head to look at the ht
object. P(object)
CBSE Term II Mathematics X (Standard) 89

Solved Examples
Example 1. In figure, a tightly stretched rope of length In right angled ΔACB,
20 m is tied from the top of a vertical pole to the AB
tan 30° =
ground. Find the height of the pole, if the angle BC
made by the rope with the ground is 30°. 1 h
⇒ =
[CBSE 2020 (Standard)] 3 30
B 30 3
⇒ h= ×
3 3
20 m [multiply numerator and denominator by 3]
30 × 3
=
30º
3
A C = 10 3 m
Sol. Let AB = h be the height of the pole. Hence, height of the tower is 10 3 m.
Given, length of rope, BC = 20 m Example 3. The ratio of the length of a vertical rod and
B the length of its shadow is 1 : 3. Find the angle of
elevation of the Sun at that moment?
20 m [CBSE 2020 (Standard)]
hm
Sol. Let AB be the vertical rod and BC be its shadow and θ be
the angle of elevation of the Sun.
30º
A C A

In right angled ΔACB,


Vertical rod

Perpendicular
sin 30° =
Hypotenuse
1 h
∴ =
2 20
20 !
⇒ h== 10 m B C
2 Shadow
Hence, height of the pole is 10 m. We have, AB : BC =1 : 3
Example 2. In figure, the angle of elevation of the top Let AB = x, then BC = x 3
of a tower from a point C on the ground, which is AB
30 m away from the foot of the tower, is 30°. Find In ΔABC, tan θ =
BC
the height of the tower. [CBSE 2020 (Standard)] x 1
⇒ tan θ = =
A x 3 3
⇒ tan θ = tan 30°
⇒ θ = 30°

Example 4. A vertical tower stands on a horizontal


30°
plane and is surmounted by a vertical flag-staff of
C height 6 m. At a point on the plane, the angle of
B 30 m
elevation of the bottom and top of the flag-staff are
Sol. Let height of a tower be AB = h m 30° and 45°, respectively. Find the height of the
A tower. (take, 3 = 1.73) [CBSE 2020 (Standard)]
Sol. Let BC = h be the height of the tower, CD = 6 m be the
height of the flag-staff and A is any point on the ground.
h
Consider, AB = x m.
30° Given, the angle of elevation from point A to the points C
B 30 m C and D are ∠CAB = 30° and ∠DAB = 45°.
90 CBSE Term II Mathematics X (Standard)

D Also, let AB = x m.
In right angled ΔABD,
6m
Perpendicular BD
C tan 60° = =
Base AB
BC + CD
⇒ 3= [Q tan 60° = 3 ]
h x
h + 1. 6
⇒ 3=
30° 45° x
A xm B 3x = h + 1 . 6
In right angled ΔABC, ⇒ h = 3x − 1. 6 …(i)
BC
tan 30° = In right angled ΔCBA,
AB BC
1 h tan 45° =
⇒ = AB
3 x h
⇒ x= 3hm …(i) ⇒ 1= [Q tan 45° = 1]
x
Now, in right angled ΔABD, ⇒ x=h
BD
tan 45° = On putting x = h in Eq. (i), we get
AB
h = 3 h − 1. 6
6+ h
⇒ 1= [QBD = BC + CD = 6 + h ] ⇒ h ( 3 − 1) = 1. 6
x
⇒ x=6+ h …(ii) 1. 6 3+1
⇒ h= × [rationalising]
⇒ 3h = 6 + h [from Eq. (i)] ( 3 − 1) 3+1
⇒ h( 3 − 1) = 6 1 . 6( 3 + 1)
= [Q ( a + b )( a − b ) = a 2 − b 2 ]
6 3 +1 ( 3 ) 2 − (1 ) 2
⇒ h= × (rationalisation)
( 3 − 1) 3+ 1 1 .6
= ( 3 + 1)
6( 3 + 1) 2
= [Q( a − b )( a + b ) = a 2 − b 2]
( 3 ) 2 − (1 ) 2 = 0. 8 (1.73 + 1)
6(1.73 + 1) 6 × 2.73 = 0.8 (2.73)= 2.184 m
= =
3 −1 2 Hence, the height of the pedestal is 2.184 m.
= 3 × 2.73 = 8.19 m Example 6. From a point on the ground, the angles of
Hence, height of tower is 8.19 m. elevation of the bottom and the top of a transmission
Example 5. A statue 1.6 m tall, stands on the top of a tower fixed at the top of a 20 m high building are
pedestal. From a point on the ground, the angle of 45° and 60°, respectively. Find the height of the
elevation of the top of the statue is 60° and from the tower. (use 3 = 1.73 )
same point the angle of elevation of the top of the Sol. Let AB = 20 m be the height of the building and BC = h m be
pedestal is 45°. Find the height of the pedestal. the height of transmission tower. The angles of elevation
(use 3 = 1.73) from a ground point D to the points B and C are
Sol. Let BC = h m be the height of the pedestal and CD = 1.6 m ∠ADB = 45° and ∠ADC = 60°
be the length of the statue, which is standing on the C
pedestal.
hm
D
B

C 20 m

45° 60°
h A
D

A In right angled ΔADB,


xm B AB
tan 45° =
Again, let point A be a fixed point on the ground such that AD
the angles of elevation of the top of the statue and bottom of 20
⇒ 1=
the statue (i.e. top of the pedestal) are AD
∠DAB = 60° and ∠CAB = 45°. ⇒ AD = 20 m …(i)
CBSE Term II Mathematics X (Standard) 91

Now, in right angled ΔADC, ⇒ h = 25 3 m


AC Hence, height of the pole is 25 3 m and distances of the
tan 60° =
AD point from the poles are 25 m and 75 m.
AB + BC
⇒ 3=
20
[Q from Eq. (i)] Example 8. The angle of elevation of the top of a
building from the foot of a tower is 30° and the
⇒ 20 3 = 20 + h
angle of elevation of the top of a tower from the foot
⇒ h = 20( 3 − 1) = 20(1.73 − 1) of the building is 60°. If the tower is 50 m high,
= 20 × 0.73 = 14. 60 m then find the height of the building.
Hence, height of the transmission tower is 14.6 m. Sol. Let AB = 50 m, CD = h be the height of the tower and
building. Then,
Example 7. Two poles of equal heights are standing B
opposite to each other on either side of the road,
which is 100 m wide. From a point between them D
on the road, the angles of elevation of the top of the 50 m
poles are 60° and 30°, respectively. Find the height hm
of the poles and the distance of the point from the
poles. [CBSE 2020 (Standard)] 30º 60º
A C
Sol. Let AB = 100 m be the width of the road. On both sides of
∠CAD = 30° and ∠ACB = 60°
the road, poles AE = BD = h m are standing. Let C be any
point on AB such that from point C, angles of elevation are In right angled ΔACD ,
∠BCD = 60° and ∠ACE = 30° CD 1 h
tan 30° = ⇒ =
E D AC 3 AC
⇒ AC = 3 h …(i)
h hm In right angled ΔCAB,
AB
tan 60° =
30º 60º AC
A (100 – x) m C xm B 50
100 m ⇒ 3=
AC
Let BC = x m , then AC = AB − BC = (100 − x ) m 50
⇒ 3= [from Eq. (i)]
In right angled ΔCAE, 3h
Perpendicular 50
tan 30° = ⇒ h= = 16.67 m
Base 3
1 AE 1 h
∴ = ⇒ = Hence, the height of the building is 16.67 m.
3 AC 3 (100 − x )
(100 − x )
Example 9. From the top of a 7 m high building the
⇒ h= angle of elevation of the top of a tower is 60° and
3
the angle of depression of its foot is 45°. Determine
⇒ h 3 = 100 − x …(i)
the height of the tower. [CBSE 2020 (Standard)]
and in right angled ΔBCD,
Sol. Let AB = 7 m be the height of the building and EC be the
BD
tan 60° = height of tower.
BC
h A is the point from where elevation of tower is 60° and the
⇒ 3= angle of depression of its foot is 45°.
x
Here, EC = DE + CD
⇒ h = 3x …(ii)
Also, CD = AB = 7 m
Put h = 3x in Eq. (i), we get and BC = AD
3x × 3 = 100 − x E
⇒ 3x + x = 100 ⇒ 4x = 100
100
⇒ x= = 25 m
4 60°
A D
∴ BC = 25 m and AC = 100 − x = 100 − 25 = 75 m 45°
Put x = 25 in Eq. (i), we get
7m

⇒ h 3 = 100 − 25
75 3 75 3
⇒ h= × = B 45° C
3 3 3
92 CBSE Term II Mathematics X (Standard)

Now, in right angled ΔABC, Cʹ B + BM


⇒ 3=
AB PM
tan 45° = 10 + h + 10
BC ⇒ 3= [from Eq. (i)]
3h
7
⇒ 1= ⇒ 3h = 20 + h
BC
⇒ 2 h = 20
⇒ BC = 7 m ⇒ h = 10 m
Also, in right angled ΔADE, Now, the height of the cloud from the surface of lake
DE =BC
tan 60° = = BM + h
AD
= 10 + 10 = 20 m
DE
⇒ 3= [Q AD = BC ]
7 Example 11. From a point on a bridge across a river,
⇒ DE = 7 3 m the angles of depression of the banks on opposite
sides of the river are 30° and 45°, respectively. If
∴ Height of the tower,
the bridge is at a height of 30 m from sea level, then
EC = DE + CD
find the width of the river. (use 3 = 1 . 73)
= (7 3 + 7 ) m
Sol. Let A be a point on the bridge and points B and D are on the
= 7( 3 + 1) m opposite side of the banks. Then, angles of depression from
Hence, height of the tower is 7( 3 + 1) m. point A to the opposite banks are
∠EAB = 30° and ∠FAD = 45°
Example 10. If the angle of elevation of a cloud from a ⇒ ∠CBA = 30° and ∠CDA = 45°
point 10 m above a lake is 30° and the angle of [Q alternate angles are equal]
depression of its reflection in the lake is 60°, find
A F
the height of the cloud from the surface of lake. E
30º 45º Bridge
[CBSE 2020 (Standard)]
Sol. Let AB be the surface of the lake and P be the point of
observation such that AP = 10 m. Let C be the position of the 30 m
cloud and Cʹ be the reflection in the lake, then CB = Cʹ B.
C
30º 45º
B C D
h (River bank) (Opposite
river bank)
P 30° M In right angled ΔACB,
60°
10 m AC
10 m tan 30° =
BC
A B 1 30
⇒ =
3 BC
(10 +h)m ⇒ BC = 30 3 m
and in right angled ΔACD,
AC
Cʹ tan 45° =
CD
Let CM = h , then CB = 10 + h AC
⇒ Cʹ B = 10 + h ⇒ 1=
30
In right angled ΔCMP, ⇒ AC = 30 m
CM
tan 30° = Hence, width of the river is
PM
1 h BD = BC + CD
⇒ = = 30 3 + 30
3 PM
⇒ PM = 3h ...(i) = 30 × 1.73 + 30
In right angled ΔPMCʹ, = 51.9 + 30
Cʹ M = 81.9
tan 60° =
PM Hence, width of the river is 81.9 m.
CBSE Term II Mathematics X (Standard) 93

Example 12. From the top of a 7 m building, the angle Given, speed of plane is 720 km/h and time of flight is 10 s.
of elevation of the top of a cable tower is 60° and Also, given ∠AOC = 60° and ∠BOD = 30°
the angle of depression of its foot is 45°. Determine In right angled ΔOCA,
the height of the tower. (use 3 = 1.73). OC
cot 60° =
[CBSE 2020 (Standard)] AC
Sol. Let AB = 7 m be the height of the building and DE = h m 1 OC
⇒ =
be the height of cable tower. 3 h
E h
⇒ OC = m
3
In right angled ΔODB,
OD
60º cot 30° =
B C BD
45º hm OD
⇒ 3=
h
7m ⇒ OD = h 3
Now, CD = OD − OC
h 2h
A
45º
D
=h 3− = m
xm 3 3
2h
Then, ∠CBE = 60° Thus, distance covered by aeroplane in 10 s is m.
3
and ∠CBD = 45° ⇒ ∠ADB = 45° (alternate angle)
Distance
Let distance between two towers be Q Speed of aeroplane =
AD = BC = x m Time
CE 2h
In right angled ΔBCE, tan 60° = 5 ⎡ 5 ⎤
BC ∴ 720 × = 3 Q 1 km = m /s
CE CE 18 10 ⎣⎢ 18 ⎥⎦
⇒ 3= ⇒x = … (i)
x 3 ⇒ 40 × 5 × 10 3 = 2 h
and in right angled ΔADB, ⇒ h = 1000 3 m
AB 7
tan 45° = ⇒1 = ⇒ x = 7 m Hence, height at which the aeroplane is flying is 1000 3 m.
AD x
Put x = 7 in Eq. (i), we get Example 14. A straight highway leads to the foot of a
7=
CE
⇒ CE = 7 3 m
tower. A man standing at the top of the tower
3 observes a car at an angle of depression of 30°,
Now, height of cable tower, which is approaching the foot of the tower with a
h = DC + CE uniform speed. After covering a distance of 50 m,
= 7 + 7 3 = 7 (1 + 3 ) the angle of depression of the car becomes 60°.
= 7(1 + 1.73) = 7 × 2 . 73 = 19.11 m Find the height of the tower. (use 3 = 1.73).
Example 13. The angle of elevation of an aeroplane [CBSE 2020 (Standard)]
from point O on the ground is 60°. After a flight of Sol. Let AB = h m be the height of the tower. Let C be the initial
10 s, on the same height, the angle of elevation position of the car and D be the final position of the car, when
from point O becomes 30°. If the aeroplane is flying it covers a distance, CD = 50 m .
at the speed of 720 km/h, find the constant height at E B
which the aeroplane is flying. [CBSE 2020 (Standard)] 30°
60°
Sol. Let OX be the horizontal ground, A and B be the two
positions of the plane and O be the points of observation. h
Let height of an aeroplane from A to the ground is
AC = BD = h m
30° 60°
A B A
D C
50 m x
h
h Given, the angle of depressions from point B to the points C
and D are
∠EBC = 60° and ∠EBD = 30°
30º
60º ⇒ ∠BCA = 60° and ∠BDA = 30° [alternate angles]
X
O C D
94 CBSE Term II Mathematics X (Standard)

In right angled ΔCAB, A F


45º
AB 60º
tan 60° =
AC
h
⇒ 3=

60 "3 m
x 45º
h B x E
⇒ x= m …(i)
3
And in right angled ΔDAB, hm
AB
tan 30° =
AD 60º
1 h C x D
⇒ =
3 50 + x Given, the angle of depressions from point A are
⇒ 50 + x = h 3 ∠ FAE = 45° and ∠FAD = 60°
⇒ x = h 3 − 50 …(ii) ⇒ ∠BEA = 45° and ∠CDA = 60° [alternate angles]
From Eqs. (i) and (ii), we have In right angled ΔAEB,
h AB AB
= h 3 − 50 tan 45° = ⇒ 1=
3 BE x
⇒ x = AB …(i)
⇒ h = 3h − 50 3
and in right angled ΔADC,
⇒ 2 h = 50 3
AC 60 3 60 3
⇒ h = 25 3 tan 60° = ⇒ 3= ⇒ x=
CD x 3
⇒ h = 25 × 1 .73 ⇒ x = 60 m …(ii)
⇒ h = 43. 25 m ∴ From Eqs. (i) and (ii), we get
Hence, height of the tower is 43.25 m. AB = x = 60 m
Example 15. The angles of depression of the top and Now height of cliff,
bottom of a tower as seen from the top of a 60 3 m h = AC − AB
high cliff are 45° and 60°, respectively. Find the = 60 3 − 60
height of the tower. (use 3 = 1.73) = 60( 3 − 1)
Sol. Let AC = 60 3 m be the height of the cliff, DE = h m be the = 60( 1.73 − 1)
height of the tower and distance between tower and cliff be = 60 × 0.73 = 43.8 m
CD = BE = x m. Hence, height of the cliff is 43.8 m.
CBSE Term II Mathematics X (Standard) 95

Chapter
Practice
7. A ladder, leaning against a wall, makes an angle of
PART 1 60° with the horizontal. If the foot of the ladder is
9.5 m away from the wall. The length of the ladder is
Objective Questions [NCERT Exemplar]
(a) 10 m (b) 16 m
!
Multiple Choice Questions (c) 18 m (d) 19 m
8. A ramp for disabled people in a hospital have slope
1. The angle of elevation of the Sun when the shadow
30°. If the height of the ramp be 1 m, then the
of a pole h m high is 3 h m long is
length of ramp is
(a) 0° (b) 30° (c) 45° (d) 60° (a) 2 m (b) 0.5 m
2. If a pole 6 m high casts a shadow 2 3 m long on the (c) 2 3 m (d) 1 m
ground, then the Sun’s elevation is 9. A kite is flying at a height of 80 m above the
(a) 60° (b) 45° ground. The string attached to the kite is
(c) 30° (d) 90° temporarily tied to a point on the ground. The
3. If 300 3 m high tower makes an angle of elevation inclination of the string with ground is 60°, then the
at a point on ground which is 300 m away from its length of the string is
foot, then the angle of elevation is (a) 62.37 m (b) 92.37 m
(a) 0° (b) 30° (c) 52.57 m (d) 72.57 m
(c) 45° (d) 60°
10. The length of a string between a kite and a point on
4. From the top of a 60 m high tower, the angle of the ground is 85 m. If the string makes an angle θ
depression of a point on the ground is 30°. The 15
with the ground level such that tan θ = , then the
distance of the point from the foot of tower is 8
(a) 180 m (b) 60 3 m height of kite is
(c) 150 m (d) 30 3 m (a) 75 m (b) 78.05 m
5. The figure shows the observation of point C from (c) 226 m (d) None of these
point A. The angle of depression from A is 11. A tower stands near an airport. The angle of
[CBSE 2013] elevation θ of the tower from a point on the ground
D A is such that its tangent is 5/12. The height of the
tower, if the distance of the observer from the tower
is 120 m is [CBSE 2015]
4m
(a) 40 m (b) 50 m
(c) 60 m (d) 70 m
C B
4√3 m 12. The top of two poles of height 20 m and
(a) 30° (b) 45° (c) 60° (d) 90° 14 m are connected by a wire. If the wire makes an
angle of 30° with the horizontal, then the length of
6. A circus artist is climbing a 20 m long rope, which
the wire is
is tightly stretched and tied from the top of a
(a) 12 m (b) 10 m (c) 8 m (d) 6 m
vertical pole to the ground, then the height of pole,
if the angle made by the rope with the ground level 13. An observer, 1.5 m tall is 20.5 m away from a tower
is 30°, is 22 m high, then the angle of elevation of the top of
(a) 5 m (b) 10 m the tower from the eye of the observer is
(c) 15 m (d) 20 m (a) 30° (b) 45° (c) 60° (d) 90°
96 CBSE Term II Mathematics X (Standard)

14. The angle of elevation of the top of the tower from a (ii) Value of DF is equal to
point, which is 40 m away from the base of the h h
(a) m (b) h 3 m (c) m (d) h m
tower in the horizontal level, is 45°. Find the height 3 2
of the tower. (iii) Value of h is
(a) 70 m (b) 60 m (a) 2 (b) 3( 3 + 1)
(c) 40 m (d) 30 m
(c) 4 (d) 3( 3 − 1)
15. The angle of elevation of the top of a building
(iv) Height of the Parachute from the ground is
150 m high, from a point on the ground is 45°. The
(a) 4 m (b) 3( 4 − 3 )
distance of the point from foot of the building is
(a) 120 m (b) 130 m (c) 8 m (d) 3( 4 + 3 )
(c) 140 m (d) 150 m (v) If the Parachute is moving towards the building,
16. The angle of depression of the car parked on the then both angles of elevation will
road from the top of a 150 m high tower is 30°. The (a) remain same (b) increases
distance of the car from the tower is [CBSE 2014] (c) decreases (d) Can’t be determined
150 20. A cyclist is climbing through a 20 m long rope
(a) 150 m (b) 75 m (c) 150 3 m (d) m
3 which is highly stretched and tied from the top of a
17. From a point on the ground, the angles of elevation vertical pole to the ground as shown below
of the bottom and the top of a transmission tower
fixed at the top of a 20 m high building are 45° and
60° respectively, then the height of the tower is
(a) 14.64 m (b) 28.64 m
(c) 38.64 m (d) 19.64 m
18. A bridge on a river makes an angle of 45° with its
edge. If the length along the bridge from one edge
Based on the above information, answer the
to the other is 150 m, then the width of the river is
following questions.
(a) 107.75 m (b) 105 m
(c) 75 m (d) 106.05 m (i) The height of the pole, if angle made by rope with
the ground level is 60°, is
!
Case Based MCQs (a) 15 m (b) 10 3 m
10 15
19. There are two balcony in a house. First balcony is (c) m (d) m
3 2
at a height of 3 m above the ground and other
balcony is 6 m vertically above the lower balcony. (ii) If the angle made by the rope with the ground
level is 60°, then the distance between artist and
Ankit and Radha are sitting inside the two balcony
pole at ground level is
at points G and F, respectively. At any instant, the 10
angles of elevation of a Parachute from these (a) m (b) 10 2 m
2
balcony are observed to be 60° and 45° as shown
(c) 10 m (d) 10 3 m
below
E (iii) If the angle made by the rope with the ground
level is 45°. The height of the pole is
hm (a) 2.5 m (b) 10 m
F 45° D (c) 7.5 m (d) 10 2 m
6m
G 60° C (iv) If the angle made by the rope with the ground
3m level is 45° and 3 m rope is broken, then the height
A B of the pole is
17
Based on the above information, answer the (a) m (b) 7 m
2
following questions.
(c) 14 m (d) 7 2 m
(i) Who is more closer to the Parachute.
(a) Ankit
(v) Which mathematical concept is used here?
(b) Radha (a) Similar triangles
(c) Both are at equal distance (b) Pythagoras theorem
(d) Can’t be determined (c) Application of trigonometry
(d) None of the above
CBSE Term II Mathematics X (Standard) 97

21. A group of students of class X visited India Gate on respectively. If the distance between the peaks of
an educational trip. The teacher and students had two mountains is 1937 km, and the satellite is
interest in history as well. The teacher narrated that vertically above the mid-point of the distance
India Gate, official name Delhi Memorial, between the two mountains. (use 3 = 1.73)
originally called All-India War Memorial,
monumental sandstone arch in New Delhi, F
dedicated to the troops of British India who died in A
G
wars fought between 1914 and 1919.The teacher
also said that India Gate, which is located at the
H P
eastern end of the Rajpath (formerly called the Q R
Kingsway), is about 138 feet (42 m) in height. B D C I S
Nanda devi Mullayanagiri
(i) The distance of the satellite from the top of Nanda
Devi is
(a) 1139.4 km (b) 1119.65 km
(c) 1937 km (d) 1025.36 km
(ii) The distance of the satellite from the top of
Mullayanagiri is
(a) 1139.4 km (b) 577.52 km
(c) 1937 km (d) 1025.36 km
(i) What is the angle of elevation if they are standing
(iii) The distance of the satellite from the ground is
at a distance of 42 m away from the monument?
(a) 1139.4 km (b) 567.64 km
(a) 30° (b) 45°
(c) 1937 km (d) 1025.36 km
(c) 60° (d) 0°
(iv) What is the angle of elevation, if a man is standing
(ii) They want to see the tower at an angle of 60°. So,
at a distance of 7816 m from Nanda Devi?
they want to know the distance where they should
(a) 30° (b) 45°
stand and hence find the distance.
(c) 60° (d) 0°
(a) 25.24 m (b) 20.12 m
(c) 42 m (d) 24.24 m (v) If a mile stone very far away, makes 45° to the top
of Mullayanagiri mountain. Hence, find the
(iii) If the altitude of the Sun is at 60°, then the height
distance of this mile stone from the mountain.
of the vertical tower that will cast a shadow of
(a) 1118.327 m (b) 566.976 m
length 20 m is
20 (c) 1930 m (d) 1025.36 m
(a) 20 3 m (b) m
3
(c)
15
3
m (d) 15 3 m PART 2
(iv) The ratio of the length of a rod and its shadow is Subjective Questions
1 : 1. The angle of elevation of the Sun is
(a) 30° (b) 45° !
Short Answer Type Questions
(c) 60° (d) 90°
(v) The angle formed by the line of sight with the 1. If the height of a tower and the distance of the
horizontal when the object viewed is below the point of observation from its foot, both are
horizontal level is increased by 10%, then the angle of elevation of its
(a) corresponding angle top remains unchanged. Explain.
(b) angle of elevation 2. A straight tree is broken due to thunderstorm. The
(c) angle of depression broken part is bent in such a way that the peak of
(d) complete angle the tree touches the ground at an angle of 60° at a
22. A Satellite flying at height h is watching the top of distance of 2 3 m. Find the whole height of the tree.
the two tallest mountains in Uttarakhand and 3. Determine the height of a mountain, if the
Karnataka ,they being Nanda Devi (height 7,816m) elevation of its top at an unknown distance from the
and Mullayanagiri (height 1,930 m). The angles of base is 30° and at a distance 10 km farther off from
depression from the satellite to the top of Nanda the mountain, along the same line, the angle of
Devi and Mullayanagiri are 30° and 60°, elevation is 15°. (take tan 15° = 0.27)
98 CBSE Term II Mathematics X (Standard)

4. There is a flag staff on a tower of height 20 m. At a 15. The angle of elevation of an aeroplane from a point
point on the ground, the angles of elevation of the on the ground is 45°. After flying for 15 s, the angle
foot and top of the flag are 45° and 60°, respectively. of elevation changes to 30°. If the aeroplane is
Find the height of the flag staff. flying at a constant height of 2500 m, then find the
5. If the length of the shadow of a tower is increasing, average speed of the aeroplane. [CBSE 2013]
then the angle of elevation of the Sun is also 16. The shadow of a flag staff is three times as long as the
increasing. Why or why not? shadow of the flag staff, when the Sun rays meet the
6. A window in a building is at a height of 10 m from ground at an angle of 60°. Find the angle between the
the ground. The angle of depression of a point P on Sun rays and the ground at the time of longer shadow.
the ground from the window is 30°. The angle of 17. An aeroplane, when flying at a height of 4000 m
elevation of the top of the building from the point P from the ground, passes vertically above another
is 60°. Find the height of the building. [CBSE 2007] aeroplane at an instant when the angles of elevation
7. A player sitting on the top of a tower of height 20 m of two planes from the same point on the ground
observes the angle of depression of a ball lying on are 60° and 45°, respectively. Find the vertical
the ground as 60°. Find the distance between the distance between the aeroplanes at that instant.
foot of the tower and the ball. 18. There is a small island in the middle of a 100 m
8. If two towers of height x m and y m subtend angles wide river and a tall tree stands on the island. P and
of 30° and 60°, respectively at the centre of the line Q are points directly opposite to each other on two
joining their feet, then find x : y. [CBSE 2015]
banks and in line with the tree. If the angles of
elevation of the top of the tree from P and Q are
9. If a man standing on a platform 3 m above the respectively 30° and 45°, then find the height of the
surface of a lake observes a cloud and its reflection
in the lake, then the angle of elevation of the cloud tree. [take, 3 = 1.732]
is equal to the angle of depression of its reflection.
State true or false. Justify.
!
Long Answer Type Questions
10. From the top of a hill, the angles of depression of 19. An aeroplane is at an altitude of 1200 m. If two
two consecutive kilometre stones due East are ships are sailing towards it in the same direction.
found to be 30° and 45°. Find the height of the hill. The angles of depression of the ships as observed
[CBSE 2015] from the aeroplane are 60° and 30°, respectively.
11. The shadow of a tower is 30 m long, when the Sun’s Find the distance between both ships.
angle of elevation is 30°. What is the length of the 20. The angles of depression of two consecutive kilometre
shadow, when Sun’s elevation is 60°? stones on the road on right and left of an aeroplane
12. Two ships are there in the sea on either side of a are 60° and 45°, respectively as observed from the
light house in such a way that the ships and the base aeroplane. Find the height of the aeroplane.
of the light house are in the same straight line. 21. The angle of elevation of the top of a tower at a
The angle of depression of two ships as observed distance of 120 m from a point A on the ground
from the top of the light house are 60° and 45°. is 45°. If the angle of elevation of the top of a flag
If the height of the light house is 200 m, then find staff fixed at the top of the tower, at A is 60°, then
the distance between the two ships. [CBSE 2014] find the height of the flag staff. [use, 3 =1.73]
13. From the top of a tower of height 50 m, the angles [CBSE 2014]
of depression of the top and bottom of a pole are 30° 22. A balloon is connected to an electric pole. It is
and 45°, respectively. Find [CBSE 2015] inclined at 60° to the horizontal by a cable of length
(i) how far the pole is from the bottom of the tower. 215 m. Determine the height of the balloon from
(ii) the height of the pole. [ take, 3 = 1.732 ] the ground. Also, find the height of the balloon, if
the angle of inclination is changed from 60° to 30°.
14. A man standing on the deck of a ship, which is 10 m [CBSE 2015]
above the water level. He observes that the angle of
23. A man in a boat rowing away from a light house 100
elevation of the top of a hill is 60° and the angle of
m high takes 2 min to change the angle of elevation of
depression of the base of the hill is 30°. Calculate
the light house from 60° to 45°. Find the speed of
the distance of the hill from the ship and height of
boat.
the hill. [CBSE 2016]
CBSE Term II Mathematics X (Standard) 99

24. The angle of elevation of the top of a tower from the opposite side of the lane are found to be α and β,
certain point is 30°. If the observer moves 20 m respectively. Prove that the height of the other house
towards the tower, the angle of elevation of the top is h(1 + tan α cot β) m.
increases by 15°. Find the height of the tower. 32. The lower window of a house is at a height of 2 m
25. The shadow of a tower standing on a level plane is above the ground and its upper window is 4 m
found to be 50 m longer when Sun’s elevation is 30° vertically above the lower window. At any instant the
than when it is 60°. Find the height of the tower. angles of elevation of a balloon from these windows
26. A vertical tower stands on a horizontal plane and is are observed to be 60° and 30°, respectively. Find
surmounted by a vertical flag staff of height h. At a the height of the balloon above the ground.
point on the plane, the angles of elevation of the !
Case Based Questions
bottom and the top of the flag staff are α and β
respectively. Prove that the height of the tower is 33. A girl 8 m tall spots a parrot sitting on the top of a
building of height 58 m from the ground. The angle
⎛ h tan α ⎞
⎜ ⎟. of elevation of the parrot from the eyes of girl at
⎝ tan β − tan α ⎠ any instant is 60°. The parrot flies away
27. The angle of elevation of the top of a tower 30 m horizontally in such a way that it remained at a
high from the foot of another tower in the same constant height from the ground. After 8 s, the
plane is 60° and the angle of elevation of the top of angle of elevation of the parrot from the same point
the second tower from the foot of the first tower is is 30°.
30°. Find the distance between the two towers and
also the height of the tower.
28. From the top of a tower h m high, angles of
depression of two objects, which are in line with the
foot of the tower are α and β (β > α). Find the
58 m

distance between the two objects.


29. A ladder against a vertical wall at an inclination α to 60°
the horizontal. Its foot is pulled away from the wall 30° 8m
through a distance p, so that its upper end slides a
distance q down the wall and then the ladder makes Based on the above information, answer the
an angle β with the horizontal. Show that following questions. (Take 3 = 1.73)
p cos β − cos α
= . (i) Find the distance of first position of the parrot
q sin α − sin β from the eyes of the girl.
30. The angle of elevation of the top of a vertical tower (ii) If the distance between the position of parrot
from a point on the ground is 60°. From another increases, then the angle of elevation decreases.
point 10 m vertically above the first, its angle of Justify with girl.
elevation is 45°. Find the height of the tower. (iii) Find the distance between the girl and the building.
31. A window of a house is h m above the ground. From (iv) How much distance covers parrot covers?
the window, the angles of elevation and depression of (v) Find the speed of the parrot in 8s.
the top and the bottom of another house situated on
100 CBSE Term II Mathematics X (Standard)

SOLUTIONS
Objective Questions 4. (b) Let the distance of the foot of tower from the point be
1. (b) Let the angle of elevation of the Sun is θ. QR = x m.
P
A
30°

60 m
h
30°
Q x R
θ
B C
√3h and height PR = 60 m and ∠PQR = 30°
Given, height of pole = h PR 60
In ΔPRQ, tan 30° = =
AC h QR x
Now, in ΔABC, tan θ = =
BC 3h 1 60
⇒ = ⇒ x = 60 3 m
1 3 x
⇒ tan θ = = tan 30°
3 5. (a) In right angled ΔABC, ∠B = 90°
⇒ θ = 30° D A
Hence, the angle of elevation of the Sun is 30°. θ

2. (a) Let BC = 6 m be the height of the pole and AB = 2 3 m


4m
be the length of the shadow on the ground.
Let the Sun makes an angle θ on the ground. C B
4√3m
C Sun
Let ∠DAC = θ
Then, ∠ACB = ∠DAC = θ [alternate angles]
6m Perpendicular AB 4 1
Now, tan θ = = = = = tan 30°
Base BC 4 3 3
θ
A B ∴ The angle of depression from A is 30°.
2 √3 m
6. (b) Let AB be the vertical pole and CA be the 20 m long rope
BC such that its one end A is tied from the top of the vertical pole
Now, in ΔBAC, tan θ =
AB AB and the other end C is tied to a point C on the ground.
6 3 3 A
⇒ tan θ = = ⋅ [by rationalising]
2 3 3 3
3 3
⇒ tan θ = = 3 = tan 60°[Q tan 60° = 3 ] m
3 20
∴ θ = 60°
Hence, the Sun’s elevation is 60°.
30°
3. (d) Let AB be the tower whose height is 300 3 m,
C B
i.e. AB = 300 3 m. Again, let C be the point at a distance of
300 m from the foot of the tower, i.e. AC = 300 m. In ΔABC, we have
Here, the angle of elevation is unknown, so let it be θ. AB
sin 30° =
Since, here base and perpendicular are given. AC
B 1 AB
⇒ =
2 20
⇒ AB = 10 m
300√3 m
Hence, the height of the pole is 10 m.
7. (d) Let the length of ladder AB = h m.
θ
A C B
300 m
So, in right angled ΔBAC,
Perpendicular AB 300 3
tan θ = = = h
Base AC 300
⇒ tan θ = 3 = tan 60°
∴ θ = 60° 60°
Hence, the required angle of elevation is 60°. A 9.5 m C
CBSE Term II Mathematics X (Standard) 101

Here, AC = 9. 5 m, ∠BAC = 60° 15


⇒ sin θ =
AC 1 9.5 17
In ΔABC, cos 60° = ⇒ = B
AB 2 h
⇒ h = 2 × 9.5 = 19 m
Hence, length of ladder is 19 m.

m
85
8. (a) Let XZ be the length and YZ be the height of the ramp.
Z

θ
A C
1m
BC
In ΔABC, sin θ =
30° AB
X Y 15 BC
⇒ =
Then, ∠ZXY = 30° and YZ = 1 m 17 85
In right angled ΔXYZ, ⇒ BC = 75 m
Perpendicular YZ Hence, height of kite is 75 m
sin 30° = =
Hypotenuse XZ 11. (b) Let BC = h m be the height of the tower and A be the
1 1 point on the ground such that, ∠BAC = θ.
⇒ = ⇒ XZ = 2 m
2 XZ C
Hence, the length of ramp is 2 m.
9. (b) Let C be the position of the kite and AC be the length of
hm
the string which makes an angle of 60° with the ground. The
height of the kite from the ground is BC = 80 m.
C θ
A 120 m B
Then, AB = 120 m
80 m In right angled ΔABC,
Perpendicular BC h
tan θ = = =
60° Base AB 120
A B 5 h ⎡ 5 ⎤
⇒ = Q tan θ = , given
12 120 ⎢⎣ 12 ⎥⎦
In right angled ΔABC,
Perpendicular BC ⇒ h = 50 m
sin 60° = =
Hypotenuse AC Hence, height of the tower is 50 m.
3 80 12. (a) Here, CD = 20 m [height of big pole]
⇒ =
2 AC AB = 14 m [height of small pole]
80 × 2 3 D
⇒ AC = × [by rationalising]
3 3
160 3 160 × 1.732 6m
= = = 92. 37 m
3 3
Hence, the length of the string is 92.37 m. B 30°
x E 20 m
10. (a) Given, length of the string of the kite,
AB = 85 m 14 m 14 m
15
and tan θ =
8
8 A C
⇒ cotθ =
15 ∴ DE = CD − CE
Q cosec θ − 1 = cot 2 θ
2
⇒ DE = CD − AB [Q AB = CE ]
64 ⇒ DE = 20 − 14 = 6 m
∴ cosec2θ − 1 =
225 DE
64 289 In ΔBDE, sin 30° =
⇒ cosec2θ = 1 + = BD
225 225 1 6
⇒ = ⇒ BD = 12 m
289 17 2 BD
⇒ cosec θ = =
225 15 ∴ Length of wire = 12 m
102 CBSE Term II Mathematics X (Standard)

13. (b) Let BE = 22 m be the height of the tower and 150


⇒ 1=⇒ BC = 150 m
AD = 1. 5 m be the height of the observer. The point D be BC
the observer’s eye. Draw DC || AB. Hence, the distance of the point from foot of the building is
E 150 m.
16. (c) Let AB =150 m be the height of the tower and angle of
depression is ∠DAC = 30°.

20.5 m
A

22 m
D 30º

θ 150 m
D C
20.5 m
1.5 m
30º
A B
20.5 m C B

Then, AB = 20. 5 m = DC Then, ∠ACB = ∠DAC = 30° [alternate angles]


and EC = BE − BC = BE − AD Now, in right angled ΔABC,
= 22 − 1. 5 = 20. 5 m [Q BC = AD ] Perpendicular AB
tan 30° = =
Let θ be the angle of elevation made by observer’s eye with Base BC
the top of the tower i.e. ∠EDC = θ. 1 150
⇒ =
In right angled ΔDCE, 3 BC
Perpendicular CE 20. 5 ⇒ BC = 150 3 m
tan θ = = =
Base DC 20. 5 Hence, the distance of the car from the tower is 150 3 m.
⇒ tan θ = 1 17. (a) Let the height of the building be, BC = 20 m
⇒ tan θ = tan 45° and height of the tower be CD = x m
⇒ θ = 45° D
14. (c) In the figure, let AB be a tower which has height h m.
The angle of elevation from point C at a distance of 40 m xm
from point B is ∠ACB = 45° and BC = 40 m.
A C

20 m
hm
45°
60°
45° A ym B
C B
40 m Let the point A be at a distance y m
Then, in right angled ΔABC, from the foot of the building.
Perpendicular AB Now, in ΔABC,
tan 45° = = BC
Base BC = tan 45° = 1
h h AB
⇒ tan 45° = ⇒ 1= [Q tan 45° = 1] 20
40 40 ⇒ =1
∴ h = 40 m y
Hence, the height of tower is 40 m. ⇒ y = 20 m
15. (d) Let AB = 150 m be the height of building and C be a i.e. AB = 20 m
BD
point on the ground such that ∠ACB = 45°. Now, in ΔABD, = tan 60° = 3
AB
A
BD
⇒ = 3
AB
150 m 20 + x
⇒ = 3
20
45° ⇒ 20 + x = 20 3
C B ⇒ x = 20 3 − 20
In right angled ΔABC, = 20( 3 − 1)
Perpendicular AB
tan 45° = = = 20(1.732 − 1)
Base BC ⇒ x = 20 × 0.732 = 14. 64 m
CBSE Term II Mathematics X (Standard) 103

18. (d) Let BC be the width of the river and A, B be the ends of 20. (i) (b) Let in ΔABC, AC will be rope and AB be a vertical
river such that AB = 150 m = Length of the bridge [given] pole.
and ∠BAC = 45°. A

B edge of river

m
20
River River
45°
edge of river
A C 60°
C B
In right angled ΔACB,
Perpendicular Then, AC = 20 m, ∠C = 60°, ∠B = 90°
sin 45° =
Hypotenuse AB
In ΔABC, sin 60° =
1 BC BC 1 AC
⇒ = = [Qsin 45° = ] 3 AB
2 AB 150 2 =
150 2 2 20
∴ BC = × [by rationalising] AB = 10 3 m
2 2
150 BC 1 BC
= 2 = 75 2 (ii) (c) cos 60° = ⇒ =
2 AC 2 20
= 75 × 1.414 [Q 2 = 1.414] BC = 10 m
= 106.05 m (approx.) AB
(iii) (d) sin 45° = [ ∠C = 45° ]
Hence, width of the river is 106.05 m. AC
1 AB 20
19. (i) (b) Radha is more closer to the Parachute because the = ⇒ = AB
angles of elevation of Parachute from these balcony are 2 20 2
observed to be 45°. So, Radha is more closer to the 20 2
⇒ AB = × [by rationalising]
Parachute than Ankit. 2 2
DE
(ii) (d) In ΔDEF, ∠D = 90°, ∠DFE = 45°, tan 45° = = 10 2 m
DF
DE (iv) (a) Length of rope = 20 − 3 = 17 m
⇒ 1= AB
DF sin 45° = [∠C = 45°]
⇒ DE = DF = h m AC
1 AB 17
(iii) (b) In ΔEGC, ∠EGC = 60°, ∠C = 90° = ⇔ AB = m
2 17 2
CE
tan 60° = A
CG
CD + DE h+6
3= ⇒ 3= [Q CG = DF ]
CG DF
m
17

h+6
⇒ 3=
h 45°
6 6 C B
⇒ 3 =1+ ⇒ ( 3 − 1) =
h h (v) (c) in this, mathematical concept trigonometric ratio is
6 3+1 used here, which is application of trigonometry.
∴ h= × [by rationalising]
( 3 − 1) 3+1 21. (i) (b) Let AB be the monument of height 42 m and C is the
6( 3 + 1) point where they are standing, such that BC = 42 m.
= [Q ( a + b )( a − b ) = a 2 − b 2 ] A
( 3 ) 2 − (1 ) 2
6 ( 3 + 1 ) 6 ( 3 + 1)
= = = 3 ( 3 + 1) m
3−1 2
42 m
(iv) (d) Height of the Parachute from the ground is BE, then
BE = BC + CD + DE
θ
BE = 3 + 6 + 3( 3 + 1) C B
42 m
= 9 + 3 ( 3 + 1)
Now, in ΔABC,
=9+ 3 3 + 3 AB
tan θ =
= 12 + 3 3 BC
= 3( 4 + 3) m 42
⇒ tan θ = =1
(v) (c) If the Parachute is moving towards the building, then 42
both angles of elevation will decreases. ⇒ tan θ = 1 ⇒ θ = 45°
104 CBSE Term II Mathematics X (Standard)

(ii) (d) In ΔABC, 22. As it is given that satellite is the mid-point of the two
A mountain hills i.e. I is the mid-point of DS.
F
30° 60°

42 m
A 30°
G
60°
C B
60°
7816 m H P
AB
tan 60° = 968500 m 968500 m 1930 m
BC
42 D
3= I S
BC
42 42 3 1937000 m
BC = = × [by rationalising]
3 3 3 (i) (b) We have, AG = DI = 968500 m
42 3 Now, in ΔFAG,
= = 14 3 AG
3 cos 30° =
= 14 × 1.732 = 24.24 m AF
(iii) (a) Let AB = h be the height of the tower. 3 968500
⇒ =
2 AF
A
968500 × 2 1937000
⇒ AF = =
3 1.73
= 1119653.18 m
hm
= 1119. 65 km
(ii) (c) We have, HP = IS = 968500 m
60°
C B Now, in ΔFHP,
20 m
HP
AB h cos 60° =
Now, in ΔABC, tan 60° = ⇒ 3= ⇒ h = 20 3 m FP
BC 20 1 968500
=
(iv) (b) Let h and x be the height and length of shadow of the 2 FP
vertical tower. FP = 968500 × 2 = 1937000 m = 1937 km
A
FG 1 FG
(iii) (b) In ΔFAG, tan 30° = ⇒ =
AG 3 968500
968500
hm ⇒ FG = = 559826. 59 m
3
= 559. 82 km
θ
C B ∴Height of satellite from ground = FI = FG + GI
xm = 559. 82 + 7. 816
Now, in ΔABC, [Q GI = AD = 7816 m = 7. 816 km ]
AB h = 567. 64 km
tan θ = ⇒ tan θ =
BC x (iv) (b) Let E be the position of man.
⇒ tan θ = 1 [Q h : x = 1 : 1] A
⇒ θ = 45°
(v) (c) The angle of depression of the object viewed, is the
angle formed by the line of sight with the horizontal,
when it is below the horizontal level.
θ
Horizontal line D E
O 7816 m
Angle of depression
Then, DE = 7816 m
Li

AD 7816
ne

In ΔADE, tan θ = = =1
of

DE 7816
sig
ht

[Q height of mountain AD = 7816 m]


P (object)
∴ θ = 45°
CBSE Term II Mathematics X (Standard) 105

(v) (c) Let T be the point where mile stone is kept. 2. Let AB be the tree whose part AC breaks and touches the
P ground at D.
Then, BD = 2 3 m [given]
and AC = CD
1930 m A

45°
T S
PS
So, In ΔPST, tan 45° =
TS C
1930
⇒ 1= ⇒ TS = 1930 m
TS
Subjective Questions
60°
1. Case I Let the height of a tower be h and the distance of the B D
2 √3 m
point of observation from its foot be x.
In ΔABC, In right angled ΔCBD,
AC h BD
tan θ1 = = cos 60° =
BC x CD
⎛ h⎞ 1 2 3 ⎡ 1 ⎤
⇒ θ1 = tan −1 ⎜ ⎟ ...(i) ⇒ = Q cos 60° = and BD = 2 3 m
⎝ x⎠ 2 CD ⎢⎣ 2 ⎥⎦
A ⇒ CD = 2 × 2 3 = 4 3
= 4 × 1.732 = 6.928 m [Q 3 = 1.732]
h ∴ AC = CD = 6.928 m
Again, in right angled ΔCBD,
θ1 BC
B C tan 60° =
x BD
BC
Case II Now, the height of the tower increased by 10% ⇒ 3= [Q tan 60° = 3 and BD = 2 3 m]
10 11 h 2 3
= h + 10% of h = h + h × = ⇒ BC = 3 × 2 3 = 6 m
100 10
and the distance of the point of observation from its foot Now, AB = AC + BC
= x + 10% of x = 6.928 + 6 = 12.928 m (approx.)
10 11x Hence, whole height of the tree is 12.928 m.
=x+x × =
100 10 3. Let AB = h km be the height of the mountain. Let C be a
P point at a distance of x km from the base of the mountain
such that ∠ACB = 30° and let D be a point at a distance of
11h 10 km from C along the same line. Then, ∠ADB = 15°
10 and AD = AC + DC = ( x + 10) km
θ2 B
Q R
11x
10
h km
⎛ 11h ⎞
⎜ ⎟
PR ⎝ 10 ⎠ 30°
In ΔPQR, tan θ 2 = = D 15°
QR ⎛ 11x ⎞ C A
⎜ ⎟
⎝ 10 ⎠
h In right angled ΔBAC,
⇒ tan θ 2 = AB
x tan 30° =
AC
⎛ h⎞
⇒ θ 2 = tan −1 ⎜ ⎟ ...(ii) 1 h ⎡ 1 ⎤
⎝ x⎠ ⇒ = Q tan 30° =
3 x ⎢⎣ 3 ⎥⎦
From Eqs. (i) and (ii),
⇒ x=h 3 ...(i)
θ1 = θ 2
In right angled ΔBAD ,
Hence, the required angle of elevation of its top remains AB
unchanged. tan 15° =
AD
106 CBSE Term II Mathematics X (Standard)

h ⇒ tan θ = 3 = tan 60°


⇒ 0.27 = [ given , tan 15° = 0.27 ]
x + 10 ∴ θ = 60°
⇒ 0.27 ( x + 10) = h ...(ii) II. A same hight of tower casts a shadow 4m long from
On putting x = 3h from Eq. (i) in Eq. (ii), we get preceding shadow, when the Sun’s elevation is 30°.
0.27( 3h + 10) = h AB
In ΔAPB, tan θ =
PB
⇒ 0.27 × 3h + 0.27 × 10 = h
AB
⇒ h(1 − 0.27 × 3 ) = 0.27 × 10 =
PC + CB
⇒ h(1 − 0.27 × 1.732 ) = 2 .7 [Q 3 = 1.732]
2 3 2 3
⇒ h (1 − 0. 47 ) = 2.7 ⇒ tan θ = =
4+2 6
⇒ 0. 53h = 2.7
A
2.7
⇒ h= = 5.09 ≈ 5 km
0.53
Hence, the height of mountain is 5 km. 2√3 m
4. Let AB be the tower and AC be the flag staff on the tower.
Let D be a point on the ground such that the angles of θ 60°
P B
elevation of foot A and top C of the flag staff are 45° and 60°, 4m C 2m
respectively.
C 3 3 3
⇒ tan θ = ⋅ =
3 3 3 3
A 1
⇒ tan θ = = tan 30°
3
∴ θ = 30°
60° Hence, we conclude from above two examples that if the
45° length of the shadow of a tower is increasing, then the
B D
angle of elevation of the Sun is decreasing.
Then, we have AB = 20 m , ∠ADB = 45° and ∠CDB = 60° 6. Let QS be the building and R be the position of window.
In right angled ΔABD, Given, height of the window, QR = 10 m
AB ⎡ Perpendicular ⎤ ∠QPR = ∠XRP = 30° [alternate angles]
tan 45°= Q tan θ =
BD ⎢⎣ Base ⎥⎦
and ∠SPQ = 60°
20
⇒ 1= S
BD
⇒ BD = 20 m [Q tan 45° = 1]
and in right angled ΔCBD,
BC BC
tan 60°= ⇒ 3= [Q tan 60° = 3 ] X R (Window)
BD 20 30°
60° 10 m
⇒ BC = 20 3 = 20 × 1.732 [Q 3 = 1.732] 30°
P
= 34. 64 m (approx.) Q

Now, AC = BC − AB = 34.64 − 20 In right angled ΔPQR ,


= 14.64 m (approx.) QR
tan 30° =
Hence, the height of the flag staff is 14.64 m. PQ
5. To understand the fact of this question, consider the 1 10 ⎡ 1 ⎤
following example ⇒ = Q tan 30° =
3 PQ ⎢⎣ 3 ⎥⎦
I. A tower 2 3 m high casts a shadow 2 m long on the
⇒ PQ = 10 3 m ...(i)
ground, when the Sun’s elevation is 60°.
BC 2 3 In right angled ΔPQS,
In ΔACB, tan θ = = QS
AB 2 tan 60° =
C
PQ
QS
⇒ 3= [Q tan 60° = 3 and from Eq. (i)]
10 3
2√3 m ⇒ QS = 10 × 3 = 30 m
θ Hence, height of the building is 30 m.
A 2m B
CBSE Term II Mathematics X (Standard) 107

7. Let AB = 20 m be the height of tower and let the ball lying So, angle of depression is different in the lake from the angle
on the ground at point C. of elevation of the cloud above the surface of a lake.
Given, angle of depression, C
∠TAC = 60° = ∠ACB [alternate angles]
A hm
T
60°
θ1
P M
θ2 3m
20 m Q O

60° h
C B
In right angled ΔABC, θ R
AB
tan 60° = CM h
BC In ΔMPC, tan θ1 = =
20 PM PM
⇒ 3= tan θ1 1
BC ⇒ = …(i)
20 20 h PM
⇒ BC = = = 11.55 m RM OR + OM h + 3
3 1.732 In ΔRPM, tan θ 2 = = =
PM PM PM
Hence, the distance between the foot of the tower and the
tan θ 2 1
ball is 11.55 m. ⇒ = …(ii)
h + 3 PM
8. Let AB be the tower of height x m , and CD be the tower of
height y m. From Eqs. (i) and (ii),
D tan θ1 tan θ 2 ⎛ h + 3⎞
= ⇒ tan θ 2 = ⎜ ⎟ tan θ1
B h h+3 ⎝ h ⎠
ym So, θ1 ≠ θ 2
xm
60°
Hence, it is a false statement.
30°
A E C 10. Let AB = h km be the height of the hill and C, D be two
a a
consecutive stones such that CD = 1 km .
Let E be the mid-point of the line AC. Then, ∠AEB = 30° Let BC be x km, then BD = BC + CD = ( x + 1) km
and ∠CED = 60°.
A 30°
Also, AE = EC = a m (let) X
In right angled ΔBAE, 45°
AB x
tan 30°= =
AE a
1 x a h km
⇒ = ⇒x = …(i)
3 a 3
and in right angled ΔDCE, 45° 30°
DC y x km C 1 km D
tan 60°= = B
CE a (x+1) km
y
⇒ 3 = ⇒ y = 3a …(ii)
a Now, ∠ ADB = ∠XAD = 30° [alternate angles]
[Q Eq. (i) divide by Eq. (ii)] and ∠ ACB = ∠XAC = 45° [alternate angles]
a In right angled ΔABC,
x 1 1
∴ = 3 = = tan 45° =
Perpendicular AB
=
y 3a 3× 3 3 Base BC
Hence, x : y = 1 : 3 h
⇒ 1 = ⇒x = h …(i)
9. From figure, we observe that, a man standing on a platform x
at point P, 3 m above the surface of a lake observes a cloud at AB
Now, in right angled ΔABD, tan 30° =
point C. Let the height of the cloud from the surface of the BD
platform is h and angle of elevation of the cloud is θ1. 1 h ⎡ 1 ⎤
Now at same point P, a man observes a cloud reflection in ⇒ = Q tan 30° =
3 x+1 ⎢⎣ 3 ⎥⎦
the lake at this time the height of reflection of cloud in lake 1 h
is ( h + 3) because in lake platform height is also added to ⇒ = [from Eq. (i)]
reflection of cloud. 3 h+1
108 CBSE Term II Mathematics X (Standard)

⇒ h + 1 = 3h In right angled ΔBMP,


⇒ h( 3 − 1) = 1 PM
tan 45° =
BM
1 3 + 1 ⎛ 3 + 1⎞
⇒ h= × =⎜ ⎟ km 200
3 −1 3+1 ⎝ 2 ⎠ ⇒ 1= [Q tan 45° = 1]
y
3+1 ⇒ y = 200 m
Hence, height of the hill is km.
2 Now, distance between the two ships = AB = x + y
11. Let AB be the tower, BC be the shadow of tower, when angle = 115. 47 + 200
of elevation of Sun is 30° and BD be the shadow of tower, = 315.47 m
when angle of elevation of Sun is 60°. 13. Let distance of the pole, say AE, from the bottom of the tower,
Then, we have say BD, be x m and let the height of the pole, AE = y m
BC = 30 m, ∠ACB = 30° and ∠ADB = 60° Now, draw EC ||AB.
Now, let AB = h m and BD = x m D
A

50 m
hm 30º
E xm C
ym
30° 60° ym
C D B
xm 45º
30 m
A xm B
Clearly, in ΔABC, we have
Then, ∠DEC = 30°, ∠DAB = 45°
Perpendicular AB h
tan 30° = = = and DC = DB − BC = DB − AE [Q BC = AE ]
Base BC 30
1 h 30 ⇒ DC = ( 50 − y ) m
⇒ = ⇒h = (i) In right angled ΔABD,
3 30 3
Perpendicular BD
30 3 30 3 tan 45° = =
⇒ × = = 10 3 m Base AB
3 3 3 50
AB h ⇒ 1= ⇒ x = 50 m …(i)
Also, in ΔABD , tan 60° = = x
BD x ∴ The pole is 50 m away from the foot of the tower.
10 3 (ii) In right angled ΔECD ,
⇒ 3= ⇒ x = 10 m
x Perpendicular DC
tan 30° = =
Hence, length of shadow is 10 m, when angle of elevation is Base EC
60°. 1 50 − y ⎡ 1 ⎤
12. Let PM be the light house of height 200 m and let A and B be ⇒ = tan 30° =
3 x ⎢⎣ 3 ⎥⎦
two ships on either sides of light house such that the angles
1 50 − y
of depression of A and B are 60° and 45°, respectively. ⇒ = [Q x = 50 m from Eq. (i)]
3 50
Let AM = x m and BM = y m
Then, ∠XPB = ∠MBP = 45° [alternate angles] ⇒ 3 ( 50 − y ) = 50
and ∠YPA = ∠MAP = 60° [alternate angles] 50
⇒ 50 − y =
P 3
X Y
45° 60° ⎛ 1 ⎞
⇒ y = 50 ⎜1 − ⎟
⎝ 3⎠
200 m ⎛ 1 ⎞
= 50⎜1 − ⎟
⎝ 1.732 ⎠
45° 60°
B A = 50(1 − 0. 57737 )
ym M xm
= 50 × 0. 4226
In right angled ΔAMP, = 21.13 m
Perpendicular PM
tan 60° = = ∴ Height of the pole = 21.13 m
Base AM
200 14. Let a man is standing on the deck of a ship at point A such
⇒ 3= [Q tan 60° = 3 ] that AB = 10 m and let CD be the hill.
x
200 200 Then, ∠EAD = 60°
⇒ x= m= m = 115. 47 m and ∠CAE = ∠BCA = 30° [alternate angles]
3 1.732
CBSE Term II Mathematics X (Standard) 109

Let BC = x m = AE and DE = h m 16. Let AB be the flag staff of height h units and AC = x units be
D length of its shadow, when the Sun rays meet the ground at
an angle of 60°.
Also, let θ be the angle between the Sun rays and the
hm ground, when the length of the shadow of the flag staff is
AD = 3x units.
B

60°
A E
30° h
10 m 10 m
30° θ 60°
B C D
xm A
C
2x x
In right angled ΔAED ,
Perpendicular DE h In right angled ΔCAB,
tan 60° = = =
Base EA x Perpendicular AB
h tan 60° = =
⇒ 3 = ⇒h = 3x Base AC
x h
⇒ 3= [Q tan 60° = 3 ]
In right angled ΔABC, x
AB 1 10 ⎡ 1 ⎤ ⇒ h = 3x ...(i)
tan 30° = ⇒ = Q tan 30° =
BC 3 x ⎢⎣ 3 ⎥⎦ Now, in right angled ΔDAB,
⇒ x = 10 3 m AB AB
tan θ = = [Q AD = DC + CA ]
⇒ h = 10 3 × 3 = 30 m AD DC + CA
h h
∴The height of hill, CD = h + 10 = 30 + 10 = 40 m ⇒ tan θ = =
2 x + x 3x
Hence, The distance of the hill from the ship is 10 3 m and
height of the hill is 40 m. 3x
= [from Eq. (i)]
15. Let OX be the horizontal ground; A and B be the two 3x
positions of the plane and O be the point of observation. 1 ⎡ 1 ⎤
= = tan 30° Q tan 30° =
A B 3 ⎣⎢ 3 ⎦⎥
∴ θ = 30°
Hence, the angle between the sun rays and the ground at the
2500 m time of longer shadow is 30°.
17. Let P and Q be the positions of two aeroplanes, where P is
45°
vertically above Q and OP = 4000 m.
O X P
C D
Here, AC = BD = 2500 m , ∠AOC = 45°
and ∠BOD = 30°
Q 4000 m
In right angled ΔOCA,
Base OC
cot 45°= =
perpendicular AC 60°
A 45°
OC O
⇒ 1= [Q cot 45° = 1]
AC Here, ∠PAO = 60° and ∠QAO = 45°
⇒ OC = AC = 2500 m Now, in right angled ΔAOP,
In right angled ΔODB, Perpendicular OP
OD OD tan 60° = =
cot 30°= ⇒ 3= Base AO
BD 2500 4000
⇒ 3= [Q tan 60° = 3 ]
⇒ OD = 2500 3 m AO
Now, CD = OD − OC = 2500 3 − 2500 4000
⇒ AO = ...(i)
= 2500( 3 − 1) = 2500(1.732 − 1) 3
= 2500 × 0.732 = 1830 m In right angled ΔAOQ,
OQ OQ
Thus, distance covered by plane in 15 s is 1830 m. tan 45° = ⇒ 1= [∴ tan 45° = 1]
1830 60 × 60 OA OA
∴Speed of plane = × = 439.2 km/h ⇒ OA = OQ ...(ii)
15 1000
110 CBSE Term II Mathematics X (Standard)

From Eqs. (i) and (ii), we get We have, AB = 1200 m


4000 Let AC = x m and CD = y m.
OQ = m
3 In right angled ΔBAC, we have
∴vertical distance between the aeroplanes Perpendicular AB
tan 60° = =
4000 Base AC
= PQ = OP − OQ = 4000 −
3 1200
⇒ 3= [Q tan 60° = 3]
⎛ 1 ⎞ ⎛ 1 ⎞ x
= 4000 ⎜1 − ⎟ = 4000⎜1 − ⎟
⎝ 3⎠ ⎝ 1.732 ⎠ 1200 3
⇒ x= × [rationalising]
= 4000 (1 − 0. 577 ) 3 3
= 4000 × 0. 423 = 1692 m 1200 3
⇒ x= = 400 3 m ...(i)
18. Let OA be the tree of height h m. 3
Given, PQ = 100 m and angles of elevation are ∠APO = 30° In right angled ΔBAD , we have
and ∠OQA = 45°. AB AB
A tan 30° = = [Q AD = DC + CA]
AD DC + CA
1 1200
⇒ =
hm 3 x +y
30° 45° ⎡ 1 ⎤
P Q
⇒ x + y = 1200 3 Q tan 30° =
O ⎢⎣ 3 ⎥⎦
100 m
⇒ y = 1200 3 − x …(ii)
Perpendicular OA
In right angled ΔPOA, tan 30° = = On putting the value of x from Eq. (i) in Eq. (ii), we get
Base OP
1 h ⎡ 1 ⎤ y = 1200 3 − 400 3
⇒ = Q tan 30° =
3 OP ⎢⎣ 3 ⎥⎦ = 800 3
⇒ OP = 3h ...(i) = 800 × 1.732 [Q 3 = 1.732]
Now, in right angled ΔQOA, = 1385. 6 m
OA Hence, the distance between both ships is 1385. 6 m.
tan 45° =
OQ 20. Let A be the aeroplane and AD be its height. Again, let B
h and C be two consecutive kilometre stones on the road on
⇒ 1= [Q tan 45° = 1]
OQ the left and right of plane A and the angles of depression of
C and B from plane A are 60° and 45°, respectively.
⇒ OQ = h ...(ii)
A
On adding Eqs. (i) and (ii), we get P Q
45° 60°
OP + OQ = 3 h + h
⇒ PQ = ( 3 + 1)h [Q OP + OQ = PQ]
⇒ 100 = ( 3 + 1)h [QPQ = 100 m, given]
100 3 −1
⇒ h= × [by rationalising] 45° 60°
3+1 3 −1 B C
x km (1 – x) km
100 (1.732 − 1) D
= 1 km
2
= 50 × 0.732 = 36. 6 m Then, ∠ABC = ∠PAB = 45° [alternate angles]
Hecne, height of the tree is 36.6 m. and ∠ACB = ∠QAC = 60° [alternate angles]
19. Let the aeroplane be at B and two ships be at C and D such Also, BC = 1 km
that their angles of depression from B are 60° and 30°, Let BD = x km , then
respectively. Then, the angles of elevation of B from D and C DC = BC − BD = (1 − x ) km …(i)
are 30° and 60°, respectively. In right angled ΔADB,
B Perpendicular AD
tan 45° = =
30° Base BD
60°
AD
⇒ 1= [Q tan 45° = 1]
1200 m x
⇒ AD = x
and in right angled ΔADC,
30° 60°
AD
D C A tan 60° =
ym xm DC
CBSE Term II Mathematics X (Standard) 111

x 22. Let D be the position of the balloon, when it is inclined at


⇒ 3=
1−x angle of 60° and AB be the height of the pole.
[Q tan 60° = 3 and from Eq. (i)] D (Balloon)

⇒ 3 − 3x = x
⇒ 3 = 3x + x
C
⇒ ( 3 + 1) x = 3 B
3 3 3 −1
⇒ x= = × 30°
3+1 3+1 3 −1 60°
E A
[by rationalising]
3− 3 Given, length of cable, DE = 215 m
= In right angled ΔEAD,
( 3 ) 2 − (1 ) 2
Perpendicular AD
[Q ( a + b )( a − b ) = a 2 − b 2] sin 60° = =
Hypotenuse ED
3 − 3 3 − 1.732 3 AD ⎡ 3⎤
= = [Q 3 = 1.732]
2 2 ⇒
2
=
215 ⎢Q sin 60°= 2 ⎥
⎣ ⎦
1.268
= = 0.634 km 215 3
2 ⇒ AD = m
Hence, the height of the aeroplane is 0.634 km. 2
Hence , initial height of the balloon from the ground is
21. Let height of the tower, BC = h m and height of the flagstaff
215 3
CD = H m . m.
2
∴ BD = BC + CD = ( h + H )m …(i)
Given, AB =120 m, ∠CAB = 45° and ∠DAB = 60° Again, in right angled ΔEAD,
D Base AE AE
cos 60° = = =
Hm
Hypotenuse DE 215
C
1 AE ⎡ 1⎤
⇒ = Q cos 60° =
2 215 ⎢⎣ 2 ⎥⎦
(h+H) m
215
⇒ AE = m …(i)
hm 2
Now, the angle of inclination is changed, say ∠CEA = 30°.
45° 60° In right angled ΔEAC,
A
120 m B Perpendicular AC
tan 30° = =
In right angled ΔABC, we get Base EA
BC ⎡ Perpendicular ⎤ 1 AC × 2
tan 45°= Q tan θ = ⇒ =
AB ⎢⎣ Base ⎥⎦ 3 215
h ⇒ 2 3AC = 215
⇒ 1= [Q tan 45° = 1]
120 215
⇒ AC = m
⇒ h = 120 m …(ii) 2 3
BD
Now, in right angled ΔABD, we get tan 60° = 23. Let the height of the light house AB be 100 m. C and D be
AB the positions of man when angle of elevation changes from
h +H
⇒ 3= …(iii) 60° to 45°, respectively. The man has covered a distance CD
120 in 2 min.
[Q tan 60° = 3 and from Eq. (i)] Distance CD
Q Speed = ⇒ Speed = ...(i)
From Eqs. (ii) and (iii), Time 2
120 + H In right angled ΔABC,
3=
120 A
⇒ 120 3 = 120 + H
⇒ H = 120 ( 3 − 1) 100 m
Light house
= 120(1.732 − 1)
= 120 × 0.732 = 87. 84 m 45° 60°
Hence, height of flag staff is 87.84 m. D
C B
112 CBSE Term II Mathematics X (Standard)

Perpendicular AB ⇒ 20 = h 3 − h
tan 60° = =
Base BC ⇒ h ( 3 − 1) = 20
100
⇒ 3= [Q tan 60° = 3 ] 20 3+1
BC ∴ h= ⋅ [by rationalisation]
100 3 3 −1 3+1
⇒ BC = m ...(ii)
3 20 ( 3 + 1)
⇒ =
In right angled ΔABD, 3−1
AB 20 ( 3 + 1)
tan 45° = =
BD 2
100 ⇒ = 10 ( 3 + 1) m
⇒ 1= [Q tan 45° = 1]
BD
Hence, the required height of tower is 10 ( 3 + 1) m.
⇒ BD = 100 m
100 3 25. Let the height of the tower be h and RQ = x m
Now, CD = BD − BC = 100 −
3 Given that, PR = 50 m
⎛ 3 − 3⎞ and ∠SPQ = 30° , ∠SRQ = 60°
100 ⎜ ⎟ SQ
CD ⎝ 3 ⎠ 50 Now, in ΔSRQ, tan 60° =
and Speed = = = ( 3 − 3 ) m/min RQ
2 2 3
h h
24. Let the height of the tower be h. ⇒ 3= ⇒ x= …(i)
x 3
Also, SR = x m, ∠PSR = θ
SQ SQ h
Given that, QS = 20 m and in ΔSPQ, tan 30° = = =
PQ PR + RQ 50 + x
and ∠PQR = 30°
P S Sun

h hm

30° θ 30° 60°


Q R P xm Q
20 m S xm 50 m R
1 h
Now, in ΔPSR, =
PR h 3 50 + x
tan θ = =
SR x ⇒ 3 ⋅ h = 50 + x
h h
⇒ tan θ = ⇒ 3 ⋅ h = 50 + [from Eq. (i)]
x 3
h 1
⇒ x= ...(i) ⇒ ( 3− ) h = 50
tan θ 3
Now, in ΔPQR, ( 3 − 1)
⇒ h = 50
PR PR 3
tan 30° = =
QR QS + SR 50 3
∴ h=
h 2
⇒ tan 30° =
20 + x h = 25 3 m
h h Hence, the required height of tower is 25 3 m.
⇒ 20 + x = =
tan 30° 1 / 3 26. Let the height of the tower be H and OR = x
⇒ 20 + x = h 3 Given that, height of flag staff = h = FP and ∠PRO = α,
h ∠FRO = β
⇒ 20 + =h 3 [from Eq. (i)] …(ii)
tan θ F

Since, after moving 20 m towards the tower the angle of flag staff h
elevation of the top increases by 15°.
P
i.e. ∠PSR = θ = ∠PQR + 15°
⇒ θ = 30° + 15 = 45°
h H
∴From Eq. (i) 20 + =h 3
tan 45° β
h α
⇒ 20 + = h 3 R x O
1
CBSE Term II Mathematics X (Standard) 113

PO H 28. Let the distance between two objects is x m.


Now, in ΔPRO, tan α = =
RO x and CD = y m.
H A
⇒ x= …(i) X
tan α Y α
β
FO
and in ΔFRO, tan β =
RO
FP + PO hm
=
RO
h+H
⇒ tan β =
x α β
h+H B x y D
⇒ x= …(ii) C
tan β
Given that, ∠BAX = α = ∠ABD, [alternate angle]
From Eqs. (i) and (ii), ∠CAY = β = ∠ACD [alternate angle]
H h+H
= and the height of tower, AD = h m
tan α tan β
Now, in ΔACD,
⇒ H tan β = h tan α + H tan α AD h
tan β = =
⇒ H tan β − H tan α = h tan α CD y
h
⇒ H (tan β − tan α) = h tan α ⇒ y= …(i)
tan β
h tan α
⇒ H= and in ΔABD,
tan β − tan α
AD AD
h tan α tan α = =
Hence, the required height of tower is BD BC + CD
tan β − tan α h
⇒ tan α =
Hence proved. x+y
27. Let distance between the two towers = AB = x m h
⇒ x+y=
and height of the other tower = PA = h m tan α
Given that, height of the tower = QB = 30 m and h
⇒ y= −x …(ii)
∠QAB = 60°, ∠PBA = 30° tan α
Q From Eqs. (i) and (ii),
h h
= −x
P tan β tan α
30 m h h
∴ x= −
hm tan α tan β
⎛ 1 1 ⎞
60° 30° =h⎜ − ⎟
A xm B ⎝ tan α tan β⎠
QB 30 ⎡ 1 ⎤
Now, in ΔQAB, tan 60° = = = h (cot α − cot β )
⎢Q cot θ = tan θ ⎥
AB x ⎣ ⎦
30 which is the required distance between the two objects.
⇒ 3=
x Hence proved.
30 3 29. Let OQ = x and OA = y
∴ x= ⋅ [by rationalising]
3 3 Given that, BQ = q, SA = P and AB = SQ = Length of ladder
30 3 Also, ∠BAO = α and ∠QSO = β
= = 10 3 m
3 (Wall)
and in ΔPBA, B
PA h q
tan 30° =
=
AB x
1 h Q
⇒ = [Q x = 10 3 m] (Ladder)
3 10 3 x
⇒ h = 10 m
β α
Hence, the required distance and height are 10 3 m and S A O
y
10 m, respectively. P
114 CBSE Term II Mathematics X (Standard)

Now, in ΔBAO, H ⎛ 1 ⎞
⇒ H− = 10 ⇒ H ⎜1 − ⎟ = 10
OA 3 ⎝ 3⎠
cos α =
AB ⎛ 3 − 1⎞
⇒ OA = AB cos α …(i) ⇒ H⎜ ⎟ = 10
⎝ 3 ⎠
OB
and sin α = 10 3 3+1
AB ∴ H= ⋅ [by rationalisation]
⇒ OB = BA sin α …(ii) 3 −1 3+1
Now, in ΔQSO, 10 3 ( 3 + 1) 10 3 ( 3 + 1)
= =
OS 3−1 2
cos β =
SQ ⇒ = 5 3 ( 3 + 1) = 5( 3 + 3) m.
⇒ OS = SQ cos β = AB cos β Hence, the required height of the tower is 5 ( 3 + 3) m.
[Q AB = SQ] …(iii)
OQ 31. Let the height of the other house = OQ = H
and sin β =
SQ and OB = MW = x m
Given that, height of the first house = WB = h = MO
⇒ OQ = SQ sin β = AB sin β
and ∠QWM = α, ∠OWM = β = ∠WOB [alternate angle]
[Q AB = SQ] …(iv)
Now, SA = OS − AO Q
P = AB cos β − AB cos α

(H – h )
⇒ P = AB (cos β − cos α) …(v)
and BQ = BO − QO
⇒ q = BA sin α − AB sin β W α
(Window) β MH
x
⇒ q = AB (sin α − sin β) …(vi)
On dividing Eq. (v) by Eq. (vi), we get h h
p AB (cos β − cos α ) cos β − cos α
= = β
q AB (sin α − sin β) sin α − sin β
B x O
p cos β − cos α
⇒ = Hence proved. WB h
q sin α − sin β Now, in ΔWOB, tan β = =
OB x
30. Let the height of vertical tower be, h
OT = H and OP = AB = x m ⇒ x= …(i)
tan β
Given that, AP = 10 m
QM OQ − MO
and ∠TPO = 60° , ∠TAB = 45° And in ΔQWM, tan α = =
WM WM
T
H−h
⇒ tan α =
(H – 10) m

x
H−h
⇒ x= ...(ii)
tan α
45° H
A xm B From Eqs. (i) and (ii),
h H−h
10 m

10 m

=
tan β tan α
60°
P O ⇒ h tan α = (H − h ) tan β
xm
⇒ h tan α = H tan β − h tan β
Now, in ΔTPO, ⇒ H tan β = h(tan α + tan β)
OT H
tan 60° = = ⎛ tan α + tan β⎞
OP x ∴ H=h⎜ ⎟
H H ⎝ tan β ⎠
⇒ 3= ⇒ x= ...(i)
x 3 ⎛ 1 ⎞
= h ⎜1 + tan α ⋅ ⎟
and in ΔTAB, ⎝ tan β⎠
TB H − 10 = h (1 + tan α ⋅ cot β)
tan 45° = =
AB x ⎡ 1 ⎤
⇒ 1=
H − 10
⇒ x = H − 10 ⎢Q cot θ = tan θ ⎥
⎣ ⎦
x
H Hence, the required height of the other house is
⇒ = H − 10 [from Eq. (i)] h (1 + tan α ⋅ cot β) m. Hence proved.
3
CBSE Term II Mathematics X (Standard) 115

32. Let the height of the balloon above the ground is H ⇒ 2H = 16


and OP = W2R = W1Q = x ⇒ H=8
Given that, height of lower window from above the ground So, the required height is 8 m.
= W2P = 2 m = OR Hence, the required height of the balloon above the ground
Height of upper window from above the lower window is 8 m.
= W1 W2 = 4 m = QR 33. (i) Distance of first position of parrot from the eyes of girl
∴ BQ = OB − ( QR + RO) = AC
=H − ( 4 + 2 ) =H − 6 E C

and ∠BW1 Q = 30°


⇒ ∠BW2R = 60°

58 m
Balloon

60°
B D A
B 30° 8m
F
(H – 6)

G H

In ΔABC,
w1 30°
Q BC
xm sin 60° =
Upper Hm AC
window
4m CH − BH
⇒ AC =
60° x sin 60°
w2 R
Lower 58 − 8 100
window 2m = = m
3/2 3
P O
xm (ii) If the distance increases, then the angle of elevation
decreases.
Now, in ΔBW2R ,
(iii) Distance between girl and building = AB
BR BQ + QR
tan 60°= = Now, in ΔABC,
W2R x
BC 50
(H − 6 ) + 4 tan 60° = ⇒ 3 AB = 50 ⇒ AB = m
⇒ 3= AB 3
x DE
H −2 (iv) In ΔAED, tan 30° =
⇒ x= ...(i) AD
3 ⇒ AD = 3 BC = 50 3 m
and in ΔBW1Q,
BQ [QED = BC = 58 − 8 = 50]
tan 30° = Now, distance between two position of parrot = EC
W1Q
= BD = AD − AB
H −6 1
⇒ tan 30° = = ⎛ 50 ⎞
x 3 = ⎜ 50 3 − ⎟m
⎝ 3⎠
⇒ x = 3 (H − 6 ) ...(ii)
50( 3 − 1) 100
From Eqs. (i) and (ii), = = = 57. 80 m
1.73 1.73
(H − 2 ) Distance covered
3 (H − 6 ) = (v) Speed of parrot =
3 Time taken
3 (H − 6 ) = H − 2 ⎛ 57. 80⎞
=⎜ ⎟ m/s = 7.225 m/s
⇒ 3H − 18 = H − 2 ⎝ 8 ⎠
Chapter Test
Multiple Choice Questions Based on the above information, answer the following
1. A circus artist is climbing from the ground along a questions.
rope stretched from the top of a vertical pole and tied (i) Measure of ∠ACD is equal to
at the ground. The height of the pole is 12 m and the (a) 30° (b) 45°
angle made by the rope with ground level is 30°. The (c) 60° (d) 90°
distance covered by the artist in climbing to the top of (ii) If ∠YAB = 45°, then ∠ABD is also 45°, Why?
the pole is (a) vertically opposite angles
(a) 12 m (b) 6 m (b) alternate interior angles
(c) 24 m (d) 32 m (c) alternate exterior angles
(d) corresponding angles
2. A ladder 15 m long just reaches the top of a vertical
wall. If the ladder makes an angle of 60° with the wall, (iii) Length of CD is equal to
then the height of the wall is (a) 90 m (b) 50 3 m
15 (c) 50 / 3 m (d) 100 m
(a) 30 m (b) m
2 (iv) Length of BD is equal to
(c) 15 m (d) 25 m (a) 50 m (b) 100 m
(c) 100 2 m (d) 100 3 m
3. A kite is flying at a height of 30 m from the ground.
The length of string from the kite to the ground is (v) Length of AC is equal to
60 m. Assuming that there is no slack in the string, (a) 100 / 3 m (b) 100 3 m
then the angle of elevation of the kite at the ground is (c) 50 m (d) 100 m
(a) 30° (b) 45° Short Answer Type Questions
(c) 60° (d) None of these 7. The angle of elevation of the top of a tower is 30°. If
4. The tops of two poles of height 30 m and 24 m are the height of the tower is doubled, then the angle of
connected by a wire. If the wire makes an angle of 45° elevation of its top will also be doubled. State true or
with the horizontal, then find the length of the wire. false. Explain.
(a) 14 m (b) 3 m 8. A peacock is sitting on the top of a tree. It observes a
(c) 4 m (d) 6 2 m serpent on the ground making an angle of depression
of 30°. The peacock catches the serpent in 12 s with the
5. An observer 3.5 m tall is 38.5 m away from a tower
speed of 300 m/min. What is the height of the tree?
42 m high. The angle of elevation of the top of the
[CBSE 2015]
tower from the eye of the observer is
(a) 30° 9. The angles of elevation and depression of the top and
(b) 90° bottom of a light house from the top of a 60 m high
(c) 45° building are 30° and 60°, respectively. Find the
(d) 60° difference between the heights of the light house and
building.
Case Based MCQs
10. As observed from the top of a 100 m high light house
6. A boy is standing on the top of mountain. He from the sea-level, the angles of depression of two
observed that boat P and boat Q are approaching ships are 30° and 45°. If one ship is exactly behind the
towards mountain from opposite directions. He finds other on the same side of the light house, find the
that angle of depression of boat P is 60° and angle of distance between the two ships. [CBSE 2018]
depression of boat Q is 45°. He also knows that height Long Answer Type Questions
of the mountain is 50 m.
A 11. Two ships are sailing in the sea on either side of the
X Y light house. The angles of depression of two ships as
60° 45°
observed from the top of the light house are 60° and
45°, respectively. If the distance between the ships is
50 m
⎛ 3 + 1⎞
100 ⎜ ⎟ m, then find the height of the light house.
P Q C B ⎝ 3 ⎠
D

Answers
For Detailed Solutions
1. (c) 2. (b) 3. (a) 4. (d) 5. (c) 6. (i) (c) (ii) (b) (iii) (c) (iv) (a) (v) (a)
Scan the code
7. False 8. 30 m 9. 20 m 10. 100 ( 3 − 1 ) m 11. 100 m
CBSE Term II Mathematics X (Standard) 117

CHAPTER 06

Surface Areas
and Volumes

In this Chapter...
! Solid Figures
! Surface Area
! Volume
! Combination of Two Figures
! Conversion of Solid from One Shape to Another

Solid Figures Different Types of Solid Figures


The objects having definite shape, size and occupies a fixed 1. Cuboid
amount of space in three dimensions are called solids such as A cuboid is a solid figure having 6 rectangular faces. Let its
cube, cuboid, cylinder, cone, sphere and hemisphere, etc. length = l units, breadth = b units and height = h units.

Surface Area (SA) b

Surface area of a solid body is the area of all of its surfaces


together and it is always measured in square unit. h

e.g. A cube has 6 surfaces and each surface is in a square


shape. Therefore, its surface area will be 6 a 2 sq units, where
l

a 2 is the area of each surface of the cube. Then,


Volume (i) Total surface area of cuboid (TSA)
= 2 ( lb + bh + hl) sq units
Space occupied by an object/solid body is called the volume
of that particular object/solid. Volume is always measured in (ii) Lateral surface area of cuboid = 2( l + b )h sq units
cube unit. or Lateral surface area = Area of the 4 vertical faces
e.g. Suppose, a cube has edge of length a units. Volume of a
(iii) Diagonal of the cuboid = l 2 + b 2 + h 2 units
cube is equal to the product of area of base and height of
a cube i.e. a 2 × a = a 3 cu units. (iv) Volume of cuboid = l × b × h cu units
118 CBSE Term II Mathematics X (Standard)

2. Cube Then,
Cube is a special case of cuboid which has 6 equal square faces. (i) Curved surface area (CSA)
= CSA of outer cylinder + CSA of inner cylinder
= 2 πRh + 2 πrh
a = 2π(R + r )h sq units
(ii) Total surface area (TSA)
a = CSA of hollow cylinder + Area of both ends
a = 2π ( R + r )h + 2π ( R 2 − r 2 )
Let its length = breadth = height = a units = 2 π(R + r )h + 2 π(R + r ) (R − r )
∴ Each edge of cube = a units = 2π(R + r ) [ h + R − r ] sq units
Then, (iii) Total outer surface area = 2 πRh + 2 π(R 2 − r 2 ) sq units
(i) Total surface area (TSA) of a cube (iv) Volume of hollow cylinder
= 6 × (Edge ) 2 = 6 a 2 sq units = Volume of outer cylinder
(ii) Lateral surface area of cube = 4 × (Edge ) 2 = 4 a 2 sq units − Volume of inner cylinder
= πR 2 h − πr 2 h
(iii) Diagonal of a cube = 3 × Edge = 3 a units
= π(R 2 − r 2 )h cu units
(iv) Volume of a cube = (Edge ) 3 = a 3 cu units
5. Sphere
3. Right Circular Cylinder
Cylinder is a solid figure obtained by revolving the rectangle, A sphere is a solid generated by the revolution of a
say ABCD, about its one side, say BC. Let base radius of right semi-circle about its diameter. Let radius of sphere be r
circular cylinder be r units and its height be h units. Then, units.
B A
A
r

h r
O

r
D B
C
Then,
(i) Curved surface area (CSA) (i) Surface area (SA) of sphere = 4 πr 2 sq units
= Circumference of the base × Height = 2πrh sq units 4
(ii) Total surface area (TSA) (ii) Volume of sphere = πr 3 cu units
3
= Curved surface area (CSA) + Area of two ends
6. Spherical Shell
= 2 πrh + 2 πr 2 = 2 πr( h + r ) sq units
If R and r are respectively the outer and inner radii of a
(iii) Volume of the cylinder = Area of base × Height spherical shell, then
= πr 2 h cu units (i) Outer surface area = 4 πR 2 sq units
4. Right Circular Hollow Cylinder (ii) Inner surface area = 4 πr 2 sq units
Let R units and r units be the external and internal radii of the 4
(iii) Volume of a hollow sphere = π(R 3 − r 3 ) cu units
hollow cylinder, respectively and h units be its height. 3
R
B
r
A
h O R

r
CBSE Term II Mathematics X (Standard) 119

7. Hemisphere e.g. A combined solid is formed by joining hemisphere and


A plane passing through the centre, cuts the sphere in two right circular cone.
equal parts, each part is called a hemisphere. Let radius of
hemisphere be r units. Then,
r O

(i) Curved surface area (CSA) of hemisphere = 2 πr 2 sq units (i) Surface area of combined solid figure
(ii) Total surface area (TSA) of hemisphere = CSA of cone + CSA of hemisphere
= CSA of hemisphere + Area of one end (ii) Volume of combined solid figure
= 2 πr 2 + πr 2 = 3 πr 2 sq units. = Volume of cone + Volume of hemisphere
2 While calculating the surface area, we have not added the
(iii) Volume of hemisphere = πr 3 cu units surface areas of the two individual solids, rather we have
3
added curved surface area because some part of the surface
8. Right Circular Cone area disappeared in the process of joining them. But this will
not be in the case, when we calculate the volume.
A right circular cone is a solid generated by the revolution of
a right angled triangle about one of its sides containing the Conversion of Solid from One Shape to
right angle as axis as shown in figure. Let height of a right
Another
circular cone be h units and its radius be r units. Then,
(i) Slant height of the cone, Sometimes, we need to convert solid figure of one shape to
another. When we come across objects which are converted
l = AC = r 2 + h 2 units from one shape to another or when a liquid which is
(ii) Curved surface area (CSA) of cone = πrl sq units originally filled in one container of a particular shape is
poured into another container of a different shape or size, the
(iii) Total surface area (TSA) of a cone
volume remains same. e.g.
= Curved surface area (CSA) + Area of the base
(i) If a solid metallic sphere is melted and recast into more
= πrl + πr 2 = πr( l + r ) sq units
than one spherical balls, then volume of metallic sphere
A = Sum of volumes of all spherical balls.
(ii) If the Earth taken out by digging a well and spreading it
uniformly around the well to form an embankment in the
h
l shape of a cylindrical shell from its original shape of right
circular cylinder, then volume of embankment
= Volume of Earth taken out by digging a well.
B r O C Important Results or Formulae
1 2 If a solid of one shape is converted into solid (or solids) of
(iv) Volume of cone = πr h cu units another shape, then
3
(i) Volume of the solid to be converted = Total volume of the
Combination of Two Solids solids into which the given solid is to be converted
Sometimes, we have to find the curved surface area and (ii) Number of solids of a given shape in which a given solid
volume of a solid, which is a combination of two solids. Then, is to be converted
for finding the surface area, we add the curved surface areas Volume of the solid to be converted
=
of individual solids and for finding the volume of this solid, Volume of one converted solid
we add the volumes of individual solids.
120 CBSE Term II Mathematics X (Standard)

Solved Examples
Example 1. Three metallic solid cubes whose edges are We know that,
3 cm, 4 cm and 5 cm are melted and formed into a Volume of cylinder = πr 2h
single cube. Find the edge of the cube so formed.
Sol. Given, edges of three solid cubes are 3 cm, 4 cm and 5 cm,
respectively. 66 cm
10cm
∴ Volume of first cube = ( 3)3 = 27 cm 3
[Q volume of cube = (side) 3]
Volume of second cube = ( 4)3 = 64 cm 3 13 cm
and volume of third cube = ( 5)3 = 125 cm 3 240 cm

∴ Sum of volume of three cubes = (27 + 64 + 125)


= 216 cm 3
Let the edge of the resulting cube = R cm
∴ Total volume of iron pole = Volume of first cylinder
Then, volume of the resulting cube, R 3 = 216 ⇒ R = 6 cm
+ Volume of second cylinder
Example 2. The volume of a right circular cylinder with = π (13)2 × 240 + π (10)2 × 66
1
its height equal to the radius is 25 cm 3 . Find the = π [169 × 240 + 100 × 66]
7 = 3.14 [ 40560 + 6600]
22
height of the cylinder. (Use π = ) = 3.14 × 47160
7 = 148082.4 cm 3
Sol. Let h and r be the height and radius of right circular
Hence, total mass of the iron pole
cylinder, respectively.
Given, height of cylinder = Radius of cylinder = 148082.4 × 8 g = 1184659.2 g
[given, 1 cm 3 ≈ 8 g]
i.e. h=r
1184659.2
Q Volume of cylinder = πr 2h = kg
1000
1 22 1
∴ 25 = × h 2 × h [Q h = r and V = 25 , given] = 1184.66 kg

Q 1g =
1
kg

7 7 7 ⎢⎣ 1000 ⎥⎦
176 22
⇒ = × h3
7 7 Example 4. A spherical metal ball of radius 8 cm is
⇒ h3 = 8 melted to make 8 smaller identical balls. The radius
⇒ 3 3
h =2 ⇒ h =2 [taking cube root]
of each new ball is ……… cm.
Sol. Let radius of larger sphere be R = 8 cm
Hence, height of cylinder is 2 cm.
and radius of smaller sphere be r cm
Example 3. An iron pole consists of a cylinder of height Let number of smaller sphere be n = 8
240 cm and base diameter 26 cm, which is According to the given condition,
surmounted by another cylinder of height 66 cm Volume of larger sphere = n × volume of smaller sphere
and radius 10 cm. Find the mass of the pole given 4 4
that 1 cm3 of iron has approximately 8 g mass. ∴ πR 3 = n × πr 3
3 3
[take, π = 3.14] ∴ ( 8) 3 = 8 × r 3
Sol. Here, solid iron pole is a combination of two cylinders. ⇒ r 3 = 82 ⇒ r 3 = 64 = ( 4)3
For first cylinder,
⇒ r = 4 cm [taking cube root]
Height = 240 cm
Hence, radius of new ball is 4 cm.
Base diameter = 26 cm
26 Example 5. A solid is in the shape of a cone mounted
∴ Base radius = cm = 13 cm
2 on a hemisphere of same base radius. If the curved
For second cylinder, surface areas of the hemispherical part and the
Height = 66 cm conical part are equal, then find the ratio of the
Radius = 10 cm radius and the height of the conical part.
CBSE Term II Mathematics X (Standard) 121

Sol. Let radius, height and slant height of a cone are r, h and l, Example 7. In figure, a tent is in the shape of a cylinder
respectively. Then, radius of hemisphere will be r.
surmounted by a conical top. The cylindrical part is
2.1 m high and conical part has slant height
h l 2.8 m. Both the parts have same radius 2 m.
r Find the area of the canvas used to make the tent.
⎡ 22 ⎤
⎢⎣Use π = 7 ⎥⎦

Now, curved surface area of cone C1 = πrl 2.8 m


and curved surface area of hemisphere, C 2 = 2 πr 2
According to the question,
C1 = C 2
2.1 m
∴ πrl = 2 πr 2 ⇒ l = 2 r …(i)
Also, l = r 2 + h 2
2m
⇒ (2 r ) = r 2 + h 2 [Q from Eq. (i)]
On squaring both sides, we get Sol. Given radius of conical and cylindrical part is r = 2 m .

(2 r ) 2 = ( r 2 + h 2 ) 2 ⇒ 4 r 2 = r 2 + h 2 Slant height of cone is l = 2. 8 m


2 2 2 2
And height of cylinder is h = 2.1 m
⇒ 3r = h ⇒ ( 3r ) = h
Taking square root both sides, we get 2.8 m
r 1
3r = h ⇒ =
h 3
Hence, the ratio of the radius and height of the conical part
is 1 : 3. 2.1 m
Example 6. A solid is in the shape of a hemisphere
surmounted by a cone. If the radius of hemisphere 2m
and base radius of cone is 7 cm and height of cone
is 3.5 cm, find the volume of the solid. ∴The area of the canvas used, to make the tent = curve
22 ⎤ surface area of cone + curve surface area of cylinder

⎢⎣ take, π = 7 ⎥⎦ = πrl + 2 πrh
22 22
= × 2 × 2. 8 + 2 × × 2 × 2.1
Sol. Given, radius of hemisphere and cone is r = 7 cm. 7 7
And height of cone ( h ) = 3.5 cm = 17. 6 + 26. 4 = 44 cm 2
A Hence, the area of the canvas used to make the tent is
44 cm 2.
3.5 cm
7 cm Example 8. From a solid right circular cylinder of
B C
height 14 cm and base radius 6 cm, a right circular
7 cm cone of same height and same base radius is
removed. Find the volume of the remaining solid.
Sol. Given radius and height of cylinder are
1 2 1 22
Now, volume of cone V1 = πr h = × × (7 )2 × 3.5 r = 6 cm and h = 14 cm
3 3 7
= 179.67 cm 3
2 2 22
and Volume of hemisphere , V2 = πr 3 = × × (7 ) 3
3 3 7
14 cm
= 718.67 cm 3
∴ The volume of solid figure
= Volume of cone + Volume of hemisphere
= V1 + V2 6 cm
= 179.67 + 718. 67 = 898.34 cm 3 Also, radius and height of cone will be
Hence, volume of solid shape is 898.34 cm 3. r1 = 6 cm and h1 = 14 cm
122 CBSE Term II Mathematics X (Standard)

Now, volume of cylinder, Example 10. Two cones with same base radius 8 cm and
V1 = πr12h height 15 cm are joined together along their bases.
22 Find the surface area of the shape so formed.
= × ( 6)2 × 14 = 1584 cm 3
7 Sol. If two cones with same base and height are joined together
1 along their bases, then the shape so formed is look like as
Volume of cone, V2 = πr12h1
3 figure shown.
1 22
= × × ( 6)2 × 14 = 528 cm 3
3 7 8 cm
∴Volume of remaining solid
= Volume of cylinder − Volume of cone 16 cm
15 cm
= V1 − V2 8 cm
= 1584 − 528 = 1056 cm 3
Hence, volume of the remaining solid is 1056 cm 3.
30 cm
Example 9. An ice-cream cone full of ice-cream having
radius 5 cm and height 10 cm as shown in figure Given that, radius of cone, r = 8 cm and height of cone,
5 cm h = 15 cm
So, surface area of the shape so formed
= Curved area of first cone
+ Curved surface area of second cone
= 2 ⋅ Surface area of cone [since, both cones are identical]
10 cm = 2 × πrl = 2 × π × r × r 2 + h 2
22 2 × 22 × 8 × 64 + 225
=2 × × 8 × ( 8)2 + (15)2 =
7 7
44 × 8 × 289 44 × 8 × 17
= =
7 7
Calculate the volume of ice-cream, provided that its 5984 2
= = 854. 85 cm
1 7
part is left unfilled with ice-cream.
6 = 855 cm 2 (approx.)
Sol. Given, ice-cream cone is the combination of a hemisphere Hence, the surface area of shape so formed is 855 cm 2.
and a cone.
Also , radius of hemisphere = 5 cm
Example 11. The barrel of a fountain pen, cylindrical in
2 2 22 shape, is 7 cm long and 0.5 cm in diameter. A full
∴ Volume of hemisphere = πr 3 = × × ( 5) 3 barrel of ink in the pen can be used for writing 275
3 3 7
5500 words on an average. How many words would be
= = 261. 90 cm 3 written using a bottle of ink containing one-fourth
21
of a litre? [CBSE 2015, 14]
Now, radius of the cone = 5 cm
Sol. Given, height of cylindrical pen = 7 cm
and height of the cone = 10 − 5 = 5 cm
1 Diameter 0. 5
∴ Volume of the cone = πr 2h Radius = = cm
3 2 2
1 22 ∴Volume of barrel of a fountain pen = πr 2h
= × × ( 5) 2 × 5 2
3 7 22 ⎛ 0. 5⎞ 22
= ×⎜ ⎟ ×7= cm 3
2750 7 ⎝ 2 ⎠ 16
= = 130. 95 cm 3
21 It is given that, a pen can write 275 words by using the ink
22
Now, total volume of ice-cream cone cm 3.
= 261.90 + 130.95 = 392.85 cm 3 16
∴ Volume of ink = 275 words
1 22
Since, part is left unfilled with ice-cream. ⇒ cm 3 = 275 words
6 16
1 1 275 × 16 1
∴Required volume of ice-cream = 392.85 − 392.85 × ⇒ × 1000 cm 3 = × × 1000 = 50000
6 4 22 4
1
= 392.85 − 65.475 [Q he will use L of ink to write words]
4
= 327.4 cm 3 1
Hence, the pen can write 50000 words by L of ink.
4
CBSE Term II Mathematics X (Standard) 123

Example 12. 500 persons are taking a dip into a Sol. It is clear, from the figure, length = 20 m
cuboidal pond which is 80 m long and 50 m broad. 1
and width = m of each step.
What is the rise of water level in the pond, if the 2
1
average displacement of the water by a person is and height of Ist step which is in the bottom = m
4
0.04 m 3 ? 1 1
Sol. Let the rise of water level in the pond be h m when 500 ∴Height of second step = 2 × = m
4 2
persons are taking a dip into a cuboidal pond. 1 3
Height of third step = 3 × = m
4 4
h M M
1 10
Height of tenth step = 10 × = m
4 4
Water Total volume of the concrete used
1 1 1 2 1 3 1 10
= 20 × × + 20 × × + 20 × × + ... + 20 × ×
2 4 2 4 2 4 2 4
Given that, [Q volume of cuboid = l × b × h]
1 1
Length of the cuboidal pond = 80 m = 20 × × [1 + 2 + 3 + ... + 10]
2 4
Breadth of the cuboidal pond = 50 m
1 1 10 × 11 ⎡ n ( n + 1) ⎤
Now, volume for the rise of water level in the pond = 20 × × × Q 1 + 2 + ... + n =
2 4 2 ⎢⎣ 2 ⎥⎦
= Length × Breadth × Height 3
= 137.5 m
= 80 × 50 × h
= 4000 h m 3 Example 14. A wall 24 m long, 0.4 m thick and 6 m
and the average displacement of the water by a person high is constructed with the bricks each of
= 0. 04 m 3 dimensions 25 cm × 16 cm × 10 cm. If the mortar
1
So, the average displacement of the water by 500 persons occupies th of the volume of the wall, then find
= 500 × 0. 04 m 3
10
the number of bricks used in constructing the wall.
Now, by given condition,
Sol. Given that, a wall is constructed with the help of bricks and
Volume for the rise of water level in the pond = Average mortar.
displacement of the water by 500 persons
∴Number of bricks
⇒ 4000 h = 500 × 0. 04
⎛1 ⎞
500 × 0. 04 20 1 (Volume of wall) − ⎜ th volume of wall ⎟
∴ h= = = m ⎝ 10 ⎠
4000 4000 200 = ...(i)
Volume of a brick
= 0. 005 m Also, given that
= 0. 005 × 100 cm Length of a wall ( l ) = 24 m ,
[Q 1 m = 100 cm ] Thickness of a wall ( b ) = 0. 4 m,
= 0. 5 cm Height of a wall ( h ) = 6 m
Hence, the required rise of water level in the pond is 0.5 cm. So, volume of a wall constructed with the bricks = l × b × h
Example 13. A small terrace at a hockey ground = 24 × 0. 4 × 6
comprises of 10 steps each of which 20 m long and 24 × 4 × 6 3
= m
1 10
built of solid concrete. Each step has a rise of m 1 1 24 × 4 × 6
4 Now, th volume of a wall = ×
1 10 10 10
and a tread of m. Calculate the total volume of 24 × 4 × 6 3
2 = m
concrete required to build the terrace. 102
25
and Length of a brick ( l1 ) = 25 cm = m
100
16
Breadth of a brick ( b1 ) = 16 cm = m
1m 100
2 10
Height of a brick ( h1 ) = 10 cm = m
100
m

So, volume of a brick = l1 × b1 × h1


20

1m 1 25 16 10 25 × 16 3
4 m 3m = × × = m
2 4 100 100 100 105
124 CBSE Term II Mathematics X (Standard)

From Eq. (i), Since, the water is flowing at the rate of 5 km/h.
⎛ 24 × 4 × 6 24 × 4 × 6⎞ Therefore, length of the water flow in x h
⎜ − ⎟
⎝ 10 100 ⎠ = 5x km = 5000x m
Number of bricks =
⎛ 25 × 16⎞ [Q 1 km = 1000 m]
⎜ ⎟
⎝ 105 ⎠ We have, diameter of cylindrical pipe = 14 cm
14 7
24 × 4 × 6 10 5 ∴ Radius of cylindrical pipe, r = = 7 cm = m
= ×9× 2 100
100 25 × 16
Volume of the water flowing through the cylindrical
24 × 4 × 6 × 9 × 1000
= pipe in
25 × 16 2
22 ⎛ 7 ⎞
= 24 × 6 × 9 × 10 = 12960 x h = πr 2h = ×⎜ ⎟ × 5000x
7 ⎝ 100⎠
Hence, the required number of bricks used in constructing
the wall is 12960. = 77 x m 3
Also, volume of the water that falls into the tank in x h
Example 15. Water is flowing at the rate of 5 km/h
= l×b×h
through a pipe of diameter 14 cm into a rectangular 7
tank which is 50 m long and 44 m wide. Determine the = 50 × 44 ×
100
time in which the level of the water in the tank will rise
= 154 m 3
by 7 cm.
⎡ 7 ⎤
14 cm ⎢Q l = 50 m, b = 44 m and h = radius = 100 m ⎥
⎣ ⎦
Q Volume of the water flowing through the cylindrical pipe
in x h = Volume of water that falls in the tank in x h
⇒ 77 x = 154
7 cm
⇒ x =2
44 m Hence, the level of water in the tank will rise by 7 cm in 2 h.
50 m
Sol. Suppose, the level of the water in the tank will rise by 7 cm
in x h.
CBSE Term II Mathematics X (Standard) 125

Chapter
Practice
7. A cylindrical pencil sharpened at one edge is the
PART 1 combination of
(a) a cone and a cylinder
Objective Questions (b) cube and a cylinder
(c) a hemisphere and a cylinder
!
Multiple Choice Questions (d) two cylinders

1. Three cubes each of side 5 cm are joined end to 8. A surahi is the combination of
end, then the surface area of the resulting solid is (a) a sphere and a cylinder
(a) 250 cm 2 (b) 180 cm 2 (b) a hemisphere and a cylinder
(c) 350 cm 2 (d) None of these (c) two hemispheres
(d) a cylinder and a cone
2. A solid ball is exactly fitted inside the cubical box of
side a. The volume of the ball is 9. Two cones have their heights in the ratio 1 : 3 and
1 4 radii in the ratio 3 : 1, then the ratio of their
(a) πa 3 (b) πa 3
6 3 volumes is
1 (a) 1 : 3 (b) 3 : 1
(c) πa 3 (d) None of these
3 (c) 2 : 3 (d) 3 : 2
3. A cubical icecream brick of edge 22 cm is to be 10. The shape of a gilli, in the gilli-danda game (see
distributed among some children by filling figure) is a combination of
icecream cones of radius 2 cm and height 7 cm upto
its brim. How many children will get icecream
cones? (a) two cylinders
(a) 163 (b) 263 (b) a cone and a cylinder
(c) 363 (d) 463 (c) two cones and a cylinder
4. A right circular cylinder of radius r cm and height h (d) two cylinders and a cone
cm ( where, h > 2 r ) just encloses a sphere of 11. A plumbline (sahul) is the combination of
diameter (see figure)
(a) r cm (b) 2r cm
(c) h cm (d) 2h cm
5. If two solid hemispheres of same base radius r are
joined together along their bases, then curved
surface area of this new solid is
(a) 4πr 2 (b) 6πr 2
(a) a cone and a cylinder (b) a hemisphere and a cone
(c) 3πr 2 (d) 8πr 2
(c) cube and a cylinder (d) sphere and cylinder
6. A solid cylinder of radius r and height h is placed
over other cylinder of same height and radius.
12. A solid cone of radius r and height h is placed over
a solid cylinder having same base radius and height
The total surface area of the shape so formed is
as that of a cone. The total surface area of the
(a) 4πr( h 2 + r 2 )
combined solid is
(b) 4πr[ h + r ] (a) πrl + 2 πrh (b) πr 2( l + 2 h )
(c) 4π ( h 2 + r 2 )
2 2
(c) πr [ r + h + 2 h + r ] (d) None of these
(d) None of the above
126 CBSE Term II Mathematics X (Standard)

13. The capacity of a cylindrical vessel with a 20. A wooden article was made by scooping out a
hemispherical portion raised upward at the bottom hemisphere from each end of a solid cylinder, as
πr 2 shown in figure. If the height of the cylinder is
as shown in the figure is [ 3h − 2 r ]. 10 cm and its base is of radius 3.5 cm. Find the total
3
surface area of the article.
r cm

h cm

1 2
(a) πr 2[2 h − 3r ] (b) πr 2[ 3h − 2 r ] [CBSE 2018]
3 3
(a) 374 cm 2 (b) 370 cm 2
1 2
(c) πr [ 3h − 2 r ] (d) None of these (c) 475 cm 2 (d) None of these
3
21. A heap of rice is in the form of a cone of base
14. The diameter of a sphere is 6 cm. It is melted and
diameter 24 m and height 3.5 m. Find the volume of
drawn into a wire of diameter 2 mm.
the rice. How much canvas as cloth is required to
The length of the wire is
just cover the heap? [CBSE 2018]
(a) 12 m (b) 18 m
(a) 105.5 m 2 (b) 471.42 m 2
(c) 36 m (d) 66 m
(c) 173.5 m 2 (d) None of these
15. During conversion of a solid from one shape to
another, the volume of the new shape will 22. A mason constructs a wall of dimensions
(a) increase (b) decrease
270 cm × 300 cm × 350 cm with the bricks each of
(c) remain unaltered (d) be doubled size 22.5 cm × 11.25 cm × 8.75 cm and it is assumed
1
16. From a solid circular cylinder with height 10 cm that space is covered by the mortar. Then, the
8
and radius of the base 6 cm, a right circular cone of
number of bricks used to construct the wall is
the same height and same base is removed, then the
(a) 11100 (b) 11200 (c) 11000 (d) 11300
volume of remaining solid is
(a) 280 π cm 3 (b) 330 π cm 3 !
Case Based MCQs
3 3
(c) 240 π cm (d) 440 π cm
23. To make the learning process more interesting
17. A 20 m deep well, with diameter 7 m is dug and the creative and innovative Shavya’s class teacher
earth from digging is evently spread out to form a brings clay in the classroom, to teach the topic.
platform 22 m by 14 m. The height of the platform Surface Areas and Volumes. With clay, she forms a
is cylinder of radius 4 cm and height 18 cm. Then, she
(a) 2.5 m (b) 3.5 m moulds the cylinder into a sphere and ask some
(c) 3 m (d) 2 m question to students.
18. If the radius of the base of a right circular cylinder
is halved, keeping the height same, then find the
ratio of the volume of the cylinder thus obtained to
the volume of original cylinder. [CBSE 2009]
1 1 1 1
(a) (b) (c) (d)
3 4 2 5
19. Marbles of diameter 1.4 cm are dropped into a
cylindrical beaker of diameter 7 cm containing
some water. The water level rises by 5.6 cm. When
marble dropped into the beaker, then the number of
marble is (i) The radius of the sphere so formed is
(a) 150 (b) 160
(a) 4 cm (b) 6 cm
(c) 175 (d) 235
(c) 7 cm (d) 8 cm
CBSE Term II Mathematics X (Standard) 127

(ii) The volume of the sphere so formed is 25. The Great Stupa at Sanchi is one of the oldest stone
(a) 905.14 cm3 (b) 903.27 cm3 structures in India, and an important monument of
(c) 1296.5 cm3 (d) 1156.63 cm3 Indian Architecture. It was originally
(iii) Find the ratio of the volume of sphere to the commissioned by the emperor Ashoka in the 3rd
volume of cylinder. century BCE. Its nucleus was a simple
(a) 2 : 1 (b) 1 : 2 hemispherical brick structure built over the relics
(c) 1 : 1 (d) 3 : 1 of the Buddha. It is a perfect example of
(iv) Total surface area of the cylinder is combination of solid figures. A big hemispherical
(a) 553.14 cm2 (b) 751.52 cm2 dome with a cuboidal structure mounted on it.
(c) 625 cm2 (d) 785.38 cm2
22
(take π = )
(v) During the conversion of a solid from one shape to 7 [CBSE Question Bank]
another the volume of new shape will
(a) be increase (b) be decrease
(c) remain unaltered (d) be double
24. Geeta and Meena have 10 and 6 CD respectively,
each of radius 4 cm and thickness 1 cm. They place
their CD one above the other to form solid
cylinders.

Chattra
Dome Harmika
Toranas Balustrade

Stairs

Based on the above information, answer the


following questions.
(i) Curved surface area of the cylinder made by Geeta
is (i) Calculate the volume of the hemispherical dome if
(a) 308.17 cm2 (b) 132 cm2 the height of the dome is 21 m.
(c) 154 cm2 (d) 251.42 cm2 (a) 19404 cu m (b) 2000 cu m
(ii) The ratio of curved surface area of the cylinder (c) 15000 cu m (d) 19000 cu m
made by Geeta and Meena is (ii) The formula to find the volume of sphere is
(a) 3 : 5 (b) 3 : 2 2 4
(a) π r 3 (b) πr 3
(c) 5 : 3 (d) 5 : 7 3 3
(iii) The volume of the cylinder made by Meena is (c) 4π r 2 (d) 2 πr 2
(a) 301.44 cm3 (b) 144 cm3 (iii) The cloth require to cover the hemispherical dome
(c) 132 cm3 (d) 208.42 cm3 if the radius of its base is 14m is
(iv) The ratio of the volume of the cylinders made by (a) 1222 sq m (b) 1232 sq m
Geeta and Meena is (c) 1200 sq m (d) 1400 sq m
(a) 1 : 2 (b) 2 : 5 (iv) The total surface area of the combined figure
(c) 3 : 5 (d) 5 : 3 i.e. hemispherical dome with radius 14 m and
(v) When two CD Cassette are shifted from Geeta cuboidal shaped top with dimensions 8 m × 6 m
cylinder to Meena’s cylinder, then × 4 m is
(a) Volume of two cylinder become equal (a)1200 sq m (b) 1232 sq m
(b) Volume of Geeta’s cylinder > Volume of Meena’s (c) 1392 sq m (d) 1932 sq m
cylinder (v) The volume of the cuboidal shaped top is with
(c) Volume of Meena’s cylinder > Volume of Geeta’s dimensions mentioned in question (iv).
cylinder (a) 182.45 m 3 (b) 282.45 m 3
(d) None of the above (c) 292 m 3 (d) 192 m 3
128 CBSE Term II Mathematics X (Standard)

9. A building is in the form of a cylinder surmounted


PART 2 by a hemispherical dome (see the figure). The base
2
Subjective Questions diameter of the dome is equal to of the total
3
height of the building. Find the height of the
!
Short Answer Type Questions 1
building, if it contains 67 m 3 of air.
1. Two identical cubes each of volume 64 cm 3 are 21
joined together end to end. What is the surface area r
r
of the resulting cuboid?
2. How many shots each having diameter 3 cm can be H
made from a cuboidal lead solid of dimensions h
9 cm × 11 cm × 12 cm?
3. 16 glass spheres each of radius 2 cm are packed into
a cuboidal box of internal dimensions 16 cm × 8 cm 10. Twelve solid spheres of the same size are made by
× 8 cm and then the box is filled with water. Find melting a solid metallic cylinder of base diameter
the volume of water filled in the box. 2 cm and height 16 cm. Find the diameter of each
4. If a solid piece of iron in the form of a cuboid of sphere.
dimensions 49 cm × 33 cm × 24 cm, is moulded to 11. A solid is composed of a cylinder with
form a solid sphere. Then, find radius of the sphere. hemispherical ends. If the whole length of the solid
5. If volumes of two spheres are in the ratio 64 : 27, is 104 cm and the radius of each hemispherical end
then find the ratio of their surface areas. is 7 cm, then find the cost of polishing its surface at
6. The decorative block shown in the following figure the rate of ` 2 per dm 2 .
is made of two solids, a cube and a hemisphere. ⎡ 22 ⎤
The base of the block is a cube with edge 6 cm and ⎢⎣ take, π = 7 ⎥⎦
the hemisphere fixed on the top has a diameter of
2.1 cm, then find the total surface area of the 12. A solid metallic hemisphere of radius 8 cm is
block and find the total area to be painted. melted and recasted into a right circular cone of
base radius 6 cm. Determine the height of the
⎡ 22 ⎤
cone.
⎢⎣ take, π = 7 ⎥⎦
2.1 cm 13. A rectangular water tank of base 11 m × 6 m
contains water upto a height of 5 m. If the water in
the tank is transferred to a cylindrical tank of
radius 3.5 m, find the height of the water level in
6 cm the tank.
6 cm
14. A copper rod of diameter 1 cm and length 8 cm is
drawn into a wire of length 8 m of uniform
thickness. Find the thickness of the wire.
6 cm 15. The rain water from a roof of dimensions 22 m
× 20 m drains into a cylindrical vessel having
7. From a solid cube of side 7 cm, a conical cavity of diameter of base 2 m and height 3.5 m. If the rain
height 7 cm and radius 3 cm is hollowed out. Find
water collected from the roof just fill the
the volume of the remaining solid.
cylindrical vessel, then find the rainfall (in cm).
8. A hemispherical bowl of internal radius 9 cm is full 16. A cylindrical bucket of height 32 cm and base
of liquid. The liquid is to be filled into cylindrical
radius 18 cm is filled with sand. This bucket is
shaped bottles each of radius 1.5 cm and height
emptied on the ground and a conical heap of sand
4 cm. How many bottles are needed to empty the
is formed. If the height of the conical heap is 24
bowl?
cm, find the radius and slant height of the heap.
CBSE Term II Mathematics X (Standard) 129

17. The barrel of a fountain pen, cylindrical in shape, is 26. A medicine-capsule is in the shape of a cylinder of
7 cm long and 5 mm in diameter. A full barrel of ink diameter 0.5 cm with two hemispheres stuck to
in the pin is used up on writing 3300 words on an each of its ends. The length of entire capsule is
average. How many words can be written in a bottle 2 cm. The capacity of the capsule is
of ink containing one-fifth of a litre? 27. A rocket is in the form of a right circular cylinder
18. Water flows at the rate of 10 m min −1 through a closed at the lower end and surmounted by a cone
cylindrical pipe 5 mm in diameter. How long would it with the same radius as that of the cylinder. The
take to fill a conical vessel whose diameter at the diameter and height of the cylinder are 6 cm and
base is 40 cm and depth 24 cm? 12 cm, respectively. If the slant height of the
19. Water flows through a cylindrical pipe, whose inner conical portion is 5 cm, then find the total surface
radius is 1 cm, at the rate of 80 cms −1 in an empty area and volume of the rocket. [use π = 3.14]
cylindrical tank, the radius of whose base is 40 cm. 28. A solid toy is in the form of a hemisphere
What is the rise of water level in tank in half an hour? surmounted by a right circular cone. The height
20. A factory manufactures 120000 pencils daily. The of the cone is 3 cm and the diameter of the base is
pencils are cylindrical in shape each of length 25 cm 4 cm. Determine the volume of the solid toy. If a
and circumference of base as 1.5 cm. Determine the right circular cylinder circumscribes the toy, then
cost of colouring the curved surfaces of the pencils find the difference of the volumes of the cylinder
manufactured in one day at ` 0.05 per dm 2 . and the toy. [take, π = 3 . 14]

21. A well of diameter 10 m is dug 14 m deep. The Earth 29. A wooden toy rocket is in the shape of a cone
taken out of it is spread evenly all around to a width mounted on a cylinder, as shown in figure. The
of 5 m to form an embankment. Find the height of height of the entire rocket is 24 cm, while the
embankment. height of the conical part is 4 cm.

22. Marbles of diameter 1.4 cm are dropped into a 4 cm


cylindrical beaker of diameter 7 cm containing some
24 cm
water. Find the number of marbles that should be
dropped into the beaker, so that the water level rises
by 5.6 cm. 4 cm 6 cm

!
Long Answer Type Questions
Base of cylinder
23. If a hollow cube of internal edge 22 cm is filled with Base of cone
spherical marbles of diameter 0.5 cm and it is
1 The base of the conical portion has a diameter of
assumed that space of the cube remains unfilled.
8 6 cm, while the base diameter of the cylindrical
Then, the number of marbles that the cube can portion is 4 cm. If the conical portion is to be
accomodate is painted orange and the cylindrical portion yellow,
24. A solid iron cuboidal block of dimensions then find the area of the rocket painted with each
4.4 m × 2.6 m × 1 m is recast into a hollow cylindrical of these colours. [take, π = 3.14]
pipe of internal radius 30 cm and thickness 5 cm. 30. Two solid cones A and B are placed in a cylindrical
Find the length of the pipe. tube as shown in the figure. The ratio of their
25. A building is in the form of a cylinder surmounted by capacities is 2 : 1. Find the heights and capacities
19 3 of cones. Also, find the volume of the remaining
a hemispherical vaulted dome and contains 41 m portion of the cylinder.
21
21 cm
of air. If the internal diameter of dome is equal to its
total height above the floor, find the height of the
6 cm

A B
building?
130 CBSE Term II Mathematics X (Standard)

31. How many spherical lead shots of diameter 4 cm


can be made out of a solid cube of lead whose edge
measures 44 cm.
32. A metallic spherical shell of internal and external
diameters 4 cm and 8 cm, respectively is melted
and recast into the form of a cone of base diameter
8 cm. Find the height of the cone.
33. How many spherical lead shots each of diameter
4.2 cm can be obtained from a solid rectangular
lead piece with dimensions 66 cm, 42 cm and (i) Ram Mandir is constructed in the form of the
21 cm? cubical base of 30 cm × 20 cm × 10 cm, then find
34. Find the number of metallic circular disc with the area covered.
1.5 cm base diameter and of height 0.2 cm to be (ii) If the radius of the cylinder is 7 cm and Height of
melted to form a right circular cylinder of height the cylinder is 60 cm and the radius of the cone is
10 cm and diameter 4.5 cm. similar to that of cylinder and Height of the cone is
24 cm, then the ratio of curved surface area of
35. A heap of rice is in the form of a cone of diameter cylinder to curved surface area of the cone.
9 m and height 3.5 m. Find the volume of the rice.
(iii) Given structure in based on the concept of
How much canvas cloth is required to just cover
heap? (a) Area and perimeter
(b) Surface area and volume
36. How many cubic centimetres of iron is required to
construct an open box whose external dimensions (c) Both (a) and (b)
are 36 cm, 25 cm and 16.5 cm provided the (d) None of the above
thickness of the iron is 1.5 cm. If one cubic 39. Adventure camps are the perfect place for the
centimetre of iron weights 7.5 g, then find the children to practice decision making for themselves
weight of the box. without parents and teachers guiding their every
37. Water is flowing at the rate of 15 kmh −1 through a move. Some students of a school reached for
pipe of diameter 14 cm into a cuboidal pond which adventure at Sakleshpur. At the camp, the waiters
is 50 m long and 44 m wide. In what time will the served some students with a welcome drink in a
level of water in pond rise by 21 cm? cylindrical glass and some students in a
hemispherical cup whose dimensions are shown
!
Case Base Questions below.
38. Mathematics teacher of a school took her 10th
standard students to show Ram Mandir. It was a
part of their Educational trip. The teacher had
interest in history as well. She narrated the facts of
Ram Mandir to students.
Ram mandir is a Hindu temple that is being built in
Ayodhya, which is in Uttar Pradesh. The temple
construction is being supervised by the Shri Ram
Janmabhoomi Teerth Kshetra.
Then the teacher said in this monuments one can
find combination of solid figures. She pointed that d=7 cm
there are cubical bases and in centre cylinder with h=10.5 cm d=7 cm
the cone shape structure on the top is constructed.
CBSE Term II Mathematics X (Standard) 131

After that they went for a jungle trek. The jungle trek 40. On a Sunday, your Parents took you to a fair.
was enjoyable but tiring. As dusk fell, it was time to take You could see lot of toys displayed, and you
shelter. Each group of four students was given a canvas wanted them to buy a RUBIK’s cube and
of area 551m2. Each group had to make a conical tent to strawberry ice-cream for you.
accommodate all the four students. Assuming that all the Observe the figures and answer the questions.
stitching and wasting incurred while cutting, would [CBSE Question Bank]
amount to 1 m2, the students put the tents. The radius of
the tent is 7 m. [CBSE Question Bank]

(i) Find the length of the diagonal if each edge


measures 6cm.
(ii) Find volume of the solid figure if the length
of the edge is 7cm.
Area = 551 m2
(iii) What is the curved surface area of
r=7m hemisphere (ice-cream) if the base radius is 7
cm?
(iv) Find the slant height of a cone if the radius is
(i) Find the volume of cylindrical cup. 7 cm and the height is 24 cm
(ii) Find the volume of hemispherical cup. (v) Find the total surface area of cone with
(iii) Find the height of the conical tent prepared to hemispherical ice cream.
accommodate four students.

SOLUTIONS
Objective Questions
1. (c) Here, on joining three cubes, we get a cuboid whose 3. (c) Given, volume of brick = 22 3 cm 3
length, l = 5 + 5 + 5 = 15 cm, breadth, b = 5 cm and 1
height, h = 5 cm ∴Volume of 1 cone = πr 2h
3
5 cm 5 cm 5 cm 1 22 22 × 4
= × ×2 ×2 ×7=
3 7 3
5 cm Let number of cones = n
I II III
4
Then, n × 22 × = 22 × 22 × 22
5 cm 3
22 × 22 × 3
⇒ n=
∴ Required surface area of the resulting solid 4
= Surface area of new cuboid ∴ n = 121 × 3 = 363
= 2 ( lb + bh + hl ) = 2 (15 × 5 + 5 × 5 + 5 × 15)
4. (b) Because the sphere encloses in the cylinder,
= 2 (75 + 25 + 75) = 2 (175) = 350 cm 2 therefore the diameter of sphere is equal to diameter
2. (a) Because solid ball is exactly fitted inside the cubical box of side of cylinder which is 2r cm.
a. So, a is the diameter for the solid ball. 5. (a) Because curved surface area of a hemisphere is
∴ Radius of the ball =
a 2 πr 2 and here, we join two solid hemispheres along
2 their bases of radius r, from which we get a solid
4 ⎛ a⎞ 1
3 sphere.
So, volume of the ball = π ⎜ ⎟ = πa 3 Hence, the curved surface area of new solid
3 ⎝2⎠ 6
= 2 π r 2 + 2 π r 2 = 4π r 2
132 CBSE Term II Mathematics X (Standard)

6. (d) Since, the total surface area of cylinder of radius r and and total surface area of a cylinder of base radius, r and
height h = 2 πrh + 2 πr 2 height, h
When one cylinder is placed over the other cylinder of same = Curved surface area + Area of both base
height and radius, = 2 πrh + 2 πr 2
then height of the new cylinder = 2 h Here, when we placed a cone over a cylinder, then one base
and radius of the new cylinder = r is common for both.
∴Total surface area of the new cylinder = 2 πr(2 h ) + 2 πr 2 So, total surface area of the combined solid
= 4πrh + 2 πr 2 = πrl + 2 πrh + πr 2
= 2 πr(2 h + r ) = πr [ l + 2 h + r ]
7. (a) Because the shape of sharpened pencil is = πr ⎡ r 2 + h 2 + 2 h + r ⎤
⎣ ⎦
13. (c) We know that, capacity of cylindrical vessel = πr 2h cm 3
2
= + and capacity of hemisphere = πr 3 cm
3
= Cylinder + Cone From the figure, capacity of the cylindrical vessel
2 1
8. (a) Because the shape of surahi is = πr 2h − πr 3 = πr 2 [ 3h − 2 r ]
3 3
14. (c) We have, diameter of metallic sphere = 6 cm
∴Radius of metallic sphere, r1 = 3 cm
Also, diameter of cross-section of cylindrical wire = 0.2 cm
= + = Sphere + Cylinder
∴Radius of cross-sections of cylindrical wire, r2 = 0.1 cm
Let the length of the wire be h cm.
Since, metallic sphere is converted into a cylindrical shaped
9. (b) Let the radii of two cones are r1 , r2 and their heights are wire of length h cm.
h1 and h 2. ∴Volume of the metal used in wire = Volume of the sphere
r 3 h 1 4
Given, 1 = and 1 = ⇒ πr22h = πr13
r2 1 h2 3 3
2
1 2 ⎛ 1⎞ 4
πr h 2
⇒ π × ⎜ ⎟ × h = × π × 27
V1 3 1 1 ⎛ r1 ⎞ ⎛ h1 ⎞ ⎝ 10⎠ 3
Now, the ratio of their volumes, = =⎜ ⎟ ⎜ ⎟
V2 1 πr 2h ⎝ r2 ⎠ ⎝ h 2 ⎠ 1
2 2 ⇒ π× × h = 36π
3
2
100
⎛ 3⎞ ⎛ 1⎞ 1 3 36π × 100
= ⎜ ⎟ ×⎜ ⎟ = 9 × = = 3:1 ⇒ h=
⎝ 1⎠ ⎝ 3⎠ 3 1 π
Hence, the ratio of their volumes is 3 : 1. = 3600 cm = 36 m
10. (c) [Q 1m = 100 cm ]
15. (c) During conversion of a solid from one shape to another,
the volume of the new shape will remain unaltered.
= + + 16. (c) Volume of the remaining solid
= Volume of the cylinder − Volume of the cone
= Cone + Cylinder + Cone
⎧ 1 ⎫
= Two cones and a cylinder = ⎨π × 62 × 10 − × π × 62 × 10⎬
⎩ 3 ⎭
11. (b)
= ( 360π − 120π ) = 240π cm 3
7
17. (a)Q Radius of the well = m = 3. 5 m
2
+ = Hemisphere + Cone 22
= ∴Volume of the earth dug out = × ( 3. 5)2 × 20
7
22
= × 3. 5 × 3. 5 × 20
7
12. (c) We know that, total surface area of a cone of radius, r = 770 m 3
and height, h = Curved surface Area + area of base
Area of platform = (22 × 14) m 2 = 308 m 2
= πrl + πr 2
770
where, l = h 2 + r2 ∴ Height = = 2.5 m
308
CBSE Term II Mathematics X (Standard) 133

18. (b) Let the radius of original right circular cylinder be ‘r’ cm. 1 2
Now, volume of rice = volume of cone = πr h
and height be ‘h’ cm. Then, 3
Volume of original cylinder V1 = ( πr 2h ) cm 3 1 22 ⎡ d 24 ⎤
= × × (12 )2 × 3. 5 Qr = = = 12 m
3 7 ⎢⎣ 2 2 ⎥⎦
Volume of circular cylinder, when radius is halved
2 22 × 144 × 3. 5
⎛ r⎞ πr 2h =
V2 = π ⎜ ⎟ h cm 3 = cm 3 21
⎝ 2⎠ 4
11088
V2 πr 2h 1 1 = = 528 m3
Thus, = × = . 21
V1 4 π r 2h 4
Now, slant height l = h 2 + r 2
4
19. (a) So, volume of one spherial marble = π (0.7)3
3 = ( 3. 5)2 + (12 )2 = 12 . 25 + 144
⎡ 4 ⎤
Q volume of sphere = πr 3 = 156.25 = 12 . 5 m
⎢⎣ 3 ⎥⎦
∴The canvas required to cover the heap = πrl
1.372 3
= π cm 22 3300
3 = × 12 × 12 . 5 = = 471. 42 m2
7 7
∴ Volume of the raised water in beaker = π (3.5)2 × 5.6
22. (b) Volume of the wall = 270 × 300 × 350 = 28350000 cm 3
[Q volume of cylinder = πr 2h ]
[Q volume of cuboid = length × breadth × height]
= 68.6 π cm 3
1
Now, required number of marbles Since, space of wall is covered by mortar.
8
Volume of the raised water in beaker
= So, remaining space of wall = Volume of wall
Volume of one spherical marble
− Volume of mortar
68.6 π
= × 3 = 150 marbles 1
1.372 π = 28350000 − 28350000 ×
8
20. (a) Given, wooden article is a combination of a cylinder and = 28350000 − 3543750
two hemispheres.
3.5 cm = 24806250 cm 3
Now, volume of one brick = 22 . 5 × 11.25 × 8.75
= 2214. 844 cm 3
[Q volume of cuboid = length × breadth × height]
10 cm 24806250
∴ Required number of bricks = = 11200 (approx.)
2214. 844
Hence, the number of bricks used to construct the wall is
11200.
23. (i) (b) Since, volume of sphere = Volume of cylinder
4
Here, height of the cylinder, h = 10 cm ⇒ πR 3 = πr 2h , where R , r are the radii of sphere and
3
Q Radius of base of the cylinder cylinder, respectively.
= Radius of hemisphere, r = 3.5 cm 3
Now, required TSA of the wooden article ⇒ R 3 = r2 × h ×
4
= 2 × CSA of one hemisphere + CSA of cylinder 3 4 × 4 × 18 × 3
⇒ R = = 8 × 27 ⇒ R 3 = (2 × 3)3
= 2 × (2 π r 2 ) + 2 π r h 4
= 2 π r (2 r + h ) ∴ R = 6 cm
22 4 4 22
=2 × × 3.5 × (2 × 3.5 + 10) (ii) (a) Volume of sphere = πR 3 = × ×6×6×6
7 3 3 7
22
= × 7 × ( 7 + 10) = 22 × 17 = 374 cm 2 = 905.14 cm3
7
(iii) (c) Since the volume of sphere is equal to volume of
21. (b) Given diameter d = 24 m cylinder, then the ratio of volume of the sphere to the
A volume of cylinder = 1 : 1
(iv) (a) Total surface area of cylinder = 2πr( r + h )
22 22
=2 × × 4( 4 + 18) = 2 × × 4 × 22
3.5m 7 7
2
= 553.14 cm
B C (v) (c) During the conversion of a solid from one shape to
24 m another the volume of new shape will remain unaltered.
134 CBSE Term II Mathematics X (Standard)

24. We have radius of each CD cassette = 4 cm Subjective Questions


and thickness of each cassette = 1 cm 1. Let the length of a side of a cube = a cm
So, height of cylindrical made by Geeta, h1 = 10 × 1 = 10 cm a
and height of cylindrical made by Meena, h 2 = 6 × 1 = 6 cm a
(i) (d) Curved surface area of cylinder made by Geeta
22
= 2 πrh = 2 × × 4 × 10 = 251. 42 cm2
7 a
⎡Curved surface area of ⎤
⎢cylinder made by Geeta ⎥
(ii) (c)∴Required ratio = ⎣ ⎦
⎡Curved surface area of ⎤ 2a
⎢cylinder made by Meena ⎥
⎣ ⎦ Given, volume of the cube, a 3 = 64 cm 3 ⇒ a = 4 cm
2 πrh1 h1 10 On joining two cubes, we get a cuboid whose
= = = = 5: 3
2 πrh 2 h 2 6 length, l = 2a cm
(iii) (a) Volume of cylinder made by Meena breadth, b = a cm
22 and height, h = a cm
= πr 2h 2 = ×4×4×6
7 Now, surface area of the resulting cuboid
= 301. 44 cm3 = 2 ( lb + bh + hl )
(iv) (d) Required ratio = 2 (2 a ⋅ a + a ⋅ a + a ⋅ 2 a )
=
Volume of the cylinder made by Geeta = 2 (2 a 2 + a 2 + 2 a 2 ) = 2 ( 5 a 2 )
Volume of the cylinder made by Meena = 10 a 2 = 10 ( 4)2 = 160 cm 2
2
πr h1 h1 2. Given, dimensions of cuboidal = 9 cm × 11 cm × 12 cm
= = = 5: 3
πr 2h 2 h 2
∴Volume of cuboidal = 9 × 11 × 12 = 1188 cm 3
(v) (a) When two CD Cassette are shifted from Geeta’s
and diameter of shot = 3 cm
cylinder to Meena’s cylinder, then length of both
cylinders become equal. 3
∴ Radius of shot, r = = 1. 5 cm
So, volume of both cylinders become equal. 2
4 4 22
25. (i) (a) As, we know that hemisphere is a type of solid in Volume of shot = πr 3 = × × (1.5)3
which radius is the height. So, radius = 21 m 3 3 7
297
∴Required volume = πr 3
2 = = 14.143 cm 3
3 21
2 22 1188
= × × 21 × 21 × 21 ∴Required number of shots = = 84 (approx.)
3 7 14.143
= 19404 cu m 3. Given, dimensions of the cuboidal = 16 cm × 8 cm × 8 cm
4 ∴Volume of the cuboidal = 16 × 8 × 8 = 1024 cm 3
(ii) (b) Volume of sphere = πr 3
3 Also, given radius of one glass sphere = 2 cm
(iii) (b) Given, radius ( r ) = 14 m 4 3 4 22
∴Volume of one glass sphere = πr = × × (2 ) 3
∴Curved surface area of hemisphere dome = 2 πr 2 3 3 7
22 704
=2× × 14 × 14 = = 33.523 cm 3
7 21
= 1232 sq m Now, volume of 16 glass spheres = 16 × 33.523 = 536.37 cm 3
(iv) (c) Here, radius of hemispherical dome ( r ) = 14 m ∴ Required volume of water = Volume of cuboidal
Surface area of dome = 2 πr 2 − Volume of 16 glass spheres
= 1024 − 536.37
22
=2× × 14 × 14 = 1232 m 2 = 487.6 cm 3
7
and CSA of cuboidal shaped top = 2 × h( l + b ) + lb 4. Given, dimensions of the cuboid = 49 cm × 33 cm × 24 cm
= 2 × 4( 8 + 6) + 8 × 6 ∴ Volume of the cuboid = 49 × 33 × 24 = 38808 cm 3
= 8(14) + 48 [Q volume of cuboid = length × breadth × height]
= 112 + 48 = 160 m 2 Let the radius of the sphere is r, then
4
∴Total surface area = 1232 + 160 = 1392 m 2 Volume of the sphere = πr 3
3
(v) (d) Volume of cuboidal shape = lbh 4
[Q volume of the sphere = π × (radius)3]
= 8 × 6 × 4 = 192 m 3 3
CBSE Term II Mathematics X (Standard) 135

According to the question, Clearly, the total area to be painted = Total surface area of
Volume of the sphere = Volume of the cuboid the decorative block − Area of base of cube
4 3 = 219. 465 − 62 = 219. 465 − 36 = 183. 465 cm 2
⇒ πr = 38808
3 7. Given that, side of a solid cube ( a ) = 7 cm
22 3 Height of conical cavity i.e. cone, h = 7 cm
⇒ 4× r = 38808 × 3
7
38808 × 3 × 7
⇒ r3 = = 441 × 21
4 × 22
⇒ r 3 = 21 × 21 × 21
∴ r = 21 cm 7 cm
Hence, the radius of the sphere is 21 cm.
5. Let the radii of the two spheres are r1 and r2, respectively.
4
∴ Volume of the sphere of radius, r1 = V1 = πr13 …(i) 3 cm
3
4 Since, the height of conical cavity and the side of cube is
[Q volume of sphere = π (radius)3]
3 equal that means the conical cavity fit vertically in the cube.
4 Radius of conical cavity i.e. cone, r = 3 cm
and volume of the sphere of radius, r2 = V2 = π r23 …(ii)
3 ⇒ Diameter = 2 × r = 2 × 3 = 6 cm
4 3 Since, the diameter is less than the side of a cube that means
πr1
64
Given, ratio of volumes = V1 : V2 = 64 : 27 ⇒ 3 = the base of a conical cavity is not fit inhorizontal face of
4 3 27 cube.
πr2
3 Now, volume of cube = (side) 3= a 3 = (7 )3 = 343 cm 3
[using Eqs. (i) and (ii)] 1
r13 64 r1 4 and volume of conical cavity i.e. cone = π × r 2 × h
⇒ = ⇒ = …(iii) 3
r23 27 r2 3 1 22
= × × 3× 3×7
4πr12 3 7
Now, ratio of surface area =
4πr22 = 66 cm 3
[Q surface area of a sphere = 4π (radius) 2] ∴Volume of remaining solid = Volume of cube
r2 − Volume of conical cavity
= 12 = 343 − 66 = 277 cm 3
r2
2 2 Hence, the required volume of solid is 277 cm 3.
⎛r⎞ ⎛ 4⎞ 16
= ⎜ 1⎟ = ⎜ ⎟ = [using Eq. (iii)] 8. Given, radius of hemispherical bowl, r = 9 cm
⎝ r2 ⎠ ⎝ 3⎠ 9
and radius of cylindrical bottles, R = 1. 5 cm and height,
Hence, the required ratio of their surface area is 16 : 9. h = 4 cm
6. Here, the decorative block is a combination of a cube and a ∴Number of required cylindrical bottles
hemisphere. Volume of hemispherical bowl
For cubical portion, =
Volume of one cylindrical bottle
Each edge = 6 cm 2 3
For hemispherical portion, πr
= 3 2
Diameter = 2.1 cm πR h
2.1 2
∴ Radius, r = cm ×π ×9×9×9
2
= 3 = 54
Now, total surface area of the cube π × 1. 5 × 1. 5 × 4
= 6 × (Edge) 2= 6 × 6 × 6 = 216 cm 2 9. Let r be the radius of the hemispherical dome
Here, the part of the cube where the hemisphere is and total height of building be H m.
attached, is not included in the surface area. It is given that diameter of dome
So, the total surface area of the decorative block 2
= × Total height of the building
= Total surface area of cube − Area of base of hemisphere 3
+ Curved surface area of hemisphere 1
⇒ r= Hm
= 216 − πr 2 + 2 πr 2 = 216 + πr 2 3
22 2 . 1 2 . 1 Let h m be the height of the cylinder.
= 216 + × ×
7 2 2 1 2
∴ h = H − r = H − H = Hm
= 216 + 3. 465 = 219. 465 cm 2 3 3
136 CBSE Term II Mathematics X (Standard)

Volume of the air inside the building = Volume of air ⎡ 22 ⎤ 22


=2 × 2 × × (7 ) 2 + 2 × × (7 ) ( 90)
inside the dome + Volume of air inside the cylinder ⎣⎢ 7 ⎦⎥ 7
2 = 4 × 22 × 7 + 2 × 22 × 90 = 22 [28 + 180]
= πr 3 + πr 2h
3 = 4576 cm 2
⎡2 ⎛ 1 ⎞ 3 ⎛ 1 ⎞ 2 ⎛ 2 ⎞ ⎤
= π ⎢ ⎜ H⎟ + ⎜ H⎟ ⎜ H⎟ ⎥ 7 cm
⎝ ⎠ ⎝3 ⎠ ⎝3 ⎠
⎣3 3 ⎦
3
7 cm
⎡ 2H 2 3⎤
= π⎢ + H ⎥
⎣ 81 27 ⎦
104 cm
8 90 cm
= πH 3 m 3
81
1
Given, volume of the air inside the building = 67 m 3
21
8 1408 7 cm
∴ πH 3 =
81 21 7 cm
3 1408 81
⇒ H = ×
21 8π Then, the cost of polishing at the rate of ` 2 per dm 2
1408 × 81 × 7 4576 × 2
⇒ 3
H = =` = ` 91.52 [Q1 dm 2 = 100 cm 2 ]
21 × 8 × 22 100
⇒ H 3 = 216 ⇒ H = 6 m 12. Let height of the cone be h.
Given, radius of the base of the cone = 6 cm
10. Given, diameter of the cylinder = 2 cm
1 1
∴Radius = 1 cm and height of the cylinder = 16 cm ∴ Volume of circular cone = πr 2h = π ( 6)2 h
3 3
[Q diameter = 2 × radius] 36 π h
∴Volume of the cylinder = π × (1)2 × 16 = 16 π cm 3 = = 12 π h cm 3
3
[Q volume of cylinder= π × (radius)2 × height] Also, given radius of the hemisphere = 8 cm
Now, let the radius of solid sphere = r cm 2 2
∴Volume of the hemisphere = π r 3 = π ( 8)3
4 3 3
Then, its volume = πr 3 cm 3
3 512 × 2 π
= cm 3
4 3
[Q volume of sphere = × π × (radius)3]
3 According to the question,
According to the question, Volume of the cone = Volume of the hemisphere
Volume of the twelve solid sphere = Volume of cylinder 512 × 2 π
⇒ 12 πh =
4 3
⇒ 12 × πr 3 = 16 π
3 512 × 2 π 256
∴ h= = = 28. 44 cm
⇒ r 3 = 1 ⇒ r = 1 cm 12 × 3 π 9
∴ Diameter of each sphere, d = 2 r = 2 × 1 = 2 cm 13. Given, dimensions of base of rectangular tank = 11 m × 6 m
Hence, the required diameter of each sphere is 2 cm. and height of water = 5 m
11. Given, whole length of the solid = 104 cm Volume of the water in rectangular tank = 11 × 6 × 5
= 330 m 3
and the radius of each hemisphere = 7 cm
Therefore, the length of the cylindrical part of the solid Also, given radius of the cylindrical tank = 3.5 m
= (104 − 2 × 7 ) = 90 cm Let height of water level in cylindrical tank be h.
Then, volume of the water in cylindrical tank = πr 2h
For hemispherical portion,
Radius, r = 7 cm = π (3.5)2 × h
22
For cylindrical portion, = × 3.5 × 3.5 × h = 11. 0 × 3. 5 × h = 38. 5 h m 3
7
Radius, r = 7 cm
According to the question,
Height, h = 90 cm
330 = 38. 5 h
∴ Total surface area of the solid [since, volume of water is same in both tanks]
= 2 × Curved surface area of a hemisphere 330 3300
∴ h= =
+ Curved surface area of cylindrical part 38. 5 385
= 2 [2 πr 2 ] + 2 πrh ∴ = 8. 57 m or 8.6 m
= 2 × [2 π (7 )2 ] + 2 π (7 ) ( 90) Hence, the height of water level in cylindrical tank is 8.6 m.
CBSE Term II Mathematics X (Standard) 137

14. Here, a rod of cylindrical shape is converted into a wire of 10368


⇒ r2 = = 1296 ⇒ r = 36 cm
cylindrical shape. 8
For rod, Again, let the slant height of the conical heap = l
1
Diameter = 1 cm ⇒ Radius, r1 = cm and length, Now, l 2 = h 2 + r 2 = (24)2 + ( 36)2
2
h = 8 cm = 576 + 1296 = 1872
2
⎛ 1⎞ ∴ l = 43. 267 cm
∴ Volume of the rod = πr12h = π × ⎜ ⎟ × 8 = 2 π cm 3
⎝ 2⎠ Hence, radius of conical heap of sand = 36 cm
For wire, and slant height of conical heap = 43. 267 cm
Length = 8 m = 800 cm [Q1m = 100 cm ] 17. Given, length of the barrel of a fountain pen = 7 cm
Let r be the radius (in cm) of cross-section of the wire, then 5 1 ⎡ 1 ⎤
Volume of wire = π × r 2 × 800 cm 3 and diameter = 5 mm = cm = cm Q 1 mm = cm
10 2 ⎢⎣ 10 ⎥⎦
Since, the rod is converted into wire, so 1
Volume of wire = Volume of rod ∴ Radius of the barrel = = 0.25 cm
2 ×2
1 1
⇒ π × r 2 × 800 = 2 π ⇒ r 2 = ⇒r = cm Volume of the barrel = πr 2h [since, its shape is cylindrical]
400 20
2 1 22
Then, diameter = = cm = × (0.25)2 × 7
20 10 7
Hence, the diameter of the cross-section, i.e. the thickness = 22 × 0.0625 = 1.375 cm 3
1
of the wire is cm, i.e. 1 mm [Q1cm = 10 mm ] 1
10 Also, given volume of ink in the bottle = of litre
5
15. Given, length of roof = 22 m and breadth of roof = 20 m 1
= × 1000 cm = 200 cm 3
3
Let the rainfall be a cm. 5
a 22 a 3
∴Volume of water on the roof = 22 × 20 × = m Now, 1.375 cm 3 ink is used for writing number of words
100 5
= 3300
Also, we have radius of base of the cylindrical vessel, r = 1 m 3300
3
and height of the cylindrical vessel, h = 3.5 m ∴1 cm ink is used for writing number of words =
1.375
∴Volume of water in the cylindrical vessel when it is just full ∴200 cm 3 ink is used for writing number of words
⎛ 22 7⎞
= πr 2h = ⎜ × 1 × 1 × ⎟ = 11 m 3 =
3300
× 200 = 480000
⎝7 2⎠ 1.375
Now, volume of water on the roof = Volume of water in 18. Given, speed of water flow = 10 m min −1 = 1000 cm/min
the vessel 5 1
⎡ ⎤
22 a and diameter of the pipe = 5 mm = cm Q 1 mm = cm
⇒ = 11 10 ⎢⎣ 10 ⎥⎦
5
5
11 × 5 ∴ Radius of the pipe = = 0.25 cm
∴ a= = 2.5 cm 10 × 2
22
[Q volume of cylinder=π × (radius) 2 × height] ∴ Area of the face of pipe
Hence, the rainfall is 2.5 cm. 22
= πr 2 = × (0.25)2 = 0.1964 cm 2
16. Given, radius of the base of the bucket = 18 cm 7
Also, given diameter of the conical vessel = 40 cm
Height of the bucket = 32 cm
40
So, volume of the sand in cylindrical bucket = π r 2h ∴Radius of the conical vessel = = 20 cm
2
= π (18)2 × 32 = 10368 π
and depth of the conical vessel = 24 cm
Also, given height of the conical heap ( h ) = 24 cm 1 1 22
∴Volume of conical vessel = πr 2h = × × (20)2 × 24
Let radius of heap be r cm. 3 3 7
1 211200
Then, volume of the sand in the heap = π r 2 h = = 10057.14 cm 3
3 21
1 2 2 Volume of the conical vessel
= πr × 24 = 8 π r ∴Required time =
3 Area of the face of pipe × Speed of water
According to the question, 10057.14
=
Volume of the sand in cylindrical bucket = Volume of the 0.1964 × 1000
sand in conical heap 20
= 51.20 min = 51 min × 60 s
⇒ 10368 π = 8πr 2 100
⇒ 10368 = 8 r 2 = 51 min 12 s
138 CBSE Term II Mathematics X (Standard)

19. Given, radius of tank, r1 = 40 cm The embankment is in the form of cylindrical shell, so area
Let height of water level in tank in half an hour = h1 of embankment = π(R 2 − r 2 ) = π(102 − 52 )
Also, given internal radius of cylindrical pipe, r2 = 1 cm 22
= π (100 − 25) = × 75 m 2
and speed of water = 80 cm/s i.e. in 1s water flow = 80 cm 7
∴ In 30 (min) water flow = 80 × 60 × 30 = 144000 cm Since, Earth taken out from well is used to form
embankment.
According to the question,
∴ Volume of embankment
Volume of water in cylindrical tank = Volume of water flow
from the circular pipe in half an hour = Volume of Earth taken out on digging the well
⇒ π r12 h1 = π r22 h 2 ⇒ Area of embankment × Height of embankment
= Volume of Earth dugout
⇒ 40 × 40 × h1 = 1 × 1 × 144000
Volume of Earth dugout
144000 ⇒ Height of embankment =
∴ h1 = = 90 cm Area of the embankment
40 × 40
1100
Hence, the level of water in cylindrical tank rises 90 cm in = = 4. 67 m
22
half an hour. × 75
7
20. Given, pencils are cylindrical in shape.
22. Given, diameter of a marble = 1.4 cm
Length of one pencil = 25 cm 1.4
and circumference of base, 2πr = 1.5 cm ∴ Radius of marble ( r ) = = 0.7 cm
2
1.5 × 7
⇒ r= = 0.2386 cm 4 4
22 × 2 So, volume of one marble = πr 3 = π ( 0.7 )3
3 3
Now, curved surface area of one pencil = 2πrh 4 1.372
22 = π × 0. 343 = π cm 3
=2 × × 0. 2386 × 25 3 3
7 Also, given diameter of beaker = 7 cm
262. 46
= = 37.49 cm 2 7
∴ Radius of beaker = = 3.5 cm
7 2
37.49 ⎡ 1 ⎤
= dm 2 Q 1 cm = dm Height of water level raised = 5.6 cm
100 ⎢⎣ 10 ⎥⎦
∴Volume of the raised water in beaker
= 0.375 dm 2 = π (3.5)2 × 5.6 = 68.6π cm 3
∴Curved surface area of 120000 pencils = 0.375 × 120000 Now, required number of marbles
= 45000 dm 2 Volume of the raised water in beaker
2 =
Now, cost of colouring 1 dm curved surface of the pencils Volume of one spherical marble
manufactured in one day = ` 0.05 68.6 π
∴Cost of colouring 45000 dm 2 curved surface = × 3 = 150
1.372 π
= 45000 × 0. 05 = ` 2250
23. Given, edge of the cube = 22 cm
21. Here, a well is dug and Earth taken out of it is used to form
an embankment. ∴ Volume of the cube = (22 )3 = 10648 cm 3
Given, Diameter of well = 10 m [Q volume of cube = (side)3]
10
∴ Radius = = 5m Also, given diameter of marble = 0.5 cm
2 0. 5
Also, depth = 14 m ∴ Radius of a marble, r = = 0. 25 cm
2
∴Volume of Earth taken out on digging the well [Q diameter =2 × radius]
22 4 4 22
= πr 2h = × ( 5)2 × 14 = 1100 m 3 Volume of one marble = πr 3 = × × (0.25)3
7 3 3 7
4
10 m [Q volume of sphere = × π × (radius)3]
5m 5m
3
1. 375 3
h = = 0. 0655 cm
21
1
Filled space of cube = Volume of the cube −
8
14 m

× Volume of cube
1
= 10648 − 10648 ×
8
7
= 10648 × = 9317 cm 3
8
CBSE Term II Mathematics X (Standard) 139

∴ Required number of marbles ∴ Total volume of the building = Volume of the cylinder
Total space filled by marbles in a cube + Volume of hemispherical dome
=
Volume of one marble ⎛ 3 2 3⎞ 3 5 3 3
= ⎜ πr + πr ⎟ m = πr m
9317 ⎝ 3 ⎠ 3
= = 142244 (approx.)
0.0655 According to the question,
Hence, the number of marbles that the cube can accomodate Volume of the building = Volume of the air
is 142244. 5 3 19
⇒ πr = 41
24. Given that, a solid iron cuboidal block is recast into a hollow 3 21
cylindrical pipe. 5 3 880
⇒ πr =
Length of cuboidal pipe ( l ) = 4. 4 m 3 21
880 × 7 × 3
Breadth of cuboidal pipe ( b ) = 2. 6 m and height of cuboidal ⇒ r3 =
pipe ( h ) = 1 m 21 × 22 × 5
So, volume of a solid iron cuboidal block = l ⋅ b ⋅ h 40 × 21
= =8
= 4. 4 × 2. 6 × 1 = 11. 44 m 3 21 × 5
3
Also, internal radius of hollow cylindrical pipe ( ri ) = 30 cm ⇒ r = 8 ⇒ r =2m
= 0. 3 m ∴ Height of the building = 2 r = 2 × 2 = 4 m
and thickness of hollow cylindrical pipe = 5 cm = 0. 05 m 26. Given, diameter of cylinder = Diameter of hemisphere
= 0.5 cm
So, external radius of hollow cylindrical pipe
( re ) = ri + Thickness [since, both hemispheres are attach with cylinder]
= 0. 3 + 0. 05 = 0. 35 m ∴ Radius of cylinder ( r ) = radius of hemisphere
0.5
∴Volume of hollow cylindrical pipe (r) = = 0.25 cm
2
= Volume of cylindrical pipe with external radius
[Q diameter = 2 × radius]
− Volume of cylindrical pipe with internal radius
= πre2h1 − πri2h1 = π ( re2 − ri2 ) h1
22 0.5 cm
0.25 0.25
= [( 0. 35)2 − ( 0. 3)2 ] ⋅ h1
7
22 22
= [ 0. 1225 − 0. 09] ⋅ h1 = [ 0. 0325] ⋅ h1 2 cm
7 7
and total length of capsule = 2 cm
= 0.715 × h1 / 7
∴Length of cylindrical part of capsule,
where, h1 be the length of the hollow cylindrical pipe.
h = Length of capsule − Radius of both hemispheres
Now, by given condition,
= 2 − (0.25 + 0.25) = 1.5 cm
Volume of solid iron cuboidal block = Volume of hollow Now, capacity of capsule = Volume of cylindrical part
cylindrical pipe
+ 2 × Volume of hemisphere
⇒ 11. 44 = 0.715 × h / 7 2
11. 44 × 7 = πr h + 2 × πr 3
2

∴ h= = 112 m 3
0.715 [Q volume of cylinder = π × (radius)2
Hence, required length of pipe is 112 m. 2
× height and volume of hemisphere = π (radius)3]
25. Let total height of the building = Internal diameter of the 3
dome = 2r m 22 2 4 3
= [(0.25) × 1.5 + × (0.25) ]
7 3
22
r = [0.09375 + 0.0208]
7
r r 22
= × 0.11455 = 0.36 cm 3
7
2r
Hence, the capacity of capsule is 0.36 cm 3.
r
27. Since, rocket is the combination of a right circular cylinder
and a cone.
Given, diameter of the cylinder = 6 cm
6
2r ∴ Radius of the cylinder = = 3 cm
∴ Radius of building (or dome) = =rm 2
2 and height of the cylinder = 12 cm
Height of cylinder = 2r − r = r m ∴ Volume of the cylinder = πr 2h = 3.14 × ( 3)2 × 12
∴ Volume of the cylinder = π r 2 ( r ) = πr 3 m 3 = 339.12 cm 3
2 and curved surface area = 2πrh
and volume of hemispherical dome on cylinder = πr 3 m 3
3 = 2 × 3.14 × 3 × 12 = 226.08
140 CBSE Term II Mathematics X (Standard)

A 2 3 1 2
= πr + πr h
3 3
⎡2 1 ⎤
5 cm
= × 3.14 × (2 )3 + × 3.14 × (2 )2 × 3
⎢⎣ 3 3 ⎥⎦
h
= 16.75 + 12 . 56 = 29.31 cm 3
Hence, volume of the solid toy is 29.31 cm 3.
O 3 cm
B C Now, let the right circular cylinder EFGH circumscribe the
given solid toy. Then, radius of the base of the right circular
cylinder = HP = BO = 2 cm and its height,
12 cm EH = AP = AO + OP = 3 + 2 = 5 cm
So, volume of the cylinder = π r 2 h
= 3.14 × 2 2 × 5
E D = 62.8 cm 3
6 cm Now, required difference of the volume of the cylinder and
Now, in right angled ΔAOC, the solid toy = Volume of cylinder − Volume of solid toy
= 62.8 − 29.31 = 33.49 cm 3
h = 52 − 32
29. Here, the given wooden toy rocket is combination of a cone
= 25 − 9 = 16 = 4 and a cylinder.
∴Height of the cone, h = 4 cm For conical portion,
and radius of the cone, r = 3 cm Diameter = 6 cm
Now, volume of the cone 6
∴ Radius, r1 = cm = 3 cm
1 1 2
= πr 2h = × 3.14 × ( 3)2 × 4
3 3 Height, h1 = 4 cm
113.04
= = 37.68 cm 3 Then, slant height, l = ( 3)2 + 42 [Q l = r 2 + h 2 ]
3
and curved surface area = πrl = 3.14 × 3 × 5 = 47.1 = 9 + 16 = 25 = 5 cm
Hence, total volume of the rocket For cylindrical portion,
= 339.12 + 37.68 = 376.8 cm 3 Diameter = 4 cm
and total surface area of the rocket 4
∴ Radius, r2 = cm = 2 cm
= CSA of cone + CSA of cylinder + Area of base of cylinder 2
= 47.1 + 226. 08 + 28. 26 Height, h 2 = Total height of rocket − Height of cone
= 301. 44 cm 2 = 24 − 4 [Q total height of rocket = 24 cm]
= 20 cm
28. Here, given solid toy is a combination of a right circular
cone and a hemisphere. Here, we have to find the area of the rocket painted with
orange and yellow colours separately.
Let BPC be the hemisphere and ABC be the cone standing
on the base of the hemisphere as shown in the figure. Since, radius of base of cone is larger than radius of base of
cylinder and cone is mounted on cylinder.
For conical portion, height, h = 3 cm
∴ Area to be painted orange = Curved surface area of cone
Diameter, d = 4 cm
+ Area of base of cone − Area of base of cylinder
E A F [Q area of base of cylinder is common
in area of base of cone]
= πr1l + πr12 − πr22
B O C = 3.14 × 3 × 5 + 3.14 × (3)2 − 3.14 × (2)2
= 3.14 [15 + 9 −4]
H G = 3.14 × 20 = 62.8 cm 2
P
Now, area to be painted yellow
4 = Curved surface area of cylinder
∴Radius, r = cm = 2 cm
2 + Area of base of the cylinder = 2 πr2h 2 + π r22
For hemispherical portion,
= 2 × 3.14 × 2 × 20 + 3.14 × (2)2
Radius, r = 2 cm [Q radii of hemisphere and cone are same]
So, volume of the solid toy = 3.14 [80 + 4]
= Volume of hemisphere + Volume of cone = 3.14 × 84 = 263.76 cm 2
CBSE Term II Mathematics X (Standard) 141

30. Let volume of cone A be 2 V and volume of cone B be V. Now, since edge of a solid cube ( a ) = 44 cm
Again, let height of the cone A = h1 cm, then height of cone So, volume of a solid cube = ( a )3
B = (21 − h1 ) cm = ( 44)3
21 cm
= 44 × 44 × 44 cm 3
From Eq. (i),

6 cm
6 cm

A B
44 × 44 × 44
Number of spherical lead shots = × 21
4 × 22 × 8
h1 21 – h1 = 11 × 21 × 11
Given, diameter of the cone = 6 cm = 121 × 21
6 = 2541
∴ Radius of the cone = = 3 cm
2 Hence, the required number of spherical lead shots is 2541.
1 1 32. Given, internal diameter of spherical shell = 4 cm
Now, volume of the cone, A = 2 V = πr 2h = π( 3)2 h1
3 3 and external diameter of shell = 8 cm
1 3 4
⇒ V = π 9 h1 = h1π …(i) ∴ Internal radius of spherical shell, r1 = cm = 2 cm
6 2 2
and volume of the cone, [Q diameter = 2 × radius]
1 8
B = V = π ( 3)2 (21 − h1 ) = 3π (21 − h1 ) …(ii) and external radius of shell, r2 = = 4 cm
3 2
From Eqs. (i) and (ii), [Q diameter = 2 × radius]
3
h1π = 3π (21 − h1 )
2 8 cm
⇒ h1 = 2 (21 − h1 )
4 cm
⇒ 3h1 = 42
42
⇒ h1 = = 14 cm
3
∴ Height of cone, B = 21 − h1 = 21 − 14 = 7 cm
22 Spherical shell
Now, volume of the cone, A = 3 × 14 × = 132 cm 3
7 4
Now, volume of the spherical shell = π [ r23 − r13 ]
[using Eq. (i)] 3
1 22 4
and volume of the cone, B = × × 9 × 7 = 66 cm 3 [Q volume of the spherical shell = π
3 7 3
[using Eq. (ii)] {(external radius)3 − (internal radius)3}]
22 2 4
Now, volume of the cylinder = πr h = ( 3) × 21 = 594 cm 3
2
= π ( 43 − 2 3 )
7 3
∴ Required volume of the remaining portion 4
= π ( 64 − 8)
= Volume of the cylinder 3
− (Volume of cone A + Volume of cone B) 224
= π cm 3
= 594 − (132 + 66) = 396 cm 3 3
Let height of the cone = h cm
31. Given that, lots of spherical lead shots made out of a solid
cube of lead. Diameter of the base of cone = 8 cm
8
∴ Number of spherical lead shots ∴ Radius of the base of cone = = 4 cm
Volume of a solid cube of lead 2
= ...(i)
Volume of a spherical lead shot [Q diameter = 2 × radius]
Given that, diameter of a spherical lead shot i.e. sphere = 4 cm According to the question,
4 Volume of cone = Volume of spherical shell
⇒ Radius of a spherical lead shot (r) = 1 224
2 ⇒ π ( 4) 2 h = π
r = 2 cm [Q diameter = 2 × radius] 3 3
So, volume of a spherical lead shot i.e. sphere 224
⇒ h= = 14 cm
4 16
= π r3 1
3 [Q volume of cone = × π × (radius)2 × (height)]
4 22 4 × 22 × 8 3
= × × (2 ) 3 = cm 3
3 7 21 Hence, the height of the cone is 14 cm.
142 CBSE Term II Mathematics X (Standard)

33. Given that, lots of spherical lead shots made from a solid 4. 5
⇒ Radius of a right circular cylinder ( r ) = cm
rectangular lead piece. 2
∴ Number of spherical lead shots ∴ Volume of right circular cylinder = π r 2h
Volume of solid rectangular lead piece 2
= ...(i) ⎛ 4.5⎞
Volume of a spherical lead shot = π ⎜ ⎟ × 10
⎝ 2 ⎠
Also, given that diameter of a spherical lead shot π
i.e. sphere = 4. 2 cm =× 4. 5 × 4. 5 × 10
4
4.2
∴ Radius of a spherical lead shot, r = = 2.1 cm ∴ Number of metallic circular disc
2 Volume of a right circular cylinder
⎡ 1 ⎤ =
Q radius = diameter Volume of a metallic circular disc
⎢⎣ 2 ⎥⎦
π
× 4. 5 × 4. 5 × 10
So, volume of a spherical lead shot i.e. sphere
= 4
4 π
= π r3 × 1. 5 × 1. 5 × 0. 2
3 4
4 22 3 × 3 × 10 900
= × × (2.1)3 = = = 450
3 7 0. 2 2
4 22 Hence, the required number of metallic circular disc is 450.
= × × 2.1 × 2.1 × 2.1
3 7 35. Given that, a heap of rice is in the form of a cone.
4 × 22 × 21 × 21 × 21
= Height of a heap of rice i.e. cone ( h ) = 3. 5 m
3 × 7 × 1000
and diameter of a heap of rice i.e. cone = 9 m
Now, length of rectangular lead piece, l = 66 cm 9
Radius of a heap of rice i.e. cone ( r ) = m
Breadth of rectangular lead piece, b = 42 cm 2
Height of rectangular lead piece, h = 21 cm 1
So, volume of rice = π × r 2h
∴Volume of a solid rectangular lead piece i.e. cuboid 3
=l×b×h 1 22 9 9
= × × × × 3. 5
= 66 × 42 × 21 3 7 2 2
From Eq. (i), 6237
= = 74. 25 m 3
Number of spherical lead shots 84
66 × 42 × 21 Now, canvas cloth required to just cover heap of rice
= × 3 × 7 × 1000
4 × 22 × 21 × 21 × 21 = Surface area of a heap of rice
3 × 22 × 21 × 2 × 21 × 21 × 1000 = πrl
= 22
4 × 22 × 21 × 21 × 21 = × r × r2 + h 2
7
= 3 × 2 × 250
2
= 6 × 250 = 1500 22 9 ⎛ 9⎞
= × × ⎜ ⎟ + ( 3.5)2
Hence, the required number of spherical lead shots is 1500. 7 2 ⎝ 2⎠
34. Given that, lots of metallic circular disc to be melted to form 11 × 9 81
a right circular cylinder. Here, a circular disc work as a = × + 12. 25
7 4
circular cylinder.
99 130 99
Base diameter of metallic circular disc = 1. 5 cm = × = × 32 . 5
1. 5 7 4 7
∴ Radius of metallic circular disc = cm = 14.142 × 5. 7 = 80. 61 m 2
2
[Q diameter = 2 × radius] Hence, 80.61 m 2 canvas cloth is required to just cover heap.
and height of metallic circular disc i.e. = 0.2 cm 36. Let the length(l), breadth (b) and height (h) be the external
∴Volume of a circular disc = π × (Radius)2 × Height dimension of an open box and thickness be x.
2
⎛ 1. 5⎞
=π×⎜ ⎟ × 0. 2
⎝ 2 ⎠
π x x
= × 1. 5 × 1. 5 × 0. 2
4
Now, height of a right circular cylinder ( h ) = 10 cm x
and diameter of a right circular cylinder = 4. 5 cm
CBSE Term II Mathematics X (Standard) 143

Given that, and speed of water flowing through the pipe = (15 × 1000)
external length of an open box ( l ) = 36 cm = 15000 mh −1
external breadth of an open box ( b ) = 25 cm Now, volume of water flow in 1 h = π R 2H
and external height of an open box ( h ) = 16.5 cm
⎛ 22 7 7 ⎞
∴ External volume of an open box = lbh =⎜ × × × 15000⎟
⎝ 7 100 100 ⎠
= 36 × 25 × 16. 5 = 14850 cm 3
= 231 m 3
Since, the thickness of the iron ( x ) = 1. 5 cm
So, internal length of an open box ( l1 ) = l − 2 x Since, 231 m 3 of water falls in the pond in 1 h.
1
= 36 − 2 × 1. 5 So, 1 m 3 water falls in the pond in h.
231
= 36 − 3 = 33 cm
⎛ 1 ⎞
Therefore, internal breadth of an open box ( b 2 ) = b − 2 x Also, 462 m 3 of water falls in the pond in ⎜ × 462⎟ h = 2 h
⎝ 231 ⎠
= 25 − 2 × 1. 5
= 25 − 3 = 22 cm Hence, the required time is 2 h.
and internal height of an open box ( h 2 ) = ( h − x ) 38. (i) Length of the cuboid (L ) = 30 cm
= 16. 5 − 1. 5 = 15 cm Breadth of the cuboid (B ) =20 cm
So, internal volume of an open box Height of the cuboid (H ) = 10 cm
= ( l − 2 x) ⋅ ( b − 2 x) ⋅ ( h − x) Then,
= 33 × 22 × 15 Area covered by the cuboid
= 10890 cm 3 = L × B + 2 H (L + B )
= 30 × 20 + 2 × 10( 30 + 20)
Therefore, required iron to construct an open box
= 600 + 1000
= External volume of an open box
= 1600 cm2
− Internal volume of an open box
= 14850 − 10890 = 3960 cm 3 (ii) We know that,
3 C.S.A of the cylinder = 2 πRH1
Hence, required iron to construct an open box is 3960 cm .
22
7. 5 C.S.A of the cylinder = 2 × × 7 × 60
Given that, 1 cm 3 of iron weights = 7 . 5 g = kg 7
1000
= 2 × 22 × 60
= 0. 0075 kg
∴ 3960 cm 3 of iron weights = 3960 × 0. 0075 = 29.7 kg = 2640 cm2
In cone, height is given and radius of the cone is equal to
37. Given, length of the pond = 50 m and width of the pond
radius of the cylinder,
= 44 m Q L2 = H 22 + R 2
21
Depth required of water = 21 cm = m ⇒ L2 = 7 × 7 + 24 × 24
100
∴ Volume of water in the pond = l × b × h ⇒ L2 = 49 + 576 = 625
⎛ 21 ⎞ ∴ L = 25 cm
= ⎜ 50 × 44 × ⎟
⎝ 100⎠ 22
C.S.A. of cone = πrl = × 7 × 25 = 550 cm2
= 462 m 3 7
7 C.S. A of the cylinder 2640
Also, given radius of the pipe = 7 cm = m ⇒ = = 24 : 5
100 C.S. A. of the cone 550
(iii) (b) Given structure is based on the concept of surface
14 cm area and volume.
7 21
Pipe 39. (i) Given, r = cm, h = 10. 5 cm = cm
2 2
44 m Volume of cylindrical cup = πr 2h
22 7 7 21
= × × ×
7 2 2 2
11 × 7 × 21
21 m =
4
Tank 100
= 404.25 cm 3
50 m
144 CBSE Term II Mathematics X (Standard)

7 (ii) Given edge of cube = 7 cm


(ii) Given, r = cm
2 ∴Volume of cube = 7 × 7 × 7 = 343 cm 3
2 22 7 7 7
∴ Volume of hemispherical cup = × × × × (iii) Given, radius ( r ) = 7 cm
3 7 2 2 2
∴Curved surface area of hemisphere = 2 πr 2
= 89. 83 cm 3
22
(iii) Area of canvas provided = 551 m 2 =2× ×7 ×7
7
Area of remained after westage = 551 − 1 = 550 m 2 = 44 × 7 = 308 cm 2
So, area of conical tent = πrl (iv) Given, radius ( r ) = 7 cm and height ( h ) = 24 cm
Here, r = 7 m Slant height ( l ) = ?
22
∴ πrl = 550 ⇒ × 7 × l = 550 ⇒ l = 25 m ∴ l 2 = r 2 + h 2 = (7 )2 + (24)2
7
= 49 + 576 = 625
Now, h = l 2 − r 2 = 625 − 49
⇒ l = 625 = 25 cm
= 576 = 24 m (v) TSA of cone = πrl + 2 πr 2
40. (i) Given, edge of cube = 6 cm 22 22
∴Diagonal of cube = 3 × edge of cube = × 7 × 25 + 2 × ×7 ×7
7 7
= 3 × 6 = 6 3 cm = 550 + 308 = 858 cm 2
Chapter Test
Multiple Choice Questions (iii) The curved surface area of the conical cavity
1. How many cubes of side 2 cm can be made from a so formed is
(a) 2250 cm 2 (b) 2200 cm 2
solid cube of side 10 cm?
(c) 1800 cm 2 (d) 2400 cm 2
(a) 100 (b) 125 (c) 175 (d) 200
(iv) External curved surface area of the cylinder
2. 2 cubes, each of volume 125 cm 3 , are joined end to is
end. Find the surface area of the resulting cuboid.
(a) 2876 cm 2 (b) 1250 cm 2
(a) 100 cm 2 (b) 200 cm 2 (c) 225 cm 2 (d) 250 cm 2 (c) 4224 cm 2 (d) 3824 cm 2
3. The radius of a sphere (in cm) whose volume is (v) Volume of conical cavity is
12π cm 3 , is (a) 6232 cm 3
2/ 3 1/ 3
(a) 3 (b) 3 3 (c) 3 (d) 3 (b) 7248 cm 3
4. A solid spherical ball fits exactly inside the cubical box (c) 5380 cm 3
of side 2a. The volume of the ball is (d) 9856 cm 3
16 1 3 Short Answer Type Questions
(a) πa 3 (b) πa
3 6 7. A hemispherical depression is cut out from one face of
32 3 4 3
(c) πa (d) πa a cuboidal block of side 7 cm such that the diameter of
3 3
the hemisphere is equal to the edge of the cube. Find
5. A cone and a cylinder have the same radii but the the surface area of the remaining solid.
height of the cone is 3 times that of the cylinder, then
the ratio of their volumes.
8. The capacity of a cylindrical glass tumbler is 125.6 cm 3 .
If the radius of the glass tumbler is 2 cm, then find its
(a) 1 : 2 (b) 2 : 1
height. (Use π = 3.14)
(c) 1 : 1 (d) None of these
9. In given figure, a solid toy is in the form of a
Case Based MCQs
hemisphere surmounted by a right circular cone. The
6. One day Aakash was going home from market saw a height of the cone is 2 cm and the diameter of the
carpenter working on wood. He found that he is base is 4 cm. Determine the volume of the toy.
carving out a cone of same height and same diameter ⎡ 22 ⎤
take π =
from a cylinder. The height of the cylinder is 48 cm and ⎣⎢ 7 ⎦⎥
base radius is 14 cm. While watching this, some
questions came into Aakash’s mind. Help Aakash to
find the answer of the following questions. 2 cm

14 cm
2 cm
48 cm

Long Answer Type Questions


10. From a solid cylinder whose height is 2.4 cm and
(i) After carving out cone from the cylinder, diameter 1.4 cm, a conical cavity of the same height
(a) Volume of the cylindrical wood will decrease and same diameter is hollowed out. Find the total
(b) Height of the cylindrical wood will increase surface area of the remaining solid to the nearest cm 2 .
(c) Volume of cylindrical wood will increase 11. A well of diameter 3 m is dug 14 m deep. The earth
(d) Radius of the cylindrical wood will decrease taken out of it has been spread evenly all around it in
(ii) Find the slant height of the conical cavity so the shape of a circular ring of width 4 m to form a
formed. platform. Find the height of the platform.
(a) 28 cm (b) 60 cm ⎡ 22 ⎤
take π =
(c) 40 cm (d) 50 cm ⎢⎣ 7 ⎥⎦

Answers
1. (b) 2. (d) 3. (c) 4. (d) 5. (c) 6. (i) (a) (ii) (d) (iii) (b) (iv) (c) (v) (d) For Detailed Solutions
7. 332.465 cm 2 8. 10 cm 9. 25.12 cm 3 10. 18 cm 2 11. 1.125 m Scan the code
146 CBSE Term II Mathematics X (Standard)

CHAPTER 07

Statistics

In this Chapter...
! Mean of Grouped Data
! Mode of Grouped Data
! Median of Grouped Data

Arithmetic Mean or Mean or Average In general, for the ith class interval, we have frequency f i
corresponding to the class mark x i . The sum of the values in
The arithmetic mean of a set of observations is obtained by
the last column gives us Σf i x i , so the mean x of the given data
dividing the sum of the values of all observations by the total
is given by
number of observations.
Σf x
Thus, the mean of n observations x1 , x 2 , x 3 , K , x n , is x= i i.
defined as Σf i
n
∑ xi 2. Assumed Mean Method
x1 + x 2 + x 3 + … + x n i = 1
Mean ( x ) = = The cases, in which numerical values of x i and f i are large and
n n
computation of product of x i and f i becomes tedious and time
where, the Greek letter ‘Σ’ (sigma) means ‘Summation.’ consuming, assumed mean method is used. In this method,
Let x1 , x 2 , . . . , x n be n observations with respective first of all, one among x i ’s is chosen as the assumed mean
frequencies f 1 , f 2 , . . . , f n . This means observation x1 occurs denoted by ‘a’. After that, the difference di between a and
f 1 times, x 2 occurs f 2 times and so on. each of the x i ’ s, i.e. d i = x i − a is calculated.
n
∑ f i xi Then, arithmetic mean is given by
i =1 Σ f i xi Σf i d i
∴ Mean ( x ) = n
= x = a+
Σ fi Σf i
∑ fi
i =1 where, d i = xi − a
Method of Calculating Mode
Mean of Grouped Data The observation, which occurs most frequently among the
1. Direct Method
given observations, i.e. the value of the observation having
maximum frequency is called mode. e.g. Mode of the numbers
In this method, we find the class marks of each class 2, 3, 4, 4, 6, 6, 6, 6, 7 and 9 is 6 because it is repeated
interval. These class marks would serve as the
maximum number of times, i.e. 4 times.
representative of whole class and are represented by x i .
CBSE Term II Mathematics X (Standard) 147

Modal Class Case II If n is even, then


In a grouped frequency distribution, it is not possible to ⎛ n⎞ ⎛n ⎞
Median = Mean of value of ⎜ ⎟ th and ⎜ + 1⎟ th
determine the mode by looking at the frequencies. So, here ⎝ 2⎠ ⎝2 ⎠
we first locate a class with the maximum frequency. This class observations
is called modal class. e.g.
1 ⎡⎛ n ⎞ ⎛n ⎞ ⎤
= × Value of ⎢⎜ ⎟ th + ⎜ + 1⎟ th⎥
Class interval 0-10 10-20 20-30 30-40 40-50 50-60 2 ⎣ ⎝ 2 ⎠ ⎝2 ⎠ ⎦
Number of students 2 9 14 20 22 8 observations
e.g. If six girls of different heights are made to stand
Here, the highest frequency is of the class 40-50, which is 22. in a row, in descending order of their heights, then
Hence, the modal class is 40-50.
the mean height of third and fourth girl from either
Mode of Grouped Data end is the median height.
In grouped data, mode is a value that lies in the modal class Since, n = 6 is even.
and it is given by the formula, n 6
So, = = 3rd observation
⎧ f1 − f 0 ⎫ 2 2
Mode = l + ⎨ ⎬×h ⎛n ⎞ 6 6 +2
⎩2f 1 − f 0 − f 2 ⎭ and ⎜ + 1⎟ = + 1 = = 4th observation
⎝2 ⎠ 2 2
where, l = lower limit of the modal class ∴ Median = Mean of 3rd and 4th observations
h = size of the class intervals
(assuming all class sizes to be equal) Cumulative Frequency
f 1 = frequency of the modal class The frequency of an observation in a data refers to how many
f 0 = frequency of the class preceding the modal class times that observation occur in the data. Cumulative
frequency of a class is defined as the sum of all frequencies
f 2 = frequency of the class succeeding the modal class upto the given class.
Less than type and more than type. Formation of these two
Median distributions can be understood with the help of following
Median is defined as the middle-most or the central example.
observation, when the observations are arranged either in e.g. Consider a grouped frequency distribution of marks
ascending or descending order of their magnitudes. obtained out of 100, by 58 students, in a certain examination,
Median divides the arranged series into two equal parts, i.e. as follows:
50% of the observations lie below the median and the
remaining are above the median.
Let n be the total number of observations and suppose that Marks Number of students
they are arranged in ascending or descending order. 0-10 5
Median of the data depends on the number of 10-20 7
observations (n). 20-30 4
Case I If n is odd, then 30-40 2
⎛ n + 1⎞ 40-50 3
Median = Value of ⎜ ⎟ th observation
⎝ 2 ⎠ 50-60 6
e.g. If five girls of different heights are made to 60-70 7
stand in a row, in descending order of their heights, 70-80 9
then the height of the third girl from either end is 80-90 8
median height. 90-100 7
Since, n = 5 is odd. Cumulative frequency distribution of the less than type
⎛ n + 1⎞ 5 +1 Here, the number of students who have scored marks less
∴ Median = ⎜ ⎟ th observation = than 10 are 5. The number of students who have scored
⎝ 2 ⎠ 2
marks less than 20 includes the number of students who have
6 scored marks from 0-10 as well as the number of students
= = 3rd observation
2 who have scored marks from 10-20.
148 CBSE Term II Mathematics X (Standard)

Thus, the total number of students with marks less than 20 is Let n be the total number of observations (sum of
5 + 7 , i.e. 12. So, the cumulative frequency of the class 10-20 frequencies), then median of the data depends on the number
is 12. of observations ( n ).
Similarly, on computing the cumulative frequencies of the ⎛ n + 1⎞
If n is odd, then Median = Value of ⎜ ⎟ th observation.
other classes, which is shown in the table. ⎝ 2 ⎠
Number of students If n is even, then
Marks obtained
(cumulative frequency) ⎛ n⎞ ⎛n ⎞
Median = Mean of ⎜ ⎟ th and ⎜ + 1⎟ th observations
Less than 10 5 ⎝ 2⎠ ⎝2 ⎠
Less than 20 5 + 7 = 12 1 ⎡⎛ n ⎞ ⎛n ⎞ ⎤
Less than 30 12 + 4 = 16 = × Value of ⎢⎜ ⎟ th + ⎜ + 1⎟ th⎥ observations
2 ⎝
⎣ 2 ⎠ ⎝2 ⎠ ⎦
Less than 40 16 + 2 = 18
Here, for the value of observation, first look at the cumulative
Less than 50 18 + 3 = 21
frequency just greater than (and nearest to) the position of
Less than 60 21 + 6 = 27 required observations. Then, determine the corresponding
Less than 70 27 + 7 = 34 value of the observation.
Less than 80 34 + 9 = 43
Median for Grouped Data
Less than 90 43 + 8 = 51
Less than 100 51 + 7 = 58 In a grouped data, we may not find the middle observation by
looking at the cumulative frequencies, since the middle
Cumulative frequency distribution of the more than type observation will be some value in a class interval, so it is
For this type of distribution, we make the table for the necessary to find the value inside a class that divides the
number of students with scores, more than or equal to 0, whole distribution into two halves.
more than or equal to 10, more than or equal to 20 and so on. For this, we find the cumulative frequencies of all the classes
From the example, we observed that all 58 students have n
scored marks more than or equal to 0. and then determine , where n = number of observations.
2
There are 5 students scoring marks in the interval 0-10, it Now, locate the class whose cumulative frequency is greater
shows that there are 58 − 5 = 53 students getting more than or n
than (i.e. nearest to) and this class is called median class.
equal to 10 marks. In the same manner, the number of 2
students scoring 20 marks or above = 53 − 7 = 46 students, After finding the median class, use the following formula for
and so on. calculating the median.
Number of students ⎧N ⎫
Marks obtained − cf
(cumulative frequency) ⎪2 ⎪
Median = l + ⎨ ⎬×h
More than or equal to 0 58
⎪ f ⎪
More than or equal to 10 58 − 5 = 53 ⎩ ⎭
More than or equal to 20 53 − 7 = 46
where, l = lower limit of median class
More than or equal to 30 46 − 4 = 42
N = sum of frequencies
More than or equal to 40 42 − 2 = 40
cf = cumulative frequency of the class preceding
More than or equal to 50 40 − 3 = 37 the median class
More than or equal to 60 37 − 6 = 31
f = frequency of the median class
More than or equal to 70 31 − 7 = 24
h = class width (assuming class sizes to be equal)
More than or equal to 80 24 − 9 = 15
More than or equal to 90 15 − 8 = 7 Relationship among Mean,
Median and Mode
Median for Discrete Series There is an empirical relationship among the three measures
A series having observations x1 , x 2 , x 3 , K , x n with respective of central tendency, which is given by
frequencies f 1 , f 2 , f 3 K , f n is known as discrete series. Mode = 3(Median) − 2(Mean)
3 (Median) − Mode
Method to Find the Median of the Discrete Series or Mean =
2
Firstly, we arrange the data in the ascending or descending Mode + 2 (Mean)
order of x i , then we find the cumulative frequencies of all the or Median =
observations. 3
CBSE Term II Mathematics X (Standard) 149

Solved Examples
Example 1. Find the mean of the following data. Sol. The table for given data is

x 10 30 50 70 89 Class Frequency (fi ) Mid-value ( x i ) fi x i

f 7 8 10 15 10 2-4 6 3 18

Sol. Table for the given data is 4-6 8 5 40

xi fi fi x i 6-8 15 7 105

10 7 70 8-10 p 9 9p
30 8 240 10-12 8 11 88
50 10 500 12-14 4 13 52
70 15 1050 Σ fi x i
Σfi = p + 41
89 10 890 = 9p + 303
Total Σ fi = 50 Σ fi x i = 2750 Given, mean = 7.5
Σfi x i 9p + 303
Here, Σfi = 50 and Σfi x i = 2750 ∴ = 7. 5 ⇒ = 7. 5
Σfi p + 41
Σfi x i
∴ Mean ( x ) = ⇒ 9p + 303 = 7.5p + 307. 5
Σfi
⇒ 9p − 7.5p = 307.5 − 303
2750
= 55
= ⇒ 1. 5p = 4. 5
50 4.5
Hence, mean of the given data is 55. ⇒ p= =3
1.5
Hence, value of p is 3.
Example 2. Calculate the mean of the scores of 20
students in a mathematics test Example 4. The weights of tea in 70 packets are shown
Marks 10-20 20-30 30-40 40-50 50-60 in the following table
Number of students 2 4 7 6 1 Weight (in gm) Number of packets
200-201 13
Sol. We first, find the class marks x i of each class and then
201-202 27
proceed as follows
202-203 18
Marks Class marks ( x i ) Frequency ( fi ) fi xi 203-204 10
10-20 15 2 30 204-205 1
20-30 25 4 100 205-206 1
30-40 35 7 245
Find the mean weight of packets.
40-50 45 6 270 Sol. First, we find the class marks of the given data as follows.
50-60 55 1 55
Weight Number of Class Deviation fi di
Σ fi = 20 Σ fi x i = 700
(in gm) Packets ( fi ) marks ( x i ) ( di = x i − a )
Σfi x i 700
Therefore, mean ( x ) = = = 35
Σfi 20 200-201 13 200.5 −3 − 39
Hence, the mean of scores of 20 students in mathematics test 201-202 27 201.5 −2 − 54
is 35. 202-203 18 202.5 −1 − 18
Example 3. Find the value of p, if the mean of the 203-204 10 a = 203. 5 0 0
following distribution is 7.5. 204-205 1 204.5 1 1
Classes 2-4 4-6 6-8 8-10 10-12 12-14 205-206 1 205.5 2 2
Frequency (fi ) 6 8 15 p 8 4 N = ∑ fi = 70 ∑ fi di = − 108
150 CBSE Term II Mathematics X (Standard)

Here, assume mean ( a ) = 203. 5 Sol. Here, the number of students who have scored marks less
∑ fi di than 10 are 10. The number of students who have scored
∴ Mean ( x ) = a +
∑ fi marks less than 20 includes the number of students who
have scored marks from 0-10 as well as the number of
108
= 203.5 − students who have scored marks from 10-20.
70
Thus, the total number of students with marks less than 20 is
= 203.5 − 1.54 10 + 8, i.e. 18. So, the cumulative frequency of the class
= 201.96 10-20 is 18.
Hence, the required mean weight is 201.96 gm. Similarly, on computing the cumulative frequencies of the
other classes, i.e. the number of students with marks less
Example 5. The following distribution gives cumulative than 30, less than 40, … less than 100, we get the
frequencies of ‘more than type’: distribution which is called the cumulative frequency
distribution of the less than type.
Marks obtained
5 10 15 20
(More than or equal to) Number of students
Marks obtained
Number of students (cumulative frequency)
30 23 8 2
(cumulative frequency) Less than 10 10

Change the above data into a continuous grouped Less than 20 10 + 8 = 18


frequency distribution. [CBSE 2015] Less than 30 18 + 7 = 25
Sol. Given, distribution is the more than type distribution. Less than 40 25 + 4 = 29
Here, we observe that, all 30 students have obtained marks Less than 50 29 + 6 = 35
more than or equal to 5. Further, since 23 students have
obtained score more than or equal to 10. So, 30 − 23 = 7 Less than 60 35 + 8 = 43
students lie in the class 5-10. Similarly, we can find the other Less than 70 43 + 5 = 48
classes and their corresponding frequencies. Now, we
Less than 80 48 + 9 = 57
construct the continuous grouped frequency distribution as
Less than 90 57 + 5 = 62
Class (Marks obtained) Number of students
Less than 100 62 + 8 = 80
5-10 30 – 23 = 7
Here, 10, 20, 30,…, 100 are the upper limits of the
10-15 23 – 8 = 15 respective class intervals.
15-20 8–2=6
Example 7. In a class of 72 students, marks obtained by
More than or equal to 20 2 the students in a class test (out of 10) are given
below:
Example 6. Consider a grouped frequency distribution
of marks obtained out of 100, by 70 students in a Marks obtained
1 2 3 4 6 7 9 10
certain examination, as follows: (Out of 10)
Number of students 3 5 12 18 23 8 2 1
Marks Number of students
0-10 10 Find the mode of the data.
Sol. The mode of the given data is 6 as it has the maximum
10-20 8 frequency, i.e. 23 among all the observations.
20-30 7 Example 8. The weight of coffee in 70 packets are
30-40 4 shown in the following table
40-50 6 Weight (in gm) Number of packets
50-60 8 200-201 12
60-70 5 201-202 26
70-80 9 202-203 20
80-90 5 203-204 9
90-100 8 204-205 2
205-206 1
Form the cumulative frequency distribution of less
than type. Determine the modal weight.
CBSE Term II Mathematics X (Standard) 151

Sol. In the given data, the highest frequency is 26, which lies in Sol. In a given data, the highest frequency is 41, which lies in the
the interval 201-202 interval 10000-15000.
Here, l = 201, f1 = 26, f0 = 12 , f2 = 20 and h = 1 Here, l = 10000, f1 = 41, f0 = 26,
⎛ f −f ⎞
∴ Mode = l + ⎜ 1 0 ⎟ × h f2 = 16 and h = 5000
⎝ 2 f1 − f0 − f2 ⎠ ⎛ f −f ⎞
∴ Mode = l + ⎜ 1 0 ⎟ × h
⎛ 26 − 12 ⎞ ⎝ 2 f1 − f0 − f2 ⎠
= 201 + ⎜ ⎟ ×1
⎝ 2 × 26 − 12 − 20⎠
⎛ 41 − 26 ⎞
⎛ 14 ⎞ 14 = 10000 + ⎜ ⎟ × 5000
= 201 + ⎜ ⎟ = 201 + ⎝ 2 × 41 − 26 − 16⎠
⎝ 52 − 32 ⎠ 20
⎛ 15 ⎞
= 201 + 0.7 = 201.7 gm = 10000 + ⎜ ⎟ × 5000
⎝ 82 − 42 ⎠
Hence, the modal weight is 201.7 gm.
⎛ 15 ⎞
Example 9. Find the mode of the following distribution = 10000 + ⎜ ⎟ × 5000
⎝ 40⎠
Marks 0-10 10-20 20-30 30-40 40-50 50-60 = 10000 + 15 × 125
Number of = 10000 + 1875
4 6 7 12 5 6
students = ` 11875
Hence, the modal income is ` 11875.
Sol. Given, distribution table is
Example 11. Find the median of the following data.
Marks Number of students
Marks obtained 20 29 28 42 19 35 51
0-10 4
10-20 6 Number of students 3 4 5 7 9 2 3

20-30 7 ( f0 ) Sol. Let us arrange the data in ascending order of x i and make a
30-40 12 ( f1 ) cumulative frequency table.
40-50 5 ( f2 ) Marks Number of Cumulative
obtained (x i ) students (fi ) frequency (cf )
50-60 6
19 9 9
The highest frequency in the given distribution is 12, whose
corresponding class is 30 - 40. 20 3 9 + 3 = 12
Thus, 30-40 is the required modal class. 28 5 12 + 5 = 17
Here, l = 30, f1 = 12 , f0 = 7, f2 = 5 and h = 10 29 4 17 + 4 = 21
f1 − f0
∴ Mode = l + ×h 35 2 21 + 2 = 23
2 f1 − f0 − f2
12 − 7 42 7 23 + 7 = 30
= 30 + × 10
2 × 12 − 7 − 5 51 3 30 + 3 = 33
50 50
= 30 + = 30 + = 30 + 4.17 = 34.17 Here, n = 33 (odd)
24 − 12 12
Hence, mode of the given distribution is 34.17. ⎛ n + 1⎞
∴ Median = Value of ⎜ ⎟ th observation
⎝ 2 ⎠
Example 10. The monthly income of 100 families are ⎛ 33 + 1⎞
given as below = Value of ⎜ ⎟ th observation
⎝ 2 ⎠
Income (in `) Number of families = Value of 17th observation
0-5000 8 Corresponding value of 17th observation of cumulative
5000-10000 26 frequency in x i is 28. Hence, median is 28.
10000-15000 41 Example 12. 200 surnames were randomly picked up
15000-20000 16 from a local telephone directory and the frequency
20000-25000 3 distribution of the number of letters in English
25000-30000 3 alphabets in the surnames was obtained as follows:
30000-35000 2 Number of letters 0-5 5-10 10-15 15-20 20-25
35000-40000 1 Number of surnames 20 60 80 32 8
Calculate the modal income. Find the median of the above data.
152 CBSE Term II Mathematics X (Standard)

Sol. The cumulative frequency table of given data is ( 45 − 40 − p)


= 50 + × 10
20
Number of Number of Cumulative
⎛ 5 − p⎞ 5−p
letters surnames ( fi ) frequency (cf) ⇒ 50 = 50 + ⎜ ⎟ ⇒0 = [Median = 50]
⎝ 2 ⎠ 2
0-5 20 20
∴ p=5
5-10 60 20 + 60 = 80 ( cf ) Also, 78 + p + q = 90 [given]
10-15 80 ( = f ) 80 + 80 = 160 ⇒ 78 + 5 + q = 90
15-20 32 160 + 32 = 192 ⇒ q = 90 − 83
20-25 8 192 + 8 = 200 ∴ q =7

Total N = 200 Example 14. The median of the following data is 525.
Find the values of x and y, if total frequency is 100.
Since, the cumulative frequency just greater than 100 is 160
and the corresponding class interval is 10-15. 0- 100- 200- 300- 400- 500- 600- 700- 800- 900-
Class
N 200 100 200 300 400 500 600 700 800 900 1000
∴ N = 200; ∴ = = 100
2 2 Frequency 2 5 x 12 17 20 y 9 7 4
Here, l = 10, cf = 80, h = 5 and f = 80
⎧N ⎫ Sol. Given, frequency table is
− cf
⎪2 ⎪ ⎧100 − 80 ⎫
Now, median = l + ⎨ ⎬ × h = 10 + ⎨ ⎬×5 Class Frequency ( f1 ) Cumulative Frequency ( cf )
⎪ f ⎪ ⎩ 80 ⎭
⎩ ⎭ 0-100 2 2
⎛ 20 ⎞ 100-200 5 7
= 10 + ⎜ ⎟ × 5 = 10 + 1.25 = 11.25
⎝ 80⎠
200-300 x 7+x
Example 13. The median of the following data is 50. 300-400 12 19 + x
Find the values of p and q, if the sum of all the
400-500 17 36 + x ( cf )
frequencies is 90.
Marks Frequency 500-600 20 ( f ) 56 + x
20-30 p 600-700 y 56 + x + y
30-40 15 700-800 9 65 + x + y
40-50 25 800-900 7 72 + x + y
50-60 20 900-1000 4 76 + x + y
60-70 q
Given, total frequency is 100.
70-80 8
∴ 2 + 5 + x + 12 + 17 + 20 + y + 9 + 7 + 4 = 100
80-90 10 ⇒ 76 + x + y = 100
Sol. ⇒ x + y = 24 …(i)
Marks Frequency ( fi ) Cumulative frequency ( cf ) It is given that the median is 525.
Clearly, 525 lies in the class 500-600. So, 500-600 is the
20-30 p p
median class.
30-40 15 15 + p
Here, l = 500, h = 100, f = 20 and cf = 36 + x
40-50 25 40 + p = cf ∴ N = 100
50-60 20 ( = f ) 60 + p N
60-70 q 60 + p + q − cf
Q Median = l + 2 ×h
70-80 8 68 + p + q f
80-90 10 78 + p + q 50 − 36 − x
⇒ 525 = 500 + × 100
20
Given, N = 90
⇒ 525 = 500 + (14 − x ) × 5
N 90
∴ = = 45 ⇒ 525 = 500 + 70 − 5x
2 2 ⇒ 5x = 570 − 525
which lies in the interval 50-60. 45
⇒ 5x = 45 ⇒ x = =9
Here, l = 50, f = 20, cf = 40 + p and h = 10 5
⎛N ⎞ Put x = 9 in Eq. (i), we get
⎜ − cf⎟
⎝2 ⎠ 9 + y = 24 ⇒ y = 24 − 9 = 15
Q Median = l + ×h
f Hence, x = 9 and y = 15
CBSE Term II Mathematics X (Standard) 153

Chapter
Practice
8. If the arithmetic mean of the following distribution
PART 1 is 47, then the value of p is

Objective Questions Class interval 0-20 20-40 40-60 60-80 80-100


Frequency 8 15 20 p 5
!
Multiple Choice Questions
(a) 10 (b) 11 (c) 13 (d) 12
1. Which of the following is a measure of central
tendency? 9. The times (in seconds) taken by 150 atheletes to run
(a) Frequency (b) Cumulative frequency
a 110 m hurdle race are tabulated below
(c) Mean (d) Class-limit 13.8- 14- 14.2- 14.4- 14.6- 14.8-
Class
2. While computing mean of grouped data, we assume 14 14.2 14.4 14.6 14.8 15
that the frequencies are Frequency 2 4 5 71 48 20
(a) evenly distributed over all the class
(b) centred at the class marks of the class The number of atheletes who completed the race in
(c) centred at the upper limits of the class less than 14.6 s is
(d) centred at the lower limits of the class (a) 11 (b) 71 (c) 82 (d) 130
3. While computing the mean of grouped data, we 10. For the following distribution
assume that the frequencies are
(a) evenly distributed over all the class Marks Number of students
(b) centred at the class marks of the class Below 10 3
(c) centred at the upper limits of the class Below 20 12
(d) centred at the lower limits of the class
Below 30 27
4. If the difference of mode and median of a data is 24,
Below 40 57
then the difference of median and mean is
(a) 12 (b) 24 (c) 8 (d) 36 Below 50 75
n
Below 60 80
5. If x is the mean of x’s, then the value of ∑ x i is
i =1
The modal class is
x x
(a) (b) 2 x (c) n x (d) (a) 10-20 (b) 20-30 (c) 30-40 (d) 50-60
2 n
6. If x i ’s are the mid-points of the class intervals of 11. Consider the following distribution
grouped data, f i ’s are the corresponding Marks obtained Number of students
frequencies and x is the mean, then Σ ( f i x i − x ) is More than or equal to 0 63
equal to More than or equal to 10 58
(a) 0 (b) −1 (c) 1 (d) 2 More than or equal to 20 55
Σf d
7. In the formula x = a + i i , for finding the mean More than or equal to 30 51
Σf i
More than or equal to 40 48
of grouped data d i ’s are deviation from a of
More than or equal to 50 42
(a) lower limits of the class
(b) upper limits of the class
The frequency of the class 30-40 is
(c) mid-points of the class
(d) frequencies of the class marks (a) 3 (b) 4 (c) 48 (d) 51
154 CBSE Term II Mathematics X (Standard)

12. For the following distribution 18. The mean, mode and median of grouped data will
Number of Number of
always be
Marks Marks (a) same
students students
Below 10 3 Below 40 57 (b) different
(c) depends on the type of data
Below 20 12 Below 50 75
(d) None of the above
Below 30 28 Below 60 80
19. The mean and median of a distribution are 14 and
The modal class is 15 respectively. The value of mode is
(a) 0-20 (b) 20-30 (c) 30-40 (d) 50-60 (a) 16 (b) 17
13. A student noted the number of cars passing through (c) 13 (d) 18
a spot on a road for 100 periods each of 3 min and
summarised in the table given below.
!
Case Based Study
Number of cars Frequency 20. Analysis of Water Consumption in a Society
0-10 7 An inspector in an enforcement squad of department
10-20 14
of water resources visit to a society of 100 families
and record their monthly consumption of water on
20-30 13
the basis of family members and wastage of water,
30-40 12 which is summarise in the following table.
40-50 20
Monthly Consumption 0- 10- 20- 30- 40- 50-
50-60 11 Total
(in kWh) 10 20 30 40 50 60
60-70 15
Number of Families 10 x 25 30 y 10 100
70-80 8

Then, the mode of the data is


(a) 34.7 (b) 44.7 (c) 54.7 (d) 64.7
14. Mode of the following grouped frequency
distribution is
Class 3-6 6-9 9-12 12-15 15-18 18-21 21-24
Frequency 2 5 10 23 21 12 3

(a) 13.6 (b) 15.6 (c) 14.6 (d) 16.6


15. If the number of runs scored by 11 players of a
cricket team of India are 5, 19, 42, 11, 50, 30, 21, 0,
52, 36, 27, then median is
(a) 30 (b) 32 (c) 36 (d) 27
16. Consider the following frequency distribution
Class 0-5 6-11 12-17 18-23 24-29
Frequency 13 10 15 8 11 Based on the above information, answer the
following questions.
The upper limit of the median class is
(i) The value of x + y is
(a) 17 (b) 17.5 (c) 18 (d) 18.5 (a) 50 (b) 42
17. Consider the following frequency distribution (c) 25 (d) 200
Class 65- 85- 105- 125- 145- 165- 185- (ii) If the median of the above data is 32, then x is
85 105 125 145 165 185 205 equal to
(a) 10 (b) 8
Frequency 4 5 13 20 14 7 4
(c) 9 (d) None of these
The difference of the upper limit of the median (iii) What will be the upper limit of the modal class?
class and the lower limit of the modal class is (a) 40 (b) 60
(a) 0 (b) 19 (c) 20 (d) 38 (c) 65 (d) 70
CBSE Term II Mathematics X (Standard) 155

(iv) If A be the assumed mean, then A is always (iv) If a machine work for 10 h in a day, then
(a) > (Actual mean) approximate time required to complete the work
(b) < (Actual Mean) for a machine is
(c) = (Actual Mean) (a) 3 days (b) 4 days
(d) Can’t say (c) 5 days (d) 6 days
(v) The class mark of the modal class is (v) The measure of central tendency is
(a) 25 (b) 35 (a) Mean (b) Median
(c) 30 (d) 45 (c) Mode (d) All of these
21. As the demand for the products grew a 22. Direct income in India was drastically impacted
manufacturing company decided to purchase more due to the COVID-19 lockdown. Most of the
machines. For which they want to know the mean companies decided to bring down the salaries of the
time required to complete the work for a worker. employees upto 50%.
The following table shows the frequency The following table shows the salaries (in percent)
distribution of the time required for each machine received by 50 employees during lockdown.
to complete a work.
Salaries received (in %) 50-60 60-70 70-80 80-90
Time (in hours) 15-19 20-24 25-29 30-34 35-39 Number of employees 18 12 16 4
Number of machines 20 35 32 28 25

Based on the above information, answer the


following questions.
(i) Total number of persons whose salary is reduced
by more than 20% is
Based on the above information, answer the
(a) 40 (b) 46 (c) 30 (d) 22
following questions.
(ii) Total number of persons whose salary is reduced
(i) The class mark of the modal class 30-34 is
by atmost 40% is
(a) 17 (b) 22
(a) 32 (b) 40
(c) 27 (d) 32
(c) 46 (d) 18
(ii) If x i ’s denotes the class mark and f i ’s denotes the
(iii) The modal class is
corresponding frequencies for the given data, then
(a) 50-60 (b) 60-70
the value of Σx i f i equals to
(c) 70-80 (d) 80-90
(a) 3600
(b) 3205 (iv) The median class of the given data is
(c) 3670 (a) 50-60 (b) 60-70
(d) 3795 (c) 70-80 (d) 80-90
(iii) The mean time required to complete the work for a (v) The empirical relationship among mean, median
worker is and mode is
(a) 27.10 h (a) 3 Median = Mode + 2 Mean
(b) 23 h (b) 3 Median = Mode −2 Mean
(c) 24 h (c) Median = 3 Mode −2 Mean
(d) None of the above (d) Median = 3 Mode + 2 Mean
156 CBSE Term II Mathematics X (Standard)

Find the mean mileage.


PART 2 The manufacturer claimed that the mileage of the
model was 16 kmL −1 . Do you agree with this
Subjective Questions claim?
9. An NGO working for welfare of cancer patients,
!
Short Answer Type Questions maintained its records as follows:
1. Find the class marks of the class 15-35 and 0-20 20-40 40-60 60-80
Age of patients (in years)
class 45-60.
Number of patients 35 315 120 50
2. What is the arithmetic mean of first n natural
numbers? Find mode. [CBSE 2016]
3. Calculate the mean of the following data. 10. Find the mode of the following distribution.
Class 4-7 8-11 12-15 16-19
Class 10-15 15-20 20-25 25-30 30-35 35-40
Frequency 5 4 9 10
Frequency 45 30 75 20 35 15

4. Find the mean of the distribution.


11. Find the mode of the following distribution.
Class 1-3 3-5 5-7 7-10
Class 25-30 30-35 35-40 40-45 45-50 50-55
Frequency 9 22 27 17
Frequency 20 36 53 40 28 14
5. The following table gives the number of pages
12. Compute the mode for the following frequency
written by Sarika for completing her own book for
distribution.
30 days.
Size of items
Number of pages 16-18 19-21 22-24 25-27 28-30 0-4 4-8 8-12 12-16 16-20 20-24 24-28
(in cm)
written per day
Frequency 5 7 9 17 12 10 6
Number of days 1 3 4 9 13
13. Find the mode of the following frequency
Find the mean number of pages written per day. distribution.
6. The mean of the following data is 14. Find the value Class 15-20 20-25 25-30 30-35 35-40 40-45
of k. Frequency 3 8 9 10 3 2
x 5 10 15 20 25
14. The set of data given below shows the ages of
f 7 k 8 4 5 participants in a certain summer camp. Draw a
cumulative frequency table for the data.
7. The mean of the following frequency distribution is
18. The frequency f in the class interval 19-21 is Age (in years) 10 11 12 13 14 15
missing. Determine f . Frequency 3 18 13 12 7 27
Class interval 11-13 13-15 15-17 17-19 19-21 21-23 23-25
15. Given below is a cumulative frequency distribution
Frequency 3 6 9 13 f 5 4 showing the marks secured by 50 students of a
class
8. The mileage (kmL −1 ) of 50 cars of the same model Below 20 Below 40 Below 60 Below 80 Below
was tested by a manufacturer and details are Marks 100
tabulated as given below.
Number of 17 22 29 37 50
Mileage (kmL −1 ) 10-12 12-14 14-16 16-18 students

Number of cars 7 12 18 13 Form the frequency distribution table for the data.
CBSE Term II Mathematics X (Standard) 157

16. The following table shows the cumulative 19. From the following distribution, find the median
frequency distribution of marks of 800 students in
Class Frequency
an examination.
500-600 36
Marks Number of students 600-700 32
Below 10 10 700-800 32
Below 20 50 800-900 20
Below 30 130 900-100 30

Below 40 270 20. Size of agricultural holdings in a survey of 200


Below 50 440 families is given in the following table
Below 60 570 Size of agricultural Number of
holdings (in hectare) families
Below 70 670
0-5 10
Below 80 740
5-10 15
Below 90 780
10-15 30
Below 100 800
15-20 80
Construct a frequency distribution table for the 20-25 40
data above. 25-30 20
17. The following distribution of weights (in kg) of 30-35 5
40 persons.
Compute median and modal class of the holdings.
Weight (in kg) Number of persons
21. If median = 137 units and mean = 137.05 units, then
40-45 4
find the mode.
45-50 4
50-55 13 !
Long Answer Type Questions
55-60 5 22. The weights (in kg) of 50 wrestlers are recorded in
60-65 6 the following table.
65-70 5 Number of
Weight (in kg)
70-75 2 wrestlers
75-80 1 100-110 4
110-120 14
Construct a cumulative frequency distribution (of
the less than type) table for the data above. 120-130 21

18. Form the frequency distribution table from the 130-140 8


following data 140-150 3

Marks (Out of 90) Number of candidates Find the mean weight of the wrestlers.
More than or equal to 80 4 23. If mode of the following series is 54, then find the
More than or equal to 70 6 value of f .
More than or equal to 60 11 Class
interval 0-15 15-30 30-45 45-60 60-75 75-90
More than or equal to 50 17
More than or equal to 40 23 Frequency 3 5 f 16 12 7
More than or equal to 30 27 Find the modal class in which the given mode lies
More than or equal to 20 30 and find the value of f by using the formula,
More than or equal to 10 32 ⎧ f1 − f 0 ⎫
Mode = l + ⎨ ⎬×h.
More than or equal to 0 34 ⎩2 f 1 − f 0 − f 2 ⎭
158 CBSE Term II Mathematics X (Standard)

24. Find the mode of the following distribution. 29. Weekly income of 600 families is tabulated below
Classes 0-20 20-40 40-60 60-80 80-100 Weekly income (in `) Number of families
Frequency 10 8 12 16 4 0-1000 250
1000-2000 190
25. The following are the ages of 300 patients getting 2000-3000 100
medical treatment in a hospital on a particular day 3000-4000 40
Age (in years) 10-20 20-30 30-40 40-50 50-60 60-70 4000-5000 15
5000-6000 5
Number of 60 42 55 70 53 20
patients Total 600

Form Compute the median income.


(i) less than type cumulative frequency distribution. 30. A survey regarding the heights (in cm) of 51 boys of
(ii) more than type cumulative frequency distribution. Class X of a school was conducted and the following
data was obtained:
26. Find the unknown entries a, b, c, d, e and f in the
following distribution of heights of students in a Heights (in cm) Number of boys
class Less than 140 4
Cumulative Less than 145 11
Height (in cm) Frequency
frequency Less than 150 29
150-155 12 a Less than 155 40
155-160 b 25 Less than 160 46
160-165 10 c Less than 165 51

165-170 d 43 Find the median height.


170-175 e 48 31. Find the missing frequencies in the
following frequency distribution table, if N = 100
175-180 2 f
and median is 32. CBSE 2019
Total 50
Marks obtained 0-10 10-20 20-30 30-40 40-50 50-60 Total
27. The maximum bowling speeds (in km/h) of Number of
33 players at a cricket coaching centre are given as 10 ? 25 30 ? 10 100
students
follows
32. The table below shows the salaries of 280 persons.
Number of
Speed (in km/h)
players Salary (in ` thousand) Number of persons
85-100 11 5-10 49
10-15 133
100-115 9
15-20 63
115-130 8
20-25 15
130-145 5 25-30 6

Calculate the median bowling speed. 30-35 7


35-40 4
28. Obtain the median for the following frequency
distribution. 40-45 2
45-50 1
x 1 2 3 4 5 6 7 8 9
Calculate (i) median of the data,
y 8 10 11 16 20 25 15 9 6
(ii) mode of the data.
CBSE Term II Mathematics X (Standard) 159

!
Case Based Questions (i) Estimate the mean time taken by a student to
finish the race.
33. The men’s 200 m race event at the 2020
Tokyo Olympic took place 3 and 4 August. Time
Number of Students
(in seconds)
A stopwatch was used to find the time that it took a
group of Athletes to run 200 m. 0-20 8
0-20 20-40 40-60 60-80 80-100 20-40 10
Time (in seconds)
8 10 13 6 3 40-60 13
Number of Students
60-80 6
80-100 3

(ii) What is the sum of lower limits of median class and


modal class.
(iii) How many students finished the race within
1 min?

SOLUTIONS
Objective Questions 8. (d) Let us construct the following table for finding the
1. (c) Mean is the measure of central tendency. arithmetic mean
2. (b) In computing the mean of grouped data, the frequencies Class Frequency ( fi ) Class mark
are centred at the class marks of the class. ( xi )
f i xi
interval
3. (b) While computing mean of grouped data, we assume that
0-20 8 10 80
the frequencies are centred at the class marks of the classes.
4. (a) We have, Mode − Median = 24 20-40 15 30 450
We know that, Mode = 3 Median − 2 Mean 40-60 20 50 1000
∴ Mode − Median = 2 Median − 2 Mean
60-80 p 70 70p
⇒ 24 = 2(Median − Mean)
⇒ Median − Mean = 12 80-100 5 90 450
5. (c) We know that, Total Σfi = 48 + p Σfi x i = 1980 + 70p
n

∑ xi Now, x=
Σfi x i
i =1
Q x= Σfi
n
n 1980 + 70p
=
⇒ ∑ xi = n x 48 + p
i =1 1980 + 70p
Σ fi x i ⇒ 47 =
6. (a)Q x= 48 + p
n
⇒ 2256 + 47 p = 1980 + 70p
∴ Σ ( fi x i − x ) = Σ fi x i − Σx = nx − nx [Q Σx = nx ]
⇒ 276 = 23p
=0
⇒ p = 12
7. (c) We know that, di = x i − a
9. (c) The number of atheletes who completed the race in less
i.e. di ’s are the deviation from a of mid-points of the classes. than 14.6 = 2 + 4 + 5 + 71 = 82
160 CBSE Term II Mathematics X (Standard)

10. (c) f1 − f0
Q Mode = l + ×h
2 f1 − f0 − f2
Marks Number of students Cumulative frequency
23 − 10
Below 10 3=3 3 = 12 + ×3
46 − 10 − 21
10-20 12 − 3 = 9 12 13 13
= 12 + × 3 = 12 + = 12 + 2. 6 = 14. 6
20-30 27 − 12 = 15 27 15 5
15. (d) Arranging the terms in ascending order,
30-40 57 − 27 = 30 57
0, 5, 11, 19, 21, 27, 30, 36, 42, 50, 52
40-50 75 − 57 = 18 75 Here n = 11 (odd)
50-60 80 − 75 = 5 80 ⎛ n + 1⎞
Q Median = ⎜ ⎟ th
⎝ 2 ⎠
Here, we see that the highest frequency is 30, which lies in
the interval 30-40. ⎛ 11 + 1⎞
Median value = ⎜ ⎟ th = 6th value = 27
11. (a) Given, the distribution table ⎝ 2 ⎠
16. (b) Given, classes are not continuous, so we make
Marks obtained Number of students continuous by subtracting 0.5 from lower limit and adding
0-10 63 − 58 = 5 0.5 to upper limit of each class.
10-20 58 − 55 = 3 Class Frequency Cumulative frequency
20-30 55 − 51 = 4 −0.5-5.5 13 13
30-40 51 − 48 = 3 5.5-11.5 10 23
40-50 48 − 42 = 6 11.5-17.5 15 38
50 42 17.5-23.5 8 46

Frequency of the modal class 30-40 is 3 from the above table. 23.5-29.5 11 57
12. (c) Let us first construct the following frequency distribution N 57
table. Here, = = 28. 5, which lies in the interval 11.5-17.5.
2 2
Marks Number of students Hence, the upper limit is 17.5.
0-10 3 17. (c)
10-20 9 Class Frequency Cumulative frequency
20-30 16 65-85 4 4
30-40 29 85-105 5 9

40-50 18 105-125 13 22
125-145 20 42
50-60 5
145-165 14 56
Since, the maximum frequency is 29 and the class
165-185 7 63
corresponding to this frequency is 30-40.
So, the modal class is 30-40. 185-205 4 67
13. (b) Here, modal class is 40-50. Since, it has maximum N 67
frequency which is 20. Here, = = 33. 5 which lies in the interval 125 -145.
2 2
∴ l = 40, f1 = 20, f0 = 12, f2 = 11 and h = 10
Hence, upper limit of median class is 145.
⎛ f −f ⎞
Q Mode = l + ⎜ 1 0 ⎟ × h Here, we see that the highest frequency is 20 which lies in
⎝ 2 f1 − f0 − f2 ⎠ 125-145. Hence, the lower limit of modal class is 125.
⎛ 20 − 12 ⎞ ∴ Required difference = Upper limit of median class
= 40 + ⎜ ⎟ × 10
⎝ 40 − 12 − 11⎠ − Lower limit of modal class
80 = 145 − 125 = 20
= 40 +
17 18. (c) No, the value of these three measures can be the same, it
= 40 + 4.7 = 44.7 depends on the type of data.
14. (c) We observe that the class 12-15 has maximum frequency. 19. (b) Given, mean = 14 and median = 15
Therefore, this is the modal class. By using empirical relationship,
We have, l = 12, h = 3 , f1 = 23 , f0 = 10 and f2 = 21 Mode = 3 Median −2 Mean = 3 × 15 − 2 × 14 = 45 − 28 = 17
CBSE Term II Mathematics X (Standard) 161

Case Based Study 27.10


(iv) (a) Approximate time =
20. (i) (c) Given x and y are the frequencies of class intervals 10
10-20 and 40-50, respectively. Then, = 2.710
10 + x + 25 + 30 + y + 10 = 100 ~
− 3 days
⇒ x + y = 25 …(1) (v) (d) Measure of central tendency are mean, median and
(ii) (c) Median is 32, which lies in 30-40. mode.
So, the median class is 30-40. 22. (i) (b)∴Required number of employees = 18 + 12 + 16
∴ l = 30, h = 10, f = 30, N = 100 = 46
and cf = 10 + x + 25 = x + 35 (ii) (a)∴Required number of employees = 12 + 16 + 4
⎛N ⎞ = 32
⎜ − cf ⎟ (iii) (a) The maximum frequency is 18 and the corresponding
Now, median = l + ⎜ 2 ⎟ ×h class is 50-60.
⎜ f ⎟
⎝ ⎠ Hence, modal class is 50-60.
⎡{50 − ( x + 35)} ⎤ (iv) (b) Consider the table
⇒ 30 + × 10 = 32
⎢⎣ 30 ⎥⎦
Salaries received Number of Cumulative
(15 − x ) (in %) employees ( fi ) frequency ( cf )
⇒ 30 + = 32
3
50-60 18 18
⇒ (15 − x ) = 6 ⇒ x = 9
Put x = 9 in Eq. (1), we get 60-70 12 18 + 12 = 30
y = 16 70-80 16 30 + 16 = 46
Hence, x = 9 and y = 16 80-90 4 46 + 4 = 50
(iii) (a) Since, the maximum frequency is 30, so the modal
class is 30-40. Total Σfi = 50
Hence, upper limit of the modal class is 40. N 50
(iv) (d) The value of assumed mean can be less, more or equal Here, = = 25
2 2
than the actual mean.
The cumulative frequency more than 25 lies in 60-70.
(v) (b) The modal class is 30-40.
Hence, median class is 60-70.
30 + 40 70
∴ Class mark = = = 35 (v) (a) As we know, Mode = 3 Median − 2 Mean
2 2
∴ 3 Median = Mode + 2 Mean
21. (i) (d) Class mark of class 30-34
30 + 34 Subjective Questions
=
2 Lower limit + Upper limit
1. We know that, Class mark =
64 2
=
2 15 + 35 50
∴ Class mark of 15-35 is = = = 25
= 32 2 2
(ii) (d) Let’s make the table 45 + 60 105
Class mark of 45-60 is = = 52.5
2 2
Class Class marks Frequency ( fi ) x i fi Sum of all the observations
( xi ) 2. Arithmetic mean =
Number of observations
15-19 17 20 340 1+2 +K+ n
=
20-24 22 35 770 n
n
25-29 27 32 864 [2 × 1 + ( n − 1)1]
= 2
30-34 32 28 896 n
[Q1 + 2 + K + n is an AP series whose first term is a = 1 and
35-39 37 25 925
common difference is d = 1. We know that, the
Total Σfi = 140 Σx i fi = 3795 n
sum of n th term of an AP is Sn = [2 a + ( n − 1)d ]]
2
Σx i fi = 3795 2 + n −1 n + 1
Σx f = =
(iii) (a) Mean time ( X ) = i i 2 2
Σfi 3. Since, given data is not continuous, so we subtract 0.5 from
3795 the lower limit and add 0.5 in the upper limit of each class.
=
140 Now, we first find the class mark x i of each class and then
= 27.10 proceed as follows
162 CBSE Term II Mathematics X (Standard)

Class marks Given, mean = 14


Class Frequency (fi ) f i xi Σfi x i 10k + 360
( xi ) ∴ = 14 ⇒ = 14
Σfi k + 24
3.5-7.5 5.5 5 27.5
⇒ 10k + 360 = 14( k + 24)
7.5-11.5 9.5 4 38 ⇒ 10k + 360 = 14k + 336
11.5-15.5 13.5 9 121.5 ⇒ 14k − 10k = 360 − 336
15.5-19.5 17.5 10 175 ⇒ 4k = 24
24
Σ fi = 28 Σ fi x i = 362 ∴ k= =6
4
Σfi x i 362 Hence, the value of k is 6.
Therefore, ( x ) mean = = = 12 . 93
Σfi 28 7. Table of given data is
Hence, mean of the given data is 12.93. Class interval Frequency (fi ) Mid-value x i fi
4. We first, find the class mark x i of each class and then (x i )
proceed as follows.
11-13 3 12 36
Class marks Frequency f i xi
Class 13-15 6 14 84
(xi ) (f i )
1-3 2 9 18 15-17 9 16 144
3-5 4 22 88 17-19 13 18 234
5-7 6 27 162 19-21 f 20 20f
7-10 8.5 17 144.5
21-23 5 22 110
Σ fi = 75 Σ fi x i = 412.5
23-25 4 24 96
Σ fi x i 412.5
Therefore, mean ( x ) = = = 5.5
Σ fi 75 Σfi x i =
Total Σfi = 40 + f
Hence, mean of the given distribution is 5.5. 704 + 20f
5. Since, given data is not continuous, so we subtract 0.5 from Σfi x i
the lower limit and add 0.5 in the upper limit of each class. Q Mean =
Σfi
Class mark Mid-value (x i ) Number of days f i xi 704 + 20f
(f i ) ∴ 18 = [Q mean = 18, given]
40 + f
15.5-18.5 17 1 17
⇒ 720 + 18f = 704 + 20f
18.5-21.5 20 3 60 ⇒ 16 = 2 f
21.5-24.5 23 4 92 ⇒ f=8
24.5-27.5 26 9 234 Hence, missing frequency in the given data is 8.
27.5-30.5 29 13 377 8.
Mileage Class marks Number of cars f i xi
Total 30 780
(kmL −1 ) (x i ) (f i )
Σ fi x i 780 10-12 11 7 77
Q Mean ( x ) = = = 26
Σ fi 30
12-14 13 12 156
Hence, the mean of pages written per day is 26.
14-16 15 18 270
6. Table for the given data is
16-18 17 13 221
xi fi fixi
Total Σ fi = 50 Σ fi x i = 724
5 7 35
10 k 10k Here, Σ fi = 50
15 8 120 and Σ fi x i = 724
Σ f x 724
20 4 80 Q Mean ( x ) = i i = = 14. 48
Σ fi 50
25 5 125
Hence, mean mileage is 14.48 kmL −1.
Total Σ fi = k + 24 Σ fi x i = 10 k + 360
No, the manufacturer is claiming mileage 1.52 kmL −1 more
Here, Σfi = k + 24 and Σfi x i = 10k + 360 than average mileage.
CBSE Term II Mathematics X (Standard) 163

9. In the given table, the highest frequency is 53 and


corresponding class of this frequency is 35-40.
Age of patients (in years) 0-20 20-40 40-60 60-80
Thus, 35-40 is a modal class.
Number of patients 35( f0 ) 315( f1 ) 120( f2 ) 50 Here, l = 35, f1 = 53, f0 = 36, f2 = 40 and h = 5
f1 − f0
Q Mode = l + ×h
Here, maximum frequency is 315 and the class 2 f1 − f0 − f2
corresponding to this frequency is 20-40. So, the modal 53 − 36
class is 20-40. = 35 + ×5
2 × 53 − 36 − 40
∴ l = 20, f1 = 315, f0 = 35, f2 = 120 and h = 20
17 × 5 85
⎛ f −f ⎞ = 35 + = 35 +
Now, Mode = l + ⎜ 1 0 ⎟ × h 106 − 76 30
⎝ 2 f1 − f0 − f2 ⎠
= 35 + 2 . 83 = 37 . 83 (approx)
⎛ 315 − 35 ⎞ Hence, mode of given data is 37.83.
= 20 + ⎜ ⎟ × 20
⎝ 2 × 315 − 35 − 120⎠
12. Given frequency distribution table is
⎛ 280 ⎞
= 20 + ⎜ ⎟ × 20 Size of items (in cm) Frequency
⎝ 630 − 155⎠
280 0-4 5
= 20 + × 20 4-8 7
475
= 20 + 11.79 = 31.79 8-12 9
Hence, average age of maximum number of patients is 31.79. 12-16 17
10. Given, distribution table is 16-20 12
Class Frequency 20-24 10
10-15 45 24-28 6
15-20 30( f0 ) The maximum frequency in the given distribution table is
20-25 75( f1 ) 17, which lies in the class interval 12-16.
∴Modal class = 12 -16
25-30 20( f2 )
So, l = 12 , f1 = 17, f0 = 9, f2 = 12 and h = 4
30-35 35 ⎛ f −f ⎞
Q Mode = l + ⎜ 1 0 ⎟ × h
35-40 15 ⎝ 2 f1 − f0 − f2 ⎠
The highest frequency in the given data is 75 and the ⎛ 17 − 9 ⎞
corresponding class is 20-25, which is a modal class. = 12 + ⎜ ⎟ ×4
⎝ 2 × 17 − 9 − 12 ⎠
Here, l = 20, f1 = 75, f0 = 30, f2 = 20 and h = 5
f1 − f0 ⎛ 8 ⎞
= 12 + ⎜ ⎟ ×4
Q Mode = l + ×h ⎝ 34 − 21⎠
2 f1 − f0 − f2
75 − 30 32
= 20 + ×5 = 12 + = 12 + 2 . 46 = 14. 46
2 × 75 − 30 − 20 13
45 × 5 Hence, mode of given distribution is 14.46.
= 20 + 13. Given, frequency distribution table is
150 − 50
225 Class Frequency (fi )
= 20 + = 20 + 2 .25 = 22 . 25
100 15-20 3
11. Given, distribution table is 20-25 8
Class Frequency 25-30 9
25-30 20 30-35 10
30-35 36 35-40 3
35-40 53 40-45 2

40-45 40 The maximum frequency in the given distribution table is


10, which lies in the class interval 30-35.
45-50 28
∴ Modal Class = 30-35
50-55 14
So, l = 30, f1 = 10, f0 = 9, f2 = 3 and h = 5
164 CBSE Term II Mathematics X (Standard)

f1 − f0 17. The cumulative distribution (less than type) table is shown


Q Mode = l + ×h
2 f1 − f0 − f2 below
10 − 9
= 30 + ×5 Weight (in kg) Cumulative frequency (cf )
2 × 10 − 9 − 3
5 Less than 45 4
= 30 + = 30 + 0. 625 = 30. 625
8 Less than 50 4+ 4=8
Hence, mode of given distribution is 30.625. Less than 55 8 + 13 = 21
14. The cumulative frequency of first observation is the same as Less than 60 21 + 5 = 26
its frequency, since there is no frequency before it.
Less than 65 26 + 6 = 32
Now, the cumulative frequency table is
Less than 70 32 + 5 = 37
Age Frequency Cumulative frequency
Less than 75 37 + 2 = 39
(in years) (f i ) (cf)
Less than 80 39 + 1 = 40
10 3 3
11 18 3 + 18 = 21 18. Here, we observe that, all 34 students have scored marks
more than or equal to 0. Since, 32 students have scored
12 13 21 + 13 = 34 marks more than or equal to 10. So, 34 − 32 = 2 students lies
13 12 34 + 12 = 46 in the interval 0-10 and so on.
Now, we construct the frequency distribution table.
14 7 46 + 7 = 53
Class interval Number of candidates (f i )
15 27 53 + 27 = 80
0-10 34 − 32 = 2
15. Here, we observe that, 17 students have scored marks below
20 i.e. it lies between class interval 0-20 and 22 students 10-20 32 − 30 = 2
have scored marks below 40, so 22 − 17 = 5 students lies in 20-30 30 − 27 = 3
the class interval 20-40 continuting in the same manner, we
30-40 27 − 23 = 4
get the complete frequency distribution table for given data.
40-50 23 − 17 = 6
Marks Number of students
50-60 17 − 11 = 6
0-20 17 60-70 11 − 6 = 5
20-40 22 − 17 = 5 70-80 6− 4 =2
40-60 29 − 22 = 7 80-90 4
60-80 37 − 29 = 8
19. The cumulative frequency table for given distribution is
80-100 50 − 37 = 13
Class Frequency (fi ) Cumulative Frequency
16. Here, we observe that 10 students have scored marks below (cf)
10 i.e. it lies between class interval 0-10. Similarly, 50 500-600 36 36
students have scored marks below 20. So, 50 − 10 = 40
600-700 32 36 + 32 = 68
students lies in the interval 10 - 20 and so on. The table of a
frequency distribution for the given data is 700-800 32 (f) 68 + 32 = 100
800-900 20 100 + 20 = 120
Class interval Number of students (f i )
900-1000 30 120 + 30 = 150
0-10 10
10-20 50 − 10 = 40 N 150
Here, = = 75, which lies in the cumulative frequency
20-30 130 − 50 = 80 2 2
100, whose corresponding class is 700-800. Thus, modal class
30-40 270 − 130 = 140 is 700-800.
40-50 440 − 270 = 170 Here, l = 700, cf = 68, f = 32 and h = 100
N
50-60 570 − 440 = 130 − cf
Q Median = l + 2 ×h
60-70 670 − 570 = 100 f
70-80 740 − 670 = 70 75 − 68 700
= 700 + × 100 = 700 +
32 32
80-90 780 − 740 = 40
= 700 + 21. 88 = 721. 88
90-100 800 − 780 = 20 Hence, median of the given distribution is 721.88.
CBSE Term II Mathematics X (Standard) 165

20. By assumed mean method,


Σ fi di
Size of agricultural Number of Cumulative Mean ( x ) = a +
Σ fi
holdings (in hec) families ( f i ) frequency (cf )
( − 80)
0-5 10 10 = 125 +
50
5-10 15 25 = 125 − 1. 6 = 123. 4 kg
10-15 30 55 23. Here, given mode is 54, which lies between 45-60.
Therefore, the modal class is 45-60.
15-20 80 (f) 135
∴ l = 45, f1 = 16, f0 = f , f2 = 12 and h = 15
20-25 40 175 ⎛ f −f ⎞
Q Mode = l + ⎜ 1 0 ⎟ × h
25-30 20 195 ⎝ 2 f1 − f0 − f2 ⎠
30-35 5 200 ⎛ 16 − f ⎞
∴ 54 = 45 + ⎜ ⎟ × 15
⎝ 2 × 16 − f − 12 ⎠
I. Here, N = 200 16 − f
N 200 ⇒ 9= × 15
Now, = = 100, which lies in the interval 15-20. 20 − f
2 2
⇒ 9(20 − f ) = 15(16 − f )
Here, l = 15 , h = 5, f = 80 and cf = 55
⎛N ⎞ ⇒ 180 − 9f = 240 − 15f
⎜ − cf ⎟ ⇒ 6f = 240 − 180 = 60
∴ Median = l + ⎜ 2 ⎟ ×h
⎜ f ⎟ ⇒ f = 10
⎝ ⎠
Hence, required value of f is 10.
⎛ 100 − 55⎞
= 15 + ⎜ ⎟ ×5 24. The given distribution table is
⎝ 80 ⎠
Class Frequency ( f )
⎛ 45⎞
= 15 + ⎜ ⎟
⎝ 16 ⎠ 0-20 10
= 15 + 2. 81 = 17. 81 hec 20-40 8
II. In a given table 80 is the highest frequency.
40-60 12 ( f0 )
So, the modal class is 15-20.
60-80 16 ( f1 )
21. Given, median = 137 units and mean= 137. 05 units.
We know that, 80-100 4 ( f2 )
Mode = 3(Median) − 2(Mean) The highest frequency in the given distribution table is 16,
= 3 (137 ) − 2(137. 05) whose corresponding class is 60-80. Thus, 60-80 is the modal
class of the given distribution.
= 411 − 274. 10
Here, l = 60, f1 = 16, f0 = 12, f2 = 4 and h = 20
= 136.90 f1 − f0
Q Mode = l + ×h
Hence, the value of mode is 136.90 units. 2 f1 − f0 − f2
22. We first find the class mark x i , of each class and then 16 − 12
= 60 + × 20
proceed as follows 2 × 16 − 12 − 4
Number of Deviations 4 × 20 80
Weight Class = 60 + = 60 +
wrestlers marks ( x ) d i = x i − a, f i di 32 − 16 16
(in kg) i
(f i ) a = 125 = 60 + 5 = 65
100-110 4 105 −20 −80 Hence, mode of the given distribution is 65.
25. (i) We observe that the number of patients which take
110-120 14 115 −10 −140 medical treatment in a hospital on a particular day less
120-130 21 a = 125 0 0 than 10 is 0. Similarly, less than 20 include the number of
patients which take medical treatment from 0-10 as well
130-140 8 135 10 80 as the number of patients which take medical treatment
from 10-20.
140-150 3 145 20 60
So, the total number of patients less than 20 is
N = Σ fi = 50 Σ fi di = − 80 0 + 60 = 60, we say that the cumulative frequency of the
class 10-20 is 60. Similarly, for other classes, which is
∴Assumed mean ( a ) = 125, shown below the table.
Class width ( h ) = 10 and total observation (N ) = 50
166 CBSE Term II Mathematics X (Standard)

(ii) Also, we observe that all 300 patients which take medical 27. First we construct the cumulative frequency table
treatment more than or equal to 10. Since, there are 60
patients which take medical treatment in the interval Number of Cumulative
Speed (in km/h)
10-20, this means that there are 300 − 60 = 240 patients players ( f i ) frequency ( cf )
which take medical treatment more than or equal to 20. 85-100 11 11
Continuing in the same manner, which is shown below 100-115 9 (f) 11 + 9 = 20
the table.
115-130 8 20 + 8 = 28
(i) Less than type (ii) More than type 130-145 5 28 + 5 = 33
Number of Number of
Age (in years) Age (in years) It is given that, N = 33
students students
N 33
Less than 10 0 More than or 300 ∴ = = 16. 5
2 2
equal to 10
So, the median class is 100-115.
Less than 20 60 More than or 240
equal to 20 Here, l = 100, f = 9, cf = 11 and h = 15
⎛N ⎞
Less than 30 102 More than or 198 ⎜ − cf⎟
⎝2 ⎠
equal to 30 Q Median = l + ×h
f
Less than 40 157 More than or 143
equal to 40 (16. 5 − 11)
= 100 + × 15
9
Less than 50 227 More than or 73
5. 5 × 15
equal to 50 = 100 +
9
Less than 60 280 More than or 20
82. 5
equal to 60 = 100 +
9
Less than 70 300
= 100 + 9.17 = 109.17
26. Hence, the median bowling speed is 109.17 km/h.
Cumulative Cumulative 28. Here, the given data is in ascending order of x i .
Frequency Cumulative frequency table for the given data is
Height (in cm) (f i ) frequency frequency
(given) ( cf )
x
i
fi cf
150-155 12 a 12
1 8 8
155-160 b 25 12 + b
2 10 18
160-165 10 c 22 + b
3 11 29
165-170 d 43 22 + b + d
4 16 45
170-175 e 48 22 + b + d + e
5 20 65
175-180 2 f 24 + b + d + e
6 25 90
Total 50
7 15 105
On comparing last two tables, we get
8 9 114
a = 12
9 6 120
∴ 12 + b = 25
⇒ b = 25 − 12 = 13 Here, n = 120 (even)
22 + b = c 1⎡ ⎧⎛ n ⎞ ⎛n ⎞ ⎫⎤
c = 22 + 13 = 35 ∴Median = Value of ⎨⎜⎝ ⎟⎠ th + ⎜⎝ + 1⎟⎠ th ⎬⎥ observations
⇒ 2 ⎢⎣ ⎩ 2 2 ⎭⎦
22 + b + d = 43
1⎡ ⎧⎛ 120⎞ ⎛ 120 ⎞ ⎫⎤
⇒ 22 + 13 + d = 43 = Value of ⎨⎜⎝ ⎟ th + ⎜ + 1⎟ th ⎬⎥ observations
⇒ d = 43 − 35 = 8 2 ⎢⎣ ⎩ 2 ⎠ ⎝ 2 ⎠ ⎭⎦
22 + b + d + e = 48 1
= [Value of 60th observation + Value of 61th observation]
⇒ 22 + 13 + 8 + e = 48 2
⇒ e = 48 − 43 = 5 Both 60th and 61th observations lie in the cumulative
and 24 + b + d + e = f frequency 65 and its corresponding value of x i is 5.
⇒ 24 + 13 + 8 + 5 = f 1
∴ Median = ( 5 + 5) = 5
∴ f = 50 2
CBSE Term II Mathematics X (Standard) 167

29. First we construct a cumulative frequency table. Here, N = 51


N 51
Weekly income Number of families Cumulative frequency ∴ = = 25.5
2 2
(in `) (f i ) ( cf )
Since, the cumulative frequency just greater than 25.5 is 29
0-1000 250 250 and the corresponding class interval is 145-150.
1000-2000 190 = f 250 + 190 = 440 ∴ Median class = 145-150
= mid class Now, l = 145, f = 18, cf = 11 and h = 5
⎧N ⎫
2000-3000 100 440 + 100 = 540 − cf
⎪2 ⎪ ⎧25. 5 − 11 ⎫
∴ Median = l + ⎨ ⎬ × h = 145 + ⎨ ⎬×5
3000-4000 40 540 + 40 = 580 ⎪ f ⎪ ⎩ 18 ⎭
4000-5000 15 580 + 15 = 595 ⎩ ⎭
72.5
5000-6000 5 595 + 5 = 600 = 145 + = 145 + 4. 03 = 149. 03
18
It is given that, N = 600 Hence, the required median height is 149.03 cm.
N 600 31. Given, median = 32
∴ = = 300
2 2 and N = Σf = 100
Since, cumulative frequency 440 lies in the interval Let f1 and f2 be the frequencies of the class interval 10-20
1000 - 2000. and 40-50, respectively.
Here, l = 1000, f = 190, cf = 250 and h = 1000 Since, sum of frequencies = 100
⎧N ⎫ ∴ 10 + f1 + 25 + 30 + f2 + 10 = 100
⎨ − cf ⎬
⎩ 2 ⎭×h ⇒ f1 + f2 = 100 − 75 ⇒ f1 + f2 = 25
Q Median = l +
f ⇒ f2 = 25 − f1 …(i)
( 300 − 250) Now, the cumulative frequency table for given distribution is
= 1000 + × 1000
190
50 Class Cumulative
= 1000 + × 1000 Frequency ( f i )
190 interval frequency ( cf )
5000
= 1000 + 0-10 10 10
19
= 1000 + 263.15 10-20 f1 10 + f1
= 1263.15 20-30 25 35 + f1
Hence, the median income is ` 1263.15. 30-40 30 ( f ) 65 + f1
30. To calculate the median height, we need to convert the given
40-50 f2 65 + f1 + f2
data in the continuous grouped frequency distribution.
Given, distribution is of less than type and 140, 145, 150, …, 50-60 10 75 + f1 + f2
165 gives the upper limits of the corresponding class Total N = f1 + f2 + 75
intervals. So, the classes should be below 140, 140-145,
145-150, …, 160-165. N
Here, N = 100 ⇒ = 50
Clearly, the frequency of class interval below 140 is 4, since 2
there are 4 boys with height less than 140. For the frequency Given, median = 32, which belongs to the class 30-40.
of class interval 140-145 subtract the number of boys having So, the median class is 30-40.
height less than 140 from the number of boys having height
Then, l = 30, h = 10, f = 30 and cf = 35 + f1
less than 145.
⎧N ⎫
Thus, the frequency of class interval 140 - 145 is 11 − 4 = 7. − cf
⎪ ⎪
Similarly, we can calculate the frequencies of other class Q Median = l + ⎨ 2 ⎬×h
intervals and get the following table ⎪ f ⎪
⎩ ⎭
Cumulative ⎧ 50 − 35 − f1 ⎫
Class interval Frequency ( f i ) ∴ 32 = 30 + ⎨ ⎬ × 10
frequency (cf ) ⎩ 30 ⎭
Below 140 4 4 15 − f1
⇒ 32 − 30 =
140-145 11 − 4 = 7 11 3
145-150 29 − 11 = 18 = f 29 ⇒ 2 × 3 = 15 − f1
⇒ f1 = 15 − 6 = 9
150-155 40 − 29 = 11 40
On putting the value of f1 in Eq. (i), we get
155-160 46 − 40 = 6 46 f2 = 25 − 9 = 16
160-165 51 − 46 = 5 51 Hence, the missing frequencies are f1 = 9 and f2 = 16 .
168 CBSE Term II Mathematics X (Standard)

32. First, we construct a cumulative frequency table = ` 12.727 (in thousand)


= 12.727 × 1000 = ` 12727
Salary (in ` Number of Cumulative frequency
Hence, the median and modal salary are ` 13421 and
thousand) persons ( f i ) ( cf )
` 12727, respectively.
5-10 49 ( f0 ) 49 ( cf ) 33. (i)
10-15 f1 = 133 133 + 49 = 182
Time Number of Class mark f i xi
15-20 63 ( f2 ) 182 + 63 = 245 (in seconds) students ( f i ) ( xi )
20-25 15 245 + 15 = 260
0-20 8 10 8 × 10 = 80
25-30 6 260 + 6 = 266
20-40 10 30 10 × 30 = 300
30-35 7 266 + 7 = 273
40-60 13 50 13 × 50 = 650
35-40 4 273 + 4 = 277
60-80 6 70 6 × 70 = 420
40-45 2 277 + 2 = 279
80-100 3 90 3 × 90 = 270
45-50 1 279 + 1 = 280
Σfi = 40 Σfi x i = 1720
N = 280
N 280 Σfi x i 1720
∴ = = 140 Mean ( x ) = = = 43
2 2 Σfi 40
(i) Here, median class is 10-15, because 140 lies in it. ∴ Mean time is 43s.
∴ l = 10, f = 133, cf = 49 and h = 5 (ii)
⎛N ⎞ Time Number of Cumulative
⎜ − cf⎟
⎝2 ⎠ (in seconds) students ( f i ) frequency ( cf )
Q Median = l + ×h
f 0-20 8 8
(140 − 49)
= 10 + ×5 20-40 10 8 + 10 = 18
133
91 × 5 40-60 13 18 + 13 = 31
= 10 +
133 60-80 6 31 + 6 = 37
455
= 10 + = 10 + 3. 421 80-100 3 37 + 3 = 40
133
= ` 13. 421 (in thousand) Σfi = 40
= 13. 421 × 1000
Modal class is a class having highest frequency.
= ` 13421
So, 40-60 is modal class
(ii) Here, the highest frequency is 133, which lies in the
To find median class, we find cumulative frequency
interval 10-15, called modal class.
N 40
∴l = 10, h = 5, f1 = 133, f0 = 49, and f2 = 63. = = 20
2 2
⎛ f −f ⎞
∴ Mode = l + ⎜ 1 0 ⎟ × h ∴ 40-60 has cumulative frequency greater than 20.
⎝ 2 f1 − f0 − f2 ⎠
Thus, 40-60 is the median class.
⎧ 133 − 49 ⎫ ∴Sum of lower limits of median class and modal class
= 10 + ⎨ ⎬×5
⎩2 × 133 − 49 − 63 ⎭ = 40 + 40 = 80.
84 × 5 (iii) Students finished the race within 1 min
= 10 + = Students between 0-20 + Students between 20-40
266 − 112
84 × 5 + Students between 40-60
= 10 + = 10 + 2.727 = 8 + 10 + 13 = 31
154
Chapter Test
Multiple Choice Questions (ii) The modal value of the given data is
(a) 150 (b) 150.91
1. A survey conducted by a group of students is (c) 145.6 (d) 140.9
given as
(iii) The median value of the given data is
Family Size 1-3 3-5 5-7 7-9 9-11 (a) 140 (b) 146.67 (c) 130 (d) 136.6
Number of families 7 8 2 2 1 (iv) Assumed mean method is useful in
determining the
The mean of the data is (a) Mean (b) Median
(a) 6.8 (b) 4.2 (c) Mode (d) All of these
(c) 5.4 (d) None of these (v) The manufacturer can claim that the
2. The relationship among mean, median and mileage for his car is
mode for a distribution is (a) 144 km/charge (b) 155 km/charge
(a) Mode = Median − 2 mean (c) 165 km/charge (d) 175 km/charge
(b) Mode = 3 Median − 2 mean Short Answer Type Questions
(c) Mode = 2 Median − 3 mean
(d) Mode = Median − mean 5. Find the median of the first ten prime numbers.
3. For the following distribution 6. An aircraft has 120 passenger seats. The
number of seats occupied during 100 flights is
Class 0-5 5 -10 10 -15 15 - 20 20 - 25
given in the following table.
Frequency 10 15 12 20 9
Number of seats 100-104 104-108 108-112 112-116 116-120
The sum of lower limits of the median class and Frequency 15 20 32 18 15
modal class is
(a) 15 (b) 25 (c) 30 (d) 35 Determine the mean number of seats occupied
Case Based MCQs over the flights.

4. A Tesla car manufacturing industry wants to 7. The following distribution gives the daily
declare the mileage of their electric cars. For income of 50 workers of a factory:
this, they recorded the mileage (km/charge) of
Daily income (in `) 100-120 120-140 140-160 160-180 180-200
100 cars of the same model. Details of which
are given in the following table. Number of workers 12 14 8 6 10
Mileage 100-120 120-140 140-160 160-180 Write the above distribution as ‘less than type’
(km/charge)
cumulative frequency distribution. [CBSE 2015]
Number of Cars 14 24 36 26
8. Find the mode of the following frequency
distribution. [CBSE 2019]

Class 0-10 10-20 20-30 30-40 40-50 50-60 60-70

Frequency 8 10 10 16 12 6 7

Long Answer Type Questions

Based on the above information, answer the 9. Find the mean of the following frequency
following questions. distribution using assumed mean method.
(i) The average mileage is Class 2-8 8-14 14-20 20-26 26-32
(a) 140 km/charge
Frequency 6 3 12 11 8
(b) 150 km/charge
(c) 130 km/charge
(d) 144.8 km/charge

Answers
For Detailed Solutions
1. (b) 2. (b) 3. (b) 4. (i) (d) (ii) (b) (iii) (b) (iv) (a) (v) (a)
Scan the code
5. 12 6. 109.92 8. 36, 9. 18.8
170 CBSE Term II Mathematics X (Standard)
Mathematics (Standard)
Class 10th (Term II)

Practice Paper 1 *

(Solved)
Instructions Time : 2 Hr
Max. Marks : 40
1. The question paper contains three sections A, B and C.
2. Section A has 5 questions with 3 internal choices.
3. Section B has 4 questions with 3 internal choices.
4. Section C has 1 Case Based MCQs comprises of 5 MCQs.
5. There is no negative marking.
* As exact Blue-print and Pattern for CBSE Term II exams is not released yet. So the pattern of this
paper is designed by the author on the basis of trend of past CBSE Papers. Students are advised
not to consider the pattern of this paper as official, it is just for practice purpose.

Section A (3 Marks Each)


This section consists of 5 questions of Short Answer Type.
1. Find the value of k for which the quadratic equation (3k + 1 )x 2 + 2( k + 1 )x + 1 = 0, has equal roots . Also find
these roots.
2. Write the expression a n − a k for the AP a, a + d, a + 2 d, …
Hence, find the common difference of the AP for which 25th term is 10 more than the 23rd term.
Or If two towers of heights x m and y m subtend angles of 45° and 60°, respectively at the centre of a line
joining their feet, then find the ratio of ( x + y): y.
3. The length of common chord of two intersecting circles is 30 cm. If the diameters of these two circles are
50 cm and 34 cm, then calculate the distance between their centres.
Or Given, a line segment AB. Divide it in the ratio m : n by construction, where both m and n are positive
integers and let m = 4 and n = 3.
4. From a solid cube of side 7 cm, a conical cavity of height 7 cm and radius 3 cm is hollowed out. Find the
volume of the remaining solid.
5. The mode of the following series is 36. Find the missing (x ) frequency in it.
Class interval 0-10 10-20 20-30 30-40 40-50 50-60 60-70

Frequency 8 10 x 16 12 6 7

Or The 8th term of an AP is 17 and its 14th term is 29. Find its common difference.
CBSE Term II Mathematics X (Standard) 171

Section B (5 Marks Each)


This section consists of 4 questions of Long Answer Type.
6. Construct a tangent to a circle of radius 4 cm from a point on the concentric circle of radius 6 cm and measure
its length. Also, verify the measurement by actual calculation.
Or A decorative block as shown in figure is made of two solids, a cube and a hemisphere.
4.2 cm

cm
5
5 cm

5 cm

The base of the block is the cube with edge of 5 cm and the hemisphere attached on the top has a diameter of
4.2 cm. If the block is to be painted, then find the total area to be painted. [take, π = 22 / 7]
7. From the point, 36 m above the surface of a lake, the angle of elevation of a bird is observed to be 30° and angle
of depression of its image in the water of the lake is observed to be 60°. Find the actual height of the bird above
the surface of the lake.
Or Prove that the intercept of a tangent between two parallel tangents to a circle subtends a right angle at the
centre.
x − 1 2x + 1 5 1
8. Find the solution of the equation + = , x ≠ − , 1 by factorisation method.
2x + 1 x − 1 2 2
Or Find the median for the following frequency distribution.

Height (in cm) Frequency


160-162 15
163-165 117
166-168 136
169-171 118
172-174 14

9. A girl of height 120 cm is walking away from the base of a lamp-post at a speed of 1.2 m/s. If the lamp is 4.8 m
above the ground, then find the length of her shadow after 6 s.

Section C (1 Mark Each)


This section consists of 1 Case Based comprises of 5 MCQs.
Σf i x i
10. In a mathematic class, a teacher explain the concept for determine the mean by defining the formula x = .
Σf i
172 CBSE Term II Mathematics X (Standard)

Further, a teacher give one example for explaining the above (ii) Find the cumulative frequency value in the
concepts. interval (40-55).
The marks obtained by 30 students of class X of a certain (a) 5 (b) 12
school in a mathematics paper consisting of 100 marks are (c) 2 (d) 18
presented in table below (iii) Through cumulative frequency table, which
central measurement can be determined.
Class interval 10-25 25-40 40-55 55-70 70-85 85-100
(a) mean (b) mode
Number of 2 3 7 6 6 6 (c) median (d) None of these
Students (iv) Find the lower limit of the median class.
(i) Find the average marks obtained by the students. (a) 55 (b) 40
(a) 61 (c) 70 (d) 25
(b) 62 (e) Find the upper limit of modal class.
(c) 63 (a) 40 (b) 55
(d) 64 (c) 70 (d) 25

Solutions
1. Given quadratic equation is On subtracting Eq. (ii) from Eq. (i), we get
( 3k + 1) x 2 + 2 ( k + 1) x + 1 = 0 a n − a k = [ a + ( n − 1) d] − [ a + ( k − 1) d]
On comparing with ax 2 + bx + c = 0, we get ⇒ a n − a k = a + ( n − 1) d − a − ( k − 1 ) d
a = 3k + 1, b = 2 ( k + 1), c = 1 ⇒ a n − a k = ( n − 1 − k + 1) d
Since, the roots are equal, so b 2 − 4ac = 0 ⇒ a n − a k = (n − k) d
∴ 2
[2 ( k + 1)] − 4( 3k + 1) (1) = 0 Now, a n − a k = ( n − k ) d
⇒ a 25 − a 23 = (25 − 23) d [put n = 25and k = 23]
⇒ 4( k + 1)2 − 4( 3k + 1) = 0
⇒ 10 = 2 d [Qa 25 − a 23 = 10, given]
⇒ 4( k 2 + 2 k + 1) − 4( 3k + 1) = 0 ⇒ d=5
⇒ 4( k 2 + 2 k + 1 − 3k − 1) = 0 Hence, the common difference is 5.
⇒ k 2 + 2 k + 1 − 3k − 1 = 0 Or
⇒ k 2 + 2 k + 1 − 3k − 1 = 0 [Q 4 ≠ 0] Let AB = x m be the height of a tower and CD = y m be the
height of other tower and ∠AEB = 45° and ∠CED = 60°.
⇒ k2 − k = 0
D
⇒ k ( k − 1) = 0 B
⇒ k = 0 or k = 1 y
We know that, if roots are equal, then roots will be the form of x
−b −b 45° 60°
, . A E C
2a 2a a a
−b −b
Thus, roots are , . Let E be the point (centre) on the line AC.
2a 2a
− ( k + 1) − ( k + 1) i.e. AE = EC = a m
∴ Equal roots are , In right angled ΔBAE,
( 3k + 1) ( 3k + 1)
AB ⎡ perpendicular ⎤
− ( 0 + 1) − ( 0 + 1) tan 45° = Q tan θ =
When k = 0, equal roots are and AE ⎣⎢ base ⎥⎦
0+1 0+1
x
i.e. − 1 and − 1. ⇒ =1 [Q tan 45°= 1]
a
− (1 + 1 ) − (1 + 1 )
When k = 1, equal roots are and ⇒ x=a …(i)
3+1 3+1
Again, in right angled ΔDCE,
1 1
i.e. − and − . DC
2 2 tan 60° =
CE
2. Given, first term = a and common difference = d
y
∴ a n = a + ( n − 1) d …(i) ⇒ 3= [Q tan 60°= 3 ]
a
and a k = a + ( k − 1) d …(ii) ⇒ y = 3a …(i)
CBSE Term II Mathematics X (Standard) 173

∴ ( x + y ): y = ( a + 3a ): a 3 = (1 + 3 ): 3 (iv) Now, through the point A 4( m = 4), draw a line parallel


to A 7B by making an angle equal to ∠AA 7 B at A 4
Hence, the requred ratio ( x + y ): y is (1 + 3 ): 3. intersecting AB at a point C.
3. Let, PQ be the length of the common chord of two Then, AC : BC = 4 : 3
intersecting circles. Alternative Method
∴ PQ = 30cm [given] (i) Draw any ray AX making an acute angle with AB.
Diameters of two circles are 50 cm and 34 cm. (ii) Draw a ray BY parallel to AX by making
Join AB. ∠ABY = ∠BAX.
(iii) Locate the points A 1 , A 2 , A 3 , A 4 (for m = 4) on AX and
P
similarly B 1, B 2 , B 3 (for n = 3) on BY, such that
A O B A A 1 = A 1A 2 = A 2A 3 = A 3A 4
= BB1 = B1B 2 = B 2B 3
Q X
A4
Since, AB bisects the common chord PQ perpendicularly. A3
1 1 A2
∴ OP = OQ = × PQ = × 30 = 15 cm A1
2 2
C
1 A B
Radius, AP = × 50 = 25 cm
2
B1
1
and radius, PB = × 34 = 17 cm B2
2
B3
In right angled ΔAOP,
Y
OA = ( AP )2 − ( OP )2
[by using Pythagoras theorem] (iv) Join A 4B 3. Let it intersects AB at point C.
2 2 AC 4
= (25) − (15)
Then, =
BC 3
= 625 − 225 = 400 = 20 cm
4. Given, side of a solid cube, a = 7 cm
In right angled ΔPOB, Height of conical cavity, i.e. cone, h = 7 cm
OB = (PB )2 − ( OP )2 = (17 )2 − (15)2 Radius of conical cavity, r = 3 cm
[by using Pythagoras theorem]
= 289 − 225
= 64 = 8 cm 7 cm
∴ Distance between centres,
AB = OA + OB = 20 cm + 8 cm = 28 cm
3 cm
Or
Given A line segment AB, m = 4 and n = 3. Now, volume of cube = a = (7 )3 = 343 cm 3
3

Steps of construction 1
and volume of conical cavity = π × r2 × h
(i) Draw any ray AX making an acute angle with AB. 3
1 22
(ii) Locate 7 (i.e. m + n ) points A1 , A 2 , ...,A 7 on AX, =× × 3 × 3 × 7 = 66 cm 3
3 7
such that AA 1 = A 1A 2 = ... = A 6 A 7 .
According to the question,
(iii) Join BA 7.
Volume of remaining solid
X
A7 = Volume of cube − Volume of conical cavity
A6 = 343 − 66 = 277 cm 3
A5
A4 Hence, the required volume of solid is 277 cm 3.
A3 5. Since, the mode of the given series is 36, which lies in the
A2 class 30-40.
A1
So, the modal class is 30-40.
A Then, l = 30, f1 = 16, f0 = x, f2 = 12
C B
and h = 10
174 CBSE Term II Mathematics X (Standard)

Also, mode = 36 ∴ (PM )2 = ( OP )2 − ( OM )2 [by Pythagoras theorem]


⎧ f1 − f0 ⎫ ⇒ (PM )2 = ( 6)2 − ( 4)2 = 36 − 16 = 20
Q Mode = l + ⎨h × ⎬
⎩ 2 f1 − f0 − f2 ⎭ ⇒ PM = 20 = 4. 47 ≈ 4.5
⎧ 16 − x ⎫ Hence, the length of tangent is 4.5 cm.
∴ 36 = 30 + ⎨10 × ⎬
⎩ 2 × 16 − x − 12 ⎭ Or
10 (16 − x ) Given, edge of cube = 5 cm
⇒ 36 = 30 +
(20 − x ) and diameter of hemisphere = 4. 2 cm
10 (16 − x ) 6 10(16 − x ) 4.2
⇒ 36 − 30 = ⇒ = radius of hemisphere = cm = 2.1 cm
(20 − x ) 1 (20 − x ) 2
Clearly, total surface area of the cube
⇒ 10 (16 − x ) = 6 (20 − x )
= 6 (Edge) 2
⇒ 160 − 10 x = 120 − 6x
⇒ − 10x + 6x = 120 − 160 = 6 × 5 × 5 = 150 cm 2
⇒ − 4x = − 40 Now, area to be painted on the cube
− 40 = Total surface area of cube
∴ x= = 10
−4 − Base area of hemisphere
Hence, the missing frequency is 10. 2 22 4.2 4.2
= 150 − πr = 150 − × ×
Or 7 2 2
2
Given, = 150 − 13.86 = 136.14 cm
ε 8 = 17 , ε14 = 29 Area to be painted on the hemisphere
Let be a first term a = Curved surface area of hemisphere
ε n = a + ( n − 1 )d 22 4.2 4.2
= 2 πr 2 = 2 × × × = 27.72 cm 2
17 = 9 + ( 8 − 1)d 7 2 2
⇒ 17 = a + 7 d …(i) ∴ Total area to be painted
Similarly, 29 = a + 13d …(ii) = Area to be painted on the cube
Subtract Eq. (i) from Eq. (ii), + Area to be painted on the hemisphere
a + 13d − a − 7 d = 29 − 17 = d = 2 = 136.14 + 27.72 = 163.86 cm 2
6. Given, two concentric circles of radii 4 cm and 6 cm with 7. Let QR be the surface of the lake and P be point above the
common centre O. surface such that PQ = 36 m. Let B represents the bird and
Here, we have to draw two tangents to inner circle C1 from a Bʹ be its image in the lake.
point of outer circle C 2. B
Steps of construction x
(i) Draw two concentric circles C1 and C 2 with common 30°
centre O and radii 4 cm and 6 cm, respectively. P S
36 m 60° 36 m
(ii) Take any point P on outer circle C 2 and join OP. Q
(iii) Now, bisect OP. Let Mʹ be the mid-point of OP. R
(iv) Taking Mʹ as centre and OMʹ as radius, draw a dotted
circle which cuts the inner circle C1 at two points M (36+x) m
and Pʹ.
(v) Join PM and PPʹ. Thus, PM and PPʹ are required
tangents. Bʹ

M ∴ ∠BPS = 30° and ∠Bʹ PS = 60°


Also, SR = PQ = 36 m

P O Let BS = x
C1 C2
⇒ Bʹ R = BR = ( 36 + x ) m
Pʹ ∴ Bʹ S = SR + BR
= 36 + 36 + x = (72 + x ) m
On measuring PM and PPʹ, we get In right angled ΔPSB,
PM = PPʹ = 4. 5 cm BS
= tan 30°
Calculation In right angled ΔOMP, ∠ PMO = 90° PS
CBSE Term II Mathematics X (Standard) 175

x 1 ⎡ 1 ⎤ 1
∴ = Q tan 30° = = ( ∠1 + ∠1 + ∠ 3 + ∠ 3)
PS 3 ⎣⎢ 3 ⎥⎦ 2
⇒ PS = 3x …(i) 1
= ( ∠1 + ∠2 + ∠ 3 + ∠ 4)
Again, in right angled ΔBʹ SP , 2
Bʹ S 72 + x [from Eqs. (i) and (ii)]
= tan 60° ⇒ = 3 1
PS PS = (180°) = 90°
2
[Q B’S = (72+x) m and tan 60° = 3 ]
[Q QR is a straight line, therefore
72 + x
⇒ PS = …(ii) ∠1 + ∠2 + ∠ 3 + ∠ 4 = 180°]
3
From Eqs. (i) and (ii), we get Hence proved.
72 + x x −1 2x + 1 5
3x = 8. We have, + =
2x + 1 x −1 2
3
x −1
⇒ 3 ⋅ 3x = 72 + x Let y = , then given equation becomes
2x + 1
⇒ 3x − x = 72 ⇒ 2 x = 72
1 5 y2 + 1 5
⇒ x=
72
⇒ x = 36 y+ = ⇒ = ⇒ 2 y 2 − 5y + 2 = 0
2 y 2 y 2

∴ Height of bird above surface of the lake, This is a quadratic equation.


By using factorisation method,
BR = BS + SR = 36 + 36 = 72 m
2 y 2 − 4y − y + 2 = 0
Or
Let, AB and CD are two tangents to a circle and AB || CD. ⇒ 2 y( y − 2 ) − 1 ( y − 2 ) = 0
⇒ (2 y − 1)( y − 2 ) = 0
Tangent BD subtends ∠BOD at the centre.
⇒ 2 y − 1 = 0 or y − 2 = 0
To prove ∠BOD = 90° 1
⇒ y = or y = 2
Construction Join OP, OQ and OR. 2
x −1
A Q Put y = , we get
B 2x + 1
x −1 1 x −1
= or =2
2 2x + 1 2 2x + 1
O 1 ⇒ 2 x − 2 = 2 x + 1 or x − 1 = 4x + 2
3 ⇒ −2 = 1, which is not true.
4 P Consider, x − 1 = 4x + 2
⇒ 3x = − 3 ⇒ x = − 1
C R D
Or
Proof Here, OP ⊥ BD The given series is in inclusive form. Converting it to
exclusive form and preparing the cumulative frequency table
[Q a tangent at any point of a circle is perpendicular to the
is given below
radius through the point of contact]
In right angled ΔOQB and Δ OPB, Frequency (fi ) Cumulative
Class interval
frequency
BQ = BP
159.5-162.5 15 15
[Q the lengths of tangents drawn from an
external point are equal] 162.5-165.5 117 132
OQ = OP [radii] 165.5-168.5 136 268
OB = OB [common] 168.5-171.5 118 386

∴ ΔOQB ≅ Δ OPB [by SSS congruency] 171.5-174.5 14 400


Then, ∠1 = ∠ 2 [by CPCT] …(i) Total N = Σ f i = 400
Similarly, in right angled ΔOPD and ΔORD, Here, N = 400
∠ 3 = ∠4 …(ii) N 400
Now, = = 200
1 2 2
∴ ∠BOD = ∠1 + ∠ 3 = (2 ∠1 + 2 ∠ 3)
2
176 CBSE Term II Mathematics X (Standard)

The cumulative frequency just greater than 200 is 268 and the 7.2
⇒ 3x = 7.2 ⇒ x = = 2. 4 m
3
corresponding class is 165.5-168.5.
Hence, the length of her shadow after 6 s is 2.4 m.
Thus, the median class is 165.5-168.5.
10. (i) (b) Let us make the following table for the given data.
∴ l = 165.5, h = 3 and f = 136 and C = 132
⎧ N ⎫ Class marks Cumulative
− C. f Class f i xi
⎪ ⎪ Frequency (xi ) frequency
∴ Median = l + ⎨h × 2 Interval
⎬ (1/2) ( cf )
⎪ f ⎪
⎩ ⎭ 10 + 25
10-25 2 = 17. 5 35.0 2
⎧ (200 − 132 ) ⎫ 2
= 165.5 + ⎨3 × ⎬
⎩ 136 ⎭ 25 + 40
25-40 3 = 32. 5 97.5 5
3 × 68 2
= 165.5 + = 165.5 + 1.5 = 167
136 40 + 55
40-55 7 = 47. 5 332.5 12
Hence, the median height is 167 cm. 2
9. Let AB be the lamp-post, CD be the girl and D be the 55 + 70
55-70 6 = 62. 5 375.0 18
position of girl after 6 s. 2
Again, let DE = x m be the length of shadow of the girl. 70 + 85
70-85 6 = 77. 5 465.0 24
A 2
85 + 100
= 92. 5
85-100 6 2 555.0 30
4.8 m
C Σfi x i
Total Σfi = 30 = 1860. 0
E
B D x

Given, CD = 120 cm = 1.2 m, AB = 4. 8 m Here, Σfi = 30 and Σfi x i = 1860. 0


and speed of the girl = 1.2 m/s Σf x 1860. 0
∴ Distance of the girl from lamp-post after 6 s. Q Average, x = i i = = 62
Σfi 30
BD = 1.2 × 6 = 7.2 m [Q distance = speed × time]
Hence, average marks obtained by student is 62.
In ΔABE and ΔCDE,
(ii) (b) The cumulative frequency value in the interval
∠B = ∠D [each 90°] 40-55 is 12.
∠E = ∠E [common angle] (iii) (c) Through cumulative frequency table, median can be
∴ ΔABE ~ ΔCDE [by AA similarity criterion] determined.
BE AB N 30
⇒ = …(i) (iv) (a) Here, = = 15, which lies in the cumulative
DE CD 2 2
On substituting all the values in Eq. (i), we get frequency 18, whose corresponding frequency is 55-70.
7.2 + x 4.8 Hence, lower limit of the median class is 55.
= [Q BE = BD + DE = 7.2 + x ]
x 1.2 (v) (b) In the given data, the highest frequency is 7, whose
7.2 + x corresponding interval is 40-55.
⇒ =4
x Hence, upper limit of the modal class is 55.
⇒ 7.2 + x = 4x
CBSE Term II Mathematics X (Standard) 177
Mathematics (Standard)
Class 10th (Term II)

Practice Paper 2 *

(Unsolved)
Instructions Time : 2 Hr
Max. Marks : 40
1. The question paper contains three sections A, B and C.
2. Section A has 5 questions with 3 internal choices.
3. Section B has 4 questions with 3 internal choices.
4. Section C has 1 Case Based MCQs comprises of 5 MCQs.
5. There is no negative marking.
* As exact Blue-print and Pattern for CBSE Term II exams is not released yet. So the pattern of this
paper is designed by the author on the basis of trend of past CBSE Papers. Students are advised
not to consider the pattern of this paper as official, it is just for practice purpose.

Section A (3 Marks Each)


This section consists of 5 questions of Short Answer Type.
1. Find the roots of the quadratic equation 9x 2 − 9 (a + b )x + (2 a 2 + 5ab + 2 b 2 ) = 0.
1 2
2. Which term of the progression 19, 18 , 17 , ... is the first negative term?
5 5
Or From the top of a 10 m high building, the angle of elevation of the top of a tower is 60° and the angle of
depression of its foot is 45°. Determine the height of the tower.
3. In the given figure, AB is the diameter of a circle with centre O and QC is a tangent to the circle at C. If
∠CAB = 30°, then find ∠CQA and ∠CBA.

30°
A B Q
O

Or A copper wire 4 mm in diameter is evenly wound about a cylinder whose length is 24 cm and diameter 20
cm so as to cover the whole surface. Find the length and weight of the wire assuming the specific density to
be 8.88 gm/cm 3 .
4. Draw a circle of radius 6 cm. Take a point P on it. Without using the centre of the circle, draw a tangent to
the circle at point P.
178 CBSE Term II Mathematics X (Standard)

5. The mean of the following frequency table is 50 but the frequencies f 1 and f 2 in class intervals 20-40 and 60-80
are missing. Find the missing frequencies.
Class interval 0-20 20-40 40-60 60-80 80-100 Total
Frequency 17 f1 32 f2 19 120

Or Find the volume area of the largest right circular cone that can be cut out of a cube whose edge is 10 cm.

Section B (5 Marks Each)


This section consists of 4 questions of Long Answer Type.
6. A cone of maximum size is cut-out from a cube of edge 14 cm. Find the surface area of the remaining solid left
out after the cone is cut-out.
Or The angle of elevation of a jet plane from a point A on the ground is 60°. After a flight of 15 sec, the angle of
elevation changes to 30°. If the jet plane is flying at a constant height of 1500 3 m, find the speed of the jet
plane.
7. If α and β are the zeroes of the quadratic polynomial f (x ) = 3x 2 − 4x + 1, find a quadratic polynomial whose
α2 β2
zeroes are and .
β α
Or If m times the mth term of an AP is equal to n times its nth term, then show that ( m + n ) th term of the AP is
zero.
8. Construct a tangent to a circle of radius 1.8 cm from a point on the concentric circle of radius 2.8 cm and
measure its length. Also, verify the measurement by actual calculation.
9. In the given figure, PT is a tangent and PAB is a secant. If PT = 6 cm and AB = 5 cm, then find the length of PA.

O
P
A B

Or Find the mean, mode and median of the following data.


Class 0-10 10-20 20-30 30-40 40-50 50-60 60-70

Frequency 3 4 7 15 10 7 4

Section C (1 Mark Each)


This section consists of 1 Case Based comprises of 5 MCQs.
10. In one corner of the drawing room, a flower basket is kept inside the glass, lies on the table. The basket is
designed in such a way that every one pleases to see it.
CBSE Term II Mathematics X (Standard) 179

The shape of flower basket is hemisphere with radius 60 cm and upper shape is conical with height 120 cm from the bottom
surface.
P
F E

120 cm

180 cm
O
D

60 cm 60 cm


A B

(i) Find the capacity of the glass.


14.256 3 12.256 3 142. 56 3 14.256 3
(a) m (b) m (c) m (d) m
7 7 7 5
(ii) Find the volume of the cone.
(a) 0.54 m 3 (b) 0.45 m 3 (c) 0.25 m 3 (d) 0.52 m 3
(iii) Find the curve surface area of hemisphere.
(a) 0.201 m 2 (b) 0.104 m 2 (c) 0.102 m 2 (d) 0.401 m 2
(iv) The volume of two combined figure is equal to the sum of
(a) two individual volumes (b) two individual curve surface area
(c) volumes and curve surface area (d) None of these
(v) If the cost of painting the glass outside is ` 1.20 per m 2 , find the total cost of painting the CSA of the glass.
(a) ` 55 (b) ` 55.02 (c) ` 57 (d) ` 57.02

Answers
(2a + b ) (a + 2b )
1. , 2. 25th term or 10( 3 + )1 m 3. ∠CQA = 30° and ∠CBA = 60° or 4.21 kg
3 3
5. f1 = 28 and f 2 = 24 or 261.9 cm 2 6. 1022 + 154 5 cm or 720 km/h 2

⎛ 2 28 1⎞
7. k ⎜ x − x + ⎟ where k is any non-zero real number 8. 2.14 cm
⎝ 9 3⎠
9. 4 cm or (i) 37.4, (ii) 36.15, (iii) 37.3 10. (i) (a) (ii) (b) (iii) (c) (iv) (a) (v) (d)
180 CBSE Term II Mathematics X (Standard)
Mathematics (Standard)
Class 10th (Term II)

Practice Paper 3 *

(Unsolved)
CLICK HERE TO GET MORE TERM 2 AND NTSE MATERIALS
Instructions Time : 2 Hr
Max. Marks : 40
1. The question paper contains three sections A, B and C.
2. Section A has 5 questions with 3 internal choices.
3. Section B has 4 questions with 3 internal choices.
4. Section C has 1 Case Based MCQs comprises of 5 MCQs.
5. There is no negative marking.
* As exact Blue-print and Pattern for CBSE Term II exams is not released yet. So the pattern of this
paper is designed by the author on the basis of trend of past CBSE Papers. Students are advised
not to consider the pattern of this paper as official, it is just for practice purpose.

Section A (3 Marks Each)


This section consists of 5 questions of Short Answer Type.
x − 3 x + 3 48
1. Find the solution of the equation − = , x ≠ 3, x ≠ −3.
x+3 x−3 7
2. Write the expression a n − a k for the AP a, a + d, a + 2 d, …
Hence, find the common difference of the AP for which 25th term is 10 more than the 23rd term.
Or A statue 1.6 m tall stands on the top of a pedestal. From a point on the ground, the angle of elevation of the
top of the statue is 60° and from the same point the angle of elevation of the top of the pedestal is 45°. Find
the height of the pedestal.
3. In the given figure, PQ and QR are tangents to the circle centre O, at P and R, respectively.
Find the value of x.
R

20°

S O 50° Q


P

Or Draw a circle with the help of circular solid ring. Construct a pair of tangents from a point P to the circle.
4. For grouped data, if Σf i = 20, Σf i x i = 2 p + 20 and mean of distribution is 12, then find the value of p.
CBSE Term II Mathematics X (Standard) 181

5. A vessel is in the form of a hollow hemisphere mounted by a hollow cylinder. The diameter of the hemisphere is 14
cm and the total height of the vessel is 13 cm. Find the inner surface area of the vessel.
Or Two pillars of equal height are on either sides of a road, which is 100 m wide. The angles of the top of the pillars
are 60° and 30° at a point on the road between the pillars. Find the position of the point between the pillars.
Also, find the height of each pillar.

Section B (5 Marks Each)


This section consists of 4 questions of Long Answer Type.
6. A solid toy is in the form of a hemisphere surmounted by a right circular cone. Height of the cone is 4 cm and
the diameter of the base is 8 cm. If a right circular cylinder circumscribes the solid. Find how much more space
it will cover?
Or The angle of elevation of the top of a tower from certain point is 30°. If the observer moves 20 m towards the
tower, the angle of elevation of the top increases by 15°. Find the height of the tower.
7. Construct a tangent to a circle of radius 1.8 cm from a point on the concentric circle of radius 2.8 cm and
measure its length. Also, verify the measurement by actual calculation.
Or From an external point P, two tangents PA and PB are drawn to the circle with centre O. Prove that OP is the
perpendicular bisector of AB.
8. Solve the following quadratic equation by factorisation method.
1 1 1 1
= + + , a + b ≠ 0.
a+b+x a b x
Or The angles of a triangle are in AP. If the greatest angle equals to the sum of the other two, then find the angles.
Also, conclude that find these angles are multiple of which angle.
9. The median of the distribution given below is 14.4. Find the values of x and y, if the total frequency is 20.
Class interval 0-6 6-12 12-18 18-24 24-30

Frequency 4 x 5 y 1

Section C (1 Mark Each)


This section consists of 1 Case Based comprises of 5 MCQs.
10. Suppose, there are two windows in a house. A window of the house is at a height of 1.5 m above the ground and
the other window is 3 m vertically above the lower window.
C

h – 4.5 m

B 30°
(Shyam) D

3m 3m

(RAM) A E

1.5 m 1.5 m

P Q
182 CBSE Term II Mathematics X (Standard)

Anil and Sanjeev are sitting in the two windows. At an instant, the angles of elevation of a balloon from these
windows are observed as 45° and 30°, respectively.
(i) Find the height of the balloon from the ground.
(a) 6.8 m (b) 8.6 m
(c) 9.4 m (d) 9.6 m
(ii) Among Anil and Sanjeev, who is more closer to the balloon?
(a) Sanjeev (b) Anil
(c) cannot say (d) None of these
(iii) If the balloon is moving towards the building, then will both the angles of elevation remain same?
(a) cannot say (b) Yes
(c) No (d) None of these
(iv) If the height of any tower is double and the distance between the observer and foot of the tower is also doubled,
then the angle of elevation
(a) remain same (b) become double
(c) become triple (d) None of these
(v) Suppose a tower and a pole is standing on the ground. And the angle of elevation from bottom of pole is θ1 and
elevation from top of pole to the top of tower is θ 2 .

θ1
B

θ1
A

Choose the correct option.


(a) θ1 > θ 2 (b) θ1 = θ 2
(c) θ1 < θ 2 (d) None of these

Answers
9
1. −4, 2. 5 or 0.8( 3 + 1) m 3. 45° 4. 110 5. 572 cm 2 or 25 m, 43.3 m
4
128
6. π cm 3 or10( 3 + 1 ) m 7. 2.14 cm 8. x = −a or x = −b or ` 30°, 60° and 90° angles are the multiple of 30°
3
9. x = 4 and y = 6 10. (i) (b) (ii) (a) (iii) (c) (iv) (a) (v) (a)

You might also like